You are on page 1of 325

2010

Secondary School
Examination
Papers

Secondary 4 Express
Physics
Paper 1 & 2

1 Anderson Secondary School SA2


2 Anglican High School SA2
3 Bukit Panjang Government High School SA2
4 Cedar Girls Secondary School SA2
5 Chij St. Nicholas Girls School SA2
6 Chij St. Theresas Convent SA2
7 Holy Innocents High School SA2
8 Hua Yi Secondary School SA2
9 Hwa Chong International School SA2
10 Kuo Chua Presbyterian School SA2
11 Methodist Girls School SA2
12 Nan Chiau High School SA2
13 Queeensway Secondary School SA2
14 St. Margarets Secondary School SA2
15 St. Patricks School SA2
16 Temasek Secondary School SA2
ANDERSON SECONDARY SCHOOL
SCIENCE DEPARTMENT
PRELIMINARY EXAMINATIONS
An astronaut conducted a pendulum experiment on the moon. The acceleration due to
gravity of the moon is one-sixth that of the Earth. The angle of oscillations of the fixed
CANDIDATE length pendulum is 20 and each complete oscillation takes two seconds. If the angle
NAME of the oscillations is reduced to 1o, each oscillation will lake
ClASS J
!NDEX
NUMBER Ll J A 0.3-s B 1.0 s c 1.3s b 2.0s
2 The diameter of a clay ball is measured using a micrometer screw gauge. Dean takes
5058/01 an intial zero error reading and then a reading of the diameter. The diagrams show an
PHYSICS 13"' September 201 o
1 hour enlargment of the screw gauge readings.
Paper 1 'l'heory
iero reading d iameter reading
Addlllonal Materials: MuKIP.I Choice Answer Sheet
READ THESE INSTRUCTIONS FIRST
WMte In soft pencil.
Do not use staples. paper clips, higlightets, glue or correction nuld.
WMte your name, Class and Index number on the Answer Sheet In the sp11ces provided unless \/Vnat is the diameter of the ball?
lhis has been done for you. A 1.42 mm B 1.92 mm C 1.98 mm D 2.04 mm
Thefe are fOrty questions on this paper. Answer all questions. For each question there ~re four
possible answers A , B , C and 0. . . It takes 36.5 s for a pendulum to swing from X to Y and back again twenty times.
Choose the one )'OU conisder correct end record your choece m soft pencil on the separate
Answer Sheet
Read the Instructions on the Answer Sheet very ~refully.
Each cotTect answer Will score one mark. A malll Wid nQt be deducted for a wrong answer.
Any rough worl<ing should be done In this l>ookleL
Take g , ecceler11tlon due to gravity (on earth) to be 10 ms" and c, the speed of light In 81' lo be 3
1o' ms"unless otherwise slated. What is the frequency of the pendulum?
A 0.274 Hz B 0 .548 Hz C 1.83 Hz D 2.74 Hz
4 A runner lakes 9 minutes to complete a 2.4 km route. Wnat is his average speed?
1
A 0.26 ms1 B 3.75 ms1 C 3.9 ms D 4.4 ms
TI1is document consists of il printed pages including this cover peQo
PR.liLIMJNJ\KY EXMINJ\Tf0NS 2010
J\NDf.R$0N SECONUA t<Y SCHOOL
ScUer 1'llfr J-;r,,ut Yu
Ph,..su:' P11pu I
5 A toy car is traveling at a constant speed to the left. Balls are projected \lertically
upwards from the toy, one at a time, at one-second intervals.
.
3
Neglecting air resistance, which diagram best shows a possible snapshot of tre toy car ; The density of a meta'.bar is 12 g cm.J. When a hole of volume 1.0 cm is drilled into
and the balls after the third ball has been projected? the bar, what wili be its density be?
A

B
,. e 12.0 g cm-3 D 13.0g cm-3
el' CD e A w ind;nlll is pushed by four external forces as shown.
~ ~ 10~ 2m
2m
. m
c
D
I 8N
2m
G
e
F
IM'lat are the possible sets of forces for F and G for the windmill to be stationary?
~ ' ~ SetA:F= 10N, G.,.8N
Set B: F = 8 N, G = 10 N ,
Set C: F = 2 N, G = 16 N
5 A skier is traveling downhill. The acceleration on hard snow is 4 ms2 and on sott snow
is 2 ms-2 Vllhich graph shows the motion of the skier when moving from hard snow to A Set A only.
soft snow? B Set Conly.
A s B c Sais A and B only.
0 -Sets A, B ar.d C.
:> The diagram below shows a lamina of 1Jn:form thi<:M~ss.
lime/a Thnol
s 10 5 10
c Speed/
D
Spe
Where is the centre or gravity of the lamina most likely to be located at?
' 2
A lift is moving downwards at a constant acceleration of 2 ms . Given that the li't has a
20 mass of 500 kg, what is the tension of tre cable supporting the lift?
A 2500 N B 4000 N C 5000 N D 6000 N
<)
Tlrnol
5 10 5 10
ANDSRSONSEC0N0ARYSGHOOI PRel.IMJNARY F.XMil"'AT!ONS 2010
l'h~1n Paper
I Sr:ttcr: Mr SnMTt Y11
ANDERSON SECOtlOARY SCH<Xll. l'IU-J.IMINARY EXMiNATIONS 2010
r1.,,.fic.r/41~rl s~"". "'' ~3<l r11
11 A floor has a maximum loading capacity of 5 kN m-2. Vl/hich of the following is allowed
on this flOO<?
15 Clara observes the Brownian motion of smoke particles in air with a microscope. She
An elephant weighing 50 000 N stands on 4 legs with each foot covering 400 cm2 sees moving points of light. These points of ligt)t comes from
A of the floor.
A 50 kg lady wears a pair of high heel shoes. Each shoe has a contact area of 9 A air particles only moving random ly.
6
cm2 with lhe floor.
B smoke particles only moving randomly.
1,; A 100 kg washing machine stands on a surface area of 6500 cm2 ,...
C smoke particles only vibrating.
A designer's table weighing 1000 N stands on 3 legs. Each leg has a contact area
0 2 D both smoke and air particles moving randomly.
of 0 .1 cm with the floor.
12 A ir is trapped in a narrow tube sealed al one end. \/Ihlen the atmospheric pressure is 113 A fixed mass of gas is heated while kept at constanl volume. How do the properties of
75 cm Hg, the length of the trapped air column is 10 cm as shown. the molecules of the gas change?
~=~f.::
Frequency o f Average
Average Speed
collision with walls distance apart
A Unchanged Increases Increases
B Increases Increases Decreases
Assume temperature is kept constant throughout, wh!JI is_the pressure of the trapped c Increases Unchanged Unchanged
air? D Increases Increases Unchanged
A 6cm Hg B 85 cm Hg ~ 90 cm Hg D 100cm Hg 17 \Miich of the following can be used to measure the temperature of boiling mercury?
1~ A metal block or mass 20 kg is pushed up a rough inclined plane as shown below. 1 mercury-In-glass thermometer
2 alcohol-iniJlass thermometer
~
3 thermocouple themlometer
A 3 only B-1only C 1 and--2 only D 1 and3 only
....
The total work done on the block from A to Bis 1000 J.Vl/hat is the work done against 18 Expanded polystyrene is often used to make containers for storing ice-cream because
friction? the trapped air reduces loss of thermal energy by
A 20 J B 400 J c 500 J D 600J A radiation only.
14 Bryan runs to the top of the science block in 1 minute. There are 50 steps, each 0.2 m
B conduc tion only.
in height. What is Bryan's power if his weight is 750 N? C conduction and convection.
D conduction, convection and radiation.
A 2.SW $ 125W C 625W D 1250 W
AND!:,RSON SEC0 Nl)ARY SCHOOi. PRELIMJNAR Y EXMINA TIONS 2010
1'hy..0<$Popt1 I Scflcr: Mr Eni a.tl Yu
ANDERSON SGCONIJARY SCHOOL PRliLIMINARY GXMlNATIONS 2010
Ph)'1'CI Peru I Srltt'f', M r .r,.c.rl J'u
19 A beaker contains 200 ~f water at 5 ,e_ St
cold water as shown below earn at 100 C is blown into the beaker of
ir the beaker reaches 90 oc_The process stops when the final temperature of the water
~
Z.2 A painter standing at the centre of a rectangu~c room looking into a 1 m long mirror at
the opposite end of the room. 1 ,.1onq ,,,.,;.;
0 .,..,....
Calculate the f1rtal mass of watert n the beaker, 1;iven the following:
Spe<:!flc heat capacity of wat~ 4.20 J g-'K"' How much of the painted wall can he see through the 1 m long mirror?
Spec~f1c latent heat of vapourlsation of water: 2260 J g-
Specific latent heat of fusion of ice: 336 J ., A 1m B 2m C 3m D Sm
9
A 31.Sg B 538 23 Given that the refractive index of glass is 1.5 and the refractive index of air is 1.0. what
. g C 231.6 g. D 253.8 g is the critical angle when a light ray tra_vels from glass to air?
2C ~;~v~~~-ner~tor which makes 5 oscillations a second is used to produce waves in a A 22.6 a 3o.o c 41 .8 o 4a.s
24 Which of the following statements about ultrasound is/are correct?
1 Ultrasound has the same frequency as the sound wave that we hear.
2 Sound waves that human being cannot hear is known as ultrasound.
3 Ultrasound travels much faster than the sound wave that we hear.
12om
VVhich of these is correct for-the wav_es produced? A 2 only B 2 and 3only C 1 and 3 only D None of the above
Frequency/Hz W avel ength/cm 25 A positively-charged sphere and an uncharged sphere Q , each mounted on an
Spe ed/cm s 1
A 0.5 insulating stand. are placed in contact Q is also connected to earth by a wire.
12 6
E s 3 15
c 5 12 240
D 20 3 60
~~a~~ station broadcasts at 93 800 MHz. VVhat is the wavelength or the radio wave in
71
Vv'nen the wire is removed and \hen P and Q are separated, Q 1s found to be
negatively-charged. What is the nature of materials P and Q?
A 3.20x1oJm B 313m C 320m
D 2.8x1019 m p Q
A Conductor Conductor
El Conductor Insulator
c Insulator Conductor
D Insulator Insulator
PREl.IMINARY f:XMINA TIONS 201 O
St!llffr_ Mr Emut 1-u
ANDERSON SEC.)ONDARY SCHOOL l'RJ;.L.lMINARY b").'MJNAl10NS201<J
PhJ:Jfcs l'apw I Stll'-,. M,. .U"'e.ti }'u
26 Which diagram best illustrates the electric field around a positive point charge?
A
29 A 2 m long wire with a cross-sectional area of 1 mm2 has a resistance of 16 0. What is
the resistance of the wire if ii has a cross-sectional area of 2 mm2?
* *
A 4Q 8, 80 c 32'2 D 64Q
:\0 A household a.c. supply is connected to the circuit as shworibelow.
c D
'Mlich of the following diagram show tt>e voltage V across the resistor?
27 40 J of energy was withdrawn from a battery as ii was used for 20 s. If the e.m.f. of the
battery was 2 .0 V, how much of electrical charge was withdrawn during'.that time
period?
A 0.1 C B 2C C 20C D SOC
Ti\/". ~~ .
O
VIV
t Is.
D
tis
~.;.
28 The d iagram below shows how the current, I, through a filament lamp changes wit!" the
p.d. applied across it.
v
tis
0
It is not a straight line graph because
31 The diagram shows a thermistor in a circuit.
A electricity is used to produce light.
8
C
D

the resistance of the filament increases.


the filament of the lamp is of uneven lhickness.
the potential difference from the battery is not stable. Q
What happens as the temperature of the surroundings increases?
A The light bulb will flash intermittently.
B The light bulb will get dimmer.
q The light bulb wlll get brighter_
D The light bulb is unaffecfed.
ANDEHSON S1'C'.ONPARY ~CIJOOJ, l'RIB .tMINA.R Y EXMINA TIONS lOIO
Plt)diCJ l,.npu I St1ter: Nf, l\,.ne.it Yu
ANDERSON SECONDARY SCHOOL PRELIMINARY EXMINATIONS 201o
Pi.y,tcs Pop<r I S~ttu: Mr Ern~st Yu
1:
32 \Nhen th~ term~nal~ T1 and 2 of the Y-lnput of a CRO are connected to points X and y
o~ the g1~en c1rcu1t respectively, the bright spot is deflected to P as shown in the
display gnd. 34 If 1k~ of electricity cost $0.20, how much does it cost to switch one spot light rated
Time bae at 2 kWfor6 h?
10~?
A $0.60 {I $2.40 C $15.00 0 $60.00
1~ 3!) Three st\eels of material are placed between magnets and iron nails as shown in the
mscmt
10*"

Y11aln
~m
1~

vcm-1
' \ , 1ten nal '\, kon nan ~ lton nail
VVhich of the following desaibe the force on the nail for each material?
Lead Iron A l uminium
AttracLion Attraction Attraction
If T1 is next connected to Z (T2 still at Y), where will the bright spot be deflected to? A
B Attraction Attraction Repulsion
AO BR CS O T Attraction
c Attraction No Force
D No Force No Force No Force
X3 A 3-p~n plug. is wrongly wired as shown in the diagram. It is connected to a washing
machine which has a metal case.
36 Plotting compasses X and Y are placed on the right and on the left of a current
carrying wire respectively as shown in the diagram.
In which oirections will the compass needles point?
Yellow and green wire A B D
CUJrent dtctlon C~tonl diteehon
~t~ ~t~
V\otlich of the following is TRUE?
A
Iii
The washing machine will not function when it is switched on.
The metal case of the washing machine will become 'live'.
cDjaS cDjaS
C The 13 A fuse in the plug 1s likely to blow as soon as the socket is s witched on
0 The main circuit breaker will trip as soon as the appliance 1s switched on.
ANOnRSONSEC<>NDARYSCHOOL II l'RELlMlNARY EXMINATlONS 20 10
I
Plr)'$JCI Popu
ANDERSON Sl,CONDA RY SCUOO\, 12 l'l<F.LIMIN"RY EXMlNATIONS 2lll0
Sclkr: >h Emr..IJ Trt
Ph..mc.sr/J/"r I
37 A positive charge enters a region of magnetic field as shown below.
A conducting wire cuts perpendicularly through a region of magnet.ic field at a speed v
:"~lot"~ge troYeling into a
40
as shown.
o-..

The charge will
A not be affected and continue to travel straight through.
B experience an upwards force.
C experience a downwards force.
D experience a force into the paper.
11vt11ch of the following lncre~ses the induced e.m.f. ?
36 Vllhich of the following increases the turning speed of a simple d.c. motor?
~ increasing v
1 Increasing the number of tums of the coil 2 decrease the gap netween the bar magnet
2 Increasing the input d.c. voltage '3 use a pair of stronger magnets
3 Inserting a copper core 1n the coil
A 1and2 only
A 1 and 2 only. B 1and3 only
B 1 and 3 only. C 2and 3only
C 2 and 3 only. D 1,2and3
D 1, 2 and3.
39 Which of the following is/are done to minimise heat loss by industrial power
transmission cables?
~ Increase the cross-sectional area of the cable
2 Step down the voltage of the wire
3 Use a wire with better insulation
.A 1 only
B 1and2 only
C 1and3 only
D 1,2and3
END OF PAPER
l\t'DERSON SF.CONDA II V ~CHOOL 13 1'1<1'.l.IMINl\RV EXMfNA'l'lONS 2010 PIUil.!MlNl\HY EXMINATIONS 2010
Phy.riaPapr I Scilttr Mr Ern~s1 ru ANDERSON SECONl) l\K Y SCHOOL 14
s~11er' 1\fr P,mt!..11 J'11
l'Jryuu Pper I
5S
Section A
2010 Prelims Physics Paper I 505811
ANSWER - andcrson Answer all the questions in this section.
Mr Koay used a data-logging system to record the speed of a falling
D 11 c 21 A 31 c brick
2 B 12 c 22 c 32 c Motion sensor
3 B 13 D 23 c 33 B LJ
Data-logger Concrete brick
4 D 14 B 24 D 34 B
~ CJ
I
5 D 15 B 25 c 35 c
6 A 16 D 26 A 36 c
7 B 17 A 27 c 37 B I
8 D 18 c 28 B 38 A bench
9 B 19 c 29 B 39 A
10 B 20 B 30 c 40 D
ground
~
The concrete brick of mass 2 kg was released from rest under a
motion sensor. The data logger obtained the following data.
speedfms 1 0.00 0.00 0.00 0.45 0.85 1.50 1.70 2. 10 I 2_55 2.95
time/s 7.80 7.85 7.90 7.eS a.oo a.o5 a.10 s. 1~ e.20 I 8.25
(a) Using the data from the t able above, plot a speed-time graph in the
grid given. [4]
This document consisrs of JR printed pages including lhis cover page.
ANDERSON SECONDARY SCHOOL PRlllMARY EXMINATIONS 2010
Phy.nc.'r Poper 1 Seller: Mt E'J'ne.11 Yu
Speed/mis
(e} Explain why there is a difference in the values obtained in (c} and
3 50 Ou:C..:r..TimaT4 and a (d). [1]
decom~essor
are ~ea to see t:-:is picture
There is energy loss due to air resistance resulted in falling object
Cannot accept 1ust friction.
300
2 A metal electric kettle, rated 230 V, 8.0 A , contains some water. It is
2 50 placed on a balance as shown in the diagram.
150
1. The kettle is switched on and when the water is boiling, the reading on
the balance is found to decrease by 0. 16 kg in 3 m inutes.
0 (a) Describe in terms of the motion of particles, how thermal energy is
conducted through the metal walls of the kettle into the water.
[3)
000 --~~+-~~of'---+---+---+---+---+----+----+-~ When the kettle is heated up, the metal molecules of the kettle
780 185 790 7 95 00 805 810 815 8.20 e2511mels
walls gain energy and vibrate faster about their fixed positions,
(b) From your graph, estimate the d istance travelled by the brick
[2] [1] and trasnferring energy to their adjacent molecules [1).
y, x 2.95 )( 0.35 0.516(0.490 to 0.542) These vibrating molecules in turn transfer energy to the water
molecules in ~ntact with the_k
_e~
tt_
le
~ w_
a_lls
_.~[1~]_ _ _ _ _ _ _ __
Distance travelled by the brick is 0.516 m
- ------ ---------
(c) Determine the loss in potential energy of the brick during the (b} Calculate
journey in (b}. [1) (i) the power rating of the kettle, [ 1]
p IV
=
P.E. mgh 2 x 10 x 0.516 10.32
The loss of potential energy is 10.3 J 8.0 x 230
1 840
(d) From your graph, find the maximum kinetic energy of the brick
(2) The power rating of t he kettle Is 1 840 W
Max velocity= 2.95 mis
KE. = Y,mv2 = 'h x 2 x 2.962 8.70
The maximum kinetic energy of the brick is 8.70 J
ANDERSON SFC'ONDARY SCHOOL PREL!MARY EXMINAT IONS 2010 ANDERSON SECONDARY SC'HOOL PRELIMARY EXMJNATIONS 2010
Plty~fl'.fPa1>fr } Stlf("r Mr t.r1~j'I >'- Ph)..~ics Po~r 1 Se11er: Mr Er11es1 >'11
(ii) the specific latent heat of vaporisation of water. state one (b) Why must the bird work out the depth of the fish" before it dives?
assumption made. [3) [1]
Assume no heat loss to the s urrounding of kettle [1]
a ml, The bird would not be seen at the real depth of the fish . Fish would
Iv (1840x3x60) + 0.16
2.07 look nearer than it 1s.
The specific latent heat of vaporization of water is 2.07 MJ/kg (c) On the diagram draw a ray to show how light from the fish travels to
the kingfisher. [2]
(c) Explain why the surface of the kettle is highly polished (1)
(d) Sometimes the kingfisher might not even see the fish especially
Shiny surfaces are bad emitter of heat. So water in the kettle when the fish is at a particular position. This is a phenomenon
common to light when it travels from water to air. Explain what this
will be kept warm for a longer period time [1] phenomenon is and how it can occur. [2)
3 K ingfishers usually hunt from a perch a few meters above the water. Total internal reflection has occurred.[1)
swooping down at up to 100 km/h to snatch their prey, before returning
to the perch. The whole process is an extrordinary feat of calculation, Light must travel from the water to air (denser to less dense
not only must the bird work out the depth of the fish before it dives, but
it also compensate for the effect of refraction in the water. In addition, medium). The angle of incidence must be greater than the critical
these birds have special features in their eyes to cut out reflection and angle.[1]
see their prey better.
[Extract from Focus magazine, Issue 219} 4 The digram below shows a wave approaching a beach at a speed of
2.0 m/s and the depth of the seabed at the beach.
2.0 mis
air
Fresh water
A B c
(a) The wave takes 3.0 s to travel from A to B.
(i) What is the distance between A and B? [1)
distance = speed x time
2.0 x 3 6.0
The distance between A and B is 6.0 m
(a) Explain what is meant by the refractive index of water? [1)
(ii) What is the wavelength of the wave? (1]
It is the ratio of speed of light in (vaccum)air and the speed of lig.!!.L
Wavelength = 6.0 / 2.5
in water 2.4
The Wavelength of the wave is 2.4 m
ANDERSON SECONDARY SCHOOL PREllMAAY EXMl:"<ATIONS 2010 ANDERSONS~CONPARYSCHOOL Pl\l,ll\1ARY EXMINATIONS 2010
l'hy"''h'\P(lpc"r1 Slllltr: Mr &nesr Yi1 Phys1cJ Pnr>r 1 ~ll~r- Mr Erm~..'fl Yt1
(b) State what happens to the wavelength as the wave travels from B (b) Using a scale of 1 cm representing 20 cm for object and image
to e. [1) distance, draw a ray diagram in the grid below. In your diagram
show relevant rays. Label the focal points, image, object, and
The wavelength of the wave decreases gradually as it appraches object and image distance from the lens. [3]
C. Focal length of lenses is 30 cm from question. Scale 1 cm rep 20 cm
(c) A object is seen floating on the water. State the direction (if any) in
which the object will move in the next instant. [1 ) Image
Labels
ray
Upwards
(d) Determine the frequency of the wave. [2)
Frequency speed I wavelength
2.0 12.4
0.833
The frequency of the wave is 0.833 Hz
5 !star performs an experiment to investigate the relationship between
obj ect and image distance by using means of a thin converging lens.
The setup is shown below. (c) 'From your ray diagram, determine the distance between the object
and the lens. [ 1)
120 cm (accept 116 to 124 cm)
6 When spray are used to paint the metal car body panels, the result is
sometimes patchy, with some areas being missed. One way to
overcome this problem is to use 'electrostatic spraying' as shown in the
diagram below.
00 00 nleo z. z r
0 00 -
o :o:
The lens is first placed such that it is 60 cm from the object. The screen
is moved to obtain a clear image that is an inverted replica of the
object.

Paint droplets
(Positively charged) np int
The lens is now moved a certain distance away from the object. When (a) Explain why the droplets spread out as they leave the nozzle.
the translucent screen is moved 40 cm away from the lens, a clear [1]
image is formed on it.
The droplets spread because they are of the same charge
(a} Draw, in the box below, the image observed from behind the
~
[1] {positive). Like charges repel each other.
'""""''"''a""
ANDERSON SF('ONDARV SCHOOL PRELIMARV EXMINATIONS 2010 ANDERSON SECONDARY SCHOOL l'REUMAAY f.XMINATIONS 2010
Physics Poper': Sef/er: Mr /}n<'SI Y11 />11y.\i<:.'i Poper 1 Seflt>r: Mi- Erne.vi fo
(b) State one advantage of using electrostabcs in this way to pamt the (c) s, and S2 are closed. [2)
metal parts. (1)
Total current in the circu it (A1 and A2) =5 + (1O + 10) =0.25 A
Less paint is neeeded. Paint more evenly spread out. Either one = =
Reading on V1 V2 5 + 2 2.5 V =
(c) Suggest how the paint droplets become positively charged upon
8 Ultrasonic waves are emitted by a transmitter to measure the depth of
leaving the nozzle. [3)
the sea. A receiver is used to detect the reflected waves and the
signals are shown on a cathode-ray oscilloscope or CRO.
The paint droplets rub against the nozzle as they leave[1] causes
:--m-
~ ,---r-; 1-T-ji -_- ----;;ifl
electrons from the paint droplets to be transferred to the
t-+- I __1_L+--1I ~.
1 I I I
' 'i '-t - -+-1
! 11
nozzle[1). Droplets has less electrons becoming positively I I ' ; I J: l , . : i ~W
T~-,,~1_ 1rr-: 11 1 11
...E!!a~---------~--------- ~ ...--!-----, I -,- ,-- - -.. -,--,
1 1 ! j I t ; ' I
~
1 I
l____j ____ : _I _!
I '
- ---
I !
7 The digram below shows a DC circuit. You can assume the voltmeters (a) What do the two pulses shown on the CRO represent? [2]
have infinite internal resistance and the ammeters have negligible
internal resistance. The bigger pulse represents tge uktrasonic signal emitted bt the
source. (1) The smaller pulse represents the signal reeflected
from the seabed.[1 ]
(b) If the speed of sound in seawater is 1200 mis and the time base of
the CRO is set to 0.2 second per division, calculate the depth of the
~- ~
Determine the reading in all voltmeters and ammeters when
(a) s, and S2 are opened, (2} Time between the two pulses 10 x 0.2
No current will flow through V1 and Az so they will have O value 2s (1 )
V2 has high resistance so reading will be = e.m.f 5 V =
A 1 = O since current will be negligible as resistance is almost
infinite. Distance travelled by the wave 2 x 1200
2400m [1 J
(b) s, is opened and S2 is closed, [1)
The depth of the sea 2400 /2
V1 and V2 has almost infinite resistance hence negligible current 1200m [1 }
= =
flowing through. Hence V1 OV V2 2.5 V
=
Hence A 1 and A2 0.25A
ANDERSON SECONDARY SCHOOL 9 PR!:LlMARY EXMINATIONS '.lOIO ANDF.RSON SECONDARY SCHOOL 10 PRELIMARYEXMINATIONS 2010
Phy:c1c.w1'1TJJf!rZ St1uer: Mr ti-nest l'i1 Phyt;ic.J l'~r 1 S1111r: Mr Erwu H1
Section B (a) Why must an alternating current be used by the coil to produce
heat in the pot? (2)
Answer all the questions in this section. A.C. is needed so that an a changing magnetic file beproduced
Answer only one of the two alternative questions in Question 11.
In the coil.[11 This field will induce an alternating current in the
9 Read the following passage about the induction cooking stove.
pot.(1 I
Cast Iron pot
(b) State two ways in which heat can be generated in the pot.
(2)
Current flowing in the pot will generated heat loss through
the resistive metallic pot (12 R). [1)
Heat produced by hysteresis losses.~----------
(c) Jenelle uses a pot made of copper as a cooking vessel as he
Figure 9 thinks that since copper is almost a perfect conductor of
electricity he will save energy by decreasing the time to cook his
An induction cooker uses a type of inductten heabng for cooking. It is chiefly distinguished food using his pot. 1s he right? Explain. (2)
from other common forms of stovetop cooking by the fad that the heat is generated
directly in the cooking vessel, as opposed to being generated in the stovetop (as in the No, [1] heat produced depends on resistance in th~otc...._ _ _ _
case of electrical coils or burning gas) and then transferred to the oooking vessel.
Material with no resistance would not generate as much heat loss.
In an induction stovetop, a localized coil of copper wire is placed underneath the cooking
pot. An alternating electric current is made to flow through the coil, which produces an [1) Material is not a magnetic material.
oscillating magnetic field. This results in heat being generated in the co0k1ng vessel over
it.
The cooking vessel or pot for 1he above case must be made ol a magnetic material
(ferromagnetic) and etectricaDy conductrve Principally, the coil induces an electric current
(d) On Figure 9, draw the magnetic field lines generated by the coil.
in the pot, which produces resistive heating proportional to the square of the current and to Must draw field through the pot (1)
the electrical resistance of the vessel.
(e) The cooker requires a voltage of 110 V. It is connected to a
Secondly, It also creates magnetic hysteresis losses in the pot due to Its ferromagnetic mains of 230 V a.c. via a step-down transformer. Determine the
nature. The first effect dominates: hysteresis losses typically account for less than ten
percent of the total heat generated.
turn ratio of the transformer. (1)
Induction cookers are faster and more energy-efficient than tradrtional electric cooktops; Turn ratio - 23 : 11 (23/11)
moreover, they allow instant control ol coolong energy, which no energy source other than
gas olfers Bee.use induction heats the cooking vessel itself, the poss1bdrty of bum injury
is significantly less than with other methods only skin contact with the cooking vessel itself
(or. when high heat has been used, the stovetop for a while after the vessel has been
removed) can cause harm.
ANDERSON SEC01''DAAY SCHOOi. It PRF.LIMARY EXM!NATIONS 2010 Al'IDERSOSSECO~UARYSCHOOL 12 PREUMARY EX"llNAT!Ol'S 2010
PhJJic."' PCT/Xr 1 Yu~r Afr F.rr1u1 );, Ph~ /#ilJNr :! Srll~r: Mr fJJwM )"
(f) The coil draws a current of 8.0 A Given the efficency of the 10 The figure shows a U-tube manometer connected to a gas cylinder of
transformer is 80%, determine the primary current of the large volume. The atmospheric pressure is 76 cm of mercury.
transformer. (2)
230 x Ip
Ip
= 0.8(8 x 110)
3.06 -........ .
~
~1
-
A
..
The primary c urrent of the tra ns forme r Is 3.06 A
>O
"' ""
30
.,,
"'
$0
(g) State 2 possible reasons why a transformer suffers from energy "'
loss though heat. (2)
Heat loss through Eddy(induced) current. [any two)
Heat loss through resistance in the cable. (a) What is the pressure at point A and point B in the right and left tube
respectively? (2)
Heat loss through hysteresis
Pressure at point A = 76cm Hg
Pressure at point B = 76 + 30 106 cm Hg
(h) There is a safety feature missing from the induction cooker to
reduce the risk of harm caused by current leakage. Explain what
this safety feature is. (1) (b) The tap is opened and mercury ran out until the level in the left tube
drops to the 50 cm mark.
Earthing should be implemented or 3 pin plug w ith the earth cable (i) Assume that the pressure in the gas cylinder remains constant,
what is the new position of the level in the right tube? (2)
connected to the metal chassis of the cooker.
=
Level at Left hand tube dropped by 50 - 40 10 cm
Level al Right hand tube will increase by 10 cm to 20 cm mark.
(ii) Explain how you arrived at your answer. [3)
The pressure of the gas c~inder and the atmospheric J>ressure
Remains the sameJ1], hence the difference between the levels
Of the mercury in A and B remains constant. [1 J Since B drops by
10 cm, A will also drop by 10 cm to 20 cm mark.[1]
ANDER.SON SECONON<Y SCllOOL 13 PR.ELIMARY 1:.XMINATIONS 2010
Phyr.:11.,, J'aper 1 s.mer: /vfr Erm:.,1 Y11
11 EITHER
A crane with 2 beams attached, is seen lifting a load on its side. Both
beams can be adjusted_ Beam A is adjusted such that it is (c) The crane has a "wing" attachment deployed as shown in the
perpendicular to the ground. Beam B which consists of a pulley at its diagram below_
end is adjusted such that it is parallel to the ground. The mass of the
crane without the 2 beams is 10 x 103 kg. Each uniform beam has a
mass of 500 kg.
pulley
--
10m
1m BeamB Steel cable
Beam A
Explain the purpose of this "wing" attachment. State what else can
10m be done tl> the crane (without add-on modifications to the crane)
BeamA Sm
such that the neccessity of using the "wing" attachement can be
reduced . (3)
The attachement is to provide the crane with better stability.[1J
It increases the base area of the crane as well as lowering the CG
Of the crane.[1)Adding load on to the crane at the base of the
15 m
beam A or adjust Beam A and B such the resultant
Frant View Side View
The weight of the crane, W is positioned at the center of the metal moments by the load is reduced .[1]
frame of the crane.
(d) On a particular windy day, the load experience an easternly gale
force of 50 kN as shown in the diagram below.
(a) Define moment of a force_ (1)
Ille product of a force and its perpendicular distance about
about a point or pivot
3
(b) The load has a weight of 25 x 10 N. Ex.plain with mathematical
evidence whether the crane could support the load. [3) Using means of a scaled vector diagram, determine the tension in
the steel cable [3]
Total anti-clockwise moment by weight of the load and Beam B Scale I cm rep I 0 kN
=
[25000 x (10 - 2)) + (5000 x (5 - 2) 215 000 Nm 50cmor50kN
cml~
or2SkN~
(1)
Total clockwise moment by weight of crane only:
2.5
=
100 000 x 2.5 250 000 Nm (1] Not drawn to scale
cm or 57 kN
Hence the moments by the weight of the crane is greater than the
load and beam B so the crane is able to support the load. (1) Tension of the cable 1s S7 kN. (accept SS kN to 59 kN)
ANPFRSONSECONOARYSCHOOL 15 PRELtMARY EXMINi\TlONS 20 10 ANDERSON SECONDARY SCllOOL 16 PRELIMARY EXM!N ATIONS 20 I 0
Physics /'~p~r J Sttncr. Mt ErtH''llt Yrt PhyUct PQfWr] Stf11r : M r limt."I./ )'11
(b) Wai Ji designs a simple doorbell as shown in the diagram below.
OR
When sw itch S is pressed and then released, two notes of identical
frequency 'dong-dong' are produced.
{a) The diagrams below show a simple d.c motor. The ends of the
single loop coil ABCD are soldered to copper pieces X and Y which
make contact with springy metal strip P and Q A cell is connected
across P and Q. Figure 10a.2 shows an end view of the
arrangement.
~~...
LJ ________.
)s
,. . . .,.,___] .11l----_J
r-----., '<.>-1
I 11--__j (i) Explain how the two notes are produced. [2]
When switch S is dosed, iron core magnetised attracting ham mer
Figure 1 Oa.2
Figure 1 Oa.1 Hammer strikes bell and make first dong(1). Switch is
releasedCircuit is broken iron core loses magnetism, hammer
(i) Show with arrows on Figure 1Oa.1 the direction of the current in
the loopABCD and label it 1. [1] swings to other metal plate producing second dong [1 ]
{ii) State the direction of rotation of the coil. (1)
Anti clockwise direction from the front.
{1i)Wai Ji makes the following statement.
"T he bell will still work if the battery is replaced with an a.c.
(iii) Explain the working operation of this motor. Include in your source."
explanation the purpose of the comm utator. (3)
Do you agree? Explain your answer briefly. [2)
When current flows into the coil through the magnetic field of
No, (1] Current will alternative too quickly. Hammer will move
the magnet, forces will be prodcused such that the coil will turn.
A little not enough to strike metal plates.
[1 Jthe commutator resverse the current m the coil every half-
cycle[1] so that coil can turn continously in 1 direction.[1]
{iii) Suggest one way to increase the loudness of the bell. [1]
Increase current, or increase turns around core
END OF PAPER
ANDRSONSEC0NDARVSCHOOL 17 PRELIMMV EXMINATIONS lOIO ANDERSON SECOl'iDARV SCHOOL 18 PRUIMARV EXMINATIONS Z<llO
Plt)'\lt.'."'i l'OfNT 1 .'kuu: Mr &JJt>_u YN Ph)-'SJPOf1'r1 S~lh:r: Air nwst y.,
Answer ALL the questions on the answe< sheet (OMS) provided.
A cyhndncal can 1s rolled along the ruler as shown 1n the diagram below. It rolls over
Anglican High Sch ool twice before reaching the final position.
Secondary 4
Preliminary Examination 2010 mark o:.\Jtart
ing position finQal
position
cylinder
~li-tifT1TJTlr1r1r1r1r1r1r1r1r1T111~\~1~1~1....-1r-rj'l'i,..,'j'l'l'JTJ;-.,...i.-,.:;;.__
0 cm 5 10 15 20 25 30 35 4-0
What ts the radius of Ille can?
Name : Index Number : ...... - ................
A.30cm B. 6.0cm c. 12.0cm 0. 24.0cm
Class :
2. The average speed between X and Y 1n the speed-time graph shown is
PHYSICS speed I ms
Signature of P11cnt I Guardian
5058101
PAPER 1 Multiple Ch oice l)ate~ - -- - --
Fnday 27 August 201 O 1 hour
0.'--;----2-1---<31----1~ 1ime Is
A. 6.7ms' a 133ms' C. 150ms' D. 20.0ms'
Additjonel Material:
Multiple Chotee answer sheet (OMS)
:; The diagram shows a model hot-aW' baUoon. Why will the bag and the box rise in a cold
room?
Instructions to candidates:
DO NOT TURN OVER THE PAGE UNTIL YOU ARE TOLD TO DO SO
Do not use staples, paper clips, highlighters. glue. correction fluid or correction tape.
Write your name, class. index number and subject on the separate OMS provided.
Answer ALL the questions on the answer sheet (OMS) in soft pencil.
Take g 2 10 m/s2 unless otherwise stated. A. Air pressure is smaller inside the bag than outside.
B. The candle flame blows up the bag.
C. The box is lighter than air.
O Wasm air is less dense than cold air
This Question Paper consists of 13 pmted pages including this cover page.
[Turn over)
2
4 Two Identical balls X and Y were dropped from a ~Ill bUlduig at lhe same tme_ x fell 7. A car driver stepped on the accelerator when the traffic light turned green. The graph
from the top of the building while Y fell from haJ way up as shown below. below shows how the resuttant force on the car vanes wllh tme_
x
ODO
DOD
ODO
aoa
y
'""'LJ=_
o nm.
'NhlCh ol lhe folowing speed-time graph shows the corre-::t relationships between velocity Which graph shows the variation of the car's speed With trne?
-
and trne for baUs X Md Y?
speed A B
A
o
~ time 0 time
0
1
~...

0
Sp-

L::=
k~. l~~--
D speed
t
! )(
1--------
----Y 0 ~ 0
time
8 Four blocks are placed on a plank. The end of the plank, x. is then slowly ralsd. If the
5 INhen a honzontal force o1.4N is applied to a wooden btcck of mass 2 kg on a horlzontal blocks do not slip on lhe plank. which block would be the first to topple over?
surface. the block moves with a constant velocity. If the lorce 1s increased to 10N. what
win be the acceleration of the block? 1
A. 2 m/ s2 B. 3 ml s 2 C. 4 ml s' D Sm/s2
6. The densities of metals X and Y are 7.6 glcm' and 3.0 gcm' respectively. The metals of
equal mass are melted and mixed to form an alloy.
What Is the density of the alloy?
A 12Sgtcm3 9 If au the potential ene<gy of an iron ban that falls from a height ol 100 mis conllel1ed into
a 21sg1cm'
.; 4 30 gtcm'
thennal energy. what IS the nse in temperature Ol the bag? (Specific heat ca1>3crty of "'"
is 590 J/kgK)
D. 5 30 g/cm3
A. 0.5 K
9 1.69K
t. 273.5 K
D. 274_7 K
,~ In the diagram below, 3 identical uniform metre rules are pivoted at the mid-poi~I 0. In An Immersion heater 1s usea to b<>il off water at 100 c.
The total mass or the beaker and
0<der to balance the tules horizontally, weights W , A, Band C are placed on me rules as water Is found to change from 270 g to '50 g with the kilowatt-hour meter showing
shown. d~ference In lhe readingof 0.07 kWh.
1
Arrnnge in ascending order, the magnitudes of the reaction torces ( 1}. (2) and (3) at 0 . The specttic latent heat of vaporn:ation of water in kJ kg .s
(2)..., A. 4704 B. 3024 c. 2100 D 1680
T
w
~ O T
A
I l B
! !
'VI
0
l I
T
c

o. The diagram below shows a gas cyti'lder wtuch contains a constant mass or gas at a
temperature ot 30 c.
A. (1), (2). (3).
B. (1), (3), (2)
c. (2), (1). (3),
gal re-- c:ytn:ler
l
0. (2), (3), (1) .. ..----i
When the p'ston is pulled upwards, the pressure decreases. Why 1s this so?
"i. What Is the total weight of the atmosphere pressing down on a fiat horizontal roof or
dimensions 4 .0 m x 1.0 m 7 (Given: atmospheric pressure a 10,000 Fa). A. There Is a larger a ; ea or cylinder i~ contact with lhe gas
B. The Intermolecular r0<ces between gas molecules decrease.
A. 250N B. 1250 N C. 20000 N D 40000N C. The gas molecules collide with each other less often.
ti. The ga~ mc1ecules coUide with the cylinder wau less often.
1:. Wh.ch quanoty must be the same for two bodoes ,f !hey are to be"' thermal equilibrium?
'l, A thetmometer uses a p.iysical property that varies W11h temperature. Which of lhe
"- inte~I energy 13. potenl181 energy following could not be used as lhe basis f<>" a thermometer?
C temperature D. mass
A. e m I developed by two metals joined together
B. length of a lhread of mercury
~ .>. When a metal Is heated, which of the following will occur? C. volume of a frxed mass of air
?> weight of a fD<ed mass of air
I The atoms can move freely ..
II The atoms have a larger amplitude of vib<atron.
1,1 The average kinetic energ y of the atoms Is Increased. '&. The diagram shows t11e path of a ray of light as II strikes the water-to-air boundary.
A . I and II only B. II and Ill on~/ C. I aid 1:1only C. I, II and Ill only air
1 ~. A faulty thermometer roads 10'C and 90"C whon placed In melOng ice and steam
respectively. If !his lhermometer is uniformly graduated, fond the !Ne temperature "f tt>e
lhermometer reads 40'C.
A. 27 S'C B. 32.0"C C 375"C 0 42.0-1,; ll\lha1" th P99d of light in water 7 (Given f"""f nf hQhl 3 0 x 1o" mis).
A. :93x1<fmts
B. 2.30 x 1cm1s
C. 3.0C 10 mi s
D. 3.2C x 1o" mi s
1S. Two plane mirrors X and Y make an angle of 60' with each other. If a light ray PQ strikes '" A 200 m race is run on a straight track and the timing is taken using an electronic device
the mirror X at Z , making an angle of incidence of 60', the angle of reflection at mirror Y as well as manually by a timekeeper at the finishing line_ The timekeeper starts the watch
is as soon as he hears the sound from the pistol and stops the watch when the w inner
crosses the finishing line.
~-z;;
Given Iha: the speed of sound in air is 300 ms', and the electronic device records the
Q correct 11-.e as 24.0 s, whal is the time recorded by the timekeeper?
- :eo"
- 'A. : p A. 22.5 s B. 23.3 s c 24.7 s D. 25.S s
'% '
'
2~. Three energies are listed ~elow_
~-"'y (i) the energy of a 3 m wavelength radio wave
(i~
the energy of X-ray
(ii~ the energy of yellow light from
a sodium lamp
A P" B. 30" c. 60" 0. goo
Which of the following puts these energies in order of increasing magniluc.e?
2C' A ray, XY, parallel '.o the principal axis of the lens L passes through Las shown in the A_ i, ii,iii 3. I, iii, ii C. ti,iii, i D. iii, H, i
diagram below_Where w ill the other three parallel ra1s converge after passing through L?
:-T- -- -r-: - - -- ---,-,-- TTTi--~ i-TTT - 2~. What type of image is produced at the back of the human eye?
1 : 1 I I i i
! I I I , I \ ' I l ! I j A. real , upright, same sze
B real, hverted, diminished
C. virtual, upright, diminished
D. virtual. Inverted, magnnied
l i . Cl' ?S. X and Y are two light metal balls suspended using nylon threads. When a positively
Charged rod was placed between these two balls, ball X was attracted towards the
charged rod but the ball Y w&s repelled away from the rod_
I ! J i
I ! ! ! *0-~-
i ; ! i f i I
t.:=:.:.- -.J:.:~.: ~~ =1:-H-- :::i~-H----- -t++- --
r. 'N.lich of t~e following statements correctly describes the similarities between water
waves and sound waves?
A. Both travel at the same speed. W'lich ol lhese is the correct conclusion about lhe charge on each ball?
~- Both can propagate In space.
G Both can be reOected. BallX BallY
I'.>_ Both travel fastest in solids. A. negative only positive only
B. negative or neutral '
positive neutral
c_ negative or neutral positive only
2~. When souna waves travel from one medium into another, Jie wavelength changes positive or neutral
D. negative only
because
'\. the fre~uency changes.
R the speed changes.
c_ the attraction between the molecules of the medum changes_
o_ different media have different temperatures_
7 8
27. Two wires P and Q, each of the same length and the same material, are connected in 31 Which of the following is not a form of a variable resistor?
parallel to a battery. The diameter of Pis half that of 0 .
VI/hat fraction of the total current passes through P?
A. 0.20 8 . 0.25 c. 0.33 0 0.50
28. T he diagram belOw shows t he current-voltage relationships for four different types of
ohmic conductors of the same length.

-1 A
TB'
t c
~
D
32. Why can birds stand on an overtiead transmission line without being electrocuted?
A Thel" bodies have very high res1star.ce.
B' There is no potential difference between ther feet
C . Their feet are very good l(ISulators.
0 . The spaces between their feathers act as insulators.
33. The diagram shows two wires X and Y. Each wire carries current Of the same magnttude
but in opposite directions. The wires are placed in a region ol megnetic f ield, which acts
Suppo!ie 311 four conductors are connected in parallel And .Q h:.ttMy is connected across at right angles to the plane of the wires.
them.
WireX WireY
Which of the condUClors (A, B, C or 0) will allow the least amount of cha<ges to pass
through 4 at any one bme? x x x x x x p
..
Q
x x x x x x
29. A 2.5 kO resistor is connected in series with a light
dependent resistor (LOR) and a 15 v d.c. power supply.
When light shines on the LOR, its resistance drops from
12oon to 500 n.
The voltage across the LOR when light i$ shining on it is

LDRU25k0

15V

x
x
x

x
x
x

x
x
x

x
x
x

x
x
x

x
x
x
Mag n<1t: field acts
into the ~lane of the
paper
A. 2.5 v B SOV
C. 7.SV 0 . 125V IMlat will be the directions of forces on wires X and Y, 11 terms of d.-ectJon P and
dl"ec11on Q?
30. Two resistors X and Y have the same resistance and are connected in series to a battery Force on wire X Force on wire Y
of negligible intemal resistance. When the key K 1s opened, the power dissipated by X A p. p
alone is P. a p a
c. Q a
0. Q p
34. A relay is an electrical component
A. whose resistance increases W4h 11Cteasing temperature
B that allows current to flow 11 one direction only.
C wtiose resistance decreases wh increasing light intens ty
If the key K 1s closed, what is the po..er dissipated across X? O that uses a low-current circun to switch a high current ClfCUll on or on.
A 4P B. 2P
C. O.SP D. 0.25P
9 10
35. Diagram 1 shows the magnetJC held pattern near a car magnet and an object XY. 3lt Tte figure below shows the display on a CRO With the tllTl&-base and vottage sensitivity
Diagram 2 shows the pattern that is oblafled when XY is timed around. 'Mlat 1s XY? set at 2 ms/div and 4 V/d'N respect'Nely.
dag<am 1 diagrarr 2
00
A. copper rod
B A magnet with the North pole at X
C. A magnet wtth the North pole at Y
1:1. A rod of sort iron

V.hich of the following best shows the display when the settings are changed lo 4 rnS1d1v
and 2 Vldiv?
A
"39. In which situation is 1 kWh of energy expended?
.A a 40 W lamp used fer 2\'I hours
"' a 2000 W heater used for Vi hour
+
C. a 100 0 resistor with a constant potential difference of 200 V for Y. hour
D. a 100 n resistor carrying a constant current of 0.5 A for 4 hours C. 0
37 A lamp JS connected to the secondary coil Of the transformer by long leads which have a
resistance of 2.5 Oas snown in tne diagram. (In the diagram, the resistance of the leads
1s shown as a single resostor of vaiue 2.5 0.)
39. T NO bulbs P and Q are connected to an~~transformer as shown below
II tne current flOWrlg through the tamp Is 2.0 A. Nhat is the vohage across the resistance
of the leads and the lamp? Which of the following statements Is true when switch S Is dosed?
Voltage across the leads Voltage across 1he lamp (~ The brightness of bulb P decreases. .).
A 2.5V 9.SV (i) The reading of the ammeter Increases, X..
8 5.0V 7.0V (ii) The reading of the voltmeter remains unchanged
C. 5.0V 19V
D. 5.0V 115V A. (I) only
e,.. (iii) only
C. (i) and (i~ oniy
D. (ii) and (iii) only
11 t2
46. A 240 V 5() Hz supply is connected through a transfomier to a diode as shO'Ml below.
240VlL
-----4
_J\1r1-J~ R
50Hz t_
l J
What Is the resulting voltage acrnss R?
,13 a VoltageN
I
0.
0.02 Tlrne/s
l'tPr~~
c VoltagoN ~ VoltageN
ro~ 0.02 Tme/s ll+,A_
-- END OF PAPER - -
@@@
Answers
l==I 2
B
3
D
4
B B
s 6
c
7
D
8
c
9
B l !f I
11 12 13 14 15 16 17 18 19 20
D c B c c D D B A c
21 22 23 24 25 26 27 28 29 30
c B B B B c A D A A
31 32 33 34 35 36 37 38 39 40
B B B D D B B A - B c
13
Se ction A (50 m arks) gravrtatJonal force: .. ... ..: ..................... (2)
Answer all the questions in this section. 2. (a1 Stae the principle of moments tor a body 1n equ1hbrn 1m
Fig 1 1 shows a trolley P oo a horizontal table. Fig. 1.2 stows an ldenbcal trolley Q , whlCh is
loaded with steel bars. on the same table. A horizontal force of 10 N is applied to each of the
trolley for a bme or
~s. Bot!' trolleys are initially ot rest. .._,
(2)
(b) A cesk lamp is illustrated m Fig. 2.1.
w=~-'~;:~,~~<"""'
Fig. 1.1 Fig. 1.2
(a) After 0 .2 s, trolley P moves to the right with a speed of 5 mis but trolley Q moves to the right '''
with a speed of 1 rl]/s. El<plain why these speeds ore dit!elent.
,/ 2.0N t ' -.., li(lht bUtb
,( 2.0N G.ON
__________..c=- - --==--..-.--~
.(2)
The lamp must be constructed so that it does not topple over when fully extanded as shown
(~J Trolley P is now turned over so that it is resting on its top surface. as shW.-0 in Fig. 1.3. in Fig. 2.2. The bose of the lamp is Cffcular and has a radius of 10 cm. Othe dimensions ore
shown on the figure. The total weight of the light bulb and shade 1s 6 .0 N and eoch of the two
unifonn arms has weight 2.O N.
Fig 1.3
Explain why a force of 10 N acting for 0.2 s now results in a very small speed after O 2s and
why that speed rapidly reduces to zero.
{'i On Fig. 2.2. draw on arrow to represent the weight of the base (1)
c;r, The lamp will rotate about the point P If the base. rs not heavy enough. Usg the principle of
.... ................ ....................................... ........................... ................................. (2) moments to calculate the minimum weight of the base required to prevent toppling .
1C) The mass of trolley P Is 0.50 kg. Calculate the gravitational force acting on trolley P when the
trolley Is near the surface of the Earth. (The acceleration of free tall is 1o m/s2 at the surface
of the Earth.)
3
minimum weight ='"' ~ ........... .. . [3]
3 A sphygmomanometer is commonly used by doctors to measure a patienrs blood pressure. The F1~ 4 shows two si'nil2r round-bottomed flasks placed at the same distance above a 100 W
inflatable blood pressure cuff 1s wrapped around the pal,enrs upper anm, and the cuff 1s inflated lamp. Flask A is blackened while flask B 1s coated with white pamt. Both flasks are filled
by squeezirg on the rubber pump repeatedly until the desired pressure is obtained. The patient's completely with water. The flasks are sealed with stoppers connected by a U-tube. The water
blood pressure is then determined by the doctor listening to the blood flow in the patient's arm le;els in the U tube are at the same level when the lamp 1s not switched en. The space above
wll~e the pressure is released the water level in the U-tube 1s filled with air.
For a particular patient. the air pressure in the cuff was 111Cfeased such that the mercury level
rises as shewn in Fig. 3 (not drawn to scale)
U-tube - -- ---t stoppers
mm Hg
150
Fig. 3
100
A
~ to pump and blood
50 .J L pressure cuff Fig. 4
(a) The mercury rises to 150 mm above the mercury level on the right. The atmospheric The lamp is then switch on for 3,Q minutes. After 30 minutes, the water level from nask A is
pressure for the day is 760 mm Hg observed to have risen more than the water level from flask 8 .
(~ Calculate the pressure difference (in SI units) between the air pressure in the blood (~I Explain the difference 1n the rise of the water levels from the two flas<S.
pressure cutf and the external pressure. (Density of mercury =13600 kgtrn>.)
pressure difference = ....,.... _ .. ....... (2]
(i~ Calculate the pressure (:n mm Hg) at a pomt 5.0 cm under the surface of the mercury . ...... " ................ ...........(3)
on the left tubP
ib) Explain, using the kinetic model of matter. why the pressure of the trapped air in the U-
tube increases as its volume decreases. Assume that the temperature of air remains
constant
~sure=:-:-. ,., ... ............;,1 .... . [2'.
(b) Explain why mercury is use2 in the sphygmomanometer even l!10U9h ii is poisonous.
.... ....... ........ .. ... ..... ..... ................... ...... . ........ ........ .......... .. ........ ..... .. ...... .. [2]
4 5
. (3] (ill ::>raw the charges on the metal sphere on Fig. 5 111
5. (a) A polyethene rod may be charged negatively by rubbing rt Vllth a cloth but a copper rod
held in the hand cannot be charged in this way.
. Tiie circuit shown in Fig. 6 .1 is used to produce the 1-V charactenstic of the lamp. Tlns
(ii State clearly v.tlat have happened when the polyelhene rod is bemg charged. characteristic is shown 111 Ftg. 6. 2
I/A
3.0
2.0
............ rLJ
12V __l_
(1i) Explain why the copper rod cannot be charged in this way.
.0
Fig.6.1 0 2.0 4.0 6.0 8.0 10.0 12.0 v/V
.... (2]
Fig. 6.2
(iii) Suggest what could be done to charge the copper rod by rubbing (a) A voltmeter and an ammeter are required In order to obtain the necessary readings.
Draw on Fig. 6 .1 the appropnate posltioos of the two meters [2)
... (1)
(b) For the lamp operating under normal cond1bons Vllth a potenllat difference of 6.0 V~across i~
calculate
(i} the resistance of the lamp,
(b) A copper rod is charged negatively and is hanged freely by an 1nsulabng thread as shown
1n Ftg. 5. An earthed metal sphere is then brought near it
insulating thread
0
metal sphere
resistance =.. . [2]
Fig. 5 ~ (:r) the power supplied to the lamp.
(i) State what happens to the copper rod .
........... ............ ........................................ ........................ ....... ....... (1)
6 7
power = .. (21
.............. .................... ......... ...... ......................... ....... ............ .... [2]
7. Fig. 7 1 shows the inside of a mains-operated hair dryer. The fan can either blow hot or cold air.
Fig 7 2 is a circuit diagram of the same dryer. showing how it 1s wired up for use.
8 (;?) Fig 8 1 shows the south and north poles of two bar magnets bemg placed close to and
facing each other.
.:: CT=Q+=lrftow
....- .:: -+- Coldok
: ,
l
Irlet
Fon Fig 8.1
c:
FIQ 7 1
Draw the magnetic fiel<l pattern oeiween the tWo opposite poles. [1]
(b) State two differences (in terms of magnetic properties) between a soft and hard magnebc
materi?'
(~i Show, by placing ticks 1n the table, which switches need to be switched ON to get the results
shown. [2)
Result Switch A Switch B Switch C
A blow of hot air
(:-.j Fig 8.2 shows an ell!ctromagnel' tllat conu01s 'two lamps, />. and 8:
(b) The heater must not be switched on v.ithout the fan. Which one of the switches must always J\
be switched ON to achieve this? ,-
;
switch .... ......(1)
(~) Explain what you would expect to happen if the heater was switched on, and the fan failed
to work.
solt-ir.in
core
.[2)
Explain why lamp A goes off and lamp B lights up when the switch 1s closed.
, \d) Explain why the ha~ dryer 1s norma~ly wired to a two-pin plug rather than a three-pin plug.
8 9
magnification= .......:.................[21
.. ............. .................. ....... .............................. .[3]
Section B (30 m arks)
Answer a ll the questions in this secbon.
Answer only one of the two alternative questions in Question 11.
9. (a) A uun converging lens with pnncip;il fuci di F 1 and F, fcrms a real image I at the pcx:otion
shown in Fig 9.1. Five small squares of the graph represents 1.0 cm 9. (b) Fig. 9.2 below gives the speed or water waves in metres per :>e<..lJIHl f0< various wavelengths
an:! for 4 different depths of water.
Wavelength Im
0.001 0,01 0.1 10 100
Depth /in 0.1 0.67 0.25 0.40 0.93 0.99 0.99
1 0.f)7 0.25 0.40 1.25 2.95 3.13
Fig. 9.2 \0 0.67 025 0.40 1.25 3.95 9.33
100 0.67 0.25 0.40 1.25 3.95 12.5
(i) Describe. in general terms. how the speed varies with the wavelength, r~ a depth of 1 om.
...... ..................... ................[ 1)
Fig. 9.1 (ti) State the relationship between the speed of water waves and the depth of water for a
~ertain wavelength of 0.1 m or less.
(t) Draw appropriate rays to IOCate the position ot the object and l<llit!I U>e object O (3]
.................................. ... ............... ..[1)
(ii) Measure and record the distance of the object from the lens.
object distance =...._ ... ......(1) (m}Calcutate the frequency or a water wave of wavelength 10 m ata depth of 100 m.
(iii) Determine the magnification of the image.
10 11
Tap water at 20 c is used to fill the tank. One litre of water has a mass of one kilogram.
(i} Calculate how much energy must be absorbed by the waler 10 raise the temperature to
its boiling point
frequency = ..................... (21
energy=" .1.(2)
10 (aJ Define specific heat capacity of water. 10. (b) M Calculate the fime taken for the water to begin to boil.
.......... ....(2)
(b) A contractor, Mr. Phua CK, uses the steam wallpaper stopper, shown in Fig 10, to help him
prepare his ciienrs living room for re-OecorabOn
plate
Fig. 10
time= ...~ ... ~ .... (2)
(c;} The appliance is used un\ll so % of the water has been tume<I into steam. How much energy
is required to bnng about this change of state?
heot i ng element
\Nhen the appliance 1s used, water is heated until 11 boils and produces steam Tl:e plate is
then held against the wau and the stean released from the pipe slackens the paper. The
following information is shown on the appliance: -
Power rating 2 .5 kW
Voltage 240 V energy =~.....,....... .. . .:. [21
Capacity 20 hires
Specific heat capacity of water= 4.2 Jg c1
1
(d) Explain latent heat of vaporization in terms of molecular motion.
Specific latent heat of vaporization = 2260 Jg'
12 13
{2]
f,lit) Besides increasing tile number of turns of the left COtl and increasing the voltage of the A .C
.................. .............................. .......................... ................................. [2} mains, suggest another way to make the lig~t bulb glow brighter
....[1}
11 EITHER
11 EITHER
(b) A real transformer
(a) Fig. 11.1 shows a U-shaped soft-Iron core. The right side ol the core has a 500-turn coll The table contains data about a real transformer. The load resistance, connected to the
connected to the 240 V A.C. mains. The left side of tile core has a 5-tum coil formed by a secondary coil. is varied to obtain the readings.
long wire. The eiiaS'Ol the long wire are connected to a bulb When the A.C. mains is
switched on, the bulb glows. The number of turns of the coll on the left side ot the core is column 1 2 3 4 5 6 7 9
then increased to 20 and the bulb glows brighter.
primary secondary input output toad output input output
turns turns Voltage voltage resistance curren: current power
/V /V IQ /A /A IW
l<:IW 1 120 240 6.00 0 .57 1.0 057 1.14 0 .33
,_ 2
120 240 6.00 4 .00 10.0 0.40 0.80 1.60
3 120 240 6.00 6.00 20.0 0.30 0.60 1.80
4 120 240 6.00 7.20 30.0 0.24 0.48 1.73
5 120 240 6.00 7.64 35.0 0.22 0.44 1.67
6 120 240 6.00 12.0 verv larQe 0.00 0.00 0.00
(i) State the effect of increasing the load resistor on the input current.
(i) Explain why the light bulb lights up when the A.C . mains is swHrhorl on?
........................ (1)
~ii) When the load resistor Is 10.0 n, calculate the percentage of the input ~<:!Wer that is wasted in
the transformer.
......................[2}
(ii) Why do more turns make the bulb glow brighter?
14 15
percentage wasted = ....\ ..... [2)
r~;; Discuss whether columns 1. 2, 3 and 4 suggest that the transformer is a step-up or a step-
down transformer?
[2) nme =.L .121
11 OR
11 OR (~) Sketch a graph to show how the displacement ot the material of the ciisc varies with time as
one pulse IS created. 'ndicale the bme involved
Ultrasound techniques are used in tile cllarncal engineering 1ndustJy for measunng and
detecting the levels of liquid in closed vessels. In one such method. the Pulse Echo method,
pulses of ultrasound are passed through the liquid under test By timing the echo of a renected
pulse it is possible to calculate the depth of liquid in the vessel.
The apparatus is shown 1'.l Fig. 11.2.
The source of ultrasound is a transducer. Separate
pulses of ultrasound are created using a thin disc of
material which is made to oscillate rapidly at a sealed
vessel (2)
freouency or 6 Ml-iz. Each pulse contains Just a few
~~~~~~ madeof
cycles ot the osc1:iation. with the amplitude rapidly (c) Explain why. when each pulse 1s sent out 1nlo the metal,
deaeasing wrtll lime. metal (')a large pulse returrs almost immediately,
The ultrasound passes into the metal of the vessel but,
at the metal/liquid boundary, onty about 12% of the
ultrasound passes into the liquid, the other 88% is ........................... ...................................... ... ..................................... ,(1)
reftected. The pulse is highly directional a~d does not
spread out transd ~ ttien a much smaller pulse (abou1 1.4% of that initially passing into the metal) returns.
At the boundary between the liquid and Ille gas 1n the
vessel very nearly 100% of the ultrasound Is reflected.
The reflected signals are plciled up by another transducer similar to the one that generated it,
indeed it is usuaUy convenient to i,se tre >ame transducer for emrtting and receiving the
ultrasound
T~e time taken for the sound to travel through tne IK1uid is measured and the po:.1liur1 ur U1e
liquid surface can be calculated. ........................... ...... ......... ..................... ........... " ................... "' .... (2)
[Adapted from Ultrasonic Transducers for Chemical and Process Plant (R.C Asher Physics r.fi Calculate how long the ultrasound w'll take to return to the transducer given that the
Technology, vol14)) thickness of the metal is 1 O cm and the depl'1 of the liquid Is 30 cm. Take the speed ot
ultrasound in the metal as 5600 mis and in the hquid 1500 mis
Af'!swer the following quesbons based on the 1nformatioo given above.
(a) Calculate the time taker for one oscillation of the disc In the transducer.
16 17
Anglican Hig h School Prclim Exam 2010
Pure Physics Paper I & 2 suggested answers 3(a) (i) Difference in pressure = ISOmmHg
= ( l 50x Io3) ( 13600)( I0) Pa
~ 20400 Pa
Paper 1 Answers
(ii) Pressure at the point = atmospheric pressure + pressure due to mercury
I 2 3 4 5 6 7 8 9 10 I I 12 I 13 14 15 16 17 I 18 19 I 20
B c D c IB c c D D i B A c = 760 + 50 mm I-lg
~ ~- D B B c D c = 810 mm Hg
22 23 24 25 26 27 28 29 30 31 32 I 33 34 35 36 37 138
21 39 140
cB B B B c A D A A B B IB D D B B IA D IC
(b) As density of mercury is very high. the column would be shorter
Paper 2 answers
I (a) The acceleration of trolley Q is lower than that of trolley P.
4 (a) Black surface is a better absocber of radiation than white surface.
This is because trolley Q has a larger mass than trolley P.
Hence water in flask A is hotter thu.n the wntcr in flask B.
QR:
Water in flask A expands more than water in flask B.
The net force on trolley Q is less than that of trolley P
as friction force acting on it is higher.
(b) As volume decrease, the number of molecules per unit volume increases I
l (b) The net force is much smaller during the 0.2s as the friction acting on trolley Pis space for the molecule to move decreases.
The frequency of collision of the air particles with the walls o f the tube
much higher now. This results in a smaller speed.
After 0.2s, the net force on trolley P is now negative as friction is the only force increases.
Force exerted on the walls per unit area increases.
acting on the trolley. Hence the trolley decelerates and finally stops.
l(c) W = mg S(a) (i) The electrons are transferred from the cloth to the rod.
- 0.50 x 10 This causes the rod to be negatively charged as it has more/extra
5.0N
electrons.
2(a) The sum of clockwise moments about a point is equal to the sum of
(ii) The copper rod is a conductor in which electrons arc free to move.
anticlockwise moments about the same point.
The extra charges gained from the cloth will be trarsferred to the hand.
(b) (i) 1rs1 ...
;
(iii) Hold on to the copper rod with an insulator instead of using bare hands.
i
c
~" ,r~
must be drawn from the CG
(ii) Let the weight be W. Taking moments about P,

(b)
Mo
(i) The rod is attracted towards the metal sphere.
w x 10 = (2.0 x 5) + (2.0 x 35) + (6.0 x 50)
W =38 N
6(a)
Ammeter m series wilh lamp (c) The soft-iron gets magnetized and attracts the soft iron am1ature.
T
t2V _.._
Voltmeter parallel to lamp The lower set of contacts arc separated I the circuit for lamp A breaks and
Ammeter cannot be in senes wilh hence lamp A goes off.
the 12 V battel)'. The upper set of contacts arc closed I the circuit for lamp B is closed and
hence lamp B lights up.
9(a) (i) Ray without arrow or wrong arrow direction, minus Im max.
(b) (i) R = V / ! = 6.0/2.4
=2.5 n
(ii) P - VJ - 6.0 11. 2.4 (ecffor current value)
= 14 Wor l4.4 W
7(a) Result Switch Switch Switch
A B c
A blow of hot
air " " " I
A blow of cold
air " " I
(b) Switch C
(ii) object distance = 7.8 cm
(c) lhc heat generated by the heater would not be effectively removed.
This may cause the exterior cover of the dryer tc melt. (iii) magnification= distance.ofimagefromJhele1rs
distance.of.objectfromJhelens
(d) The live components inside a hair dryer are carefully shielded and isolated 4.8
from the user by means of double insulation/ The outer casing of the hair 7.8
dryer is made of insulator. =0.62
Hence there js no danger of the casing becoming live even when the live
wire touches the casing due to a fault.
9(b) (i) In general, speed increases as wavelength increases.
8(a)
(ii) Speed remains constant when depth changes.
(iii) frequency = speed/wavelength
= 3.95/10
= 0.395 Ilz
Straight in the middle, curve near the sides, with arrows from N to S.
(b) A soft magnetic material gets magnetized easily while a hard one does not.
A hard magnetic material retains its magnetism while a soft one does not.
I O(a) Specific heat capacity of water is defined as the amount of heat required to (i ii) It is a step-up transfom1er as the secondary tuns are larger than the primary
raise the temperature of ill of 'A'ater through~- turns. Moreover, the output voltage is larger than the input voltage for rows 4, 5
and 6 (load resistance >200).
(b) (i) Q = mce = 20 x 4200 x (100-20)
= 6 720 000 J
ll OR
(ii) Q = LVt = 2.5 x 1000 x t
6720000 l l (a) t = l= _ 1_
t = 2 .5 I OOO = 2688 S f 6x10'
(c) Q = ml = IO x 2260 x 1000 = 22 600 000 J = 1.67 x 107 s
(d) During vaporization, latent heat is used by the molecules to break. the bonds of (b) displacement
attraction between the water molecules.
Euergy i:; ahu n::y uired for the water molecules to overcome external
atmospheric pressure to escape into the atmosphere.
OR: energy is used to push on the atmosphere as the water molecules occupy a
larger volume
OR: energy is converted into potential energy of the water molecules at the
gaseous state. (c) (i) 88% of the pulse (ultrasound) is reflected back and it travels fastest in
solid.
11 Either
(ii) Only 12% orthe pulse passes through the solid container into the
I l(a) (i) There is a change in the magnetic lines of force linking the 5-turn coil/circuit liquid. As the pulse in the liquid gets reflected back to the solid
on the left due to the alternating current in the 50-tum coil.
container, only 12% of this pulse can pass throJ gh the solid container to
An induced emf is produced in the 5-turn coil, hence induced current flows and
the transducer.
the bulb glows. Hence only 1.4% (0.12 x 0.12) returns to the transducer.
(ii) When there are more turns, the rate of change of magnetic lines of force
linking the coil on the left will increase. ( d)
time . metal = 2( 1x10-2 )
taken to travel through
According to Faraday's Law of electromagnetic induction, the induced emf 5600
produced in the coil on the left wi ll also increase and hence more induced c urrent = 3.57x10-6 s
will flow and the bulb will be brighter.
time taken to travel through the liquid= 2( 3 ox1 o->)
1500
(iii) Placing an iron bridge I bar across t11c top of the U-shaped iron core I use
= 4.00x 10... s
laminated sheets for the iron core I reduce the number of turns of the coi I with
the A.C. source.
total time taken = 3.57xl0.,' + 4.0xl 0...
4
= 4.04xl 0 s
(b) (i) The input current will decrease till it becomes zero.
(ii) Input power= V x I = 6 .00 x 0.80 = 4.80 W
percentage power wasted= 4SO-t60 x 100% = 66.7%
4.80
)~.
-
.c Nu_m l ____
__b_e_r..._ N_a_m
_e_
2
Which of the following pairs of physical quantities have the same unit?
BUKIT PANJANG GOVERNMENT HIGH SCHOOL
PRELIMINARY EXAMINATION 2010 A energy a.nd power
SECONDARY FOUR EXPRESS B friction and pressure
C heat capacity and latent heat
PHYSICS 5058/01 ~ displacement and distance traveled
26 August 2010 2 The diagram below showS a thick-walled tube. The thickness of the wall ts 3 mm.
1 hr INhat IS the internal diameter O( the lube?
0800 -0900 h
Candidat es answer on the OAS
Additional Materials: ~alculator
READ THESE INSTRUCTIONS FIRST
Oo not open this booklet unm you are told to do so.
Write your namo, clas.s Md index number in the spaoes at the top o! this page and on the Optical
1111
Answe< Sheet (OAS).
There are fOf'ty ques1lons in this paper. Answer all question. A 2.8cm
c
For each question there are four possible answers A. B, and o. Choose the one you conside
e
correcs and reoord your choice in soft ~ncil on Iha OAS .
3.0cm

e 3.6cm
o 7.4 cm
The pendulum shown below \Akes 0.25 s lo move from point P to point a. Whal is
the treqUflncy of the oscillation?
INFORMATION FOR CANDIDATES
Take acceleration due lo gravity. g =to m/s2 unless otherwise stalad
Silent eledronic c:alculators may b;i used.
A 0 .25 Hz
B OSOHz
C 1.00 Hz
t7 2 .00 Hz
This document consists ct 15 prtntcd pages
4
An iron cube of side xis resting on a horizontal plane. How will the pressure P on
8
4 Three coplanar forces F 1, F2 and F, ;icting on a particle 0 are represented in the plane vary with x?
m~nilude and direction in the diagram below.
L, l=-- PL.'6=_.
To balance these forces, a single force, Fis applied. F should be equal to
A B C D
f
1 16 5 cm' was added to 20 g of water at 4'C
9 Ice at .5c of mass _15 9 and voThumel of ice and water was left on the table and
and volume 20 cm~ 1n a glass. e g ass
A F3 the temperature was aUowod to stabilize to 4 C.
8 2 F3
C F 1 -F2 What is the density or the Ice-water mixture?
0 F, + F2 A 0.95gcm.J
Q 0.96gcm"
c 1.00 g cm.J
s A particle is molling in a stralght tine. Which of the &peed-time graph is O i.1ogcm.,
impo$$1b/e?
10
If the engine of a rocket traveling in space is turned off, the rocket ,..;n
A st.oP malling. .
e continue to move "1th uniform vel<Y.1ty.
e continua to move with decreasing s~.
o continue to move with uniform accelerauon.
sk of .water usi:ig a stand and clamp as shOwn in t~
1 that the arrangement will ran eas~y.
A B c 0 11 A swdent sets up a Ila . h tells hm
diagram below. However, his tcac er
To make the arrangement more stable. he shou1d
6 A beam balance cannot be used to measure the mciss of a body if there is f\O
gravitational field. This is because
A. the beam balance measures mass by comparing weights.
ll in the absence of gravitational field, substaf\Ces cannot exist.
o in the absence or gravitational field, mass and weight are equaL
0 in the absence of 9ra11itational field, the quantity mass does not eX1st
1 A body Is molling in a straight line under the action of a constant force. Which of
the following quantity of the body wlil change? I. shorten the distance d that the flask is abQlle the base of lhe stand
A mass
\\i. ~:=~~~~e~~3 ~:~~ ~~at the n;isk of water is at a higher level
B inertia
c acceleration A I and II only
0 kinetic energy B I and Ill only
C II and 111 only
D I, II and Ill
6POHS 2010 $4 Exp;eu SOSM
,....in.My
Ph)"'k:& Eumlru1ton
s
6
12 low hows a umform
. recianglllar plate PQRS of WEll9hi 10 N and
The d iagram be
4 s led about comer P. . . I lo:e in thal
14
. es cm by 3 om. PfVO mes to rest with R iff
Frictional fcxce d er.;
from gra111tali0na i
while gravi1ational orce
acts in a
sio fme the plate ro. in a llOrizontal d11eot1on
Whata~ou~Pm~~~
A
After sW111g1 feely
ng P. e1nerQy lost by the plate? frictional force acts does not.
for~
vertically below is
e
=~!1 d:~
vertical direction. trouble but grawational gravitational force. ili!e
c frictional foroe. force exceeds wori<
I~ C::~~rmal into potentlal energy "'
.
4an
p 'Mlrl< done by f Inst gmllitatioN1lforce energy.
~I
Q worlc done aga . lo I converted in .n
n inclined plane wit
I-
k done against friot n '
15 woe
a pushing force p is applied on a block, it meves up a
s 4cm R ~!- ~
A 0 .10 J
'B- 0 .15 J
C 0.20 J
0 0 25 J
R its weight W and
The forces acting on thle llaoMng
Which ortha o
block Include
.
be~t lrepre
diagramnorma reaction
sentsforce ' body diagram of
the free
13
A uniform plank .is su~pothe below. at
symmetrically
cteddiagram AP and ~act.s on the plank at O.
foroe,
A boy walkll on the plank fristion f . o
a
from to P as shown n the blOck? c
~kJltJ~
L
t
'
p a 16 A narrow beam of. light
. i: incident
the angle of VICidence
at the from
i is increased glass;}~~ :~~~~~d~es Iha brightness of
As d vary?
The distance between the boy and point Q is x. How
C does R vary W11h '11.?
.. ....,, ra t.:,'~
1
~ h.~L
0
refleded
Incident , beam
Rl Ax .
beam I
0
L+x
B
~ L L
Brt9h~ess
A
Bdgh~tn"' 8"gh\"'~
10 "''1A
~I
. . ; L._____j__
ao / ao
80. ; eo
7
17 A small iJninous objP.ct ie placed at a point .X ir. front of a plane mirror MN. Ti'.&
object S then moved tr. a point Ynuch that '1:'f is paraltel to the mirror. Tt.c gmph below shows a sol<d JI mass 4.0 Kg M'!ich is healod by a 4CO W electric
heater. What is the ;pecific latent heat of fu~co of t~.e soltd?
Ot IM four reflected rays shown in the diagram below, two rays are cue to object
a t X while two other rays are due to object when it was at Y. Determina >. Y
2G
timo/mln
0 15
A 1.CUkg'
B 3 .P kJ k9"
c 30 kJ kg''
A 07cm i's 6Ci kJ l<,g'1
e 1.o cm
c 1.Scm x
T we blOcks and Y which are mado of ttie same metal are healed by heate<s <;>I
D 3.0 C'l\ the ~e poo.ver atirlg. ~ vanatiof'I of temperature vith time ol bO'h blC>C'<S IS
s"lown on the same graph telo,..,.
,; In the diagram below F a nd f' ere lhe cc of a dive111ing le1s. Whid'I is tne most )(
likely path of the ray XY af\or passing thrcu;h the IBtls? t.emperature /C
F'
\/\'hat is the ra~i<. of the n1ass of .X to that of Y?
A. 1:3
B 1:4
,. Path 1
c 3'1
0 4:1
II Path 2
c Pa!.h 3 A sound wave ~.as a wa.el9ngth of 30 cm ar.:l speed c' 330 ms" in. 1a1r Wilen the
0 Pa'. h4 sound wave enters a meclum, ils speed changes to HO m s . What b its
wavaleogth in the '?led.um?
) I) r,m
a 15 c~
C 30cm
o 60cm
6PC'1$ 2010 S4 ~e-'4 )OSIJt
Phop.1o l:>fffrakuy Enlhnon
BPG~ 2'.fl $4 &fitus; S05&J\
Ptl'f"lc.tPr-'nltnti'JE>~
9
10
. 22 What is the period of the wave in the diagram below?
25 Fout metal spheres, P. Q, R and S are suspended by cotton threads with P
touching Q and R touching S. A positively charged rod is brought near S and then
P is earthed, as shown in the diagram.below.
Os 5s 10s 15s 20s
A 3.5 s
'B 5.0 s
c io.os
D 20.0 s
res?
n X-rays are used in clinical diagnosis for taking X-ray films because If the ea rth and the rod. are both removed. what are the char es on the sohe
p a R s
negative neutral neutral
I. X-rays affect photographic plates. A ne;iallve
II. X-rays cause ionization. negative negative positive - negative -
ts negative
neutral
Ill. the absorption of X-rays is different for different materials. c neutral neut1al
positive <' negative -
0 neutral negative
A lonly
B II only
C lllonly 21> A positive static electric charge is placed at P and a negaiive static electric charge
0 I and Ill only is placed at Q respectively. X is equidistant from points P and Q.
2:4 The diagram below shows a ship moving from p to Q
The ship contim1ously sends ultrasonic waves to the ~ea-Oed.
p a ~-8.,..
ultrasonic " :. x
wave
oG

What is the direction of electric field at the point X due to both static charges at P
Which of the following graphs correctly shows the variation of the time interval and Q as shown in the diagram above?
between emitted pulse and ~s echo from the sea-Oed along PQ?
A_.
A
'm'b'm~mril~1
8 c D
"c - l
0
t
p Q p Q p Q p Q
Position of ship Posltlon of ship Position of ship Posltlon of ship
BPC)"IS 2GIO S4 ~~ 50S8/I
Ptiv;Ja Prellminuy et:aMll\ltl.on aPGKS 2oto s.. e~pr~u sosan
?h)'ilc\ ~cfim!Mry f.,(arnln.oeot\
11
12
~e .::liayr;;;;; !!11low sho~ a mel<:1I block of dimensions 1 cm by 2 cm by 3 c
2.1
e re~1stance of the block due to faces x, y and are
Determine the relative values of R R and R
x. 1 l

;i: R
o m
x. "rand R, respectively.
30 The diagram below shows a three-pin plug. Which wire should X. Y and Z be
connected to?
1CEJ;i:,
2cm x
~
f..
3cm
A R,> R,> R,
B Ry>R,>R, x y z
c R,> R, >Ry A neutral live earth
0 R,> R,> Ry El live earth neutral
c . earth neutral live /
28 In the circuit below, the ratio of power dissipal;-d in the resistors R'. R, and RJ is O neutral earth live....-
31 The heating element of an eledric cooker consists of two 50 O coils. The coils
may be used singly or together in series or in parallel combil".Stion. If the cooker ls
used on a 230 V supply. what is the most appropriate fuse rating for tho heating
element circuit?
SA
A R1 : R2 : R, "A
c
1-0A
13A
B R/ : R,2 : R,,' D 30A - - ---
c 1 1 3? The diagram below shows a potential divider circuit. When terminal J Is moved
R,' R; R, towards a.
which of the fo41owing stalements islare true?
0 -!... _:_
_1_ .
R, 2 R,'. R} p
The potential difference across a fixed resistor of resistance 5.0 n is 3.0 V.
J
What is the work done in carrying 2 o coulomb ~f charge through the resistor?
Q
A 2.SJ
B 6.0 J
c 10 J
0 15 J I. The brightness of the bulb increases
11. The reading in the voltmeter decreases
Ill. The current lhrough lhe bulb decreases
A II only
B I and II only
C land Ill only
o 11 and Ill only
_ ...,&_
BPGHS 10\0 S4 EJi;wtt.e WM
,.,.Y'......
13
14
:n A compass is placed at point M on the cardboard as shown in the diagram below. '
, The nor1h pole of the compass needle will 'point in the direction of 36 The diagr<im below represents a model motor.
current
A B c b
l4 An electron beam passes through a pair of solenoid as shown in the diagram When the key K is closed, as seen by observer at 0, the coil will
below. In which direction is the be<im of electrons deflected?
A remain stationary.
B rotate ctockwise.
c rot.ale anliciocl<wise. .. . . . d. Jar
oscillate about the axle until it rests in a position with its plane perpen icu
0
to the magnetic field.
-
Electron '"'am The diagram below represents a model generator. Risa fixed resistor.
A into the page
El ou1 of the page
C upward
D downward
35 The figure below shows the resultant field pattern arourid a cross-section of a
direct-current canying straight wire placed in a unif?rm.magnetic field.
.
~
Which of th e following graphs represents the variation of voltag!l with time?
wire~
c D
A a
Whal is the dlreclion of force acting on the ....;re?
A r ightward
B out of the page
C upward
C5 downward
8 PGHS 20\ O S4 EQCf.4 505611
Phy1ic4 Pn~ na:rr Eurnioilion BPGtiS 2010 $Ag~~ 50S31\
Pti(SQ Prt:Amlnacy e:1to~n irtion
2
Section A
Answer all questions in !his section.
15
two horizontal ropes A and B. whi<:" are
38 The resistanee of a thermistor a11d a light-dependent resistor depends on different A box is dragged across a floor by
l...s
1
.....,....,..ly lo the direction of motJon of
1 f 36 and 21 res.,..- -
physical quantities. Which or lhe following conditions wiH c-?use the resistance of
mclined aTIhan abonxg ~~ains in oontact with the ftoor.
both a thermistor and a light-dependent resistor to be maximized? the bolC. e
. . Determine ltle tension in rope B and the
~~
- Thermistor Li h\-<le endent resistor The tension 10 rope A 15 2oN. --'ud labeled vector diagram "' your
3
1-'ighest teml)e(eture Highest brightness resultant force on the box. 11"" e
h ighest temperature Lowest brightness answer.
IC lowest temperature Highest brightness
LQ-_ _,_;;.lowe
="'~-"tu'"-re.._._ _Lowest htness
~9 Power is supplied lo a load from an ideal transformer. The primary ooil draws a
current of 0.10 A from the 240 V mains supply in order to deliver a current or 2.0 A
to the load.
Which of \he following is lhc correct set of values?
Number of turns Number of turns Potential difference.
on orimarv coil on secondarv coil
6000
across loadN
12
.
A 300
B ,6000 300 12
c 300 6000 4800
0 6000 300 4800
40 An alternating current is passed through a circuit wi'.11 s'hitch S, diodes and twO
light bulbs L1 and L1 .
...---.----<> 40 v o--~-,
a.e.
L1
tension in rope B = .. ..
Which or the two light bulbs L 1 and L2 lights up when the switch S is (I) open; end
(ii) close,_d._? -..._ ___,,,_,,...,.....----.-----::::--,~-:---:---, resultanl force = (3)
S is dosed
A L2 only
Iii L, only .
c L 1 and L~
0 L and Li
ENO OF PAPER
8PGHS 2010 $4 Eqf'll SOSM
Phys.oo P1~ [u""n.Uon
3
2
Fig. 2. 1 shows a Jet plane which can carry six peoofe. The plane is at resl.
3 .
Asphencal uuOf m a..
b-" ..., 0 5 ,,_
"" faRs from rest in air and strikes a pood below. . For
;
variat~ kinetic energy of the bOb with the distance moved
IXl!fWl's
~~in~ ~t~~:ent. The gravitational field strength g is 10 NI kg.
the us& only
Kinetic
tnergy/J
Fig.2.1 10
c
(a) On Fig. 2.1 . draw arrows lo represent the forces acting on the plane. label
your arrows With Ifie names of the for~ (1 J
:B
When fuUy loaded, the mass of Iha plane and its passangera is 2560 kg.
During taktH>tf, the 11.W> jet engines exen a total thrust of 8000 N and the friction r
between the wheels and the ground is 340 N. .,,
~:__--..r----j~---,r---r--~~ distanc.i m
0 2 $
{bf (i) Calculate the initial acceleration of the plane during take off.
Fig. 3.1
{a) (i) V'Jhat Is the depth of the pond?
depth " .................. (l]
{IQ Find the acceleration of the bob before it covers a distanoe of 2m.
acceleration= .. -., ......... r~ ... (2)
(ii) Explain why t~e acceleration decreases as the plane speeds up.
acceleration=................. ... (I]
... ............ ...................
(iii) What is the significance of the value obtained in (ii)?
~Y " " " [lj
{iii) The average accefe<ation during take-off is 2.2 mls 2 Calculate the
time the plane takes to reach its lake-off speed o f 55 mis. -..-.............................................................[!]
(I>) Describe the energy Changes of lhe bob in AB.
............................................... y .............
................................................ - ...- ..... ,.,............[1]
Fiod lhe tenninal velocity rea~ed by the bob
$PGHS 2010S4 Ez.Pteu 5051111 time= ........... ............ (2]
f>hyua. Pr._,.fl" &~tion Ptiper J
vetoci1y =........ -~-r-111
5 6
For
A student experimented with fight on a triangular glass prism XYZ. A light ray is
s incident at 11' on the surface XZ from air. The subsequent path of the ray in the
eJ(ammu".s
U$11!1 only
4 Fig. 4 .1 snows a wooden walking-stick and has a metal head
lt balances on a pencil placed 0.50 m from its rubber fool. and a rubber fool prism is shown in Fig. 5.1.
Flg. 4 .1
(a) (i) Why is the balance of the walking stick nol affected by its weighr'
- ... - - - ""' "'"' V o[1 }
Fig. S.1
( ii) The pencil is moved along the walking-slick towards its b
State and explain the motion of the walking-stick. ru ber foot.
(a) (i) Calculate tho refracti11e index of the glass.
"Pf".... .. . - ..... .. . . ... ....... . . . ... . . . - .... ..
- -
... ........ .... ...... ....... ........ ............... .., ........ ........................ ) .' \
. . . . .. . . . ... . . . . . . . . . . .. . . . . . . . - . .. . . . . . . ... . .... . 1 . . . . . . . . . . .. . . . ............. . . . . . . . . . . . . . . . . . . . refractive index =........ -:: .........(1 )
............ ........................ ..................................................... (21
(ii) State what is happening al E.
The pencil is now placed at o 45 m from lh
n 15 m from the rubber foot balance the t~ ::i'kin;~t::"o~ :oi!~~~s is placed
bb 1
....-.u ...,,...... - ...... '--"''"'"' .. ~~- .. -11)
(iii) Calculate the least possible value for angle 'JL for this effed to occur.
{bl ;aleulate the weight of the walking slick.
The gravitational field strength g is 1o N 1 kg.
'JL= ... ..... .. ...... ... (1)
N) .=xptain. with clear catculat10C1, the subsequent path of the fight ray after ii
reaches the surface XV.
weight = ......................121 ..... ~ .......-: ...... .....,, .. :: ..... ; .............................- .....,,,.,.... .. .. ~ ............. .
.. (2]
.. . ...... ........ ........................................
9POHS 2010 S<f E,..preu SOS Mt
Phytlcs Pu .iinTY E.u1Mntion Pper 1 BPGHS 10\(1 S4. Expreu ~lnl
Ph~ Pitfll'f'<inuy &ttnin~f\o-1\ PPft 2
7
8
6 ~~al~ ~f ~ecies is placed in front of and dose to a squared paper as shown
. . . e image obseNed through the lens is as shown. .
7 A student is investigating the effect of electromagnetism in a loudspeaker. He F"'
attaches a magnet to a cardboard cone to represent parts of lhe features in a :: : : " '
I
I I I
I I I
loudspeaker.
I
" \_ _ _ The magne\ is placed dose to a ooa A. which is connected to an electrical circuit
,....
I
-1-
_,_ -- L1 .,. '--~
L2 :: '--'-
ooil A
~t:1
L
I I I I I
TT I I
I I I I I I
( ) S Fig. 6.1
a late the type and linear magnification, m, of both tens
15 v
L1 Type= .. .,..... ............. m=
(1)
Fig. 7.1
L2 Type= ......... .............
...
m= ....o........... 111 {a) The ends of oo~ A is connected to a 15 V battery. Switch S is closed.
An object placed 30.0 cm in front of State and explain the movement of the magnet.
Image which is twice the size of the obje~~nver91n9 lens produces an upright ,
... (21
lb) l"he 15 V battery is now replaced wilh a 1S V a.c supply lhat causes the
oorrent in the cott to change direction 100 times every secood. Explain why
the above set-up can produce a sound wave in air v.tien S is dosed.
.................................................................
... .... ..... ............................................... .................................. ... .
Fig. 6.2 . .. .... ..... .. .._...................... ........................................................... .
(b) In Fig . 6.2, by drawing suitable rays above. locate and label ........................... ............ ................................................ ......... [2)
(i) the lens L, (11 (cJ lhe sound wave has a frequency o f 1.0 kHz. The speed of sound in air is
(Ii) principal focus F 11 1 320 m/s. Calculate, the wavelength of sound wave in air.
(e rom Fig. 6.2. find the focal length of lens, f.
f =... ............ (1 )
wavelenglh ~ ........ :'..::.. ...'..(1]
&PGHS 201os... &prnc sosam
PhY1lct P,.itnll\#Y tk11mll'lehn P&fl" 2
BPOHS 2010 S4 qreu 50~1111
Pt\ysb Prt-lltritl.IW)' 'EnminabOn PP"'f 2
9
10
8 Flg. 8. 1 sho"."s M el~ctrical circuit containing three resistors and a 50.0 cm long ior For
resistance -Mre or resistance 10 n per metre is added across points XY. nn>ners Switch S 1 is closed and switch S2 is open. The resistance wire XY is examtne1's
VMon/y
removed and a new resistor is added across XY. The p.d across the new <tS<>Ollly
A jockey J is placed on the 40.0 cm mark of the resistance wire and the other resistor is 9 .0 V. Calculate the resistance of the new resistor.
end is connected lo a voltmeter. Switches s,
and 5 2 are used to control the
current in tile citaiil
resistance =....................(21
9 A bar magnet is suspended by a spring so ltlat i\ can osclllllta freely in and out _of
a coil as shown in Fig.9.1. The coil is connected to an oscilloscope, which has ls
time base switched on. The oscilloscope Is ad1usled so a trace is obtained.
fig. 9.2 shows an enlarged view of Iha trace at a particular instant
Fig. 8.1
~-
s, -
{) When switch is open, the ammeter reading Is zero. ,........_ r v
~late the value of the potential difference across switch s,. I \ /
potential difference =..... ~_ ..........(1 1 .----. "' \ I
(b) 'ilJ Determine the ammeter and voltmeter readings when S 1 is closed
iJ
and S2 is open.
E
II 1cm
1 ~m
Fig. 9.1 Fig. 9.2
Cat Explain why there '16 an induced alternating emf in the coil \Nhen the
magnet is oscillaljng inside the coil.
ammeter reading=... ................(1)
voltmeter readrng " ... . .......(1)
(ii> Withou! further calculation, state and explain whether there are . f .......................... . . . . . . ........ ..... .
chllnges in both meters when switches S 1 and 5 2 are closed.
............................................. (2)
ff the osciUat1ng magnet Is moving up and down at a frequency ?r 2.5 Hz
and the peak voltage is 12 V, state the Y-gain in V/cm and the time base
sensitivity in ms/cm of the CRO
Y-gain " .. ......... [1]
[2)
time-base "[1)
8,.0 HS 2010 S4 Expro.t SO~&il
Phylkl Prd'fn'li.lry EA.em.,etion Ptipe( 2
8PGHS 1010 $4 E:rpre.u 5051.JU
Phrsb ?tcfflrin'Y E1~A'linatln Pap-tr2
11
{C)
State and explain the poiot(s) f>., B, c, D or Eal which the magnet 12
(I) moves past lts equibbrium position Section B
Answer ooly one of the two altemalive questions in Question 12.
.. --
10 Read the foliowing.artiele ancs then answer the questions that follow.
(Ii) Is al the extreme point of its oscillat1on
11]
The operation of a refrigerator is-based on two physical principles. First wt.en
a substance changes from liquid to gaseous state, its temperature remains at
I
its boiling point and latent heat Is absorbed by the substance. On the other
hand, the boiling point of a substance rises under high pressure, allowing lhe
gasified substance to return to its fiquid stale, hence releasing the latent heat.
- .[1]
obs~ lo fi Fig. 10. 1 shows that a pipe located partly inside (inner pipe) and partly outside
,,! ::ruceev'!hlg.~~r
(d) If lhe student wishes
state one way In which this can freq~<:>' on the CRO. (outer pipe) a refrigeralor. A special substance called the refrigerant has a
of the CRO. Wluoout ad1ustir1g the settir1gs ver/ low bomng point and ~s continuously circulated within the pipe by a pump .
.......[1 I
Fig. 9.3 shows lhe magnet beln
through the coil g re1eased from rest and Clropp1ng venically
Door
On Fig. 9.4 sketch a graph or Ila
on lhe CRO. \/0 ge against lime n the first C.4s as observed
12)
~oillt voltage
GJ release
- --- t.=.os
Fig 10.1
A narrow hole at the lop part of the outer pipe slows down the refrigerant,
causing It lo be under high pressure. As a result, the refrigerant eJ(periences a
rise in boiling point and fiquefies. releasing the latent heal
lime
When the liquefied refrigerant enters the refrigerator. ils pressure is reduced
and its boiling point re'!Urns to its previous low level. The refrigerant then
changes from liquid to gaseous state in lhe inner pipe.
Source: HKCEE
(a) In terms of molecular motion, explain how the items in the refrigerator can
be kept cool by lht! refligerant.
- - --==c:.._____ _ t =0.4 s
Fig. 9.3
Fig. 9.4
.(1]
8POii$ ~OU:> $4 &ptu.& 5o$1M
~ Prd.mll'lIY Efllmin16on fl>ef i
13
14
(IJ) E><plain. in terms of whether latent ~.eat were released or absorbed, the
thermal process that the refrigerant undergoes in the (f)
F 10 2 below shows the cooling curves of two p.ure liquids, y and Z =::,.;_.,
(i) inner pipe; and
u~der the same cooling condition. Both Y and Z have the same mass. ..., onty.
(ii) outer pipe.
Tempe<ature
"' - -tr " '' .. " -- "'
--..-.................. ...~..........._ ................... ............ .....(11
(c) What is the purpose of the pump?
- ...... ~ ...................................
............. ................ ............ ..... ...........................................(1)
{d) How is the boiling point of the refrigerant Increased?
Fig 10.2
(i) State the significance of temperature T1 as shown in Fig. 10.2
... . .................. ......................................................................... (1)
(e](i) When the remgeranl is in operation. the outer pipe dissipates thennal
energy. Stale and explain a design of the outer pipe that increases the
rate of heat dissipation.
..............-- ... .- ........................... ... ..............[t]
(ii)
Which substance has a greater specific heat capacity in the liquid state?
Explain your answer deartv.
...... ... .. . . ................ ! .... ..... .. ; ...... ... .... .... ........ ........ ..... (1)
, ............... ... ~....... .... - .- ..................................... .....111
\!ii The average rate of heat dissipation inside the refrigerator is 120W. Find
the minimum lime requited to completely freeze 0.2 kg of water that was
(iii} Which substanceiias a greater speetfic latent heal of fusion? Explain your
placed in lhe refrigerator at an initial temperature of 20' C.
answer clearly.
specific heat capaclty of water is 4200 J kg' 1 c-1
specific lalenl heat of fusion of ice is 3.34 x 1 o J kg'' '.
....~ ......... ................... ~. .\ ............ (11
time= .............. . ... [2)
8PGliS 2010 S4 bprcM SOSMf
~Id Pt~lirninwy EumNtin Pl-P 2 aPGHS 2010 S4 &pi~ SOS&IU
Ph'plocS Pfe.tirrintrty Cum1nl1l.on Pap-cf 2
15
16
11
( ,, ) The value obtained in (i) ,changes as the level of the oil in the tank For
examintJr'S
Fig. 11.1 shows a cylindrical metal tank resting on a horizonlal surface. The tank for fallS: Expl;iin this observa!JOn. vs only
has a base area of 0.50 m 2 and contains oil of density 804 kg m-3 to an initial height elCamrner'.s
ot 0.86 m. An empty U-tube Is attached lo the tank is linked by tap T1 while T 2 is u~e Only ....................... .............................. . ............ .. .
on the opposite side of the tank.
....... ................... .
................ ......
.................. ................ ................ ... : .................................. [2}
aro side of the LI-tube\ in Fig. 11.2 to indicate
T1 (d) Mark.two cro,:>S8ths (onU aluob~ ~en aH the oil has flo~d out of the tank. (11
theod !eves 1n e - .
A simiar set-up has a U-t11be that is sealed at its end an! originally having a
vacuum space before T1 is opened as shov.n 1n Fig. 11.3 belo
sealed vao.Jum
U-tube
Fig 11.1 Fig 11.2 tankol oil
(a) And the wessure exerted by the oil on the base of the tank.
Fig 11.3
pressure= .: ............. : .. --' (2] (e) Describe and e)(IJlain what happens when T1 is opened .
(bl Tep T 1 is now opened. Draw an d label clearly on Fig.
new oil levels in the tank and the LI-tube.
t 1.2 to show the
[1]
..... ... ......... .. ................................. ~ . . . . . . . . . . . . . . . . . . . . . . . . . . . . . . . . . . . .'h
. . . . . . . . . . . 1. . . . . . . . - . ... .... .
. ...................... .. .. ................................
<cl Tap T2 is then opened and oil flows out at the rate of 1.50 x 10"' 01 3 /s.
........ ......................... ...................
(i) What is the rate of change in pressure exerted on the base af the tank?
.................................................... ........................................ (2)
rate; ............... ... ... [2}
tl~GkS 2010 $4 Expteo 5056"1
Physics Prc:lltmI')' b .ITWfiMM ,.,..r l
..
17 18
12 EITHER lb) '(ii) Fig. 12.2 shows lhat a typical ultrasound pulse consists of several vibration
cycles. Find the pulse duration. t. (1 I
Fig. 12. I shows a thin disc known as tr sd
vessel of 1.Scrn thickness' throughout a~h u~r. mounted at the bottom of a metallic
5.0 MHz. This transmits an ultrasound e ansducer oscillates at a frequency or displacement
liquid lerel surface. where it is reflected bapucklset thhrough the metallic vessel toward the
o t e transducer.
time
Fig. 12.2
t= ~-:-................. ..~... [11
(c) 'The speed of ultrasound decreases from 5000 mis in metal to 1800 mis in the
Fig. 12.1 liquid. An emitted ultrasound takes 450 s to return to the transducer.
Determine the deoth of the liquid, 0 .
Source: http://www.hitechtech.com/Sound_ol_Level.htm.
. .. .................. ... ..... ...... .. [t]
. 0 = ... ::. ~- .... " (2)
(b) (i) stale what is meant bv a frequency of 5 _0 MHz.
(d)

State a possible error in using the technique described above to fin<l the depth
of the liquid in the vessel .
.... (1]
........ {t)
"""' ... -' ... ........ ........... -- ....
BPGKS 2010 SA E.xp t~s SOSlfll
p,,ytk$ PcdrrinMY Etem'b!Wl PP"tf 2
19 20
{tl Describe an expe riment 1o find th
In your account, e speed of ultrasound in the laboratory
OR
Fig. 12.3 5hows the set-up for Faraday' s iron ring experimenl
draw~ d1ag-am of the apparatus and setup
d escnbe the procedure
explain how the speed ~f sound can be determined.
Flg. 12.3
(a) Describe and eXl!lain the obs ervation when switch S is closed.
.........................
.................... ................................... ....................................................(21
Electric.el energy distribution takes place around the countly through a network of
transmission cables. as shown in fig. 12.4. The power station generates 100MW of power
: ........ ... ........................ ....- al a voltage of 25kV.
............................. .
Tral'lsfonner A. which lil'lks the power station to the transmission cables, has 4000 turns in
... ..... - ........ - ...... ............. ........................................... the prima!Y coil and 44000 turns in the secondary coil
.......................................................... ... ............................. transformer
...... ......... ....................................... ........ ................
..........................
......... ..... ......... . ........... ............. ... .. power station
Fig. 12.4
............... ...................................... ....... ..................... ........
Find the current !n the secondary coil of Transform~r A .
~) (i)
.............. ............................. .............. ...... .- ........................... .......[3)
current "'...... .......... . .... [ti
(ii)
State ano explain if electrical transmission in the cables is al a high or low
voltage.
[I)
8POtiS 2010 $-( &ptcH 5051111
Phrtlt:I Pnl rNl'l"Y ex.min.a~ Pp 2
"
21
22
{:) Alter transmission through the cables. only 80% of lhe original power
generated at the power station reaches Transfonner B. Find the total examn.r;
' "' BLANK PAGE
resistance of the transmission cables. .,.. onlf
resistance =........................ (2)
(il>) The soft iron core of the transtonner Is laminated. What material is used to
laminate the soil Iron core and what is the purpose of the lamination?
............................. ...... ......~ ............................ (11
(e.J Describe an experiment to demonstrate the two laws of electromagnetism (Faraday's
'Ind Lenz's). In your account,
draw a diagram of the apparatus and setup,
describe the procedure,
explain how the Laws are demonstrated.
.................... '-
....... ............................(3)
END OF PAPER
BPGliS 2Cl10 SC UptG'S4 50$MI
Physic; Pfdim~ry Enmina.don Pa.per l
201 0 BPGHS PHYSICS PRB.IMINARY ExAMINATlON - SOLUTION . . rtical distance of 1.5 cm below P. ,
The C .G. is ongmally ~ 0, a~ equilibrium with the CG at 0 a
Aller oscillating, l3mina rebelow P '"~le: PR " ~l' + 4> " S cm.
,
On
0
Ex'otanation
Both diplaoement and cfistance share a common unit, metre (m}
Conoeot
Units

vertical distance of 25cm


Hence PO' = 2.5 cm
wo=
p
2 B Readings can be taken from Ille ruler matkings
J.=...r.:.~.
Reading
instruments
3 0 T 2x0.25 = 0.50 s f = lfT = 1/0.50 = 2 Hz f 1/T
4 B i:3<tTI
Resultant of F, and F2 F, Resultant of the 3 forces - 2 F, s
Vec:1or
addition ------ ~ -----
5 B The velocily of the partide cannot have two values at the same lime. .. _ ___ .............. __ .......... .J
Logic
6 A
a
The reaciion rorce due to balance on the object of mass mis given in Mass and
Newton
weight R'
f
7 0 F m11. For constan! F, COM1ant acoelefation implies increase In Force and
velocity, v. This results in an increase in the kinetic energy, since KE " energy cm ,. 1 o cm s 0.01 m,
Y..m..? Since CG Is lowered by h," 2 S 1 5
Fe ma. Decrease in GPE = mgh = 10 x 0.01=0.1 J
KE= V. mv' POM
wei ht which produces a CW
8 A m pJ=p<' 13 B Answer is not 'A' because the plank has rled9whcn x ~ L Is non-:tero
moment about P. :. minimum force exe
W:mg=f('g 'I. P=!_ since an ACW moment b needed.
F W A
P=- - m pV weight of the plank below and w. be tha weight of the
A A Lei W , be the
" pxg bOy.
A Tal<ing moments about P.
px'g
x RL=W,(~)+W.(L-x)
pgx where p and x are constant
. w. w.x
ThusP .. x
R=<-f+W.-L
9 c Oensd y of water at 4"C = 20g/20 cm' =1 9 cm~ Density or
R=- ~ x+(~ +w.)
water
OR Maximum density of water is 1.0 gJcm"
(or O 9997 gtcm"") at 4C W. . ruotJve
where gradient.m =--L isne,....
(OR. volurr.e of 15 9 of water at 4C = 1 x 15 = 15 cm' And the v&ttical intercept IS~=that when x = L, R is noo-:tero
15 g of ice melts lo form 15 g or water at 4c. One can observe from the equ. tiv& gradient and a non:tero
density of final water = (20 + 15)/(20 + 15) 1 g cm") = This results in a linear graph with a noga
10 B By Newton's Firs1 Law of Motion, an object in a linear motion continues
n.
lo travel in a straight line if there are no external forces actina on
- N1L
vertical intercept.
normal reaction fo<ce I friction Forces
14 0 Friction is always perpendic<Jlar lo the
11 A I decreases the CW moment due lo weight of flask of water can be useful (e.g. walking)
P~nciple of
II causes an ACW momenV line of action of weight to lie within the Moments Free Body
bHe (s1and) . he suriace while friction opposes
(POM) 15 C Reaction force is pe1pend1cular t(o t d hence ls.in the upward direction) Diagram
the slipping motion Of the slope an
>-- Lighl,
12 A The lamina will always come to rest with ~ CG diredly below pivot P . .. I brightness lncteases due to total roflection
il.GPE "mgt.h 16 C When; excee~s crit1ca1 ang e, ---.----1---i-i;;';;';~~-
I alter several oscillations. The mechanical energy lost by the lamina is
In terms of GPE, where the mass lies at the CG (at 0).
l ( BPGliS Phy<lo tln1t - Prclini fxarnln< Report 2010 l

17 Ic intemal reflection
Produce the 4 rays ba~lcward v~ll
I Oistanoe bet...een the imaQes d stance
re t the Images of tne two obiects.
between the obied - 1.4 em
2010

Lighl <ays \
P r ' I' rn E x I n1 ' n ~ c s ~ ~ fl o ' t
llB!>.:.iHS Phvslcs Utl t l -
~-
18 A-
Di"'!rging :eos
diverges the ray (path 1) Diverging
' The direction of tho electri<: field clue to a po$~Ne cl\alge always . Electric field
Options 8, C aod D are ...-oog as refract' th . . c
outward with the e><Ceplion ol lhe ray throo: lh~~hnt~:~;~~n~s.lens l.ens
is Special Rays
21i
originates 1rom the charge.
through thin - Resistance
19 D Q=m/1 I,"' Qlm = 400 X10 x 60:4 = 60 000 J kg'
I lens
Thermal.
27 B Use R a pl/A, wtiich results in R, : R 1 : R,
..
=~: ~ : ~
6 2 3
R=pllA
,___ Meltina
20 e- Given that both blocks ,.., bj
total energy absorbed by"p a~~ Qe~~~lo Iha same power rating, th e Heatiog
28 A Sinoe current flowing through lhe resistors are the same.
P= l'R
same over Iha same period I. Sinoe P = l' R. we have P, : P1 : P 1 "R, : R2 : R,
M,c:(GO - 20)" M,c(60 - 50) E=PI, 0=11
M.:M,c1:4 29 B E =Pl= M" It (V) = OV = 2X 3 = 6J
'-- - T iv to !1l(Tlflmber: (Of a fuse<j plug, Left p;n stands !or Uve. Recal
Wave -
21 A f= vlJ.." 330/0.3- 1100 Hz 30 B
Parallel and
>.' vii'* 110/1100" 0. 1m =10 an equation 8 Among tho possible combinations, the resistance is minimum when Ille
!-'
When 1 speed cleqeases. the wawleogth decreases V=fJ..
31
oo11s er. in paraOel. The eotnbined resistance in this case is (1/50 +
series circuits
proport1onalely. Frequency remains unchanged. =
1/50)"' 2S 0 and the current is V/R 230125 = 9.2 A. thus 10 A fuse
Coostant is U$ed.
Allernatively
.!'.'...'. ~
Since r is constant. usir>g ll'le wave equation, v f ..t frequency
When terminal J is moved towards a.
the resistance ol the circuit potentiometer
32 D
v ..t i ncreases. This causes the overall current to drop. Hence the cuuenl
and tho potential difference ac(OSS the bulb drops (resistance of bulb is
..t' 110
30
330 This yields ;., = 10 cm ci>nsla111). The brightness. i.e. power dissipalod across the bulb.
> docreasos as weu.
RH grip rllle
22 A Each cycle corresponds lo 3.5s Identifying one 33 A
By right han<I grip rule, CU(rent is to the right. Magnelk: field i s hence
<:vcle in a loop around the wire
23 D Statement I is required in explaining the formation of X -ray
mn. mase m Magnetic field
EM waves I- -
34 B When current nows. North pole acquired on lop aolenokl nearest lo
due to current
Stateme!'I Ill is required lo sllow the diffem , ol x....ay eledrOns, w!llle South pola acquired on tl'8 bottom solenoid nearest to
dellfgi~"':'
X-rays
penetralion due lo different densities to fonn a
electrons Fleming's
Statement II is eotrect but not required as the .io1112mg LHR
is harmful IQ humans. nature of X-ray By Fleming' s let\ hand rule, the direction of the magnelic fo<ce acting
on the beam of eiedrons is out of the page (cutrent lo the left,
rnaoneti<: tleld actina dawnwardl Magnetic field
24 B At a pos41ion nearer to 0. the greater dep1h causes a longer time taken Reflection ol Magnetic force acts from e region of stronger magnetic field lo a region
sound
35 c
of weaker magnetic fteld
25 A By olectrostalic induction.
posl11ve charge.
s will be i nduced with (-) ch arge and . R a
'--
DC Motor -
Electrosl atic By Fleming's Left Hand Rule (LHR). a South (S) pole is a1 t he top of
36 B
(+)charged R will now induce() charge on Q and(+) ch 1P
~e~r:i'."1hed, 1he charge will be removod by groundi~~~e:ving~
Induction magnetize<! coil since the current is flowing clockwise i n the coil. As S
Fleming's
Is attracted to N of magnot, ~ must rotate antlclocl<'Mse (and rotation i s
~ in the preS<1nce of commuta!O<S) LHR
::::~g:~; :.~~"; will
When both eanh and charged rod are re d . When viewed from the back, the coil rolation is clockwise
flow into P from Q, meaning both P. Q are
~removal of cha19ed rod causes both eharges in S a!i Rlo Altematlvely
a~ze each other.
leaving both neutral force ac\lng on lell
Ffelrin9's Left hand rule shows downward
JJ (Q?- -;u JJ? arm/upward force on righl ann of the coil, cau$lng the coil to rotate in
an anlidockwlse manner when lliewed from the front. Observer 0 , at
the back, hence sees the coij rotatir>g in a clockwise manne<.
/
1J6r Altm~tively
Using Fleming's LHR. rt win show th81 the motion is downward for the
coil nearest to the N-pole of magnet
(and upward for the other coil next to $-pole). This produce an
anticlockwise rotation
3 IBP<iH ~ Phv sl cs Unit - i'relif":l Exa m l "e ' ' Nt roJ\ 2010
4(BPGH .S Phy s i cs Un lt - Preiirn Examirtt rs R e o 1t 2010
\I.A
37 0 A.C. genera1 (i.e. current musi aHemate between poslli'le and A.C.
Question 2
negall\'e values within a cycle) Other accepted answe~:
(a) Reaction (or Upthrust. Lift (not uplift))
38 0 F thermistor, ils resistance decreases wilh tempera1ure. Resistance
~
+-ff
For LOR. when brightness increases. resistance drops. E.g. variation with Reacllon force
..
temperature/
Darkness: maximum resistance, aboul IMO . brightness
Very bri9htllght mnimum resistance. about 10on .
39 B N,IN. = IJI, = 2.00.10 = 20 Transformers
Assuming ttia secondacy power output= pcimatY power in!KJI, Reaction f ceWeighl of plane
Weigh! of plane
V,N,=1.11, Weig hi of plane
Hence V, = 240/20'" 12 V
correctlv labelled force. in the correct direction
40 c L, is In series ~ S and hence win not light up if S is open Diodes and
Nate that a di e IMll allow current 10 pass through if tt Is forwatd s'Nit~e5in
bias..S. elecirical F. 6000-340B 7660 N
(b)(i)
circuff Using a flm.
a 7660 I 2S:S0
8 = 2.99 mis
Piper 2 S!S;tlon A
Since the objecl remains !ti contact with lhe ground. lh\resultant focce must act in the dredion d (b)(ii) . 'stance This reduces the resuttanl
rnotion. In the solution below, resultant force is in the ootlzontal direction. Increasing speed cause lncreasl~g rur ; " n by the rallo of force to mass, where
"'-ce aoceteralion is ..e . s
l!!!'ir.onl!ll force. F ""' e<luoed lhe aoceterahon decrease
Scale: 1 cmto2 N ' mass is constant and lhe nel 1orce 1s t '
Triangle Law OR Parallelogram uw b)(iii) u = o (al rest) a = 2.2 mis' ' v = 55 mis
( u -nco t = (v- u)/ a = (55-0) / 2 .2
,-... ' = 25S
Ques\i0f13
3 j:>\(i) Oeplh of pond =3.0 m
8 p nc1p1e o! Conse!'Vation o( Energy,
(a)(ii)
e!
at"top =e.
(before hilling waler)
F mgh Y, mV2
v =
= "2gh ,i ( 2 x 10 x 2) = 6.3 mis
Mark Scheme 11.elh<>d 1
Coned scale faaor (show at leasl 1 fort:e represeriled correctly) ~285
Corred 20 N force and direction 6 .3' O + 2(2)(a)
Correct triangle/ parallelogram (concept) 8 =9.9mis'0<10 mis1
Tenslon in rope B" (27 :t 2)N rewltarrt force= (40 :t 2)N
Forces detennlned using the correct method
Penaltt marls (Sub!ract Y, mart. each)
Nol wrong direction sho111n ( no arrow indicated) for Iha rorces
Nol wron.g labeling lhe forces
Unil I accuracy errors
time/s
Pn\im Exmnr< R tpo rt 2010
SIB PCHS Physic~ Ut"Ht - P1elim lx~min t-r s P.t:port 20 1 0 6\BPGH S Ph ~<i<I Uni! -
Area under v4 graph = distance travelled ":z.om Angle of refraction " 14
Hence,% x 6.3 x t" 2. Thi$ yields t" 0.635 s As the angle of incidence (9'!) is< than Le.
a=( v- u)/t" (6.3-1.1) I 0.635 = 9.9 mis' lhe fight .,..;11 refract al surface XY (bend awe from the normal).
MethO<! 3
(0+6.3)12 = 3.1S mis
Question 6 . . ificati of y,
t = s I <V> "2 / 3.15 = 0.635 s (a) L 1 is a~ (or concave) lens wilh a linear magn .' ~n ....!.
a " ( v - u) It= (6.3 - O) I 0.635 = 9.9 mis' l2 is a converging (or convex) lens with a linear magnificabon or l.
(a) (iiij The value is the a<:celeration due to gravity or acceleration when objeci Is under Orawtwo rays from Oto obtain image I {'Is mark per ray)
free4all, or gravilafonal a<:eeleration Position L and F shown (arid labeled) correctly
(ocallenglh f " 10 x 6 "(60.0 :t 0.5 )cm
(b) KE is converted to~ and internal enernv (in water)
I used lo overcome watet resistance I wave energy
l
(c) The bob reaches a constant terminal speed, v. when ils KE is 10 J
As KE is Hi J similar to part {b), it can infer that the terminal speed is
also 6.3 mis.
(or using calculation) E" Y. m'>l
10 =Y. (O.S) '>l
"" .J40: 6.3m/s
Question 4
_...... - -~ -
4 (a) The line ol action or weight of the mk passes through the~ (pencil)
Q! the CG is directly above the pencil. which acts as the pivot
Henco, the perpendicular distance if> zero.
No moment due to weight of pencil about the pivot since the moment due to a force
is given by the product of the force and the perpendicu4ar distance from the pivot lo
the line of action of lhe force.
(b) The walking sticlr. will turnlrotate anUclockwlso. Question 7
{or melal head >MO move down while the rubberfQPt will move up). 7 (a) The magnet will move away. . . .
The current thal flows into the coil wiU turn lhe iron cor~ Into an
01
The posjlion Ct the centre of mass/grayity {and hence the weight) is shifted to eledromagnet / the solenoid becomes electro-magnetized.
the left side of pivot (ie pencil). Using Righi Hand Grip Rule {or Clock Rulo), the right end of core
There is n<7W an unbalanced anlicloel\wise moment will be ~ As the near ends have the same S poles ,
and bke pgles repel, the magnet will move away. ~x 3
aw.
(c) Weight (of 500 g mass)= 500 /1000 x 10" 5 N
Let weigh! of stick be W The attemating current will produce allemalioO mag~lic poles on BY.
(bl
At balanee about pencil: the right side of iron CQre BY.
Total clockwise moment by weight = Total antlcloci<wise moment by mass which wi11 anract or repel the S-pol~ of magnet.
w x (0.5 - 0.45) =5 x (0.45- 0.15)
W= 30N As the magnet moves in and out, . d y,
QUO$tiOn 5 the cone which Is attached to the magnet will also vibrate, procluCIOQ the soun 6
(a) n sin i I sin r = sin 17' I sin 11 The layers of air are being alternatively comptessed and expanded
n= 1.S3 (or compression and rarefac1ion waves are produced) which propagated the . BY,
(ii) T olal intemal reflection sound wave
81
(iii) sin c" 11n=11 1.53" ~o.a (c) >-" "1r=32Q1 iooo" 0.32. m.
Hence. the smallest angle for TIR lo oa:ur is 40.8'
('1) L><: 90- ( 180 - ( 90 + 11 + 35)} = 46
La= 90-{ 180-(44 + 55))= 9
7 I BPGHS Physics Un i t - Prellrn !xan1ln~<s Report 2010 IBPGHS Physics Unit - Prelim Exarl\lner! Report 2010
Question 8
Induced votbge
(a) 12.0 V
(b) R, = 6 + 4 + 5 = 1sn
I c VI R m 12115" 0.80 A
Ammeler reads 0.80 A
p.d acrou reslster>ee wl1e (= 4 !l) = IR = o.a x 4 " J.2 v 0.4 time
Voltmeter re.ads 3.2 V
(c) Combined resistance~
(main) current I W111 Wicrease as Io 1/ Rat constant V. (or use v"' IR)
Hence. ammeter 1eadir1g Increases. tOa) As the refrigerant changes h slllle from ~quid to vapour. .
i\s molecules absorb btat energy from the. ait molecules 10 the relngerator.
As I lricreases, tf.a p.d across the 40 em wire will also increases Hence towering lhe kinetic enemy al the air molecules
=
as Va I al constant R. (or use V IR) and hence, temperature Inside the relrigermor and the Items.
Henc, voltmeter reading inaeases.
bi) Boiling since latent he'g Is ftl>sorbed.
(d) Either
Using potential divider. RI ( R + 4 + S) = 9 / 12 ii) Condensation since lalont heal ls released.
Solving, R" 30 n
Or p.dacross4n and60=12-9.,3'1/ c) The pu~ wmpressu the yapour of the relriger:mt
Current through both reslsiors c VI R 3 / 10 0.3 A and mates a conc!eose Into ~qyid.
Hence, resistance of unknr;,,,n resistor" V /I = & / 0.3 " 30 !l Circulates the relri9emnt around the pipe.
Questiont
cl) Putting the rel~rant under h!Qh pressure.
(a) The osoUatory mollon ol the magnet in the ooi1 produces an 8ftemaling et') Cooling fins provide larger Sll!1ace 8fea
and cliano1nq magnetic f>eld in the coils. for heat to be radia!ed/!r1nsferred out laster
From Farada)"s law, an induced (and alternating) emf Is produced in the c::ools. or
or paint the pjee oulide lhe refr\ge!!lor blac:I<.
As the magnet approaclies/moves down the col~ as blaek 'obiecl ls a aooc! radiator of heal.
the magnetic Reid fines in the cois increases, inducing an emf In the coils
{using Farada)"a Law) The melill pipes is cofted
As the magnet moves up I away from ooils, the lll!IQDe!ic lleld In the coils to incroase surfaco area for laster heat release.
decreases. This Induce another emf in the coR but in the opposite cli1edlon.
Pl= mc8+m~
(b) Y-Gain 12V/3cm=4V/cm 120t "0.2Jt420Dx(20-0) + 0.2x3.34x10
Time bas4J : lel the ijme base setting be x t = 697 s or 11.6 min
ax " 0.4 s = 400 ms (1/2m deducted far absenu of unil or WJong s.f)
x400/B =SO ms/cm
f\) T1 is the same temeer@!yr! .
(c) (i) Poirit B. E: that liquid X and Y "'" a1 gyrjna the start of the fuD!ng
When moving past its rest position, the magnet must be at ns
areales\ speed and thus producing the maidmum cutting ol llekl iries ii amount
Substance Y. Under the same ()()O('V1g condillon. a s;re;rter of thermal energy needs
and io9ucing the highest voltage. to be lost t>Y
subilanco Y lor the same amount of fall in temeerature
I') Points A. ii) z.
Substance Wllh the same mass, substance Z takes a longer time to underoo a change In
Al lhe extreme swing, the magnet slaps momentarily. state. indieaiing that higher amount of l atent heal needs to be lost by Z compared to Y to
There Is no clianginq magnetic field in the coil at lhis inSlant and change lrom liquid to solid state
hence iero induced voltage .
Question 11
{d) Showe an alt erfl!!!ing von~e ( a sine wave) & 11a) P" hp9
the peak vohege for lhe 2 part of graph is greater 'IYilh a ~lime interval "0.88 x604 x10 (1]
~1015 Pa
9ISPGHS Phn1 es Unit - Pr~lim Exmlnen Repnrt 2010 lOl8PGHS Phys io U nit - Pretorn Em i oe,. R.oport 1010
= 7100 Pa (2 sJ) 111
b) Liquid levels In both tank alld U-tube are the same: (112] d) Most at the pulses pass the water-air boundiuy. Very weak reflected !"Ase will be detected
New liquid level is lower than jevel A (original poshion). [112}
(diagram 11.1 behind) reflecting surface .
o) Rate of chang& ol pressure = d>ange in PI time
W mg
ultrasound
source
---------
d
..----------
sound
~=A .A.. ,.!!!Rwte.~=pgx~
I f I At At A t
= BO~;lO x1.SOx10"4 {1)
D sensor
= 2.~Pa l s[1)
ii) As the level of oil in the tank falls. lhe oressure exerted bv the remaining liquid at the base
dllCl"eases.(1] This decreases the rate of flow~. alfectlng the rate orchange or
preH\ite (1} com uter
d)
(aoct:pl mictophone connected to CRO) diagram
--- new 1. Set up the apparatus as shown in the above diagram. The sound sensor whloh is
connected to data Jogger is d m (aboot 1 m) from the reflecting surface <e.g. wall).
2. Turn on the ukraoound source to produc:e an ultrasound pulse.
3. The computer v.;u reco<d the pulse and ~s echo.
taMaf oll 4. The time taken, t s. between the IWO poises can be meas1Jred rrom the data logger
The speed ot ultrasound In the leboralory(air) am be calcuJared from v" 2d/I
Question 12 OR
a)
Fig. 14.1 Reier to diagfam. (1 J When the a'Nilch Sis closed. anrenl nows in coil A. producing a megnelk: field in the coil A.
(both points must be Accadlng to the Right-hand grid rule, the N pole ol lhe magnetic fiekl ts at the top of coil A.
correct) The son iron ring i s magnetized. This magnetic field.Increases fTom zero to maximum. .
e) (1) The sudden inaease of the magnetic lleld ln the soft Iron dng lnducei an e.m.f. In colt B. Toh1s
Preuure exelled by the almosph!fe on the oil if ye!Y tvgb and (1/2) e.m.f. 1n ~um drive4 an Induced current to now in ooi1 B . .
cannat be balanced by the column ol Oil In Ille U-tube. (112] The polarities ol lhe illduced magnetic field can be obtained L1$ing Lenii:s law. The direclK>n
(since aim. pressure - 1.0 )C 105 Pa. a oc4umn of oi >12.5 m is req\Dred to balance the af the Induced cunenl in a dosed circuit Is always such that ~s magnetic effect opposes the
pressure. Bonus 1m if able to state this) change producing tt. "Fhus the top of coM B is N pole and the induced current flows from P lo
Question 1.:Z EllllER ~ using right hand grip rule the dlredion af the magnetic field around lhe wire PQ Is in
docklOise direction. The compass needle was eiteady in the ~redion of the magnetic fiekl.
a) A higb frequency SQ!lnd wave of more than 20 kHz beyond the audible (hearing) range of hence the compass will not show arry deflection.
human. (1)
bij V/N," V./N.
b)(i) It means sx10 complete oscillations per seoond. 25000/4000; V,/44000
V, m 275 OOOV
(ii) Period T 1JI = p cp,
" 11(5.0 x 10') 1oox1il" ' x 21sooo
=2.0x10"s l,=364 A
There were 3 pulses observed. Time taken f or 3 pulse" 3(2.0 x 10')= 6.0x101 s
II) High \IOllage. A high vollage, low current transmission wnl resutt in lower power loss during
c) time taken to travel through the metallic base" (1.51100)+5000 = 3.0x10-c s. the transmlssioo
time taken lo tr.wet in !he liquid= 4SOx10 - 2{3.0X10..) "444x10..s
V" 2d/I
d ~ 111/2 c)(i) Input power at Transformer 0 "(801100) x 100 x 10
~1I800x444x10..)/2 " 0.400 m =
80 tM>I
PDY!-er loss= 100 MW- 80 MW= 20 'I.KW
lllBPGHS Phy"" Unit - P1el i m x mlnus Report 2010
12 l 8PGHS Physi<S Unit - Prellm Emintrs Reoort 2010
P~-~ t2R
R=20x 1o'/ 3642 151 Q
I) LacquerNamlsh
To reduce power loss in the iron core due !o Eddy current
d)
rrent in magnetic field
( G
Set up the apparatus as shown in the diagram above.
Move t~ wire clown t hrough the magnetic leld, the gatvan6meter shows a dellection.
Move the v.lre dOW11 faster through the magnetic field. the galvanometer shows a bigger
~to the same side.
This demo<'lstraled the Faraday's law that the Induced e.mJ. (currem) ia dlreclly proportional to
the rate ot cuttt1111 the magnetic field rones. {1]
the
The currenl nov.ing ln the .,,.;re in magne!lc fteld will produced an upward force on the wire
(obtained by using Flemlng's Left hand rule) which opposes the direction of motion or Iha wlre.
This demonstra1ed Lenz's law. [1)
(Pushing a pie<00 or magnet into a sotenDld la accepted.)
Biggei induced a.rent is produced witn nigher speed of puslVng Iha magnet into the solenoid
demonstrated the Faraday's law.
Direction of induced current and hence the polariti es of the magnelk: field iocb:ed in the
solenoid demons1tated the Lenz's Law.
lllE ENO
131 SPGHS P h yJtcs Unit - Prtlom Emrnrs Rtport 2010
CEDA R GIRLS' SECONDARY SCHOOL 2
Preliminary ExaminatiOn 2
Section A (40 m arb)
Secondary Four
Thele r$ forty qve.stlor\s in thr!i ecibn. Maw-er an quostkms. For oachqueaUon. thete are
4 posclble NWllR, A, B, C and 0. Choose tne On& you conoide( correa and record Y'JU'
cholce in oott pencil on the Optical Mark Reader (OMR) Fom1.
PHYSICS 5056/01
Paper1 21 August 2010 A pair ol vernier celipe.s is ued to meaoU(e the outer cfllltn8ter or a melll! ring. Wilh
the juws cloed and no rnetel ring, the reading is shown in dlagnim 1. Wl!h the jaws
1 hour cloed around tll8 melal ~og, Iha reading le &!\Own In diagram 2.
READ lliE:SE INSTRUCTIONS FIRST
1) Do nol open thia bookieI untl you te lold to do so.
2) Chect. tM number of ptinted pageo on both 11d11S al the pepe r.
3j Answer all quelllions.
4} E.cll ccrrect *>IWOl W~I scon1 ono mark. A merit will nol be deduc1e<I for a v.rong
ll$Wt:.'!f.
'5) Han<! in OMR Form separately.
What io 11\e ou\et diameter of the rinQ7
Mdluon11M!l!lr! t.
A l .ODcm B 108cm C 110cm o 1.18cm
O MR Fonn
1nfo!!!!ftK>n to Cndkfataa l The graph ohows \he peed-time graph ol a peMdukJm.
Tak thewolgnt of 1 kg lo be 10 N.
Tako apee<i of liglll to be J x 1o" m/a.
"
Whal is Ille period of Illa 03Cilllioo?
A UJOs B 200 c 3.00 s 0 0.00 s
An object I moving to tne righl in a straight '""' wnlt a c:onsianl peed
Which of lhe following statement is 1rue1
A There are no lorces acting on il10 object.
8 Triere I a larf18< number of foroea acting on the objed to the right than lo tho len..
i; There Is only one roroe acting on tne object and ~ is aciing lo tt>e right
0 The reoUltanl '""' acting on Ille obj't ia ~.
This document consists of 1e ptWed paues.
rrurn over
3
For Que.st/Ms- 4 a.tJd ' celer to tbe following. Three bou P, Q and R, e-e 1>e1ar>ee<S on uoifonn bem with lh<t pivot I !he
m;dpolt>l
A tone Is lhrD'JM ertica~ upwards at &peed of 60 rrJs ne11 the eoge ot a cun end h falls
to the boltom ol 111&elil! aller ;ro seconda.
~~.,:'; or Ille fo!lo....lng grapll& r"P'e&enls the va~alloo al 1he veloclty of Ille alone
A P,Q,R
6 P.R,Q
'I; Q, R,P
0 R,Q,P
A b8Hery OIN>nt\ed motDr llh a block ot wood ot conOloril speed.
A
batt....,
0
c 0
w ... ii U.e height or the cliff? What io lh4 OYef81 energy chqe lIW>I> place?
A 1$0m B 800m c 980m o 11eom ot- chemital enargy -+ G'*"iLWUDral petenl al -lllY
8 gravflatloMI '"'rtial - -+ elec1ricol energy
c; gm'lllational polet>llol en6f9Y -+ kinollc MtOY
6 ROMmary rasla her to, monkey on the ed!I o1 a table Ill point P . . shown. By tepmg D lunoti<: ~-+ g1l0Vilall<)nal pclenliel 8MfV}'
!Ump DI plullcine at U.e end of lhe tai. she;. bl to balance tM IOy.
A hamm.vl\U a Mil with a a~cd of Greh and drt.ea ~ 10 cm inlo a blooil cl wood
==1~.
At what rai. doe& U.e speed of the hanvner dac,.IH7
A 1.8 ml8' a 60m'' C 120mla'
The effc-et oflhe pla.a1iclne \s to
A it>cre80e 1he weight or tho toy.
B move the c.antre Of man to be k>C2ted m:actJy 8' P.
(; mcvfll tho cantra of mass dire.ctty &bove P.
C move thocer>tre of mass 10 bo di<ectly below P _
ffum over
e
tG The diagram 1howa the wheel Q( e car Iha! Is acce1ete!1n9 along a road In 1he dire< Two ro.-ces F, and F, act on pattick. Th length or each loree iS ~opor1i0Mi lo 1he
a!IOwn. magntude or the force. .
Which 1tatement llbollt the lon:e of hlc''1 exerted by lhe road on the l).'l"e is corre
A
8
C

The dtt9Cllon of Vie fofoe of ltiction 11 opposite IO lhe <ireetion ot llavel.


Tl\efot'CC ol lriclon Utables 1hc certo ~
The smootlief t\41 road surface Iha 9rM!er is Iha fQf'Ce of frlceioo.

>F,
Wblcll one Q( 1he following diagrams .i......s II\ megmooe and d~lOl'I of the ron:e
f, lhal would keep lhe partide $brtlona<y7
o Smaller wea ol c:onl8cl between llle road .,,d 1he tyre reduce$ lho force or fT1<
1~ A column of gas I slow1y compressed by a piston.
Diston
i IE
gu
h
>[ 1~ The diagram shoWs tl\e levels In a tlquld.manomeier when the lap b opened.
Which one ol 11le followtn11111'1phs showw 1he relalion$hip batweon l:he pre$$ure of
gas, P and lhe lengt'l of lhe eas column. h?
p
91lSfrom
.~.
cylnder
liquid
B
p Whal is the pressure ~ lhe gas in lhe cyinc!et?
.~.
A 6 cm below a1mospllerk pressure
a 6 cm at>ove almosphel1C p<Ml'fC
C 12 cm belc:m atmosphetic J)fe5SUfC
D 12 cm aoove e!mOlpheric pressure
c 0
(Turn [Tum over
8
14 In thermocoupte, tho lwmperal\n difference between tile !WO junctions Rand S,
cauau en e m.l. lo be developed, resulting in a dellac:don on the meta<.
;, A bcaket conlaiM cenaln amount of llquld Tiie g,..ph tllOOMl Ill dilferenl 1peods at
whiOh Ill molewles In a liquid are moving.
. At which poinl (A, e, c or OJ are the moleculet o1tel)l to escape f<o;n Ille Jlquod?
meter 120
--
I I I./ number ol molecu!H
A
R
f
s

8-
0
0
./
./
10

I
: ../
_,,, I
I
I
20

./(
..'
:
I I
30
delle<:lionlmv

l B
If R 11 pieced In 1uwtance ol 30'C ands I& plac<ld In another subelllnce of a higher
tempe1eturc. denection Is obsorved to be :!5 mv.
19 The diagram e/iowe l!le crcss-sectlon of a aolar P~el f(l( o water heeling syatem.
Whal is lhe lemperalure of S?
&unllght
A 30 "C e 1oc C) 130-C
0
16 A cnrte of mesa 1 kg movu ale conIJ\nl 11)e6d of 400 dn/s v..f>en a is pushed by a
force or 20 N.
How much power II required to m.ove the crate?
Which row bc<t deocribes the waler p4po& and the met<>rlQI holdirig the pipea?
A 5W B 2DW ~W D 800W
16 A liquid I& t11ted to decide If it I& eunel>le for u1e ln a r;quld41>-9lau thermometer. II le
f~ lllal tho quid does not expend uniformly whh temperetute. ,A

Water olnee
made of
Co"""""
Dllnllid
..,..,..
white
- M11A1rf1I holding plptl
lnaulAtor
OOf\ductOI
Tn'1 liquid win
/\ cau19 the martc.lngs lo be unevenly 1paced.

8
c
0

Iron
olainlcst sleet
inc
bt.ck
sill/er
- lnaui..tor
conduclor
8 decrease Ille aen11tlvlly or Ille lhlll'IT'lometer.
C increase the n>ng or lhe ll11tmetne'ler. W A &\udent &lu<!ies some equations:
o met.e lh thetmometer mole ,..pon1"'11.
f~e ~ mes x ~lion
power o energy I limo
17 A piece ol lr011 It healed redhot. When l is Crans(elnld into bolling wa19C', lhe velocily dkplecementltime
or
lempenuure Ille boflin11 water Wiii
How many ollfereot vec1ot quenl~oe.s are contained In tho equations?
A d.CrMSe.
II~. A 1 9 2 c 3 0
C reme!n the same..
0 reach \he temperaWr& or ll!e Iron.
(Tum over (Tum over
9
' 10
21 A laser in a c0mpecl disc (CO) player genorale lignt lhal has a wavelengttl or 780 nm A son er 1ignel 1ent V<!rllcally d<WmWn:la from I I hip is re!\ected from Ille ocean l'ooof
l'\elr. 111d detected by a mic:rophono oti I/le 1hip 0 8 s eller cranamisslOn.
Tho lollowing lnformetion ii glvon refracilva index of tile plastic of a CO 1.55 11 t11_11 apeed or 1011nd In w11tlf ls 1000 m.'I, Who! ls the d9Plh o!ltle a<:oan?
wavelenglh ofred light" 7 .o x ur' m
wavclenglll of violet light ~.Ox 101 m, B IOOm c 1200 m O 2~00 m
A 300m
Whl:ll o l Ille loilowlno tt.atemonta lslete conecl?
25 A plane waler wve 1ravel1 In a npplft lank u at>own
I The lasef light in' ii' ls visible.
M Tiie wavele~lh of Ille laser Mght In pllslk: i$ S03 nm.
Ill The frequancy of \he laser llgM in Iha plaalic is 3.85, 10,. ~.
A I only.
11 1- ll only.
C II end UI only.
0 I, II el1d HI.
22 Tht<mal onergy Is eupplied to nxed '"""'or gM 1t constont ~ Whl<;h of the lollowtng dlagremo 1howa tlle cro..soctlon of the tank?
Which o1 the following 1tatementa le l"'rrect?
A
er The
The average speed of the gas moleculoe Increases.
evenige dlJtaneo botwcon ttio g111 molocules irn:reases.
C The QB3 molecules make more froquenl ootrisions with I.tie conlBlner wall.
0 Th" temperature or Iha gas lnortatol.
A B c 0
al In 1111 experiment. a ttiln bum of fight from a rrt box ;, inc:iclent on a mirror The mlrTo<
t$ 1.00 m away ar>d petallel to ac:ale ST. By rotatltlg Ille mirror lhrough en angle 8. All object le placed hallway boiween a diYergln9 tons *1d its i.icat poinl
shown in tho diagram. the Hghl U11 bl - n to move ocroH I/lo ceale from s toward
T Whic:I> row besl dlliCltbts lhe image ltlflt it fotmeel?
r
~~ 9t======a=e======::JT
Sile of lm g Nature ot Image
en4isrged real
~
t ~a enlarged v111ua1
1.00m
J, 04.
st ,c
O
diminished
diminished
real
'11rtual
Howler ffom S w ould lhe llghl move If 8 - 1O' ?
B 34.2.cm C 3S.4 cm o 88.-' cm
(Tum over (Turr1 over
11 12
A unlro!m metnl Nie has 1 piece of eoft llOn ltxe<t at one end. n Is pivoted at poin1 P x nd y are two tphefec posill\oely eharged end have N Hmc amount al electric
l1
end la kepi hortzontal by paulng a amen! thtcug/l a col as shOwn. charge. A poaitive test charge paced
equld11Lant al point P upe<lences !Ila at!ect of
en electtic field,
a P What Is the diredon of this field?
I A
08
~
When flw pivot point it hilted lo Q, his obsccved that the rod slar1; to tih. To maintain
equilibrium of Ch& rod, the lolloWlng steps ean be taken t11:ept
.&. Adjult Ill witch to eonMCt to polnl 2 ol lhe col. 32 Two ldantlcal ffght bulbs, each v.i\h a volt&ge raUng o( 120 V, Gro wrongly wired In
B Increase th& resls1ance In lh6 ~I circuit sel1e$ and connected to a 120 V power supply.
C Move Ille coil downwe<da.
D Move the sot\ Iron rod In th& coB downward&. Wtth this oonneciion, the power dls~ted by each bYlb will be reduced by
~8 '
lhe table ahow. Iha usage ol eleclJlcity or an HOB apaf1ment per day. A 25% S llOV @)7&% 0 80%
Time used A copper wire is !aped on two wooden stands on a senettllle balance. When n current
I! 1 hou" is passed through tho wire from o de power aupply and a megnet la held In the Po61110n
2 shown, the balance reeding Increases.
II one unit ol electricily costa 20 cents, what is the total electrical bUI for each dtl'f?
A ' $2.52 e ss.1e c $28.80 0 $964.80
n Thnte ldendcal realatcn are conoecied in parsliel The equivalent resistance increaoes
by 700 o when one ltlllslo< is removed 1111d connected;,, series with the rematnln9
\'wo, wtikh are alllt In parallel.
The realatance or each resistor in ;..
A 3000 e 3820 c aooo 0 7000
view !rgm the 1!d1
ylew lrom Ille IOI!
lO When cw~t ol 30 mA ~ lhtough a parli<:ular resis1ot wMn a polenlial dillervnee
ol12V Whet are the polarities of the tennlnals P and Q and \he magne(ic poles labelled X and
Y?
How much energy ls eonver1od lo heal in 1 minute?
Potmv of the U"""' tennlnal Mo anode ""'e I
A 2.5 J e 21.aJ c 1soJ 0 360J .' p Q x y
I neoaUve oosllNo no<lll so\1!!!....._
MQst!ve ~lllve south nonh
11
~itive rtMWlive north south
Ill
soulh nor1h
rv oosltlve ntGatlve
A l only B II only C II and Ill only D I nnd IV only
(rum over (Turn ovor
13
14
3S The dlagnim shOwl lh<ee ldenficel lamps connec:lcd to fold 2 v eels.
2V 2V
loop ot wire pulled 11 ~
conelllnl speed
Which ooe of the following grap~ shows how tho lnduood voltilgo In tho bop or wire
vari Wllh Ima?
~F-- ~h When the witches are OJ)Oll. tho 1mme1or reada 10 mA
Whal w1ll bo the reedJn~ at Ille anvnele< 11<1d voltmeter Mien lhe wile""- 8(0 elosed?
A 8 Ammet.r reading Vollmot.r reading
6 40mA 4V
indOCl<I vottnge lnd<Jald vollagoo D l!OmA 4V
I: olOmA 2V
'Po-~ fh-.
0 80mA 2V
,. The d109ram ahowa 1 trensfotmu whiel1 conai<ta of coll ol 1200 1utns and anolll<>r
coll wtth a total ol 120 tum tt Is connec\ed in lhe clrc:ua ea 3hown.
c 0
Input
vollage
~r shOuld polnl T be connecte(I d tte wavelonms of lh lnpll\ vol1agc Olld tho
olltpul voltage are 8S shown be.law?
sv!(VV\
ltlputvollagc
(fwrn o ver
ff um over
15 16
37 tn ll\e diagram below, uniform magnellc field Is applied throu11hout lheshaded area. '9 The clrc"I shows the <;0mponan1S X and Y oon11ecledoo1 poraNei obrMches
A beam of'J)Ositivety charged pettlcie& paues lhraugh a pelt of cl\eJged ,1a1es PQ 81'1d
when tt enlets lnlo region:fl, ~moves to lhe rlgtll in clrclJlar pelll.
x
Wlla1 will heppeft to the ~logs of the ..,.,.,,olera ..tien the intensify o/ lhe tighl Md
the temperaltJfe of tho 1um>11ndJng ineteate7
Ammet<or A 1 Ammeter At Ammtl<lr A,
Which one of theofcllowlnt Is ooneet? A dec("a&e decrease decrease
8 decreaH !ncreese Increase
p It positive. Q Is negative, th maonedo fteld Ines j)Cint nonnally lnlo 1/18 page c increaao decree&e lnctuse
8 P It pc9ilt. Cl I negative. the 11'\aQMllc field 6nu point llOl!nally cut Cf Ille ....Qe 0 ncttase rnciease Increase
C !-
P negative, 0 Is posj!i8. Ille mawneUc field 1i11as point normelly Into the page.
0 Pi$ negab\oo, Q Is posilNc, Ille tnll(,lnollc fletd Nnos point 1101meUy out of the page.
.ia The diagram shcms a loudspeaker.
38 AIn waveform has a peak Vl!W~ o! 4 vons and a frequoriey ol 500 Hz la fed into a
cathode-ray 0&eilloscope. TheKfiplay on the s~n is as shown.
1Um1 or
ll1e col
I
....cm . ---..--
I cmt i _,_ ~
~. '\ ' I
! I I 11 I I I I megne1
I I1 I I I I I
I I
Whon en altea11rtir'11eumintof10 tez i& paased 11\rolJllh the ooil. lhe coll v1b111les Iott
~ _LL! J and riglt~ eauslng lh PlllM'r cone le vibrate es wen
Which ct lhe1lol1owing 5lalemen1 i.s co1Tect?
~hat are the time-base &ellln~ ~nd lbe venieal 111in se11in9?
A A low frequency aound con be heard.
T1me-baeo aetting Vert1celove1n l1fn9 8 A V&f'i ~ft sound cnn llo hoord.
A 1 rna/cm 1 V/crn C Longlludlnal wnv09 oro produced In the elr.
B 1 mslcm 2Vlcm- D Tain1Ytne wevea are produced in lhe 11r
c 2ma/cm 1 Vlcrn
D 2 ms/cm
2Vf'Cm -
End of Pape< 1
[Tum<Ntr
2
CEDAR GIRLS' SECONDARY SCHOOL
Preliminary Examination Two S.etlon A (SO mark )
Secondary Four Answer all the queslions in 1hls sect>on.
A bail is released from rest from a !>eight of 12.0m. Take the weight or a 1 kg man to
CANDIDATE
[
NAME
Class:_
. Register Number: 3__
be 9.8 N.
('ili What i.s the ~peed at which the ball h1ts'1he ground?
PHYSICS 50 58/0
Paper 2 19 August 20 Ans: ,-~ - r=: [2}
1 flo~r 4 5 mlnul (t>l What is the time itJak.es to hit IMO ground?
Addltionol Materlal>: Writing papers
lNSf RUCTIONS TO CANOIDATES
Do not o~n ttii booklet entu you 1cre told to do so. (2)
Wrb your name. c:las.. and index number in the $pace$ ptovided unle$$1his has already been done
for you.
(q What ls the speed or lhe ball ~m II is haKWOY down?
Sectlon A
Answe1 111 questions. Wt'ile your aJ\$wers In the spaces provided on Che que.stion paper.
Section B
Answer II quetlion& from !his section on the wrtiig paper provide<!.
Ouut.oone 9, 10 and 1 l(a) are compulsory.
ChooseeMer 11(b)0< 11(c:).
8egineoc:h!4'uostlon on tr..sh pge. Ans ; ------'--'-~ {21
At tbe end of the enminallOn, submit Section A a,,d B separately. (~ on ~ng the ground the baP rebound but 20"1. of the energy is lost during the
!I boooc:e. On the axes beloW, s~eteh the spoodtime graph or the baP rr0<0 the
time~ is released to the bme<I rebounds to itl greatest hetght
INFORMAnON FOR GANDIOATES Speed/ ms'
t .
The Intended number Of manes is given in brackets ( ) at the " "d of
each question or prt question.
This document consists of 12 printed pages.
[2)
[Tum over (Tum Ovor
l 4
A~ simple pendulum bob is pulled sid ..ways by 11 horizofltal force F such that tho 3 Flg 3. 1 $hows tho direction at which a sound wave lrom an unde<Water explosion is
string of the simple p<tndulum makes 1111 ao;ile of 20 ""1h the vertical. Take the weight Incident at the water-air boUndaty. The ang.le ol lnddence is 211 . The speed of aound
or a 1 kg mast to be 10N. . it\ wsler it 1SOO mis and the speed in air Is 340 mis. The ftequency ot the sound wave
Is 12 tlrM the lowe1t eudible frequency.
In the space below, draw a vee1or diagram to scale lo determine tho magn~Udes of the
tension In~ atrifg and Ille horlconlaJ foroe f. f4J
Scale 1cm represonts _ _ _ I
Fig 3.1 ( not to acal )
fal What is the wavelength of the sountl wave In water an~ 1n air? (2)
Tension 1n string : _ Wavelength 111 water : - - -- -- -
Horizontal foroe applied : - -- - -- Wevelengl~ In air : _ _ _ ___.__
(b! On Ftg 3.1, draw the d1roctlon of the sound waves as !hey emerge into air 121
er~
I (~i On Fig 3. 1. draw three w3vefronts in water an<f three wavefronts 1n air (21
(Tum Ovc

6
4 lat When 0.201\1 of mercury at ho c wa' added to o.ooso kg of waier at 20 c
in a
container ol negligible lhennal capacity, the temperature of water~ to 30 c. !S In Fig 4.1, a boy Billy stands al point B in lront of a piano mirror. A girl Ginny stands el
What is the specific heat c~pacity of mercury? (The specific heat capacity of another point G In front of the plane mirror. W Is a wall between Billy and Ginny.
water = 4200 J/(kg K)
..
An~: .....--~-~
(b} A lead bullet ol mass 64 g moving at 400 mis 3n<l at a temperature of 27 c htts
a
target and three.quarters ol the bullet meh.
Given : Mellill!l point of lead 327 c
Speccfic heat capacity of lead" 130 J/{kg K)
Specific latent heal olf\Jslon ollead = 2.48X 10' .Jilc.g
B

'
:ll Whal 1s the energy required lo raise the temperature o1 the lead bullet to the
melting poil'll ,.. -..i?
w
G
Ag 4.1
Ans. _ _ _ .....__ _ (a) GIMy is 110t able to see Billy In the mirror. Is Biiiy abte lo see Ginny in the mhrOf? (2)
Explain.
(w) \IVhat is the energy requiredto me!! tJ11ee-quart= of the bullet?
q )
!
Ans:_.__ _ __
(b) Gi"irly walks directly to the mirror aJocig the dotted tine.
On Fig 4.1, mark with a cross (X) the fltSI position on the dotted line at which
Whal percentage of lhe original energy o( the bullet is converted to lhennal Ginny Is able to see B~ly in tho mirror. sirw dearly on the diagram how you 121
energy? amve at your answer
Ans : _ _ _ _ __
rrurn 0
'urn Ovar
9
10
I In a c:;ithode ray tube, 10" electrons are emitled from a ho1 C81hcde each second.
These electr00$ then =sh into a metal blod< after being accelerated through a Secdon 8 (30 marks)
potenUal dltteren<:e of 1000 V. The charge on each electron ;, 1.6 X 10 " C.
Answer alt questions from this aectlon on the writing paper provided.
'I What Is the current carried by the electron beam? (2) Nole that for Que5tion 11. 11(a) I compulsory and c hoose it11ar11(b) o r f1(c).
9 An inve$ligallon is done involving two transparent medium P and Q . A n1y of ~ght is
a
directed from p into and the Incident angle I ( In P ) and the refracted angle r ( in Q) are
measured. TM re$ult is shown In !he graph below In Fig. 9.1.
rr
tb) How mucn energy is delivered to ti'>e bl.:>cl< each second? (2)
if' .
Ans : ~~~~~~~
(~)'What Is the rate et which the blodc lemperature rises If !he block's heait capacUy is Ag9.1
u~ m (21
! I An optical fibre ts 10 be manufac:IUred using the medium P and Q . The optical fibre
has a central core surrounded by a daddlng. Which medium should be used for the
core and which medium should be 1Jaed for 1he cladding? El(lllain.
(b l Wl!hlh& aid of a diagram. eK!>fain how a light ray can be transmitted thtough the (2)
optical fibre rrom one end to Ille othor end.
(cl Whal is the refractive Index al the P-Q boundary? 121
Sugges1 a suitable Incident angle in the c;c;ro If a light ray~ to be transmitted as (21
described in (b). Show YOU/ WOlking
State two advantages or optiul fibres rathet lhan copper ""res '"' (2]
{eJ
telacommuoieetions
End of Section A
[Tun (Turn OV
Jver
12
11
Either
1G lnc!ucld em.fa c.n be generated in two ways: Fl 11 2 showS a uniform oopper rod XY with a mass of 800 g hanging at rut in a
11 {bt
(I) by l'Dlaliv movement u~i~ magnetic field pcjnting loto the paptr. The two contaondcts X ~ Y ::~~~~of
(ii) by I changing magnetic field. the copper rod cen move smoothly along two ftxed vertleal c uc1 .,, ra
RS connecled toe d.c power eupptv.
Describe briefly an experiment to illustrate how indooed e.m.ls can be generated In Take th<= weight ol a 1 kg mass to be 10 N.
a.ch a( th41se 1wo wa~. (One experiment for each way.) 141
!bl Oescrtie IH1efty also how you can illustrate that lhe lllduced e.m.f Is proportional to
tho rate of change of ftux linkage or,..,. of flux cutting. (2)
( \ A phone oh.,ger requires 6.0 V of dlree1 cumml Draw a di~rarri to show how this
can be obtained from the 240 V a.c mains. Your diagram should Include any 141 insulal1"g thread
electrlc91 components necesaary. There should also be a brief statement on the x R
p )(
purpose or functloll ol each of the tlectTical components. )(
'- . )(
11 (a) The World Cup was Mid in Cape Town, South Africa and the television signal of the y
game was carried by eleclromagnetic wave$ which traveled from South Africa lo x )( )( X K )( )( X X
)(
Singapore via a salel!He positioned 35 000 l<m ebo\/e the surface ot the Earth.
The spood of nghl In vacuum Is 3 x 1o mf. )( x x x )( x )( x x s
a
(IJ Name the radiation Involved In the signal transmission to Singapore. copper rod
(II) The troquency of Iha slgna! used 11112 GH~. Calculate the wavelonglh of the (11
waves used ln tho transmiulon. (11 Fig. 1i.2
(1}
(Ml) Flnd the time taken for a sent tlgnal to be reftected back down and received by !! the reading on the spring balance ls 6 N, etata th dlrec"on of \ho current
(i)
dish an\eMa. (2) RowinXY. . flit chesa {3)
'lo\ T'le TV signal was received in Sir\gapore using a dish antenna shown In
(Ill Sugge~ thre.o methods to decrease the spril'IO balance value un roa
F1g 11,1. Explaln how the curved dish and \he position of the dot.ctor ensure '121 value of z.ero.
that a strong signal is picked up.
; (c) During 8 Physla lesson, 5tudonts asked to esllmato tho man of a retort 3tar.d.
Jasmine sell up the apparatus as shown In Fig. 11 3.
mply pan with a mass
or so o
CG of
retort
stand
'
~>+-----+'
15 cm 35 cm Fig. 11.3
When Jasmine added 400 g to the pan. she noticed that the retort stand was
121
usl about to overturn. Calculate the mass of tho retort stand (2]
fig . 11 .1 !Hi tist two method& to m3Xe the llllor\ stand remain slable wtien more than
.otOO g is adclod .
l TurnS>ve1 End of Soclion 8
c 21 c ---
B 22 B Section A
0 23 c 1 a 15.3 m/s
4 c
--
8
24 B b 1.56 s
5 25 c c 10.8 mis
6 0 26 0 d
7 A 27 A Speed/ml '
8 A 28 B
9 0 29 c 15.3
10 B 30 B
11 c 3l A 13.7
12 D 32 c
13 D 33 c
14 0 34 B
15 c 35 A llmel
16 A 36 B
17 c 37 B
18
_1_9_ 0 38 B
A 2 Tension in strin 0.68N horizontal force is 0.23 N
3!? D
3 a Wavelenglh in water= 6.25 m
20 D 40 c Wavelen h In air" 1.42 m
b
b i 2496 J or 2500 J 3s
l! 1190 J
Iii Total energy to raise the temper11ure to 327 c +melt Y. of bullet
=3686.4 J
Initial energy::: KE upon impac1 = 5210 J
Percentage converted =70.8 %
--~~--- ---" 1
J- i- ~'1 Incident ray from Ginny wi~ !lO blooked !1Y th~!!)_- ------1 ~
c
D
1.49
>42.
!
e Optical fibres ca11y much more lnf0fl11ati0n than.copper wires Ii
..:' ...
Image of B Less signal Joss in optical fibres
Carry information faster than oopper wires
I
I
Li<:hter and cl\&a""r to manufacture !state anv two\
. : ' ' ... 1G a I
I Diagram should involve o.ie :on and a permanent magoet
Coil is coMected to a datalogger or galvanometer to '.le!ecl any Induced
emf.
" When 1na9net is moved Into \he co", datatogger or galvanometer will
8
f w
N
G

register an induced emf.


When magnet is placed in the con and remains stationary. dat.alogger or
galvanometer WIO not register an induced eml.
II Olagram 91\0uld lnl'Olvo !WO cois that are placed near eac:h other
One coil connected to ac supply and the 01her coil ci>nnecled to CRO
_ S~O will r~ister an altematlng emf :icross th~. coil ..
6 a I - - b use diagram
- . A magnet is dropped Into a vertical coil at different helgh\s
..
'-:: , ,
' ---
: Coll is connected \o a datatogger lo measure the emf induced
' '
- '
.....\ Al a greater height, the magnet will enter the coll with a higher speed.more
'
-
"'-n
. emf wm be induced
When the magnet Is placed Inside con and remains 5\atlonary. no emf is
induced
c A step down trsnsfomil!r with a tum ratio 40:115Usedi0stcp down the
. voltage from 240 I/ to 60 V
The altemaUng supply is connected to the primary coil of the vansfom1er is
. to set up a changing magnetic field.
The secondary coll Is cut the lines of the changing magnetic field from the
primary coil so that there is a con\inuous change in magnetic nux linkage to
Focal lenmh =: 3.4 cm
b
II Virtual uonollt and ma~ifi,!d
Real inverted and maonifled
. induced an emf all the Ume
Secondary coll ls connected to a lu~ wave rectifier (four diodes) to C?nvert
6.0 v a.c to 6.0 v d.c.
7 a 8.0A
i--+'b'-+-"947368 or 947000 J(3sl) a i microwaves
c More expensive. ------------------=] 11
i1 0.025m
U s .,, power consumed= v'/R
Re~lance for 200 tums coll is g1eater than 190 turns coil iii 0.23 s
IY The olSh antema allows the elecii'omagnelic waves to be reftected and the
8 a ll'IR,- Is oreater than V'/R- hence higher
1.6 mA - power
is consumed for 190 tums
I sha""' the dish l:l!'tenna wiR direct ttie renec1ed waves towards the detector.
b 1.6J --- b I From Xto Y
Increase the current through the copper rod
-
c 0.32 Cls -------------~ ii
Increase the strength of the magnetic field into the paper
Section B - - -- - - - - - - - - - - - --
19J';t'f"M;;;fr.;mi)i;7.;;;;<4~-.::-:o:-::--:;;----~ ---------~ -- Use a liohter coee!' rod
""'""'' "' """ """'""" . """=='"""'~l
9 a Medium P ls used for \he cladding c i 10500
Medium Q Is used tor the core ii
M!!dlum Q Is opllcally dell$er th n d'
allows light rays to be transmltt!i :~u~:t Pta~~d when used ~ optical fibre 1t
tha cg Is more than 15 cm
0 Use a heavier base for the retort stand as such the antjclockwlse moment
Olag111m must show Ii hi ra 1 - ... internal reflection.
b the optical fibre 9 Y s transmitted tllrough total ir1temal r::e;;fle':".c~i"'o-
t n"'"in--< caused bv the weiaht of Iha retort stand Increases.
~--
:: ft__ ... 1.Ir.. x tt ?it Which of the following is not a SI unit?
CHIJ ST NICHOLAS GIRLS' SCHOOL -
A Leader in Every Student A metre
ll. ,._"
B second
C kilogram
., degree CAlsius
Prelminary Examination for the
General Ce~icate of Educa1t0n Ordinary level
2 A cylindrical can is rolled along the ruler as shown in the diagram below. It
PHYSICS rolls over twice before reaching the final position.
5058/1
PAPER 1 Multlple Choice
What is lhe radius of the can?
2nd SEPTEMBER 2010
Class: Sec 4
TIME 1 hour
---
INSTRUCTIONS TO CANDIDATES
~
3.0cm
'
Write your name, class and shade your class regis1er number on the answer sheet 6.0cm
provided. c 12.oem
D 24.0cm
There are forty questions in this paper. Answer all questions. For each question, there
are lour possible answers, A,B,C and D. Choose the one you consider correct and
record your choice in soft pencil on the separate answer sheet John uses a simple pendulum to record the time taken tor his classmate
3
James to complete running a 1DO m track. The time taken for .the pendulum
to move from one of i1s maximum displacement to the, other r~ 0.50 s ..The
INFORMA,.ION FORCANDIDATES time taken for James to complete running the 100 m distance is 20 penods
of the pendulum.
Each correct answer will score one mark. A mark will nol be deducted for a wrong
answer. Any rough working should be done lri this question paper. What is the averago speed of the student running the t 00 m track?
A 5.0 mis
Take the acceleration due to gravity to be 1Oms' or 1ONil<g
B 6.7 mis
c 8.0m/s
0 10m/s
4 Mary studies the following equations.
Power work/time
Fo~e = mass x acceleration
Work s F<irce x distance moved in the direction ot lorce
How many vector quanlities are contained in the three equations?
This question paper consists of 15 printed pages including this cover page. A 1
B 2
c 3
D 4
[Tum over
1 2
S !=rom the diagrams below, find tho density of Llie s.:nd {densily of waler is
1.0 g cl"'.)
7 The graph below shows the velocity that varies with hme of a stone thrown
vertically upwards from the lop ol a building. The stone reaches the ground
IOOg 2509 after 3 s. What is the height .:>f the building?
A-------------- v/ms
o---------- 20
Empty contc.:mer Con tamer with sond 10
2009 300g
0 2
-r-------- --- t/s
-10
Ccntoiner lilied with waier Contoine; v..ith sond and water -20 ------ -- ... \L.
A 1.5gcm4
e 2.0g cnf3
c 2.5gcrn.:i A Sm
0 3.0gcm-3 B 10m
c 15m
D 20 rn
6 The speedtime graphs ol car A and car B are shown below. When will the
cars meet? 8 Two blocks M and m aro connectod by a light string passlng over a smooth
pulley as shown below.
v/msl
Car B o :s:: 1 rns2
15 - -----------
5
t/s
0 2 3 4
When mis released, tho blocks move with acceleration as shown. Mer a wh~e.
A 1.8 s
the string is cul in between. Which of the following correctly describes the
e 2.6 s
motion of lhe blocks immediately after the string is cut?
c 3.4 s
0 4.0 s -
A Stoos movino
M
-- m
Moves with constant soeed
--
e decelerates accelerates
c decelerates Moves with consrant sneed
D Moves with conslant sn<>ed accelerates
3
4
9 Two objects X and Y are identical in size and shape but X has 3 times the 12 A wooden block of mass 4 0 kg moves across a table top with uniform speed
mass of Y. When they are both released at the same time from the same when pulled by a force of 20 N as shown in Fig 1.
height in an evacuated container, tl)ey reach the floor of the container at the
same time. Which one of the following statements is NOT correct?
Uniform speed - a = 2.0 ms 1
A On reaching the floor, the kinetic energy of Xis greater than that of Y.
~
C
On reaching the floor, the speed of Xis the same as the speed ol Y.
The rate of change of velocity is the same tor X and Y. r-{__ 20 N ~~ic 11---- 20 N.
D The size of the Jorce acting on X is the same as the size of the force
acting on Y.
11hnr ~m-
1
-
1
\\ Fig .I Fig:2 .
If the wooden block is pulled by the same forco on top of the table now
1O The figure below shows the foot of an athlete as he is about to start running. covered with plastic beads as shown in Fig 2, it moves with a constant
In which direction does the hictional force act on the solo of his shoe? acceleration 2.0 ms7 . What is the friction acting on the block in each case?
-
Friction on block in Fi . t ~ction on block in Fi 2
f)irtclion of running
~ 20 N 20 N
s ::?Cf~ 12 N
_____J
-j
~- SON 20 N
LQ..-1. SON
- 12 N I
J
13 A car of mass m has an engine which can deliver power p. Whal is the
minimum time in which the car can be accelerated from res\ lo a speed v?
mv
A
p
a L
11 The d iagram shows a box of mass 6.0 kg is pulled by a SO N force from an ffiY
in11Jal speed ol 4.0 ms at the loot of ramp to a final speed o! 6.0 ms al the
top o l the ramp: What is the magnitude ol opposing force along the ramp? mv
l
~
2p
2p
0
m\T
14 The diagram below shows a vase placed on a slope sor1ace. The vase does
not tilt over. Where is the most probable centre of gravity of lhe vase?
A 10 N
B 24N
C 32 N
D 38 N
5
6
15 A metal ball of mass 2 .0 kg is re.leased from the top ol a track of length 30 m
as shown. It finally stops al point Y because ol fnclion between tho metal t 18 The diagram below shows a simple hydraulic jack.
and the track.
x
/Hy\bUIC &.Jid
::'.'< :.~: !i~,:~~;~.:~?
Which ol the following alteration will enable heavier loads to be lifted?
Diameter of W Diameter of Z
A Doubled Same
B Doub!ed Halved
Find thG magnitude of the average frictional force .
c Samo Halved
D Halved Doubled
A 20 N
~ 30N
t 40N
19 Which of the following describe the characteristics of gas molecules?
D SON
1 All molecules move with the same velocity.
2 The average.velocity of lhe molecule remains constant if temperature of
16 A uniform rec1angu!ar board (6 m x 2 m) is acled on by three torces at the
the gas is constant. -
~M< ~:~. ~O"d ~ r~
3 The kinetic energy each molecule possesses is not constant. _
4 When a gas is cooled. molecules lose energy arid clusler together to
: ' ' : oootro o N. become molecules in liquid phase. -
A (2) and (3) only
B (1). (2) and (3) only
c (2), (3) and (4) only -
O (1), (2). (3) and (4) -
...
Whal should be the value ol F so Iha! the board remains on oquil1b11um? 20 In a Brownian motion expenment involving smoke panicles in air, heavy
particles settls quickly but very small particles remain suspended for long
A 40N periods of llme.
B 45 N Wtuch statement e>eplains why the sman smoke panicles do not settle?
l- 85 N
D 90N A Air pressure has a greater effect on smaller tanicles.
B Random molecular bombardment ~Y air molecules keeps the particles
suspended.
A beaker filled with some water is placed in an inverted bell-jar which is C The Eanh's gravitational field does not act on very small panicles.
connected to a vacuum pump. Before the vacuum pump is set in operation, D The small smoke particles have the same density as the air.
lhe thermometer in water reads room temperature. As air is gradually
pumped out of the belljar, what would you observe?
21 Heat energy is supplied al \he same rate to 100 g of paraffin and to 100 g of
A The water stans to boil while the thermometer registers room waler in similar containers. Why does the temperature of the paraflfn rise
temperature . more quickly?
B The water stans lo boil while the thermometer registers a value higher
than room temperature. A The parallin has a larger specific heal capacity than water.
c The water starts lo boil while the thermometer registers a vatue lower B The paraffin has a smaller specific heat capacity than water.
than room temperature . C The paraffin is tess dense than water.
D The water stans lo freeze and the thermometer registers O c. D The paraffin is more dense than water.
7 8
22 Ice cubes al a temperature of ct>C are pul into a drink of lemonade of mass 26 The figure below shows how a ray of light lr?IT' an obie:t 0 enters the eye .,
200 g at a temperature of 2fi>C. After a short time during coofing, some of after being rellected twice. Using the data 9111en ~ the hgure, find the '
the ice cubes have melted. What is the mass of ice melted? d istance ol lhe final virtual image of 0 from the mirror X.
[Specific latent heat of fusion of ice is 330 kJkg, specific heal capacity of
lemonade is 4000 Jkg"1 K" 1 )
Mirror X ~eve
A 22g
B
c
40g i
0
48g
68 g
12.0cm
1 Mirror Y

.4.0cm
i
23 A thermocouple thennometer is calibrated such that it shows 2 mV at 0C
and 26 mV at 100C. What is the temperature of a substance when the
thermocouple shows a reading of 17 mV?
A
B
62.5K
335.5 K f

l
c 336.9 K A 8.0cm
0 343.8 K B 12.0 cm
24 A beam vibrator oscillates In a ripple tank. There is a glass plate placed in
c 16.0cm
the middle of the tank. Whal win happen to the water wave moving pass the
o 20.0 cm
glass plate?
... 27 Which of the following statements about radio waves is/are correct?
Soeed Freauencv I wavelenalh
A increases increases I increases
B
t'!
decreases
i ncreases
-,.. decreases
same
- I
I
decreases
Increases

1. They travel slower than miciowaves in vacuum.


2. They travel taster than ultrasound.
3. They have higher frequencies than microwaves.
0 decreases same I decreases ~ 4. They have lower frequencies than gamma rays. /
A (1 ). (2) and (4) only
B (2). (3) and (4) only ~
25 The figure below shows a ray ot light moving lrom diamond to air. c (1) and (3) only~
0 (2) and (4} only
incident light
28 A sound wave travelling at 340 ms' produces the waveform shown below.
I Il
diamond
air
,..~111 111 I
Determine the critical angle of diamond.
A. 21 Whal is the frequency of the sound wave?
B 27
c 63 A 130 Hz
D 65 B 260 Hz
C 340Hz
D 880Hz
10
29 A radio pulse is transmitted verticaliy upwards 10 measure the height ol the
ionosphere. The outgoing pulse T , and its reflection R lrom the ionosphere, 31 Three identical resistors are wired L'P 10 the mains electncal supply in tnree
are recorded on the screen of an oscilloscope wi1h time-base set at 50 s ways shown below.
por division.
+.
..
I x
i ~
I I -- ---
I I !
! . I
I I I
What is the decreasing order of current drawn lrom the mains supply oi
these arrangements?
T R hest current to lowest current drawnlrom mainssu --,
x 1o ms, what is the approximate height
! ~ =1-.. ~ --=-----------~~
It speed of radio pulse in air is 3
ol the ionosphere?
A u.Skm ,.,,..-+=---:t----~----~---f--::---------.::.~=-1
B 13km
c 15km
0 75km 32 Which of the foiiowing isJare the advantage(s) of connect:ng household
power outlets in a cing-maln circuit?
1 The connected appliances can work inde;:>en<.lonlly.
30 Kelly used a CJ.o to study the waveforms produced by tuning loO<s. She 2 Thinner and cheaper cables can be used.
used one tuning fork to play a note of 256 Hz and used another tuning fork 3 The chance of overloading the circuit is reduced.
which pro<tuced a l<Xder 5 12 Hz. The diagrams show the osclhoscope
!races produced by diHeren1 tuning forks. Which diagrams correspond 10 the A (1) and (2) only
notes produced? 1:1 (1) and (3) only
c (2) and (3) only
D (1 ), (2) and (3j
33 The 'di agram represents the circuit for lhe side lamps of a car.
Lamp L2 suddenly goes out, but lamp L1 remains lit.
Which of the following would be the most llkety cause of ihe fault?
(1) (2) (3) (4)
A (1} and (2) only
B (2) and (4) only
<.: (1) and (3) only
D (1) and {4) only
A The battery is badly connected.
tf The lamp Lz has blown.
c The fuse has blown.
D The switch has jumped open_
11
12
34 Which of the following describesthe electromo tive force of a cell?
A The ditterence in energy between that needed to drive unit charge
lhrough the load resistl)rs and through the cell. 37
.
The diagram shows a magnet being used to pick up steel needle. The nonh
B The energy used to drive current through all load resistors in the circuit. pole ol the magnet is close to t he centre Y of the needle.
C The energy used to drive unit charge th rough the resistance of the
circuit.
0 The total energy used to d rive unit charge round 1he complete circuit.
35 The diagram below shows a potential d ivider circuit.
C:Sl
x y z - needle
What are the poles in duced in the needle at X, Y and Z?
Dole induced at X Dole Induced at Y n<>le induced at Z
A N
N
- N
s
N
N
Whal happens lo the brightness of lanw L 1 and lamp l2 as the jockey X is
moved away from end P of the potentiaf divider? -{ s N s
D s s s
Lamp L1 Lamp L2
A brighter brighter
~ brighter stays the same
C' brighter dimmer 38 A bar magnet is fixed in a container ol mercury as shown. A copper wire PO
t> d immer brighter is connected to a cell and freely suspended at P.
What will happen to the end Q, as seen by the observer. when the switch ls
An uncharged metal rod M is placed on top of
a glass beaker. A neutral metal closed and remains cfosed?
coa.t ed ball hanging trom a thread just touches one end of the metal rod.
Another charged metal rod N is brought into contact with the other end ol the
metal rod M and then quickly removed .
lhreod
+ _.
Me1olbol1 M
~ ~ .: ':~-.:;:.i~~: - . -:- -~
beaker
. - . t'
.. ; .,;.:.
:_;:._:. ; merc:ury ..
":- .. - - ;r
Which of the following will take place?
+ !
A The ball remains still throughout.
B The ball moves away only when the two rods are touching. . A Kick into the page
c The ball moves an d stays away even after the charged rod has been a Kick out of the page
removed. C Rotate clockwise
D The ball swings back and lorth white the rods are touching. o Rotate anticlockwise
14
39 X and Y are two metal rings of equal weighl but a sll: is C'J! in \', They are
dropped betweGn two magnets as shown. Which of 1he foUowlJ'\g stater:'16tn.s
is correct?
A Both fall with the same acceleration.
B X falls w11h unlform velocity while Y fails w:th acceleration.
C Both fall w ith accele ration but X has a larger acr-.,eleration.
O Both fall with acceleration but Y has a larger acceleration.
40 A transformer consists ol a coil of 4800 turns and another coil with a total o f
280 rums. l rom which oulput coukl be lapped al various termi11als. Between
w hich terminals would ycu tapped for a ~. 12 W lamp?
S T U V W
l=l:t:]"
A ST

F:::J Z40 V
B SU
C TV
D TW
END OF PAPER
.. :,, -. -. - 15
k ft_-{}-~,,_*""'
CHIJ ST NICHOLAS GIRLS' SCHOOL
A Leader in Every Stude nt

ti. Section A [50 Marks)


Answer all the questions In this section.
ltM_~~
Name I
Register No IClass 1. A motorist approaches a traffic light at 15 mis. The traffic r.ght ch311ges lo red when
he Is 33 m rrom the junction. The graph below shows how l~e velocity of the car
cl"\anges with time from the moment the driVor sees the red lighl
CHIJ ST NICHOLAS GIRLS' SCHOOL V I mis Thinl<ing
Braking Time
SecondaJy 4 Preliminary Examination nme
PHYSICS 5058 Thursday 2"" September 201 o
Paper2 1 hour 45 minutes
f
Ai;lditkmal ~terlaJs : Geometrical instruments
Elecltonic CabAator
RE.AO THESE INSTRUCTIONS FIRST t is
Write your name end Index number on all tho wor1< YoU hand in. 0 o:s
Write in dark blue or black pen in tho spaces p rovided on the Question Paper.
You may uso a pend for any diagrams or graphs.
(a) What is the dislance \ravened by the car before the driver reacts by applying the car
Section A (50 marks) brakes? 121
AnsweraO~
Section B (30 marl<s)
Answet all quesliol\$. Question 1 \ has a choice of parts to 8!\$Wer.
Write yoJr answers on the spaces provided. Calculate the deceleration of the car. (21
(b)
Information tor studellts:
Students are reminded that all quant-ative answers should include approp(late units and
should be given to a sensible number of signilicant llgUtes. Errors in units and numbets of
significant figures wil be penalised.
II world"' is need9d lor any queslJOn, It must be shown In the space provided. Can the driver slop '" time? Explaln your answer. (21
(c)
Omissiotl of essenhaf wor1<1ng win result in toss of mar1<s.
Take the value o( acceleration due lo ~ravity on Earth, g =10 m~.-or 10 N/kg.
For Examiner's Use
Seclio.n A
Section B {d) Draw on the same graph above, how the speed-time Qr3.ph o1 the car wm change
the driver has been drinking alcohol and the road is s ippery. 121
Total
(e) Stale one factor other than the conditions of the road, that will aflect !he braking
distance of a car (1)
This document cons'sts of 17 printed pages induding this OCN~ page. [Turnover]
2010 SNGS Phys/a P2 21'10 SNGS Physics P2 2
2. The diagram shows 3 ropes a11ac:hed to a ring R. Three cylindors x. y and? are
supported by these ropes from two pulleys. Tho ring R is in equlilbrium. Using a 3. A uniform sheet of cardboard ABCD is in the shape of a rectangle. The catdboa1d is
scaled vector diagram, determ;ne the tension X. suspended by a lflread frorn one end as shown in the diagram below. Tho diagram is
(4) not drawn to scale.
The cardboard is heki horiz.ontaUy and then released. The catdboard eventually
comes to rest.
(a) Draw oo the space below. the linal posit.on ol the cardboard and mal1C its centre ot
gravity, G. [2j
(b) A piece ol plaslicine is then ar.ached to comer 8.
(i) State the difection of tho turniog motion ol lhe cardboard. _ _ _ _ [1)
(Ii) Estimate the new position ol the centre or gravity and mark It with a loller Non
your diagram. I 1I
2010 SNGS Physics P2 3
2010 SNGS Ptry$k:s P2 4
4. The dagram below shows a device for comptessing materials The chart below shows how !he rate ol heat loss from a 1ogoer running at a sieady
5.
pace changes wilh air temperature.
120~~
Air temperature at 29'C
Ii----
-seomm-- I-
Air temperature at 20'C
:r> 800
................................... H
...e 700
i ..,.
:D 500 2.
"'0 n lo
600
--- +----plunge( 2.
~
400 He at by
e n pore Oon of
e 500
0
e s-we al
~ 4 00

-
300
5
~ 200
Heal loss by ~ 300
:;- conduc'*'- ~ 200
--cyfinde<

~
~
. conwc:tlon aJMt
....... 1 00
0
(a) Use Iha data from the Table to calculate the amount ot heal eno;rgy In Joules, Uie
(a) A vertical lo<e11 F of 20N is appfoed at one end of a leY i;ystttm. Tha lever is pM>ted Jogge< will lose by evaporation of sweat in one hour when the air temperature IS
on a hinge H. The plunger compresses the material In the cyfonde<. 2o'C. 121
(i) On the d iagram. draw and label two other foroes acllng on the lever arm. [2]
(ii) By laking moments about H. calculate the force acilng on the plunger. The weight
of the lever arm may be neclected. {2]
(b) The data shows that the jogger loses more heal by evaporation of sweat al 29'C
thana120c. Explainwhy. {11
(b) (i) The eross..gectional area of the plunger 1s 4.0 x 104 m>. Calculate the pressure in
pascal. exened by the plunger on the material in !he cylinder. [21
(cl The data shows that the ,ogger loses less heat by conduc1;on, conve<:tion and
radiallon et 29C than at 20 c. Explan wt-rf. [2)
(ii) Suggest two lrtelhods of increasing the pressure exerted by the plunger. (2)
2010 SNGS Physa P2 5 6
2010 SNGS PhyslCS P2
6{a) Tl>e diagram shows a ray of monoch<omatic iight incident ~mally on a lroar.gular
glass prism ot crihc.:.t angle 39.5. (i) Calcu'ate lhe refraC1ive lndax o! 'Niolter . (2)
State the changd i111ha waveler;glh of the light as it !eaves the water. (1)
(iij) If the studont decides to use dirferent thickness of g la:;s block Instead o f water.
(I) E~plain why the ray will not emerge Into air from th-::> face XY. [2]
state aod explain how its graph of real depth against apparent depth would
d iffer from that' of water. (21
Complete lhe path of the tight ray until it emerges inio the air acein Show the
workJngs clearly in the space given belcw and all relevant angles on lhe
diagram. (31 7 AA elee1ric circu;t consists of a fu6e. a battery of 6 V, a main sv.i<ch an<! tlVee lamps X.
y and z : The three lamps are opcr111ed al their rated vo:tagc and powe<. The f:.ise
protee1s all the three lamps
Lamp X is rated 6V. 24 W
Lamp Y is rated 3V. tZW
Lamp Z rs rated 3V, l'ZW
!a) (2)
(b) The cb9ratn below shoWs a student watching a swimming pool being frlled w~h wa:er
He looks vertically down at an object placed al the bottom ol lhe pool He notices thal
Che ob,ect appears lo be neaier to IWn than ii really is. He estimates the real and
apparent depth of lhe water and plots a g;apn.
fWal-
!1tc":!t::r~-::::;:rn:r.:::rr.::--:irri:r.;;:!
(b) Wlucn one ol the following tuses;, rated lA, 3A, 9A and 13A, could be used as lhe
fuse for !ho above circuit? Show worktrlg 11 any. (2)
"
0.0 0-2 b.< 0.$ O.a' . 1.0
Appwttnt d@tlYrn
2010 SNGS Phys1CS PZ 7
2010 SNGS Phys.cs PZ 8
a. The cfagram shows a~ ck. motor. AB and CD are solenoids wound around lron
cores. The solel'IOids and cool are oonl'lected in parallel to a baltery. Section B
x Answ~ all the questions. an has a choice of section 10 answer.
9.
The mictowave oven Is now commonly used in lulchens lo quickly heat up pre
prepated and fresh food. It produces microwave radiation of frequency 2500
MH~ that Is absotbed by walet molecules. The microwaves cause the water
molecules In food to vibrate. Al this agitation al molecular level creates friction
which heats up the food.
The micfowaves can only penetrate a short distance Inside the food_ The typical
power In the microwave beam is 750 W. Over each dstance of 3 mm. the
power available from the microwave decreases by sew..
Thicker food can.
however, be cooked In the microwave oven using processes other than the
absorption of microwave energy.
(a) Calculate tt>e wav9iength of the microwaves used in the microwave oven. Take the
speed ol lighl as 3 .0 x 1(f mis. {1 I
(a) When a current P8"8S tlvough the circuit. the solenoids beoor:ia electromagnets.
State the polarity of the solenoids at Band C respectiver)-.
B:_____ C: [1]
(b) Give another use of miCfowaves ti.sides the heating enect. j1J
{b) Whal is the cfirectJon ol rolabOn of the coil as seen from Y? - - - - -- - - 111
(c) Stale the luncbon ol E. (2)
(C} Use Information i:\ the passage 10 skel<:h a graph showmg how the power available
lrom lhe mictowave vanes with deplh. Plol points at depths of 0, 3, and 6 mm. l3}
(d) 11 the banery Is _replaced by a 50 Hz a.c. supply, the coil will only oscillate 10 and fro.
Do You agree With this statement? Explain your answer. (2)
power available
in micfowave
beamlW
0 3 6 9 depth withln food I mm
2010 SNGS Pt>ysics P2
9
;zo 10 SNGS Physics P2 10
(d} Since microwaves can OlllY penetrate a shon distance mside food, staie 1h 1oug~ wh at Explain the operation o: this dovice. l31
(ii}
olher process ol heal transfer by 1,111ich food thicker than 9 mm can bs cooked
thoroughly in the microwave oven. E>plain this process. 121
(e} Estimate the minimum time it will take a 750W microwavo oven to thaw 0.25 kg of
frozen soup. Tl1e soup ls initiay ai 16'C and Is lo be j~sl turned into liquld at O'G.
The soup can be assumed lo be made entirety of waler. Take the specific heat
c
capacity of ice as 21 00 .JI kg and ihe specific latent hea: of fusion of water as
34-0000 J.11<g. {3)
(b) In a iree tall experimeo:, an object attached lo Ille end ol a tape passing through a
tici<e1 timer is allowed lo lall lreely. Tapes ol a Hick length are cul_ lo lorm a !ape
chart as shown in the di agram beiow. The irequency ct the licker timer is 50 Hz.
26.3.!.--------n
Lenglhicm zo 2-- .
14. l ~-
8.0--
1O.(a) The diagram below shows lhe slru<:Me of a ticker timer. Tho e!eciromagnct
consisting of an ir>sulated coppor wire ooiled around a LI-shaped ~o n core, ls
connected to a 50 Hz a.c. supply and a diooe. It has a springy iron strip with a pin
allached to.one end. A paper tape passes under the pin.
(i)
"
Find the average vetoci:ies of the first and fourth strips. {2]
(ii) Hence, calculate the acceleration of U1e tailing object, g . 121
{i) Sketch a graph of current against time to show the variation ot the curren1
passing through the eleclromagnel [2)
(iii) Why is the experimental value ol g different lrom the expecte<:I value of
10 mis'? (11
2010 SNGS PhysiC$ P2 II 201() SNGS Physics P2 12
11. EITHER (d) Desctlbe an experiment IO determine the ~peed of sound in air using a cathode ray
osciUoscope.
(a) The dlagram below represents lhe screen of a calhode ray oscilloscope (c.r.o.). In your account,
draw a labelled diagram of the setup,
~
desctibe the procedure,
explain how the speed Is calcUle\ed !tom the readings taken. (4)
I
'
The tim&-base setting is 0.5 ms cm ancl the Vi)ain seUlng is 2.0 mV cm. A
microphone connected to !he y-platM of the c.r.o. dete<:ts a sound wave note of
frequency 500 Hz.
(i) Calculate the period of the note. 111
(u) The M!ptilucle of the signal from the mlcraphone is 6.0 mV. Draw the trace seen
on the screen of the c.r.o. when tho microphone detects the sound wave. Draw
the maximum number of waves seen. i3)
(b) What happens lo an air particle when a sound wave passes through air? (I)
(c) Whal happens lo the air particle when lhe sound is louder? II I
2010 SNGS Physics P2 13 2010 SNGS Physics P2 14
11. OR EJ<plaJn why there is a deltecllon in the galVanometrH. (21
(i)
(a) f' stJt wue XY is connected to a galvanome1e1 by :lexible v.ires so !ha! 11 can swing
lreely between rwo magnets as shown in Figure 1 below. Ooe mayr~t 1s placed
above and it'e oth111 IS plac<Jd below the w;re XY.
Why does the galvanometer reading decrease with lime? {2)
(ii)
s
The wire XY ls set sw;nging as shown ln Figure 2 by di$pladng it lo one side at (Ii} In wtlich direction Wll tile current flow In wire XY when lhe wire swings frOnl R
positlOn R and then releasing it from R. toS? \II
Whicl\ of the stages I to 5 corresponds lo the reading shown :!ls "A' on the
graph? Explain your ch<.>JC8. (I )
Fogu1e 3 shows how the reading t'rl the galvanometet varies with tJtne alter the wire
has been set swirog1ng. (v) Which of the 5tages 1 to 5 corresponds to the leading shown as a on the
graph? Oescnbe the molJon of the wire at this slage. (1)
-
2010 SNGS Phyaiu P2 15 16
2010 SNGS Physics P2
(b) How would you mod~y :tie set14> shown in (a) to obtain a relalively C<)fl$1anl a.c.
supply.
1n your account,
draw e. labelled diagram of Iha setup,
describe the procedure. (3]
End of Paper
2010 SNGS Physica P2 17
Answers for ~0 1 0 S N GS Physics Prellm Papers
3
(a)
Paoer1 B
10 2A 3A 4C 50 6C 7C 89 90 10 A
11 0 12 B 13C 14C isc 16 B 17 A 18 D 19C 20 B
21 B 22C 23 B 24 D 256 260 27 D 28 B 29 D 300
31 B 320 33 B 340 35C 36C 37 B 38c 390 40C
Decote<alion "' 15 / (4.0 0.8) (b) (ii)
~ 4.6875 ~ 4.7 m/s2 10 2 s.f.
(c) Staking distance , 'h x 15 x 3 .2 24 m =
Total travelling distance 12 + 24 = 36 m
Driver will not stop In lime.
(b){i) Anti-clockwise .
(d)
4(a}
Braking Time (i) Reaclion Reaction
force force
is
I l
I I
fs
;
(ii)

weight
Taking moments about H,
F x 120 a 20 x 50(l

___J I
(e) Speed ol car I mass or weight of car/ tyre pressure I efficiency of brakes/ F 83 N
smoothness of e I I~ a ied to b<akes. (b)(i) Pa FIA 8314.0 x 10
s 2.1 x 1o Pa or 21 kPa
2
(ii) Increase length of lever arm.
Increase force Fon end of lever arm.
Decrease cross-sectional area of plunger.
Decrease distance between H and the plunger.
360N S(a) E "' Pt (500) (60 x 60)
1
s1.8x10 J
53 (b) The body will sweat moce to cool down the body to prevent overheating as
evaporation produces cooling.
The rate of evaporation inaeases with higher temperature of surroundings.
2010 SNGS Physics Prelim 2010 SNGS Physics Prelim 2
I
I
f iemoeia!Ure-----.
Rate -of heat t ransfer depends on temperature difference. A s
. difference between surroundings and jogger is
1 t1ansfe1 1s reduced.
less at 29C than at 20 oc. heat
1
No. When a_c. reverses directionrnthe coi(-OOfarities of theSoienOlds are-also
reversed.
I
I~a-)- ~~ternal th~a The forces acting on the sides of the coil continue to a~l upward on the right
- Total reiiecii"On occurs because the ir.cldent angle of 60 Is more side of the coil and downward on lhe left side so the coil continues to rotate
(i) the cflhcal~ anti-clockwise.
750 -
300 .--: l
(ii)-
(iii)

-
The slope of Jhe graph is steeper. Because the refractive index of glass is
higher than that of water (or glass is optically denser than water).
f-=------~-'2_0~----._---:_------6~~~-~~~-J
(cl) Conduction. Hea! is tr;p1s1erred by the vibration of food molecules to the ~
neighbouring molecules.
7(a)- ------------ -
I
(bl

y
'
x
_,.._z.
-<::,,
Current through lamp Xe 24/6 = 4A.
1

I
II
( e"'"
i--.,..
i
10(a)(if
t
)--+::Pc:-1-=-mc9--,~+
I==94450
125.9
r = 130s

ml
1501 a (0.25)(2100)(18). (0.25)(3400001
= 9450 + 65000
(to 2 s.fJ__ -- 1

I
I
Total current ~ 8 A
z
Current through lamp Y and = 1213 ~ 4A.
Fuse rating is SA because fuse rating must be $lightly higher than the
operating current of the circvit.

1
I l
0
8(a) S : North C: South -- 0 .02 0.04
8(b) Anti-clockwise .
1O{ii) When a current flows in the circuit, the electromagnet attracts lhe iron strip
pulling the needle down lo m ake a dot on the tape.
(C) E ~split-ring commutator) reverses direction of current in the coil whenevei~ Whan there is no current flow, the electromagnet loses its magnetism. The
cod passes through its vertical position, to allow the coil to continue rotating in strip is no longer attracted and it springs back.
the same dir~tion. The strip moves up and down continuously to make dots on the tape at regular
time interval.
2010 SNGS Physics Prelim 3
1
10 (b)(i) rime I<>' each strip 4 x 1/50 ~.Oas
1 strip: Average velocity= 8.0 I {4 x: 1/50) = 100 cm/s
na>ro 1Ra!e-01 chariQeoi-magne11G r1ux ~!'lkaiJes ioo_-ces an e.m.i. in lhe wire. -1
motion ol )('(slow:; ~tm. The rate or ch~ of maonetic llux &iJ<aoes
Ihm---J-The j
4" strl : Avera e veloci!x= 26.3 / (4 x 1/50) 329 cmls
,. , ~ <h!cr.,ases so lhe induced e.rr..f. also decreases.
I (iii) : Current 11ows lrom Y IO X. - 1
~CiVf__ JStago 3 because XV ir; moving. at !he 1.1realesi speed. ______ !
~----e~~~ ~--Wire
fcbf
x: is movi.ig from S 10 R. - - - - - - - - -- ___ !
Diagram:
1
~ II
I
I!_
~eFlc'
i N I oo 1 S m"'"' I
I _j~L o - 1
y~ ~P;~ngs
'1 .
a good I
llOci.ngl
wa
, k,Q-0
~oon~h
a 5~rp;and
lllleose pulse
as inc:>de"t s~
Procedure: "
1. Selup the apparatus as shown above.
2. The eo<I is made lo rotate al a COl\S!ant speed in a magnetic :;ekt
...11ecied sound
The rate c;f Change of magnetic r~... x linkages induce an e .m.f. in lhe
CQi(.
1 i - - - long distance -
___...._,
3. The slip-nngs 1ranster an allernatir.g e.m.f. to the eX1emal circuit.
IPr~r~l~:rnphone

is connected to the y-input of the CRO and placed at a


distance d from the wan.
2. Switch on the CRO. Adjust the time-base and y-gain to suitable values.
3. Clap once f Fire a gunshot at the same distanced from the wall.
4. The microphone wiD detect the sound produced and the echo reflocted
from the wall. Two plAses wm be seen on the CRO screen. .
5. Read the distance x between the 2 pulses and calcufale the time taken
t =xx time-base.
6. Use the formula v =2dfl to calculate speed of sound in air.
OR 11 W ire XV cuts the ma netic !luxes
2010 SNGS Physics Prelim s 2010 SNGS PhySIC$ Prelim 6
2
When the 1aws of a pair of vernier caflpers are closed, the rea ding 1s s~n in Fig. 1.1
CHIJ St. Theresa's Convent When 1t is used to measure the diameter of a circular disc, the reading 1s shown in Fig. 1 2.
Preliminary Examination 2010
Secondary Four (Express) 0 1 4 5
PHYSICS
PAPER 1 Multiple Choice

5058/01 l0

1
1111\ IJl! 1il I
10
I om
I.LL.JI
...LI.I I IL,Y,
1 1
0
1
ll II\++ii
1 1......,.;
1
1/T'r'-l
I\ I-cm
10
Fig. 1.1 Fig. 1.2
4 August 2010
What 1s the actual diameter of the disc?
1 hour
A 4.43cm
B 4.46 cm
READ THESE INSTRUCTIONS FIRST c 4.49cm
Write in soft pencil. D 4.52 cm
Do not use staples, paper-clips, highlighters, glue or correction fluid .
Write your name, class and index number on the OTAS Sheet in the spaces provided
unless this has been done for you. 2 Which of the following describes work done, force and distance as scalar or vector
quantities?
There are forty questions on this paper. Answer all questions. For each question there
are four possible answers A, B, C and D. force distance
work done
Choose the one you consider correct and record your choice in soft pencil on the
separate OTAS Sheet A scalar vector scalar
Read the instNctions <:'n the Answer Sheet very carefully. B scalar scalar vector
c vector vector scalar
Each correct answer will score one mark. A mark will not be deducted for a wrong answer.
Any rough working should be done in this booklet. scalar vector
D vector
~ A stone is being thrown vertically upwards with a speed of 15 mis. What is the acceleration
of the stone at one second after its release?
A O ms- 2
B 15 ms-2 downwards
C 10 ms., downwards
o 10 ms- upwards
This question paper consists of 15 printed pages
[Turn Over
5058101/Prelim/2010
5058101/Prelim/2010
Sc.t1t..r: Mr N~ Knk Hua1
3
4
The figure shows the variation of velocity v with time t of a bicycle.
vims The figure shows a pendulum bob hung in equilibrium In a string by a pulling force F. The
weight of the bob Is Wand the tension in the siring Is T.
5
4
3
I/
I/
2
v F
I/ Us
Which is the correct free-body diagram showing the forces acting on the pendulum bob?
0 2 3 4 5 6 8
B
Whal are the displacemenl and ai celeralion of the b'cycle alt= 5 s ?
displacement/m acceleration/ ms-2
-
"B
c

8
8
12

1
0
1

/
w
w
0 12 0 ;,
c 0
5 A force of 15 N acting on an object of mass 500 g causes it to move with a constant
acceleration of 10 ms"' Whal is the frictional force acting against the object?
A ON
B 5N
C 10 N w
D 15N
7 A cube of volume O 06 m' is made of material with density 200 kgm-> What is the wcoght of
Ille cube?
A 0.004 N B 16 K C 160N 0 2500 N
s Which of the following methods cannot be used to Increase the slabohty of a baby hogh
chaor?
A increasing the base area of the chair
B adding wheels to the legs of the chair
C attaching weights to the base of the chair
D lowering the seat of the chair
50Sa/01/Prehm/2010
.. 5-068/01/Prelim/2010
5
4 6
A rectangular lank as shown below weighs 500 N when empty and 4500 N when it is 11 A student applied a force of 20 N to push an object with a constant velocity of 0.40 ms'
completely filled w~h oil. across the floor. What is the work done by the s tudent in 5.0 s?
A 1.6J 6 8 .0 J c 10 .0 J D 40.0 J
6m
t 0.S m
A body falls freely under the action of gravity. Which of the following statements concerning
its energy is incorrect?
A Its gravitational potential energy at the ~nd of its flight is all converted to kinetic
energy.
6 Its gravitational potential energy remains constant throughout its flight.
C Its total energy remains constant throughoul its flight.
What is the pressure exerted by the oil at the base of the lank?
D It gains kinetic energy while falling.
A 222 Pa
Over a particular temperature range, a volume of liquid expands more than the same
B 250 Pa
volume of solid because liquid molecules. compared to solid molecules.
C 1330 Pa
A are less dense.
D 2670 Pa
B are larger in size.
C move faster.
A column of liquid X, floats on water in a U-tube of un~orm cross-section area.
I) exert smaller forces on surrounding molecules.
The diagram shows a tube containing gas and mercury in a beaker of waler at room
10cm~ 14
temperat ure.
gas
mercury
If the density of water is 1000 kglm', what is the density of liquid X?
A
B

500 kgm'
800 kgm"' )

wa ter
C 1000 kgm" room temperature
What will happen lo the volume and pressure of the gas as lhe beaker of water is heated lo
D 1500 kgm' a higher steady temperature?
volume pressure
A decreases decreases
fl increases increases
c increases no change
D n o change mcreases
5058101/Preiim/20 10
5058101/Prehm/2010
7
8
1!:'. When one end of a rod is healed, heal energy is transferred down the rod by
1!1 A small heater, rated at 24 w. ls embedded in a large block of ice at 0 'C.
A hot n1olecules moving along 1he rod.
After the beater has been switched on for 672s, rt 1s found that 48 9 of ice had been melted
B hot molecules ohanging places with cool ones. into water at 0 c.
C the air around the rod, which moves after being heated. What is the specific latent heat or ice?
D transfer of energy from one molecule to the next. A 2!j0 Jig
B 336 Jig
16 Which types of surface are the best emitters and the best absorbers of infrared radiation?
c 1344 J/g
best emitter best absorber
D 2260 Jig
A
B
black and dull
black and po~shed
. black and dull
white and dull 20 The diagram show6 a water wave travelling in the direction of the arrow, at a particular
c white and dull black and POiished instant.
0 while and polished white and polished
...
t7 Which of the foUowing is not an advantage of a thermocouple thermometer?
A It has a high temperatvre nrnge.
B 1t does not require any electrical devices to calibrate.
C It can measure tempera)ure at a poinl What wiU bappen to points X aod Y as the water wave travels forward?
D It C8fl measure rapidly changing temperatures.
A Xwillgoup, Ywillgodown.
B Xwillgod<Mn, Ywillgoup.
18 A lhennometer with no scale is taped to a ruler. When it is placed in steam. the mercwy
level roses to 26 cm. When it is placed in pure melting ice, the mercury level drops to 2 cm
C Xwill go up, Ywill go up.
c:m
30
n X win go down. Y will go down.
28
!6
22
The diagram shows four rays of light from a lamp below the surface of some water.
20
t6 What is the critical angle for light in water'?
IS
I<
12
tO ,......1!){>(c:ury
8
ak
waler
Whal is the temperature shown by the mercury level in the diagram?
A 25 C B 29 'C C 33'C D 40'C lamp
5056101/PrehmllO10 '
5058101/Prehm/2010
9
22 The diagram shows a ray of light passing along a simple optical fibre. " 10
ZS Which of the following oscillosoope traces ol different sounds correspond to the softest
sound of the lowest pitch?
A B c D
What Is the refractive index of glass?
A 0.684
Whicl\ of the following statements Is true about electrostatic charging by rubbing between a
e 1.46 glass rod and a silk clolh?
c 1.73 A It involves a transfer of neutrons.
D 2.53 B It involves a transfer of protons.
c It involves a transfer of free electrons.
2J Which of lhe following types of electromagnetic waves are usually employed In industrial
heating end drying? o It involves a transfer of electrons.
A infra- red rays
Which of the following correctly depicts the electric fiel~ lines produced by two isolated
21
negative point charges?
B microwaves
C VIStble light
D X-rays
Which of the followiog shows visible light, ultraviolet rays and infra-red rays corree1ly
arrang~ in the order of illCfeasing frequency?
Lowest frequency----> highest frequency
A v1s1ble hg hl ultraviolet rays Infra-red rays
B ultraviolet rays visible light Infra-red rays
c
,_ri:r-,
Infra-red rays visible light ultraviolet rays
D Infra-red rays ultraviolet rays visible light
2~ Two 6 V batteries are connected as shown in the diagram.
~y 6V
Whal is the voltage between X and Y?
A OV B3V C 6V D 12V
5058101/Prehm/2010
5058101/Prehm/2010
\\
11
12
Four pieces of wire made of the same rraterial are connected in turn between t~.e terminals
P and Q, in the circuit shown below. ~ A lamp is connected in series with a light-Oependent resistor (_DR) and a battery. The
barrier shields the LDR from the lamp.
rVZ?ZZZ
barrier
p Q
Whic~ wire will give the smallest reading on the ammeter?
\...
A I
length
0.Sm '
. diameter
0.5mm
When the LDR is wrapped with a black cloth, the lamp glows very dimly.
When the black cloth is removed and the light Intensity on the LOR increases, what
B 0.5m ..... 1.0mm happens to tJ-e lamp?
c 1.0m /
1.0mm
A rt gets brighter.
ll 1.0 m ,. 0.5mm ,1
B It gets dimmer but does not go out.
C It goes oul
Which graph shows the current-voltage (/V) relationship for a filam@nt lamp?
D II stays the same.
A B
I
With the 1 :l Q resistor (X) in the circuit show~. the ammeter reading is 1.0 A A 20 0
~'
resistor (Y) is also available.
20 0
~
c D Which change to the circuit will give a reading of more than 1 A?
.A . reverse the connections to x
B place Y on the other side of the ammeter
C replace X with Y
D connect Yin parallel with X
An electric heater is rated al 2 kW. Electrical energy costs 8 cents per kWh. What is the
cost of using the heater for 3 hours?
A 6 cents B 16 cents c 24 cents b 4'B cent~
5058101/Prelim/2010
.. 5058101/Prelim/2010
13
"' 14
The diagram shows the three wires of an electrical mains supply connected ta a waler
healer. 37 A d.c. motor consists of a coil al many turns rotating in a fixed magnetic field. The coil is
live connected ta a d.c. supply through a split-ring commutator.
mains .___~n=e=-ut"'ra"'l_ __J heater axlo
supply
earth
What are the currents in the live, neutral and earth wires when the heater draws a current
of 5 A from the mains supply?
live neutral earth
coll of many turns
A 5A 5A SA
SA P, Q: halves of split-ring
B SA OA
c SA OA OA
0 SA 5A OA Same changes are made. one al a time.
The d.c. supply is reversed.
The coil is turned before switching on. so that P starts an the right and Q on the left.
The pales of the magnet are reversed.
What material could be used to make the needle of a plotting compass? The turns on the coil are increased in number.
A aluminium How many of these changes make the coil rotate in the apposite direction?
B brass A 1 B2 c 3 D 4
C Iran
38 The diagram shows lwo identical bulbs lighleo with normal brighlrtess using 240 V s.upply
0 steel through an ideal transformer.
36 In me cl1agram PQ is a fixed wire while RS is a movable wire. Bath wires are connected ta
a cell and dipped into a mercury reservoir. 24W, 12V
P.-----------~
R~-----~
24W, 12V

I_ _ _ _ _ _ _ _ JI
transformer
II the primary coil has 1200 turns, how many turns are there in the secondary coil.
A60 B 120 c 240 D 6000
When the switch K is closed.
:19 Electricity is transmitted at high voltage rather at low voltage because
A S will move away from PQ.
A ii wastes less energy.
B Swill move towards PQ.
B it is easier to generate_
C Swill out of the paper.
C ii requires less insulalion.
D Swill move into the paper.
D ii allows lransfarmers to be used.
5058/01/Prelim/2010
5058101/Prelino/2010
A simple a.c. generalor pr~uces a voltage that varies with time as s hown.
I I
* l@ ~.,.
Class Index Number
~-~/V=IP l
Name:
CHIJ St. _Theresa's Convent
Preliminary Examination 2010
Secondary Four (Express)
Whh ic~ ~raph shows how the voltage varies with time when the generator rotates at twice
t e oog1naJ speed?
PHYSICS 5058/02
,_.JV; kf I ?f'J .
A
PAPER 2 Theory
o I 2 I 4 Ume/s
Candidates answer on the Question Paper.
11 August 2010
=~ J
No Additional Materials are required. 1 hour 45 minutes
I I
B READ THESE INSTRUCTIONS FIRST
Write your name, class and index number in the spaces at the top of this page.
Write in dark blue or blacl< pen on both sides of the paper.
You may use a soft pencil for any diagrams or graphs.
Do not use staples, paper cli~s. highlighters, glue or correction fluH:I.
Section A
Answer all questions.
c Section B
Answer all questions. Question 11 has a choice of parts to answer.
Candidates are reminded that all quantitative answers should include appropnate units
Candidates are advised to show all Uieir working in a clear and orderly manner, as more marks are
awarded for sound use of phystCS than for correct answers.
Al the end of the examination, fasten all you work securely together.
The number of marks IS given in brackets [ I at the end of each question or pan question.
D
For Examiner's Use
Section A / 50
Section B
-/ 30
time/s -
Total / 80
-----+-- -
~----~--- -
- End of Paper -
This question paper consists o f 18 printed pages
[Turn Over
5-058/011Prehm/2010
Sch:r Mr Ng Kok Iluo.1
" 5058102/Prelim/20 10
2
Section A 3
Answer all the questions in this section. 2 Fig. 1.2 shows the essential features of a trolley used for lifting and transporting heavy
buckets in a factory. The force to lift a bucket is derived from the downward force, F,
the opera1or exerts at the handle Hat lhe end of the lifting bar. The centre of gravity or
A vessel of mass 5000 kg moves horizontally at a constant acceleration ol 1.2 ms"' the lifting bar ls at P. The mass of the bucket Is 60 kg.
when it is being towed by two tug boats with tension~ Twhich are inclined at
30 and 60 respectively with the horizontal, as shown in Fig. 1.1.
r.
-.......
/
vessel moving horizontally
al oonstant acceleration of 1.2 ms
e
5000 kg
(chagram not drawn to scale)
Fig. 1.1 Bucket. 60 kg
{a) Calculate the resultanl force acting on the vessel. [2)
Flg. 2.1
\a) Calculate the force F necessary to hold the bucket steady, in the posit.on shown.
lb ) By drawing a scaled diagram, determine the tensions T, and Te.
[2]
(b) The bucket is lifted to a height of 0.12 m in 4.0 s .
....
(I) Calculate the power fe<1uired to lilt the bucket
power= [2]
tii) Suggest a reason why the power actua~y developed by the operat' during
the 4.0 s must be greater than your calculated value in (1).
. . ' . [1]
Scale=
Explain why the device Is designed so that the bar PH Is 1.5 m long rather than
0.3m.
[3)
"" -
(1!
SOSll/02/Preln\1201 O
5058/02/Pre!un/2010
4
A train lravel!ing at a constant speed of 22 m _,I .
s s approaclung a terminal station 5
When lhe driver applies lhe brakes lh .
d istance of 150 m before It comes to e train s1otWs down at a uniform rate to travel a 4 Fig. 4.1 shows a hydraulic press with a force of 85 N being exerted on piston K with a
a comp1e e stop. . cross-sectional area of 8 cm'.
(a) Sketch a speed-Um~_ graph of the train.
As piston K is depressed, Piston L moves up and compresses an air-filled nexible
plastic container attached to it. A glycerine manometer Is connected to the plastic
container to measure the pressure difference between the air inside and the
atmosphere outside.
air.filled
nexibla plastic glycerine
piston Kol container
area 8 cm2
(2J
(b) manometer
Calculate the time taken for the train
begins. to come to a complete stop after braking
hydraulic press
Fig. 4.1
{a) Explain how pressure is transmitted in the hydraulic press.
[2]
lime taken=
;-------- (2)
(b) Calculate the pressure exerted on the nexible plashc container.
(c) What is the uniform retarda!IOIJ of the train?
pressure= (2)
(c) Given that Ill e atmospheric pressure Is 1x10' Pa. calculate the density of
glycenne.
unirorm retardation =
(2)
5058102/Prelim/201 ~ densily of glycerine = (31
5058/02/Prelom/20 I 0
6
5 Some delails about benzene are iven below.
benzene 6 A tuning fork vibraling al 550 Hz produces a so_und wave of wavelength 600 mm. wilh
6C moving compressions and rarefaclJons In the air.
soc
uid benzene 1.s JI c (a) Explain what is meant by compressions and rarefactions.
250Jt
---- ~"'"..'- ---4"'. . . .
(al_, In the following graph, sketch the cooling curve or benzene from Oc to 80 "C,
lndicabng clearly its states, boifing anomelting point.
temperature
- ~ -
[21
--'"''''''-~# ------- --------...,,
--------
~--- (b) Deduce the distance from the centre of a compression to the centre of the
nearesl rarefaction In the sound wave.
distance= [1J
(c ) Calculate the speed or sound.
speed of sound = . ..o.o-.~----'--u
[2)
[1)
i Fig. 7.1 shows a circuit connected to a battery of 6 V.
(b} During boiling, thermal energy is required without a change in temperature. 6V
(i) What is the thermal energy required during boding known as?
'
[1)
........... -~,,--------------------- --------
(ii) Using ideas about molecules, explain why thermal energy is sttll required
during boiling, even !hough there is no change in temperature.
..... - ..................., .. t .- - ............................ ----v--.-.......... .
[1]
--- ..- R
(c) Calculale the lotal energy required to heat up 7.0 g of liquid benzene from 10 C Fig. 7.1
lo ils boiling point at 80 c.
(a) Determine lhe currenl reading In the ammeler.
total energy = I~! current reading = . [2]
5058/02/Prelim/2010
5058/02/Prehm/201 O
(b) Calcula te the potential differences between
(I) Pend Q,
Name component X and state its function.
---r
...----- , ...-....~---
---~-.:: ----- [2]
potential difference between p and Q =
[2] (,) At time, t = 0 s, the coil is rotated from its horizontal position at a constant rate of
(ii) Pand R . 5 revolutions per second. The maximum output voltage obtained from the
generator Is 12 V.
On the CRO, the time-base Is set to 50 ms per cm a,d the Y-galn is set lo
6 V per cm.
On Fig. 8.2 below, sketch the graph of lhe output voltage against time as
poten~al difference between P and R = displayed on the CRO, starting your graph from time f =Os. Indicate your voltage
----------- [2} and time scales clearly. (Each smaU square on the grid Is 1cm1 cm)
(c) T_h o points Q and Rare now joined b 8 I .
direction In which Y wre. Indicate, wtth a reason the
you would expect currenl lo flow in wire QR. '
- .... ----- ------ - ---------
..............M.................. ...
---------- ------ - --------
(1]
8
Fig. 8. 1 shows the structuro of a simpl
external circuit with its output t . I e a.c generalor wilh its coil connected to an
ermina s attached to a cathode ray oscilloscope (CR.OJ.
a~oEf
cuil rotabon
:l
,_~ ____.__..___.._~___..__.___.__..__.__.,__.___.__..__, 11 ...........
cm
/ magnet 1 cm
Fig . 8.2
/ s (31
/
x
'j.i---oo=tpu-c-t- j D
terminals
CRO
Fig. 8.1
Ii') Explain Why an alternating emf d
is rotated. . IS Pro uced at the output terminals when the coll
...... ,,. -----..,1.
- - -" ll!"'.'r.. . . , \ ,

. ... . . . . . . ,Y.M-.....,
,....... .
- ..........._______
---"'"" ..;-t---
-----.------------
.., ........................
[2]
5058/021Pr&im/201 o
5058/02/Prelimf201 O
10
Section B
Answer all lho questions in this section Some information provided by a manufacturer of wind turbines is given in Fig. 9.3.
Ans1.,er only one of the two alternative questions in Question 11.
height of tower from ground to hub 56m
9 rotor dianeter 44m
Win~ power can be used for the generation of eleclrrc power. Fig. 9.1 illustrates one number of blades 3
particular lype of wind turbine.
voltage 690V
frequency 50 HL
rotor blade output powerlkW
generator housin
500
wind speed/ ms1
0 10 15 20 25 JO
Fig. S.3
(a) (ii Use t'1e graph of tne manufacturer's data to give values of
Fig. 9.1 1. the maximum oulput power,
The ".'.""d causes t.he :otor bl~des lo lurn and these drive an electric generator The
~~~~'.~.generator is situated in the housing al the lop o' the tower, as illustiated in maximum power :: (1]
2. the wjnd speed for this maximum power.
WlllO speed; (1)
(i:i Air of density p and speed vis incidenl normally on a rolor of radius r. r1e
incident "'ind power Pwof the air incident on the rotor" given by
Pw "!..trr'v 3 p
2
The aor has density 1.25 kgm'.
Calculate the incident "Ind power on the rotor, when the wind turbine is
c;pe:ahng a! maximum output power.
generator fixing fer
housing blale
Fig. 9.2
ltlcident wind power ; ' 31
5058/02/Prelim/2010
5058102/l'relim12U 10
12 13
(Ill) At high wind speeds, the turbine Is 'cul out', that is the generator Is no Describe how the resistance of an unknown metallic conductor can be measured
longer turned by the blades_ 10 (a)
using a voltmeter and an ammeter. In your account,
1. Use FJQ. 9.3 to determine this 'cut-out' speed. draw a labelled diagram of the apparatus, Including any other equipment
'cut-out' speed = [1) thal you may use.
slate the readings to be taken. .
2. Suggest one reason why it Is necessary to have a 'cut-out' speed. explain how the resistance is calculated from the readings.
-. ~-a...-..1.. ir......:r...._____ ----------l"'
--------------- ------------------ [1)
(ll) Slate whether the generator produces direct current or alternating curren~
explaining how you came to your conclusion
---..~ ---'-----.. . ---r------.:.-----------..t.------ ......... - -------------------..------
[1)
(e) The wind turbine must be protected from lightning stn'lte.
(I) Suggest, with a reason, which part of lhe wind turbine Is most likely to be
struck by lightning.
----------------.0.:............ ---------
(1]
(Ii) Suggest how the risk of damage by lightning may be minimised. (4)
.., ...7_...1.. ,. ......... -- ---- -- - ----
. 1. I d 12 V 24 Wand Is connected directly across a 12 v
(bl A small heat1nl) co1 1s ra e
accumulator.
[1) I' 300 s
(I) Calculate the amount of charge passing through the heating COt an
[2}
amount of chorge =
(ii) Calculate the resistanco of lhe heating coil.
(2)
reslstance :::
5058/02/Prelim/201o 5058/02/Prelim/2010
- 1
14
15
(c) In Fig. 10.1, a heating coll and a componenlX are connected in series across a 12 V
accumulalor. 11 ElTHER
In Fig. 10.2, the component X has been replaced by a different component Y. (a) Fig. 11.1 shows lhe positions of the eye of a car driver, lhe wing mirror of the car
-and the front of a lorry.
12 v 12V
~-_..j l J1--~ _ __,lj1-~
win_y mirror
f<:LUUI
With component X in the circuit, the coil does not heat up, but on reversing the
connections to X , the coil heats up.
With component Yin the circuit, the heat produced per second by tile coil can be
varied, and reversing the connections to component Y makes no difference lo the heat

car drive(s eye


R---;-c:fro--,nt-,of"'lo=rry-,----I 3\
produced per second. Fig. 11.1
ldenbfy components X and Yand explain the effect in eaeh case.
(<)
On Fig. 11.1, draw two light rays to indicate clearly the section of the front of
- - - - - -.......- ---.,. -"' OOOH00AooooooO OOOOUOOOOOOOOOO O --OO OOOOOHO OHO ~OO_ _..., _ _ the lorry which can be seen in the wing mirror by the car driver. (2]
(fj) The wing mirror of a car is often curved "with a surface of the shape shown
in Fig. 11.2.
....................... ............................................ (2]
,~urvedm
irror ,
' '
'' ''
./ \,
'
~ ''
!' '\
normal
normal
Fig.1 1.2
Using the laws of reflection, explain how the curved mirror would enaiJle
more of lhe front of the lorry to be .ten.
[2)
5058102/Prelim/2010
5058102/Prelim /201 U
16
(b ) An object 15 mm high is placed 70 .
length 50 mm. mm on front of a converging lens o' focal 17
{i) Using a scale diagram, draw a ray dia . OR
the distance o.Ltlle imaoe fr!llll.lbe len~'.31'1 lo detonmne omagtlllightand
Fig 11.3 shows a circuit with a steel bar that is being magnetized in a solenoid.
solenoid
~eel bar
Si;ale;
Fig.11.3
image height ;
- .......-..........._______ _
distance of the Image from the lens ; (al On Fig. 11.3, draw the magnetic field pattern created by the solenOtd. (2)
[4]
(II) Describe the nature of the image formed. {bj Fig. 11.4 shows a piece of r.ermanent magnet being placed near one end of the steel
bar.
------------------- solenoid
._,,,_~OOOooO- OO-oOO. . .Oooooo-
-o o > o o u o . ,, . ...... . . . O o o H - O Hoooo
----
------..----........................ - ...................................... - ...... (1)
(!ii)
If the object ts moved to a distance f 60
image distance be affected?
..

o mm from the lens. how will the


. . .................. -..!'. --

9 permanent magnet
----- --- -- [1]
Fig. 11.4
Stale and eplain what will happen to the permanent magnet.
5058i02JPrelim12010
(2j
5058/02/Prelim/20 tO
18
CHI~ St Theresa's Convent
(~) Describe a method In which the circuit can be modified to de-magnetise the steel bar. Secondary 4 Express Physics Preli ms
Answer Scheme for SA1 2010
------- --------- ..}..------------------ 4
0
------------------------------------------------------.......................______________________ [2)
t"l Describe an experiment to show how a beam of electrons travelling into a magnetic
field can be deflected. In your account.
draw a labelled diagram showing the beam or moving electrons and the magnetic
field,
indicate clearly the direction of the electron beam before entering. within and
beyond the magnetic fteld.
Explain what causes the deflection of the electron beam.
----------- -. ---- - ------------- ---......... ---- .... .. ----------------------.... ------ ---- ... - ----- [4)
- End of P~per -
5058/02/PreLm/2010
5058/01IPrehm/2010
CHfJ St Ther esa's Convent
Sec 4 Express Physlc.s CHIJ St Theresa's Convent
A nswer Scheme for 20 10 Prellms Sec 4 Express Physics
1a.F,. 5000 x 1 .2 [C1] Answer Scheme for 2010 Prelims
= 6000 N [Al )
lb. correct scale and diagram [B1)
r.=3000N (A1]
Sa.
To = 5200N [A 1]
T/<'C
(Allow ECF and error allowance to 10%).
=
2a. F x 1.5
F = 120 N
600 x 0.3
(C1 )
[A1]
2bi. Power= (60x10x0.12)/4 [C1)
(81]
= 18W [A1]
5bi. Latent heat/specific latent heat of vapourisationnatent heal of vapourisation (81]
2bii. Energy or power is required to overcome the frictional fcirces. [81 ]
... 5bii. Dur1ng boiling from a liquid to gaseous state, thermal energy is required to overcome
2c. Less force is required to achieve the same turning moment with a longer bar. (81)
the attractive forces o f the liquid molecules to allow the molecules to separate and
(moment = Force x perpendicular distance.)
disperse into the gaseous stale.
3a.
Sc. o, = 7 x 1.8 x (80-10} [C1]
&.peedJ'm5I = 882 J
"~..
0 2 = 7 x250 [C1]
= 1750 J
Total energy= 882 + 1750 [C1)
= 2632 J [A1)
6a. Compression Is the moving of air molecules together as tuning fork vibrates forward [81]
Graph is able to display:
Rarefactions is lhe de-compression of air molecules as tuning fork mov es backward [811
Uniform speed of 22 m/s (61)
Constant retardation (6 1]
6b. 300 mm [A ll
3b. 0.5 x 22 x t" 150 [C1]
6c. speed = 550 x 0.6 [C1J
I= 13.6 s [A1]
= 330 mis [A 1)
3c. Retardation = 22/13.6 (C1 J
= 1.61ms' 2 \A 1) I I [C1 ]
7a. current= 6x( + 4 "2+1)
2
4a. The hydraulic press makes use of the property that liquids are nol compressible [6 1] =3A [A1]
and thus pressure can be transmitted equally, through the trapped liquid, [B 1]
to the big piston upon pressing the small piston. 2
7bi. pd b/w PQ = - - x6 [C1J
2+4
4b. p = 85/(0.0008)
= 106250 Pa or 1.0625 x 105 Pa
[C 1] = 2V [A1]
[A 1]
4c. pressure difference= 0.0625 x 105 Pa [81] 2
(0.8 - 0.3)x px10 = 0 .0625 x 105 [C1 ) 7bii. pd b/w PR= - - x 6 [C1 )
2+I
p= 1250 kgm-> (A1] [A1]
=4V
le. R to Q because R is at a higher potential than Q_ [81]
CHIJ St Theresa'~ Convent
Sec 4 E><press Physics CHl l St Ther esa's Convent
Answer Schem e for 2010 Prelims Sec 4 Express Physics
Answer Scheme for 2010 Prelim5
Sa. During lhe first hall of the rolaUon th 1 10c. X: diode -') ii rectifies the circuit and allOw current to flow in one direction only when
magnets at a changing rate this i~d e co1 cuts the .magnetic nux created by the 2
~~:~i-'$%~i'~:"nde run=.
~~In one particular
ii is forward biased. [81)
direction that can be detern:ined by [611 Y: Rheostat-') It allows resistance to be varied and hence current or heat proouced
per second can be varied. {B1~
Ounng the second half of the rotation .
n1verses jts djrection. ' current continues to be induced in the coil but
(61) 11 Either
This alternate reversal of curr 1
terminals as the coil is rotateden creates an alternating emf across the output 11ai.
Sb. X is the slip ring.
i t Is used lo conduct current from ends of coil to the output te . I . [61]
rushes. rm1na s via the carbon
b [61]
Be
ITTTTT"TIElll~I,_
~
Perio.d = 0.2 s , with correct scale (61]
Amplitude = 12 V, with correct scale (61] (62]
Max. voltage at t =O , with sinusoid [Btj 2 correct rays on extreme ends of mirror/co1Tect section indicated
-1 m !or any Incorrect rays.
9ai1. 600kW (01]
9ai2. 15 mis [B1J 11aii. On the left-end of the curve mirror
angle of incidence (b/W Incident ray 1 and normal of curve surface) is reduced.
9aii. r = 4412 = 22 m [81) This means thal the reflected rey wiU have smaller angle of reflection implying
Wind power= 0.5 x 3.142 x 222 x15'x1 .25 (C1) that more of the leflslde of the lorry is reflected to the car drive(s eye. (61)
= 3.20 x 106 W or 3207 kW (A1J ...
On the righl-<!nd of the curve mirror,
9aiiJ1. 25 mis angle of incidence (b/w incident ray 2 and n01TT1al of curve surface) is increased.
9ai2 too high a wind speed . des troy uie turbine/generator/prevents
. may . damage 1IJ
(A 1
(A This means that the renected ray will have a larger angle of reflection implying
that more of the nght side of the lorry is rellecled lo the car driver's eye. [61)
9b. a c as the vollage has alternates al a frequency of 50 Hz given by the data. [611 This effectively increases the range of mirror that can be seen.
[Bl)
9ci The rotor blade as II is highest and closest to the clouds when ch arges are built
. up.(61) 11 bi. - correct scale f [01)
gcu
- correct ray diagram (dolled lines and arrows lo be shown) [01)
Using a hghlning a1Testor, or earthing the rotor blades. [61) image distance= 175 mm (10% tolerance allowed) (61]
- imago height= 37.5 mm ( 10'!. tolerance allowed)
10a. - correct labelled diagram [B1J
- co1Tect readings to be taken (61]
- repeal experiment to calculate the average of the re di [B1]
fr~mn~~ readings
~]
co1Tect description of how resistance is calculated (81]
10b1. Q = (24 x 300)112 (C1]
= 600 c [A1]
[6 1)
10b11. R c12'124 [C 1] 11bii. reallinverled/magnitied (any 2) [61]
=60 [A1] 11biii. image dista11ce is Increased
CHIJ St Theresa's Convent
Sec 4 Express Phy$lcs
Answer Scheme for 2010 Prelims
OR
11 a. correct field line direction from north to south (61)
correct field pattern (61)
f
11 b. It will be repelled away from the solenoid as north pole is induced on the solenoid at
the end closer to magnet and like poles repel. ' [B 1)
11c. Pass a.c Instead of d.c to the circuit. [81)
Pull out Ifie magnetized steel bar In e east-west direction slowly as a.c Is passed
through the solenoid. (81)
11d. labelled diagram showing correct direction of electrons and magnetic field. (81]
Electrons bending in correct direction within the magnetic field, and no bending
beyond magneUc field. [82)
State Fleming's lefl hand rule with conventional current direction is used to determine
force. Be~nd [8 1)
Holy Innocents High School
Secondary 4 Express Physics
Paper 1
1.
Object X is placed against a ruler.
~ .
A sack of weight W hangs from the end of a rope. The sack is pulled sideways by a horizontal
force F and Is held stationary. The tension In the rope Is T.
10 11 12 13 cm
.. rope lenslon
T
F
w
This ruier is placed against another ruler to measure the end correction.
Which force diagr~m gives the correct value and direction for the tension T?
What is the length of the object X?
V '\l ~
A 0.9 cm
B 1.1 cm
fl
C 1.3 cm
D 1.5cm
2.
The speed-time graph below shows how the speed of a bus varies with time. f' F
A B C D
5.
A net force Is applied on a body with its variation as shown in the figure below.
f
'\,
tima
time interval X time lotorvaJ V
-I
l'
. 31 time
Whal Is the average speed of the bus?
A
B
P+X
Q + y
What is its velocity-time graph?

'~[=?=: ~Fl:_
C (P-Q) + (X+Y)
D (P+Q} .- (X+Y)
3. I
A table-tennis ball is dropped from the same height In a vacuum and then in air. 21 31 bme
l 2t 31 time
What effect will air resistance have on its time of fall and its average acceleration?
. A
B
time of fall acceleration \'eloci~
A increase decrease
B increase same
c same decrease
D same increase it 3t ~me
0
c 2
6.
The apparatus below is set up on Earth. It shows a piece of soft iron X of mass 0.60 kg fixed on 9.
one end of a uniform beam AB which is pivoted at its midpoint. X is prevented from being pulled Elaine launches a remote control helicopter up to the sky at a constant acceleration. What are the
down by a fixed magnet by a load of 1.00 kg hung at A. changes in the gravitational potential energy and the kinetic energy?
Gravitational Potential energy Kinetic energy
A B
A decrease decrease
tJ1oad soft iron X 2=i B
c
decrease
increase
increase
decrease
1.00 kg
magnet D D
10.

increase increase
Whic h of these statements about pressure is not true?
2
The experlmenl is now conducted on the Moon where the acceleratlon due to gravity Is one-sixth A The S.I. unit of pressure Is the pascal which Is equal to 1 N m
that of the Earth. The mass of the load at A f equired to maintain the equilibrium of the beam Is B The greater the area a force acts upon the greater the pressure.
c Pressure in a liquld Increases with depth.
A 1.00 kg D Force Is equal to pressure times area.
B 3.00 kg
c 3.60 kg 11.
0 6.00 kg The U-tube shown in the diagram contains water and a liquid which are separated by mercury.
7.
'
John applied a horizontal force F on an empty light plastic container as shown below and caused it
to fall easily.
ter
liquid
p Q
Which of the following statements aboUt this arrangement is correct?
A The pressure al P is less than at Q because there is a smaller volume of liquid above it.
Which of the following aclion(s) will need a force greater than F to cause the container to fall? B The density of the liquid is less than the density of the water.
C The pressure at P and Q Is equal.
1. half fill the container with water D The pressure of the water column Is greater than the pressure of the liquid column.
2. 1111 up the container fully with water
3. apply the force al 1 cm lower than the original position 12.
The diagram shows five mercury barometers. For which of these is the external pressure greatest?
A 1 and 2 only
B 1 and3 only
C 2 and 3 only
D 1, 2 and 3
8
An engine pushes a car forward through a distance of 10 m with an average force of 500 N. There
ls a frictional force of 300 N. What is the total work done by the engine?
A 2000 J
B 3000 J
c 5000 J
D 8000 J
A B c 0
4
13.
In a hydraulic car jack, the diameter of one piston is frve limes that of the other pis:on. 16.
Some steamboat restaurants use paper pots for their customers lo boil the food themselves.
What is the reason for tho paper not to catch fire when in contact with the flame?
The force on the larger piston compared to the force on the smaller piston i s -- -- - - - -
A 5 times smaller
B the same
C 5 times larger
D 25 times larger
1. The paper is thin and thererore heal Is conducted quickly to the water in lhe paper pot.
14. 2. Water has a boiling point lower than the burning temperature of the paper.
When Gail blows into the manometer with a pressure P above a tmospheric pressure, the level in 3. The paper is thick Elnough to withstand the high temperature of the flame.
the left-hand limb rises to point X. W ha t would be the level in the right-hand limb if Gail increases
the pressure of her blowing to 2P above atmospheric pressure? A 1 and 2 only
B 1 and 3 only
c 2 and 3 only
D 1,2and3
17.
o
A tungsten wire has a resistance of 30 O al o c and 50 at 10oc.
Whal is the working temperature of the wire if its resistance is 45 O?
A 25c
1)
x B 45c
c c 1sc
[) ooc '
18.
Two copper blocks X and Y have masses M, and M,.. In an experiment the blocks are heated.
A
The results of the experiment are shown in the table.
15. thermal energy lnlUal 1e1npetalure final lemperalure
block
A student is Investigating the evaporation of waler. supplied/ J JoC Joe
The student can change:
x 2000 -10 +30
the depth of the water;
the surface area of the water; t _-_ ------- - -_-j y 2000 -10 +10
the temperature of the water.
How many of these changes, if any, would alter the rate at which evaporation occurs?
What is th" value of M, I My?
A 0
A 1 A 113
c 2 B 112
D 3 c 2
D 3
5
19.
23.
In an experiment, 2040 J of thermal energy (heat) Is used to heat 10.0 g of Ice at oc. A ripple tank is used to demonstrate refraction of plane water waves
Assume that water from the melted Ice remains at oc.
~ vr---
deep water
How much ice remains after healing? (specific latent heat of fusion of ice= 340J/g)
,_ /IJ
A O.Og
a 4.0g
c 6.0g
D 8.0 g
20.
Ice is taken from a freezer and left in a room. The Ice melts and eventually the water reaches room
temperature. Which energy transfers take place? boundary o.acm
energy transfer during melling energy transfer after melting Based on the given figure above, what Is the speed of the waves In the shallow water?
f
A from ice to room from water to room A 6.4cm/s
a from Ice to room from room to water 8 8.0cmis
c from room to ice from room to water c 9.6cm/s
D from room to ice from water to room 0 14.4cm/s
21. ' 24.
Some air Is trapped Inside a small balloon. The average kinetic energy of the air molecules in the Which of the following uses gamma rays?
balloon is increased. What remains the same?
1. sterilising
A the density of the air in the balloon 2. killing cancerous cells
B the mass of the air In the balloon 3 detecting flaws In metals
C the temperature of the air in the balloon
D the volume of the air In the balloon A 1 and 2 only
8 1and36nty
22. c 2 and 3 only
The diagrams show different views of a water wave in a-ripple tank. D 1, 2 and 3
25.
What Is a property of all electromagnetic waves?
A They are deflected by magnets.
s B They are positively charged. .
R c When they pass from air to water, their frequencies, wavelengths and speeds all increase.
p 0 They travel through vacuum.
26.
An observer O cannot see the image of target
Scale of 1d~ision 1 m
T through the mirror.
. cross-section of wave crests seen from above What Is the minimum distance he should
move to see the image of target T through the 0
Which letters represent a wavelength and a wavefront? mirror?
. ,~. _......., ..... . . .. "" .
.T.M
' .
wavelength wavefront A 1m ' \ ' 'mirror
A p R B 2m
B p s ,c 3m
c a R D 4m
D a s
8
27.
The diagram below shows three rays of light passing through a converging lens. 31. ..
The diagram shows part of an electric circuit.
Which point will the ray XY pass
through after refraction by the lens? What is the currant in the 20 resistor?
6.0A
A 1 A 0.6A
B 2 B 1.2A
c 3 C 3.0A
D 4 D 6.0A
28. 32.
A hammer strikes one end of a very long metal pipe. A detector at the other end detects two The diagram shows a thermistor connected in a potential divider circuit.
sounds at an interval of 2 s. Given the speed of sound In air and the speed of sound in metal Is
320 ms and 5000 ms' respectively, what is the length of the metal pipe?
A 2.1 m
B 684m
C 4680 m
D 9 600 m
29.
Four processes are used to charge an isolated metal sphere.
p The sphere is earthed by touching it.
Q The earth connection Is removed from the sphere.
R A charged rod is brought close to tho sphere.
s The charged rod Is removed.
The resistance of the thermistor decreases when Its temperature rises. The thermistor is heated.
In which order should these processes be carried out to charge the sphere? What happens to the potential difference across the thermistor as it is heated?
first last A It decreases but not to zero.
A p Q R s B It decreases to zero.
B p R s Q c It increases. "
c R p Q s D It stays the same.
D R p s Q
33.
All lamps In this circuit are rated at 12 V, 36 W. A, B, C and Dare 5 A fuse. Which fuse will melt
30. when the switch 1s closed?
The diagram shows an electric circuit.
y
Which pair of readings is obtained when a suitable power supply is connected between X and Y?
vottmeter ammeter
A 2V 0.5A
B 6V 0.5A
c 12V 0.5A
D 24 v 6.0A
9
10
\b~
34.
An engineer uses an allemating current supply and a diode to recharge a car battery. This is done
by passing a current through the battery in the opposite direction to that when it is discharging. -
The . WIthtlme.
graph shows how the output voltage from a simple a.c. generator vanes
Which circuit Is correct?
A B
a_c_
supply

r*l
I t -,-
When this output is passed through a component In the circuit it changes to this.
c D
~~r (\ (\ ( ~
[fi LD
lime
a.c. 8.C. What Is the component?
supply ~
supply -'"--:
A a diode
a a fuse
C a lamp
D a resistor
35.
Each of the diagrams below is a cross-section through two parallel current-carrying conductors. 1
Which diagram correctly shows the magnetic field pattern formed by the currents in the two
conductors?
! ~oilp Is connected to a centre-zero galvanometer. A second coll Q is placed next to P and Is
connected to a battery, rheostat R and switch S as shown in the diagram below. Which of the
graphs below best represents the denection of the galvanometer when the switch Is closed for a
@>f@D
few seconds and then opened ?
A
current Into plane of dl&gram
0 CUenl oul of plane of diBg<sm
B
~ s
0
@!f@D ~h~~ ~h~ ~Rwoo ~~necuon
'~
Ume .
hme lime
time
0 A B C 0
'
11
12
38.
Which transformer anangement produces an output voltage that is larger than the nput voltage. SECTION A (40 marks)
A B 7 8 9 10 11 12 13 14 15 16 17 18 19 20
2 3 4 5 6
c D D A
,,~:~, .. ~~'
B
36 37 38 39 40
24 28 29 30 31 32 33 34 35 c
B
input~ input~ D
c 0
"~'' ..~~~
input~oupu input~
39.
Figure1 below shows the magnetic field near a bar magnet and an object XY. In 'lgure 2, XY has
been turned around.
What ls XY?

m ro Figure 1 Figure 2
A a magnet wilh the north pole at X
B a magnet with the north pole at Y
C a copper rod
0 a rod of soft iron
40.
Identical metal bars X and Y are positioned in the two diagrams shown below. If there is a sl!ong
attraction shown in figure 1 and a very weak attraction shown in figure 2, what are the possible
material for X and Y?
BarX BarY
A steel bar iron bar
B steel bar bar magnet
c bar magnet steel bar
0 bar magnet bar magnet >.. ngure 1 Figure 2
14
13
Section A: Short-Answer Questions (45 marks)
Answer all the questions. Write,your answers in the spaces provided. (d) Draw simple diagrams to show the arrangements of particles of B at the temperatures
[2]
given.
l~oJ Q
1. Below is a table of information on two mixtures, A and B:
Mixture State of mixture at Technique used to separate
r.t.p '
components
A Solid Add a solvent, then filter 4oc 4ooc
B Liquid Fractional distillation
3. Elements A, B. c and o are in the same period of the Periodic Table. The following
(a) For each of the mixtures above, state the respective physical property that enables the Information is given:
components to be separated from each other. [2] An oxide A 20 exists and Is strongly basic.
Mixture A: Different solubil itiestf the components in the same solvent [1 ] The oxide of B is B,03.
Mixture B: Different boiling points [1] C produces an ion c.
o forms a covalentcompound DC!,, a liquid at room temperature.
(b) State how you would attempt to prove that a given liquid is a mixture and not a pure
liquid. [1] (a) Based on the above information, put A, B, C and 0 in their respective groups .in the
Heat the liquid. If it boils over a range of temperatures [1], it is a mixture and not [2]
Periodic Table.
a pure liquid.
Element A B c D
Group Ill VII v
2. Some properties of four elements are shown in the table below. Each element is represented
by an alphabet. The alphabet is not the symbol of the element.
[1)
(b) What type of bonding is present in 13,03?
Element Melting Boiling Electrical conductivity at Action with cold Ionic bonding
pointfC pointt'C room temperature water
A -219 -183 Nil Dissolves slightly (c) Write the for~ula for the compound formed between Band D. [1 ]
B -39 357 Good Unaffected BD
c 98 3go Good Liberates
hydrogen
D 1063 2970 Good Unaffected
--
(a) Which elements are probably metals? (1]
B, C and D (Any two)
(b) Which element is a solid at 20C and melts when dropped into boiling water? [1)
c
(c) Which element reacts most vigorously with dilute hy~rochloric acid? (1]
c
, 1 of1 0 Page 2 of 10
4. A lump of 49.6 tonnes of copper{ll) carbonate was added to excess dilute sulfuric acid as
It loses 2 electrons to form Zn' [1}
shown in the chemical equation below. The gas evolved was collected in a gas sylnge.
(b) The graph below shows the volume oP hydrogen gas produced when excess zinc granules
CuC03 (s) + H2SO, (aq) ~ CuSO, (aq) + H20 (I)+ co, (g) 3
are reacted with 10 cm3 of 1.0 mol/dm hydrochloric acid.
Vol of pas
(a) Calculate the mass of dMl.lle sulfuric acid needed to react with copper(ll) carbonate. [2] (cm')
No. of moles of copper(ll) carbonate =49.6X10'/(64+12+48) 300
=400000 mol (1/2] 240
cuco, : H,so. 180
1: 1 120
No. o f moles of sulfuric acid = 400000 mol [1/2} 60
Mass of sulfuric acid = 400000 X (2+32+64)
r . .e(s)
= 39200000 g 5 10 15 20 25
= 39.2 tonnes (1] (i) How many moles_.?' hydrogen gas were produced? {1]
No. of moles of hydrogen gas = (10/1000)X1
(b) Calculate the volume of the gas evolved at room temperature and pressure. [21 =0.01 mol
No. o f moles of carbon dioxide gas =400 000 mol (1)
Vo lume of carbon dioxide gas =400 000 X 24 (ii) On the same graph, sketch another curve you would expect if the same amount of
=9600 000 dm [1}3
powdered zinc was used instead. Label this graph A. [1 I
5. From the list of substances below, choose the most suitable answer for the following (iii) On the same graph, sketch the curve you would expect if the same amount of zinc
questions: [4) granules was reacted with 20 cm3 of 0.5 mol/dm3 of hydrochloric acid. Label this graph
silicon(IV) oxide carbon monoxide sulfur d ioxide B. ~)
magnesium oxide carbon dioxide zinc oxide
"
(iv) Explain your answer for graph A. [3]
(a) A substance which is a greenhouse gas. Carbon dioxide Powdered zi nc has smaller particle size, therefore larger surface area than zinc
(b) A gas which is neutral to damp litmus. Carbon monoxide granules. [1] Particles collide more frequently, and there are more effoctive
collisions, (1/2) resulting in a faster rate of 1 eaction (1 /2). The volume of gas
(c) A substance which reacts with both an acid and an alkali to form a salt.
Zinc oxide produced is t he same as the same amounts of reactants are used (1].
(d) A substance which reacts with rainwater to form acid rain. Sulfur dioxide
(ii) &(iii)
6(a) Zinc reacts with hydrochloric acid according to the following equation: v~:;,~.,
300
Zn (s) + 2HCI (aq) 7 ZnCl2 (aq) + H2 (g)
240
180
Name the substnnce which is oxidized, giving a reason for your answer. [2)
Zn is oxidized. (1) 120
Tlme(s)
Its oxidation state increases from 0 in Zn to +2 in ZnCl2 OR 7. The elemei!iO rubidium is in Group I of the
20 25
0
3of10 4 of 10
Periodic Table.
(a) Wri1e the balanced chemical equation for the reaction of rubidium with water. 9. Ethanol is made from sugars and can be used as an alternative fuel to petrol for car engines.
[1)
2Rb + 2H;O ~ 2RbOH + H2 (a} Complete the equation for the conversion of glucose into ethanol in the presence of
yeast.
(b} Suggest an element in Group I that reacts with water more vigorously than rubidium. (1)
Cae!jium I Francium glucose
,.... ethanol [1/2) + carbon dioJCide [112) (1]
(c) (b) Name the above process for converting glucose into ethanol using yeast. [1]
What would you observe if copper(ll) sulfate solution Is added to the solution found In
{b)? Fermentation
[1]
A blue precipitate (copper(ll) hydroxide) will be formed.
(c} State a method used to obtain ethanol from the mixture. [1]
8. The diagram below shows some properties and reactions of substancesK, L, Mand N. Fractional distillation
f
10(a)State the two methods of extracting a metal from its ore. (1)
Aqueous
solution of green Electrolysis [1/2) and reduction of metal oxides by carbon [1/2)
solid K
Add dilute nitric acid and (b) Explain why carbon can be used to obtain zinc from zinc oxide but not to obtain calcium from
aqueous barium nitrate
calcium oxide. [2)
Carbon is above zinc, but bel ow calcium in the reactivity series. (1) Therefore, it is able
to displace zinc from zinc oxide (or reduce zinc ion to zinc metal), [1) but not calcium
from calcium oxide.
OR
Calci um Is above zinc in the reactivity series I more reactive than zinc, [1) therefore
Green solution White solid L calcium oxide has strong bonds which cannot be broken down by reaction with carbon.
[1)
Add aqueous
sodium hydroxide
r---~--~ (c) Sugge_st one reason why it Is Important to recycle metals such as copper. [1)
Green
precipitate M Metals such as copper are finite resources, so it is important to recycle them.
Brown solid N
Oxidation
11. A student wishes to prepare a dry sample or barium sulfate crystals. Describe, stating the
Identify suitable starting materials, the steps to prepare the crystals. [3)
(a) white solid L barium sulfate Add aqueous barium nitrate to aqueous sodium sulfate [1.5] OR add aqueous barium
(b) green precipitate M iron(ll) hydroxide nitrate to sulfuric add {1.5) (or any suitable solutions)
(c) brOVln solid N iron(lll) hydroxide **Mentioning of 'solutions' or 'aqueous' [1/2] as precipitation requires the reactions to
(d) green solid K lron(ll) sulfate be in aqueous form
[4)
Filter the mixture. The residue collected is barium sulfate crystals. [1/2]
Wash the bari um sulfate crystals with distilled water. [1/2]
1
Dry the barium sulfate crystals between filler papers. [1/2]
Page 5of10
Page 6of10
Section B : Free Response Questions (20 marks)
Answer any two ques!ions. Write your an!>wers on the answer paper provided. H H H
I I I
l (a) The table below gives the boil,ng points of some alkanes: H - C-C- C - Cl Or
I l l
H H H
Name Formul a Boiling pointlC
Ethane ~H6 -88 H H Cl
Propane C3Ha I I I
-50 H-C-C-C-H
Butane C,H,0 -11 l I I
Pentane
H H H (1)
CsH12 28
H-CI [1)
(i) Plot a graph of boiling point against number of carbon atoms for the alkanes in the
table. 2(a) You are provided with the following substances:
(2)
Scale: 1cm 1oc aqueous chlorine
2cm 1 carbon aqueous iodine
Correct axis and scale - {1] aqueous potassium chloride
Correct points plotted - {1] aqueous potassiu m iodide
(ii) Use your graph to deduce the boiling point of hexane. C,,H,.. Use all the substances from the list to show that chlorine Is more reactive than iodine. Your
[1J
66c answer should include:
(i) details or all your observations;
(iii) A hydrocarbon has a fonnula C,H, 5 Explain why this is not an a lkane. (ii) explanations of the reaclion(s) (if any);
[1]
.Alkanes have the general formula CnHln2o and C, H,. does not fulfill this. (ii) the chemical equation(s) (with state symbols) for the reaction(s) (if any). !4]
(iv) State one other property besides boiling point that shows a gradation on moving up Cl2 (aq) + 2KI (aq) ..... 2'Kc1 (aq) + 1, (aqJ 11
through the members of the series. Observation: Yellow liquid changes to brown after some t i me. (1)
(1)
Density, viscosity, flammability
Anyone When iodine is added to potas sium chloride, the brown colour of iodine remains,
showing that no reaction has taken place. (1)
(b) (i) Construct the chemical equation for l he combustion of hexane. Chlorine Is more reactive than iodine because ii is able to displace iodine from an
[1)
2c.H,. + 190, 7 12co2 + 14H,o aqueous solution of potassium i odide, while iodine is unable to displace chlorine from
an aqueo\ls solution of potassium chloride. (1)
(ii) Besides combustion. state the olher reaction lhal alkanes undergo. W ith propane as an
example, construct the chemical equation for this reaction, then draw the full structural (b) Hearthside sails" is a nalurally occurring salt. It is soluble in water, and can be tound n1ixed
formulae of the products fonned. with sand. Describe how you would prepare pure, dry crystals of Hearthside salts from this
[4)
Substitution [1] mixture. (6]
C3H, + Cl2 --> C, H7CI + HCI [1] Add water to disso lve the Hearthside salt t o form a soluti on. (1)
Filter the mixture to obtain sand as the residue. (1)
Heat the filtrate, Hearthside solution, to evaporate the water until it is saturated. (1]
Cool the saturated solution and observe for crystal formation. (1]
Page 7 of 10
Page 8of10
Filter to separate crystals. [1]
Diagram of chloromethane:
Wash the c~tals with cold distilled water. [112)
Correct number of elements and atoms - [1]
Dry the cryst als between filter papers. (112)
Correct number of shared electrons -[1]
Correct number of electrons Jn chlorine - [1]
3(a) Explain, with the help of a suitable example and a 'dot and cross' diagram, the type of bond
formed between metals and non-metals. [4]
Ionic bond [1) is formed between metals and non-metals.
Using sodium and chlorine as examples:
Sodium atom will give away its 1 val ence electron to chlorine atom [1], forming sodium
ion and chloride ion.
~Jf@)]
Electron of chlorine
X Electron of sodium
Sodium chloride
Diagram of sodium chloride:
Correct number of electrons - [1]
Correct charges - [1]
(b) Explain the following statement
"Ionic compounds have high melting points, and conduct electricity when molten" [3]
tonic compounds have strong Ionic bonds I electrostatic attraction (1) between the
ions, therefore a large amount of heal Is needed to break these strong bonds. (1] Ionic
compounds conduct electricity when molten because the i ons are free to move. (1]
(c) Draw a 'dot and cross' diagram to show the arrangemenl of electrons in a molecule of
chloromethane. Only the valence electrons need to be shown. [3)
Electron of carbon
X Electron ot hydrogen
and chlorine
Page 9 of 10 Paoe 10 of 10
Hua Yi Secondary School
Secondary 4 Physics Paper 1 2
1 The thickness of a pencil is measured using a vernier caliper as shown in the 4 The acceleration due to gnwity on the Moon is about 1/6 that on the Earth. Which
diagram. of the following diagrams shows the correct speed-time graphs for a free falling
body when dropped respectively on the Earth's surface and the Moon's surface?
v
t~uuu-.,
B
A ,L2_,
pencil D v
What Is the thickness of the pencil? /Moon
A 0.90cm B 0.95 cm C 1.45cm D 1.50cm
2 The diagrams show three steps in an experiment to measure the density of a
metal rod.
(i) l1 (lh) 5 A boy throws a stone vertically upwards with a speed of 1O m/s. What is the
100 100
magnitude of the velocity and acceleration of the stone when it reaches the
greatest height?
Measuring
cylinder
60 reading 60 velocitv Im s acceleration I m s'
Metal rod inan3 A 10 0
\ o B 10 10
Metal rod c 0 0 '
lO
Digital-+
"' -'o 0 10 .
balance
~'----~~...L~_J.~~-'-~~~--'.......J"--L.
What is the density of the metal?
For questions 6 & 7
A 0.3 g/cm3 B 3.1 g/cm3 A water-rocket is launched vertically upwards into the air and its velocity-time graph is
c 5.6 g/cm3 D 7.0 g/cm3 shown below.
vims'
'~
3 A car travels round a circular path of radius 50 m. It completes one circle in
1 minute. F ind the dist ance traveled by the car and its average velocity.
(Take"= 3 .14)
distance Im velocity Im s
A 50 0.00
tis
R 314 - 0.00 0 I 2 3
c 314 /
5.23
..
D 7850 130.8
4E Physics P1 Prellm Exam 2010 Hua YI Secondaiy School 4E Physics P1 Prelim Exam 2010 Hua Yi Secondaiy Sc11ool
4
3
9 What will be the motion of the spaceship if the drive motors are off?
6 What is the maximum displacement made by the water-rocket?
A stationary
A 10m B 20m
B decreasing in speed
c. 30m D 60m r. increasing in speed
0 maintaining its present speed
7 Whal is the corresponding acceleration - time graph of the object?
a a 10 To slow down the spaceship, he has lo fire the main rocket motor. As the officer
A B is looking ahead, towards the Andromeda Galaxy, the direction he must lire the
motor is
A forwards, towards Andromeda.
~ backwards, towards the Milky Way.
C to the right of his path.
O to the left of his path.
c a D a
11 The spaceship has a mass of 1o kg. If the rocket produces a thrust of 105 N,
what is the acceleration of the spaceship?
A B
c 0
12 The graph below shows the varia lion or the net forcu acting on an object with
time. The object Is initially al rest.
8 An object is dropped from an airplane that is 5000 m above the ground. Which of force
the following best describes the motion of the object during the fall?
A It moves with constant speed. time
R It moves with constant acceleration.
C It moves with constant acceleration followed by constant speed.
D It moves with decreasing acceleration followed by constant speed. Which of the following velocity-time graphs correctly describes the motion of the
object?
For questions 9 to 11 A vcloci1y B
The officer in-charge of a spaceship Is travelling from our Galaxy to the Andromeda
or-~
Galaxy. He is in deep space where the gravitational field due to surrounding galaxies is
zero.
time time
Andromeda
c vclocil)'
0
Ga laxy
o~
l!'r- time time
Hua Yi SeeoncWy -
E Physics P1 Prelm Exam 2010 Hua YI Secondaty School
5 6
13 A uniform beam XY is balanced on a pivot w ith two weights, W 1 and W 2 hung at 16 The water at the foot of a waterfall has a higher temperature than the water at the
equal distances from the pivot. top even though the surrounding temperatures are equal. What is this
temperature difference for a waterfall 100 m in height, given that all the available
energy appears as heat in the water.
0 1 2 3 4 5 6 7 8 9 10 (Assume specific heat capacity of water= 4000 J/kg K, acceleration of free fall =
x L:=.~1~-1i---'----,~----'----1lc---..J__--l___J) y 10 mls2)
A 1/8 K B %K
w, c y, K D 2K
17 T he diagram shows a U-tube containing water and oil.
It is observed that the end Y moves downward w hen W 1 is removed.
W here should the pivot be in or.fer to balance the beam again?
0 2 3 4 5 6 7 B

9 10
XI 1. I Y
A B c fo 8.0
cm
W2 -.
water
14 A constant force F in the d irection shown is applied to a block. The block moves
from A to B. AC and BC are construction lines.
Given that the density of water is 1.0 g cm3 , what is the density of the oil?
A 0.80 g cm3 13 0.90 g cm'
c 1.11gcm3 D 1.25 g cm'
18 T he diagram shows a column of dry air enclosed in a narrow capillary tube by a
lhread mercury 12 cm in length.
The work done by F is
A. FxAC B FxBC Trapped dry air
c F xAB D Fx (AC+ BC)
12cm
mercury
15 A body of mass m kg Is released from rest from a point h metres above the
ground. When it reaches a point x metres above the ground, what is its kine tic
energy in joules?
A mgx B mgh
c mg(h-x) D m{h2 -x2) Given that the atmospheric pressure is 76 cm Hg , what is the pressure on the
trapped air?
A 12 cm Hg B 64cm Hg C 76 cm Hg 0 88cm Hg
4E Physics P1 Prenm Exam 2010 Hua Yi Secondary School 4E Physics P1 Prei m El<am 2010 Hua Yi Secondary $dlOOI
8
7
19 Liquid-in-glass themiometers which can measure a wide range of temperature
A "
Particle P is moving downwards.
B Particle Q is momentarily at rest.
have
c Particle R is moving to the right.
D Particle S is moving downwards.
A smaller bulbs. B thin-wallec;l bulbs.
c narrower capillaries. D shorter stems.
24 Circular water waves are produced by a dot vibrator. The wave pattern at a
certain instant is as shown in the diagram below.
20 What are the necessary-conditions for heat to flow from one body to another
when they are in contact?
1 They have different temperatures. ,
2 They have different specific heat capacities.
3 They have different internal energy.
A 1 only B 1 and 2 only If the speed of the water waves is 0.6 m s', what is the time taken for the wave
c 2 and 3 only D 1, 2and3 to travel from A to B?
A 0.050 s B 0.67 s
c 5.0 s D 6.7 s
21 Two metal blocks have their masses in the ratio 1 : 4. When an equal amount of
heat is supplied to each of them, the temperature rise of the two blocks is in the
ratio 4: 1. 25 A sound wave of wavelength of 0.5 m travels at a speed of 320 m s" in air. It
What is lhe ratio of their specific heat capacities? enters a liquid and its wavelength becomes 2.5 m. What is the frequency of the
sound in the liquid?
A 1: 1 ~ B 1:4
c 1: 16 D 16: 1 x A 128 Hz B 320 Hz
c 640 Hz 0 3200 Hz
22 A bullet of mass m hits a target at speed v. If 50% of its kinetic energy has been 26 A ship has a radar system to detect obstacles in air and a sonar system to detect
converted Into the internal energy, what is the rise in temperature of the bullet? obstacles under water-.. Whal waves are used in these two systems?
(Given that specific heat capacity of the bullet= h)
radar system sonar system
A v/2h B v2/2h microwaves
A radio waves
c
"
mv2/2h v2/4h
a microwaves ultrasonic
c ultrasonic llliCfowaves
D infra-red. rays ultra-violet rays
23 The diagram below shows the displacement-distance graph of a.transverse wave
traveling to the right at a certain instant.
displacement~ 27 A man whose eyes are 1.50 m from the ground looks at his reflection in a vertical
s~ plane mirror 2.00 m away. The top and bottom of the mirror are 2.00 m and
0 f-~---"<~_...,.__,,<--_,,'--""-" distance 1.00 m from the ground respectively. W hat distance, in m, below his eyes can the
man see of himMlf?
A 0.25 B 0.50
Which of the following statements ls correct? c 1.00 0 1.50
4E Physiti P1 Pre~m Exam 2010 Hua Yi Secondary School
4E Physics P1 Prelim Exam 2010 Hua Yi Secondary School
10
30 In the diagram shown, P and Qare two insulated uncharged metal spheres In
28 Two rays of light strike a diverging lens, as shown.
M
.
contact with each other. A positively charged object Xis brought near P. Q is
touched momentarily with a finger, and then Xis taken away. What are the
charges on spheres P and Q?
+ ++ f r. 1'
~+
+ x + p Q
. lasulator
losubtor
The distance from the centre of the lens, 0 to F, or to F2 is the focal lenglh of the
sphere P sphereQ
lens.
A zero negative .
Which statement about the rayf after they have passed through the lens is B positive negative r
correct?
c negative negative
A They appear to come from F,. D negative zero
B They appear to come from F2
c They appear to come from 0 .
D They meet at F,.
31 The diagram shows the /V characteristic of the resistors. R, and R2.
29 An eye views an object 0 by reflection in a plane mirror.
Which is the correct ray diagram?
0.3
0.2
O.l
0
The two resistors are connected in series with a power supply. The power
dissipation of R, is 4 W. What is the power dissipation of R2?
A B
A 10W B 6W
c 4W 0 2W
32 In the circuit shown, the lamps are identical.
c D
4E Physics P1PrelmElOlm2010
Hua Yi Secondary School
12
11
"
1 X is a south pole.
Which of lhe following describes lhe brightness of the lamps when K is closed?
' 2
3
The magnetic field at Pis stronger than that ata.
When a compass is put at Q, it points towards the right.
1 brightness of L1 decreases
2 brightness of lz is the same as LJ A 1&2only B 1&3only
3 brightness of l1 is greater than l2 c 2&3only D 1, 2 & 3
A 1 only B 2only
c 1 and 2 only D 2 and 3 only
35 The diagram shows a current loop.
33 A light dependent resistor (LOR) and a 500 O resistor Is connected across a 30 V
source as shown in the diagram_ The resistance of the LOR is 1000 O in the dark
but drops to 100 O In bright lighl What is the voltmeter reading In the dark and in
bright light respectively?
Which of the following combinations correctly represents the directions of
magnetic field at points P and Q respectively?
p a
A into paper into paper
B into paper out of paper
c out of paper into paper
D out of paper out of pacer
in the dark in bright light
A 2V 5V 36 Which of the following statements about the d.c. motor shown below is I are
B 10V 25V correct? ~
c 15 v 15V
D
.______ 25 v 10V
34 The di agram shows lhe magnetic field around an U-shaped magnet. Which of the
followi ng statements must be correct?

1 The motor rotates in the clockwise direction. '
2 Increase the current through the coil can increase the turning speed of
the motor. ,
3 The motor converts mechanical energy to electrical energy.
Hu YI Secondary SctlOOI
4E Physics P1 Preliln Exam 2010 Hoa Yi Secondary School
13 14
39 The figure shows the screen display of a CRO.
A 1 only B 1 &2 only
c 2 &3 only 0 1, 2& 3
37 A light aluminium ring is placed on the top of a iron core solenoid as shown.
Which of the following can keep the light aluminium ring hanging over the
solenoid?
Which of the following statements is/are correct?
)
1 The horizontal voltage Is a d.c.
2 The vertical voltage has an amplitude of 72 mV.
3 The vertical voltage has a frequency of 50 Hz.
A 1 only B 3only
1 connect an a.c. through the solenoid c 1 and 2 only D 2 and 3 only
2 connect a d.c. through the solenoid
3 replace the iron core with a bar magnet
A 1 only B 3 only 40 Brian has made a circuit with three 6 V lamps and two diodes.
c 1&2only D 1,2and3
38 Three identical light bulbs of rating '10 V, 40 W' are connected to a transformer
as shown in the diagram.
1'"' 40 vv
If the bulbs are operating at their rated power and the efficiency of the

He connects a 6 V a.c. supply lo Sand T. Which lamp/swill be lit?


A
c
all three
Yand Zonly
B
D
X and Zonly
Zonly
transformer is 80%, what will be the current in the primary coil?
A 0.6A B 0.75A
c 2.25A D 4A
End of Paper
4E Physics Pl Prelim EJcam 2010 4E Physics Pl Pldm ~ 2010
Hua Yi Secondary School
Secondary 4 Physics Paper 2
2
Section A [SO marks] ' (iii) Use your completed vector diagram to determine t he magnitude of the tension in the
string.
Answer all the questions in this section.
(1)
(a) State the difference between a vector and a scalar. (c) The student decides that she would like as little sag as possible in the string when it Is
loaded with cards. To achieve this, she tightens the string. State, with a reason. whether
-------------------- the string, loaded with cards could ever be horizontal.
----------~----
(1)
(b) In order lo display greeting cards, a student flXes a length of string between two nails 111
an~ then suspends the cards from the string. Fig. 1.1 shows the siring with one card of
weight 0.60 N suspended by a light chp at the centre of the string.
2 (a) Define the moment of a force.
nail
------------ ----------------
- - - - - - - - - [1]
(b) An electricity cable is attached to a pole at a height of 6.0 m above the ground as shown
in Fig. 2.1.
2llON
Fi g. 1.1
{I) On Fig. 1.1, mark the forces on the clip due to the lension in the siring. [1]
(ii) The resultant of the forces due to the tension in the string is 0.60 N. In the space
below, draw a vector diagram for the forces in the string and their resultant. Use a
scale of 1.0 cm to represent 0.10 N.
Fig. 2.1
The cable exerts a force of 280 Non the pole at an angle of 90 to the pole. To ensure
that there is zero turning moment on the pole itself, a wire under tension is attached to
the pole at a height of 4.0 m and it makes an angle of 35 to the pole.
Calculate
(i) the moment which the cable exerts about P, a point in the pole level with the
ground,
[2]
4E Phy>ics P2 Prelm Eqm 2010
4E Physics P2 Prrim Ellatn 2010
4
3
------- [2]
[2] (c) It Is found that the calculated value In (a) Is not equal to the observed reading In the
(ii) the tension necessary In the w ire. Gauge. Suggest a reason.
[1)
4 Fig. 4.1 shows the structure of a refrigerator. The coolant in a refrigerator transfers energy
[2] from the Inside to the outside. It evaporates as it passes through the pipes inside the freezer
compartment
On the outside of the refrigerator, the vapour is compressed back into a liquid which then
cools as ii passes through the pipes mounted at the back of the refrigerator.
Fig. 3.1 shows a syringe and a Bourdon Gauge used to investigate the relationship between
the. pres.sure an1 the. volume of a gas at constant temperature. The initial reading of the
syringe is 10 cm while that of the Bourdon Gauge Is 2 x 103 Pa. -
freezer
compartment
11-1---+- pipes
a1 back of
!!i 10 1S 20 25 refrigerator
Fig. 3.1
(a) If the syringe reading is reduced to 5 cm3 when a force is applied to the piston what is Fig. 4.1
the corresponding reading in the Bourdon Gauge? '
(a) Explain how the freezer compartment is cooled.
------------
[3)
[2)
(b) The pipes at the back of the refrigerator are coloured black. Explain now this helps to
(b) Use the kinetic theory of matter to account for the result.obtained in (a). cool the warm liquid that passes through them.
---------- - ------------------
[1)
4E Physics P2 Pretim Exam 2010 Hua Yi Seooodary S<hool
IE Physics P2 Prelim Exam 2010 Hua Yi Secondary School
2
Section A [50 marks] (iii) Use your completed vector diagram to determine the magnitude of the tension in the
string.
Answer all the questions in this section.
(1 J
(a) State the difference between a vector and a scalar. (c) The student decides that she would like as little sag as possible in the string when it is
loaded with cards. To achieve this, she tightens the string. State, with a reason, whether
the string, loaded with cards could ever be horizontal.
(1 )
(b) In order to display greeting cards, a student fixes a length of string between two nails [1]
an~ then suspends the cards from the string. Fig. 1.1 shows the string with one card of
weight 0.60 N suspended by a light clip at the centre of the string.
2 (a) Define the moment of a force.
--------------- (1)
(b) An electricity cable is attached to a pole at a height of 6.0 m above the ground as shown
in Fig. 2.1.
2l!ON
ceble
Fig. 1.1
(I) On Fig. 1.1, mark the forces on the clip due to the tension in the string. [1]
(Ii) The resultant of the forces due lo the tension in the string is 0.60 N. In the space
below, draw a vector diagram for the forces in the string and their resultant. Use a
scale of 1.0 cm lo represent 0.10 N.
Fig. 2.1
The cable exerts a force of 280 Non the pole at an angle of 90 to the pole. To ensure
that there is zero turning moment on the pole itself, a wire under tension is attached to
the pole al a height of 4.0 m and it makes an angle of 35 lo the pole.
Calculate
(i) the moment which the cable exerts about P, a point in the pole level with th'e
ground,
[2]
E Ph~ics P2 Protin Exan> 2010
4E Physics P2 Prelim Exan> 2010 Hua Yi S<!OllldafY School
4
3
[2]
[2) (c) It is found that the calculated value in (a) is not equal to the observed reading in the
(II) the tension necessary in the wire. Gauge. Suggest a reason.
(1)
4 Fig. 4.1 shows the structure of a refrigerator. The coolant in a refrigerator transfers energy
[2) from the inside to the outside. It evaporates as it passes through the pipes inside the freezer
compartment.
On the outside of the refrigerator, the vapour is c6mpressed back into a liquid which then
cools as it passes through the pipes mounted at the back of the refrigerator.
Fig. 3.1 shows a syringe and a Bourdon Gauge used to investigate the relationship between
the pressure an1 the volume of a gas at c9nstant temperature. The initial reading of the
syringe ls 10 cm while that of the Bourdon Gauge is 2 x 103 Pa.
lriii~~!!!~~~J_ freezer
comparimenc
pipes
arback of
s 10 15 20 25 refriger.l!Or
Fig. 3.1
(a) If the syringe r~ading is_ reduced to 5 cm3 when a force is applied to the piston, what is Fig. 4.1
the corresponding reading in the Bourdon Gauge?
(a) Explain how the freezer compartment is cooled.
------------------ -------------------
(3)
[2)
(b) The pipes at the back of the refrigerator are coloured black. Explain how this helps lo
(b) Use the kinetic theory of matter to account for the result _obtained In (a). cool the warm liquid that passes through them.
----------------~~-- - - - - -- - - - - - [1)
4E Physics P2 Prelm Elalm 2010
5 6
(c} Explain how movement of air at the back of the refrigerator also cools the warm liquid. Air is optically less dense than Perspex, and Perspex is optically less dense than rocket fuel.
(a} What happens to the speed of the light as it passes through the Perspex bar
(i) into the air, and (ii} into the rocket fuel?
(2]
- [1]
{d} The manufacturers recommend thal a minimum 5 cm gap be left between the pipes at
the back of the refrigerator and the wall of the room. Explain why this Is necessary and (b) What Is meant by 'critical angle' of a medium?
suggest what is likely to happen if a smaller gap is used.
[2)
[1)
(c} Complete Fig. 5.1, showing the paths of the light rays when the tank is (1} Full;
5 Fig. 5.1 shows a design for an 'optical fuel gauge' suitable for use In a rocket fuel tank. When (2) Empty; label the rays as (1) and (2) respectively. (2)
the tank runs low, the light-activated switch Is triggered.
(d) Give an explanation to your answers in part (c).
Light source producing parallel
50urce of light
Light Wires to guidance
beam vstem
[2]
Ir"'<-- - - Light-activated
switch - - - ----------------
Perspex ______._
bar 6 (a) Fig. 6.1 shows an object 00' placed in front of a converging lens. The image is formed
on the line XY x
0
~
Fuel tank Fuel tank t
o
f
y
Fig. 6.1
Fig. 5.1
(i) Draw In Fig. 6.1, construction rays from 0 to locate its image, I . Show clearly tho
position of the image. [2]
E ~ P2 Prelm Exam 2010 Hua Yi Seaindary Sdlool 4E Pllysics P2 Prelm El<a<n 2010
10
9
Section B [30 marks]
9 A beam of positively charged protons is projected between the poles of an electromagnet and
a bright spot is produced on a screen. The apparatus is enclosed in a vacuum and arranged
Answer all the questions in this section. Question 12 has a choice of parts to answer.
as shown in Fig. 9.1.
10 Table 10.1 compares the performance of the microwave oven and convention oven when
'cooking a chicken. The convention oven is first warmed up before putting the chicken in.
Table 10 1
microwave conventional
time to warm up I hour 0 0.5
D.C. power input power whilst warming up I kWh 0 1-2
sUpplY
energy to warm up /kWh 0
Fig. 9.1
input power whilst cooking I kW 1.5 0.6 (average)
(a) When the current Is switched on, time to cook chicken I hours 0.4 2.0
(i) what magnetic pole is produced at the end marked X on the above diagram? energy used whilst cooking I kWh
-------------------------- (1) total energy used I kWh
(ii) In which direction will the spot on the screen be deflected?
(a) Complete Table 10.1 to find the total energy used by each oven to cook a chicken. [2]
------ (1)
(b) Given that the energy absorbed by each chicken during the cooking process is 0.2 kWh.
Find the efficiencies of tho ovens.
(b) State one way in which the spot on the screen can be made to
(i) deflect more,
--- [1]
(ii) deflect in the opposite direction.
(1) (3)
(c) Stale two changes to the observation made in (b) if a beam of electrons is used in the (c) Compare the different cooking methods by the two ovens and account for the lower
experiment Instead of a beam of protons. energy used by the microwave oven whilst cooking.
--- ..------------------------------------ -----------------
- - - - - - - - - - - - - - - - - - - -- - - - - - -- - - - - - - - - - (2) ---------- -----------
------ (3)
4E Physics P2 Prelim E><am 2010 Hua Y1 Secondaiy School
4E Physa P2 PreUm Exam 2010 Hua Y1 Secondaty School
12
11
(c) The secondary coil Is removed and a small coil connected lo a low voltage lamp I~
(d) The shiny surface inside the microwave oven can help to reduce energy loss. Why? placed as shown in Fig. 11.2.
(11
(e) How Is the energy loss from the conventional oven minimized?
[1)
Fig.11.2
Fig. 11 .1 represents a transformer with a primary coil of 400 turns and a secondary coll of 200
turns. Explain the following observations:
(i) the lamp lights,
Resirt-c
(2)
(ii) if the coil is moved upwards, the lamp gets dimmer,
Fig.11.1
- - - - - -- - -- - - - - - - - - -- - - - - - - ------ [2)
(a) Calculate the efficiency of the transformer if the primary current is 3 A and the (iii) if an iron rod is placed through the coil, the lamp brightens again,
secondary current is 5 A.
{2)
[2)
(b) Give two reasons why the efficiency of the transformer is always less than 100%.
---------------
{2)
E Physics P2 Prelm Exam 2010
4E Physlca P2 - Exam 2010
13 14
12 Either (iii) Explain one way, other than laking the average of a number of readings, by which
(a) Fig. 12.1 illustrates an experiment to measure g, the acceleration of free fall. A ball is the reliability of the experiment could be improved.
dropped which breaks a light beam falling on a photodiode. The clock is started when
lhe ball stops the light shining on the photodiode. Vertically bolow the first light beam a
second light beam shines on another photodiode. When the ball continuing in its fall [1)
breaks the beam, the clock is stopped.
Qsau
: Photodlode (b) In an experiment to measure the acceleration of free fall, a mass of 1 kg was attached to
a piece of tape. The mass was allowed to fall freely and the tape passed through a
Lamp
I ticker timer which made 50 dots on the tape ovory second. The tape {not drawn to
I
I scale) is shown in Fig. 12.2.
I Measurements taken from the tape showed that the velocity at X was 25 cm/s and the
I
I velocity al Y was 125 cm/s.
I
tJ Clock x y
~'-----
I
I
I
I _
i . __
_! _ }
I
Lamp 6I
Photodiode
Fig. 12.2
(i) Calculate the acceleration of the 1 kg mass between X and Y.
Fig . 12.1
(I) Mark on the diagram the distance s covered by the ball. [1)
(ii) Show that I, (the time taken to travel a distance s) and s are related by the equation
s = ~ gt2
[2)
(ii) What would the result be if the experiment were repeated with a 2 kg mass on the
end of the tape. Give an explanation to your answer.
[2)
[2)
Ir s Is 5 cm and I is 0.1 s, calculate the value of g.
(Ii i) Stale one precaution you would take to ensure an accurate result.
------------------------------
- ----------------- - - - - - - - - ----------- - - - - - [1] .
(1)
4E Physics P2 Prellm E><am 2010 Hua Yi Secondary Scnool 4E Ptiyslcs P2 Prelim Exam 2010 Hua Yi Secondary School
16
15
{Ill) Another lamp L2 , also designed to run from a 9.0 V supply, has a resistance always
Or double that of lamp'L1 at all voltages.
(a) A filament lamp L, is designed to run from a 9.0 V supply. An experiment Is carried out On the same axes as in (i), draw a second graph for lamp L2. Label the new graph
to measure the current through the lamp as a variable potential difference is applied as L2 [1)
across it. The results are shown In the Table 12.1.
(Iv) State and explain whether lamp L 1 or lamp L2 is brighter at a potential difference of
Table 12.1 9 .0V.
.d. IV 0 1.0 2.0 3.0 4.0 5.0 6.0 8.0 10.0
Current/ A 0 1.0 1.4 1.7 1.9 2.2 2.4 2.B 3.2 ----------------------------------- --
------------ [2]
(i) Plot a graph of current against potential difference for lamp L 1
(b) Fig.1 2.3 shows part of the wiring for the rear lights of a car. Each rear light contains two
lamps. The side-lights are switched on when the front lights are switched on. The brake
lights are switched on when the car brakes.
Fig. 12.3
(i) Explain why the lamps are connected in parallel rather than in series.
------------------------
[2] [1]
(ii) Given that a rear side-light lamp is rated 12 V, 6 W. A brake lamp is rated at 12 V.
24W.
(ii) Calcula\e the resistance of the filament of lamp L, when p.d. across it is 9.0 v. How much current is drawn from the battery when
(A) both.brakes lights only are on?
(B) both rear side-lights and both brake lights are on?
[3] '
(1]
End of Paper
4E Physlco P2 Prollm Ex.am 2010 Hua Yi S..condary Soho~
IE Ph)'Slcs P2 Pr-elim Exam 2010 Hua YI S<!condary School
HUA YI SECONDARY SCHOOL {b) (iii) 0.43 N
PRELIMINARY EXAMINATION 2010
le) As the weight of the card i s acting downwardly, there must be a vertical 1
SECONDARY FOUR EXPRESS
component of the tension to compensate for it. When the string is horizontal,
Physics it cannot have a vertical component.
ANSWER SCHEME [Total : 6 marksf
r:' ~ >$~,,....~~._:~~... ~ :'>s.~~'\~::,~::t> ;~ ~--vr;~~:;~~~~~~, _ .. \. - . ~-
Section A C40 Marks ) 2 (a) Moment of the force is defined as the product of a force and its perpen'dicular 1
distance from the pivot.
1 8 11 A 21 A 31 B
(b) (i) Moment exerted by the cable about P 1
2 D 12 D 22 D 32 D
= 280 N x6.0 m
3 8 13 c 23 D 33 B = 1680 Nm
{ii) sin 35 =NP/ 4.0 2
4 c 14 A 24 A 34 c NP =2.29 m
5 D 15 c 25 c 35 c
Clockwise moment about P = Anti-clockwise moment about P
6 c 16 B 26 B 36 B 1680 = T x 2.29 .
7 8 17 A 27 c 37 A =
T 734 N
!Total: 4 marksl
8 D 18 B 28 A 38 B
~{~...i~~~~~~~f,~-!-;!l~:,GfJ1t..~;i~~f\.~~~--~~~,i?:;~.~Jt~~~;r!r~.~_;;!~~~~~~~~~J,r.j~-:"'(l'~.;~.'j'~~r!r$~tt.. 7-t!!WN?-
9 D 19 A 29 c 39 A 3 (a} P,V,:: P.V2 2
2x10')( 10= P2x5
10 A 20 A 30 c 40 A
P2 '"4 x 10' Pa
{b ) When the volume of the gas in the s yringe decreases, number of gas particles 2
Section B ( 50 Marks) per unit volume increases. This increases the frequency of collision between
the air particles and the wait of the syringe. Therefore, there Is an increase in
auostion Marks I the pressure of the gas.
No.
1 (a) Vector is a quantity with magnitude and direction, but scalar is a quantity with (c) Temperature of the gas in the syringe Is not constant.
(b)
ma nitude onl .
(i)
. [Total: 5 marksl
~=tt:~~1~~j~~:~0!_~;~:-~~~-"';;'~!~::.:f .<.t.:~IJf;,~::.{-fri~..... ;"t:::::-:it .'.'.f;;:!:r-~-;~-~~~~~t~~~~;~~_i~~-~r
4 (a) When the coolant in the pipe passes the freezer compartment, ii lakes in heat. 3
As the coolant has a low boiling l)Oint, it evaporates quickly and turns into
vapour state. This helps to remove heat from the freezer compartment thus
causes the temperature of the freezer to drop. f
(b) Black ,.urtace is a good emitter of infra-red radiation. This helps to cool the 1
warm liquid passes through it.
(b) (II) (c) As t he warmer air Is less dense than the cooler air, it rises. The cooler air 2
sinks to take its place. Convection current is set up to cool the warm liquid in
the oioe.
(d) The gap Is necessary to allow free movoment of air. A smaller gap would 2
reduce the airflow and less heat would be removed. This causes the coolant
not able to condense into liqui d state effectively and this will affect the
2
efficiency of the refrigerator.
rrotal: 8 marksl
4E Phyoics Malit ~ Mill-Year Exam 10 Hua Yi Secoodaiy School
:a)
. . ~ . -
(i) the light ray speeds up, (ii ) the light ray slows down.
" - - (b) The image becomes dimmer because less light passes through the lens.
[Total : 5 marks}_]
I 2
. , tiiii!'"~ , ~ ~ . ~~~~~ir.~~r;;~:,,.,,,~
Critical angle is the angle of incidence In the more dense medium where the "7 '(a) High frequency sound wave is used as it is able to penetrate a mediu m better 1
angle of refraction in the less dense medium is 90.
than a normal sound wave.
[c)
Light source i:iroducing pn-eUel
2 (b) Speed of sound in water = 2s It 2
SOUtoe of light 1500 = 2s/(1 /15)
s== 50m
Light
b<am (c) .sound travels faster in water than in air because molecules in water are more 2
U-C-1--- Ugtltar:t.ivated
tightly packed, therefore the compressions and rarefacti ons of a sound wave
switch move about more quickly In water.
(d) to determine the depth of water Any
use to study: the layers of sediments below the seafloor one
1
2 to search for objects such as wrecks, submarines and .Icebergs etc
[Total: 5 marks]
~ Fuel1ank Fu~I tank. ----1 ;4i WJ~~- "l"'"f-;,,~~~~" - :. .. . 'ir..w,..s.,.... ". ~ ii<>' , '""lzi>q';'.&.;it1:f''
8 (a) When the switch is closed, magnetic field is set up in the coil with S pole at 2
the top. The Iron bar above becomes temporarily magnetized with an i nduced
north pole at the bottom. The two unlike poles attract and hence the direction
of motion of the signal.
(b) The iron bar is magnetized by the magnetic field of the coil in the opposite
direction from that In (a). Attraction between unlike poles still takes place.
When the fuel tank is full, the light ray is traveling from a less dense medium 2 [Total : 3 marks]
to a denser medium, therefore, refraction of light takes place, and the light- ..:~!fi~~4~i"J!f!lii*-~~-t~~tlfif~;~~~~r;.F.; ~1r~i~~
activated switch ls not triggered.
9 (a) (i) N pole
(ii) Downward direction
When the fuel tank is empty, the light ray is traveling from a more dense
(b) (i) Increase the number of turns of coil on the soft iron core, increase the
medium to a less dense medium, therefore, total Internal reflection of light
voltage of the d.c. supply
takes place, thus trigger the l ight-activated switch.
(ii) Reverse the terminals of the d.c supply
(c) (i) bigger deflection (ii) deflection direction Is oppos ite to that of proton beam 2
x [Total : 6 marks]
y
(ii) Magnification = image size I object size = 1.5 cm / 1.2 cm = 1.25
4E Physics M"'1< S<hemo Md-Ye E"""' 10 H~ Yi Secondary School
4E PhysK:s Mar1< Schem e Mid-Year E xam 10 Hua YI Secondary School
(c) (i) Some of the magnetic flux around the primary coil passes through the 2
secondary coil. The changing Magnetic flux linking the coil results in an e.m.f.
Section C I Total 30 Marks]
10 (a)
. induced in the secondary coil. This produces a current in the coil and lights up
the lamp.
(ii) When the coil Is moved further away from the primary coil, the flux passing 2
microwave conventional
through it decreases. The rate of change of flux is less and the magnitude of the
induced e.m.f. is reduced. Therefore the lamps gets dimmer.
time to warm up I hour 0 0.5
(iii) The iron rod can concentrate the magnetic flux around it, this increases the 2
input power whilst warming up I kWh 0 1.2 magnetic flux passing the secondary coil, therefore, the induced e.m.f. is
increased. And the lamp becomes brighter.
energy to warm up I kWh 0 0.6
ITotal: 10 marksl
input power whilst cooking I kW 1.5 0 .6 (average) ~ft.~ --(i) .,~&\~~'f>~.tjj/1{~~~;;~~1l-l'l""1.'!;!J!.iif~~ ",.,.,.;;: -~~~~~..~ >
-
time to cook chicken I hours 0.4 2.0
12
E
(a) Qsan 1
energy used whilst cooking I kWh 0.6 1.2 ~
lamp
,
I
I
I

Ph<ltodiode
~
total energy used I kWh 0.6 1.8 I
2 I
I Start
I
I
(b) Efficiency of microwave oven - 0.2 / 0.6 x 100% = 33.3% t
I
s
/
=:J Oock
I
Stop
l
I
Efficiency of conventional oven= 0.211.8 x 100% = 11.1% 3 I
I
I
(c) When mlrcowave oven is used, the microwave pass deep into the food, some of I
thei r energy Is absorbed by the water molecules and the temperature rises, ~ I 0
lamp I Photodiode
warming the rest of the food. This process is fast.
A conventional oven cooks food using the infra-red radiation. The food ls
6
cook ed from the outside and the heat is transmitted to the inner layers through (ii) distance travelled s = average speed x ti me 2
conduction. This process is slow. 3 = (v +'\i) 12 x t
= (u + gt) /2 x t ( As v "" u + gt }
(d) The shiny surface is a good reflector of infra-red radiation thus minimised the = y. gt2 (As u =O)
heat loss through radiation. 1
s = % gi'
51100 =y, g (0.1)2
(e) The wall of the conventional oven has a air gap In between. It helps to reduce
heat loss through conduction as air is a poor conductor of heat. 1 g = 2 x (51100) + (0.1)2
= 10 rnls2
1
rrotal: 10 marks (Iii) Increases. This will Increase the accuracy of the measurement of s and the 1
=,~~ ~1~~~. ~~ir~d~t;~'.;:~m:nclf.7tE~~---~1~,;~~~-~ ~ .-.... -t~~t~~'"t"f--l~i - ~t._..~ .~~~~ t.f::._~~~ time interval t.
11 (a) VsNp = Ns/Np = 200/400 :: % 2
Efficiency of the transformer= Vsls I Vplp x 100 %= (% x5/3)x100%= 83 % (b) (i) a= v- u It = (125- 25) I (0.02x5):1000 cm/s' = 10 mis' 2
Any (ii) The same because all masses fall with the same acceleration. 2
(b) (I) Heat Is produced in the wires
(ii) Eddy current flowing in the iron core produce heat two
(Iii) Magnetic leakage, not all the magnetic flux on the primary coil reaches (iii) Increases, this w ill increase the accuracy of the measurement of sand the 1
the secondary coil 2 time interval t.
(iv) The work done in continually magnetising and demagnetising the Iron
core ITotal: 10 marksl
Hua Yi Secondary School
4E Ph~cs Mall< Sclleme Mid Year E""m 10 Hua YI Secondary School
!al (i) I/A
Iii) Res istance of L 1 =Vil = =
9.0 I 3.0 3.0 n
llill As shown In the diagram
=
(iv) Since P V' I R ,
and resistance of L2 Is double that of L,,
Power of L 2 is lower than L1
Therefore L1 is briahte r than L,. 2
lbl Ii) So that when one lamo f u se the others still can work.
(ii) (A) When both brake ligh t are on
Current through each brake light :
I= f = 24 =2A (1)
v 12
Current drawn from the battery when 2 brake lights are on
= 2A x 2 = 4A (112)
(B) When both rear side lig hts and both brake lights are on
Current through each rear light:
I= f = =0.5A (1)
v 12
Cu rrent drawn frO!'fl the battery when 2 brake lights and 2 rear lights are on
=4+1 ~ = (1/2) 3
rrotal: 10 marksl
E Physics MOtl< Scheme Mld-Y. .r Exam 10 Hua YI Secondary School
41': Physics Mark Scheme Mid Year Exam 1<1 Hua YI Se<:<>ndaoy Sct>ool
Hwa Chong International School
Secondary 4 Physics, Paper 1
1. Fig.1 shows a micrometer reading when an object was gripped between
the anvil and the spindle. If the instrument has a zero error of -0.02 mm
'
4. A space ship Is fired vertically. The velocity-time graph of the motion of the
spacecraft is shown in the diagram. The maximum height reached by the
what is the actual diameter of the object in mm? '
spacecraft is
25
20
15
10
0
10 20 T
I<:....--'----;>,.---=,---~ Tlmel h
Fig. 1
A. 10.65 A. 25 000 km
8. 10.69 e . 50000 km
c. 11.1 5 c. 75 000 km
D. 11 .19 D. 100 000 km
2. Which of the following is correct? 5. Two forces, 1 N. and 11 N, act on an object. Which of the following is not the
Scalar quantities Vector quantities possible resultant force on the object?
A power, displacement, moment work, energy, current .R.
B work, time, gravity force, displacement, moment .S!. A. 9N
C mass, power, time acceleration, weight, pressure B, 10 N
D speed, pressure, density weight, velocity, electric fi~ld c, 11 N
D. 12 N
3. Which graph shows an object, initially at rest, accelerates, and then attains
6. A horseshoe-shape is cut from a uniformly thick piece of cardboard as shown.
uniform speed?
Which letter is at the centre of mass of the piece of cardboard?
Displacement/m Displacement/m
A.
Time /s Time/s
Displacement/m Displacement/m
D
B.
Time/s Time /s
2
7. A net force is applied on a body with its variation with time as shown in the
figure below. Which of the following velocity-time graphs correctly describes 9. A uniform plank of weight 60 N is 2000 mm long and rests on a support that is
the motion of the body? 600 mm from end E.
600mm
20oQ!jm . .. :: ~
- . . .- -.
At what distance from E must a 160 N weight be placed in order to balance the
plank?
A c A. 150mm
B. 225mm
V~elocily_. C.
D.
375 mm
450 mm
O me 10.A crane lifts a load of 2000 N through a vertical distance of 30 m and moves
31 50 m across the warehouse in 50 s. What is the power developed by the
crane in lifting the load against the gravity?
B D A. t .2 kW
a. 3.2 kW
,r:-.. ~
~
c. 300 kW
D. 800 kW
8. A pta.nk of wood of negligible mass rests on a support and a weighing
machine. A man starts to walk from the support towards the weighing machine
as shown in the diagram below. X represents the distance travelled by the
man.
Additional
pressure pressure
> ) > > > > ., ; ;o )I :; l
Which of the following graphs shows the variation of the reading of the h- depth
w_ei(Jhing ma.chine with X?.
A h depth c
Additional
~
Additional
pressure pressure
o "x
h depth h depth
A B c D B D

3
12. The height of a mercury barometer is h when the atmospheric
pressure is 100 000 Pa. 15.A piece of ordinary aluminiu~ foil is used to wrap around food to be cooked in
a BBQ. The foil has a shiny side and a dull side. Which side of the foil should
be on the outside in contact with the hot charcoal? Why?
What is the pressure at X?
A 20 000 Pa A. The dull side. Since it is a better thermal conductor
B. 80 000 Pa B The dull side. Since it absorbs thermal radiation faster.
C 120 000 Pa c. The shiny side. Since it absorbs the thermal radiation faster.
0 . 180 000 Pa D. The dull side. Since it encourage the set up of convection current.
16.An airgun pellet, mass m and specific heat capacity c, hits a steel plate at
13. The figure below shows a hydraulic press made up of two circular platform at X speed v. During the impact, 50% of the pellet's kinetic energy is converted to
and Y. The diameter of piston X is 2 cm and that of piston V iSllcm. A block of thermal energy in the pellet. What is the rise in temperature of the pellet?
mass 64 kg is placed on piston Y . If the efficiency of the machine is 80 %, find A.
v'
the effort required to raise the block? effort 64 k 4c
A. 5 N
I g
B.
v'
y 2c
B. 32 N 1
mv
C. 40 N c.
D. 50 N 4c
mv'
D.
2c
17. What is the total thermal energy needed to melt 2 kg of ice at -5 C
comple.tely? Take specific latent heat of fusion of ice= 334 kJ I kg and specific
14.A slice of bread is placed under a red hot electric grill to make toast. How does heat capacity of ice = 2.05 kJ I (kg C) .
the thermal energy reach the bread?
A. 20.5 kJ
B. 87.3 kJ
c. 668 kJ
D. 689 kJ
A by conduction and radiation
I:!. by conduction and convection
C. by co~vection and radiation
0. by radiation only
5
18.A heater connected in series to two similar batteries has two identical heating
elements as shown, each with a resistance value of R. When the position of 21. When a substance, or boiling point 50 C, is cooled from BO C to 35 C,
the switch is at X, the temperature rise of the water in the beaker is t c per
second. Neglecting thermal energy lost to the surroundings, what is the
temperature rise, in 'C, or the water in one second when the switch is placed i. Thermal energy due to latent heat of fusion is given out by the
atY? substance
ii. Thermal energy due to latent heat of vaporization is absorbed by
the substarice
iii. The purity of the substance depends on whether at 50 c, the
change of state is finally completed before it further registers a
drop in temperature
iv. the average kinetic energy of the molecules of the substance
decrease and move more slowly
A (i) and (iv) only
a {iii) and (iv) only
c (ii) and (iv) only
D (ii) and (iii). only
A
2 22. Which of the following EM waves is used commonly to sterilize medica_I
0. instrument?
4
C. I A. Gamma rays
D. 21 B. Microwaves
C. X rays
19. The mass of a solid material X is 2.5 kg. Assuming the rate of thermal energy D Ultra-violet rays
transferred from melting the solid material X is 2 000 W and the time taken to
melt the en tire solid is 2.0 minutes, what is the specific latent heat of fusion for
material X?
23.A sound wave is displayed on the screen of a cathode-ray oscilloscope. The
A 1 600 J kg1 time base of the c.r.o. is set at 2.5 ms I cm. What is the frequency of the sound
B. 10 000 J kg1 wave?
C. 20 000 J kg"1
n. 96 ooo J kg1
20. In an undergrouno mine, pure water is found to be boiling at 110 c. What can I \ I \
be deduced from this statement? I . ' I
i. During the boiling process, evaporation stops I J \ I
ii.
iii.
During the boiling process, evaporation still occurs
The atmospheric pressure of the underground mine is higher than
\ J I
\ --
the normal standard atmospheric pressure on the ground surface
iv. The boiling point will be higher as one goes deeper into the
1cm
underground mine
A. 50 Hz
A. (ii) and (iii) B 100 Hz
B (ii). {iii) and {iv) C. 200 Hz
C. (i) , (iii) and {iv) D. 400 Hz
D. {i) and (iv)
7
8
24.A band member plays a note on a trumpet. He then plays a softer note of 28. The same progressive wave is represented by 2 graphs: displacement-time
higher pitch. How do the amplitude and frequency of the sound compare with and displacement-position graphs. Which of the following gives the speed of
the first? the wave?
Amplitude of second Frequency of second
displacement displacement
sound sound
A. smaller higher
B-.; smaller '
/ lower
c. larger higher
0. larger lower
time position
25.Lip Meng, 180 cm tall looks at his image in a plane mirror mounted on a
vertical wall. Althoug h the mirror is only 90 cm tall, he can j ust see the whole of
his image. He now moves back so as lo double his distance from the mirror.
He will now be able lo see
A. the whole of the image as before with no space above or below. A. pq
B. the whole of the image but with spare space al the top.
C. the whole of the image but with spare space at the bottom. B. .!!..
D the whole of the image but with spare space at top and bottom. q
c. !!...
p
26.A dipper vibrating up and down produces waves in a ripple tank. What will t
happen if the frequency of the dipper is increased? D.
pq
A. The waves will move slower across the tank.
29.The diagram shows a girl watching ripples on a pond. She counts 18 ripples hit
B. The wave fronts will be further apart.
c. The wave fronts -will be closer the shore in 1Os. The wave peaks are 12 cm _a part
o The waves will move faster across the tank. "
27 .A beam of light travel from the left to the right as shown, passing through 3
different media of unknown refractive indices, 71. Rank the refractive index of
the 3 media in ascending order.
3
A. ,.,, , ,.,,. 11, ~
a. 'h I 7li1 T/) r
c. 17., '11 ,.,,
D. 11 17,, 1/, 11,
Which of the following is correct?
Wavelength Frequency
A. 1.8Hz 12crn
a. 12 cm 1.8 Hz ,,
c. 12 cm 18 Hz
D. 18 Hz 12cm
9 10
30.When a plastic rod is rubbed with wool, the wool acquires a positive charge wn in the diagram using
33. Three identical resistors are connected as Sho
because connecting wires of negligible resistance.
A. electrons are transferred from the wool to the rod. y
9. protons are transferred from the wool to the rod.
C. electrons are trans{erred from the rod to the wool.
o. protons are transferred from the' rod to the wool.
Whal is the effective resistance between the points X and Y?
31.Three metal spheres X, Y and Z are placed on three stands so that they are in
contact as shown. A positively charged rod is brought near X. The sphere Y is A. 0 0
then earthed while the charged rod is still in position. B. 3 0
c. 60
0 90
34. The v against 1 characteristic graphs for two resistors A and B are shown in
the diagram below.
V/V
4
Which of the following statements is/are true?
3
(1) Xis negatively charged.
(2) Y is neutral.
2
(3) Z is positively charged.
A. (1) only
B. (3) only
c (1} and (2) only
D. (1) and (3) only
0.1 0.2 0.3 0.4 I/A
32.A student uses a 3 kW electrical oven for 4 hours to bake a cake and a 0.8 kW
kettle lo boil water for 30 minutes. How much the student has to pay for using
electricity if one kVl'h of electricity costs 20 cents?
A. $ 1.24 If resistors A and B were connected in parallel across a 2.0 V cell, what would
B $ 2.48 the current flowing through A and B be?
c. $ 7.20 A B
0. $ 48.0 A. 0.1 A 0.05 A'
B. 0.4 A 0.8 A
c. 0.6 A 0.4 A
D. O.BA 1.6A
11 12
35. A 3-pin plug is fitted to the lead of a 1 kW electric kettle to be used on a 250 V
a.c. supply. Which of the following statements is not correct?

37.Two metal rods are placed above copper plates next to one another as shown.
What observation would you expect if an a.c. source is used as shown?
A. A 4-A fuse is the most appropriate value to use
.,~~
B. The brow rt wire should be connected to the Jive side of the mains
c .l'.he fuse should be.fitted in the neutral lead
D. The neulra/wire is coloured blue
36. The diagram below shows how an electric bell work I I
A They will repel and attract in a periodic manner
13. Theywill be repelled
C. They will be attracted
D. Nothing happens
38. The diagram below shows an a.c. generator. If the coil starts from a vertical
position which diagram correctly represents the change of the e.m.f. with time?
4 metal rods are placed, each at a time, in a coil of wire. The table below gives
the number of paper clips picked up by the rod when placed in the coil.
Which rod would be the most suitable to use for the core in a coil of an electric
bell?
.,.,..
metal number of paper clips picked number of paper clips still ..
:'_:~ ~ '
rod up when there is a current in attracted when the current is
the coil switched off
A. 1 0
B. 20 2
c 35 0
D. 35 30
A .c
B D
13
14
39.A 250 V output transformer is supplying electrical power to operate a motor at
its rated values shown below. Given that the total resistance of the
transmission cables is 2 0, calculate the power output of the transfomer.
200V
transforrner 5kW
motor
A. 625 W
@ 1250W
c. 5000W
D. 6250W
40. The primary coil of a transformer is connected to an alternating voltage supply.
The secondary coil is connected t cross a variable resistor. Which change will
cause an increase in the potential difference across the secondary coil,
assuming 100%. efficiency?
primary .. ~-... secondafy
'.h : l ;. j'_
A. decreasing the cross-sectional area of the secondary coil
(9) decreasing the current in the primary coil
~. decreasing the number of turns of the primary coil
D. decreasing the resistance of the variable resistor
-End Of Paper-
15
Hwa Chong International School
Secondary 4 Physics, Paper 2
SECTION A (50 MARKS)
"
2. Fig. 2 shows an axle A, of radius 14 cm, fixed to a wheel W of radius 70
Answer all the questions in this section. cm, so that they rotate together, mounted on bearings which are not
shown.
1. The velocity-time graph of a moving object of mass 4.0 kg is shown
s
below. wheel W
Velocity/ mis axle A
10 .... ,.................................
''' '' ''
'' 20kg
'' ''
40 I' ''
--- -~-- ~ -
.. ,.,..,.,..,.,.,..,. .. ,. - -
Fig. 2
30 --------.. -----1----------------
'
A 20 kg mass hangs on a rope wound round the axle. A person is pulling
the string S, which is wound round the wheel.
JO 20 30 Time/s a. State what is meant by the principle of moment (1]
a . During the first 30 s, what is the displacement of the object from the
~~ ~
b. Calculate the force, f acting on the string S which produces an equal
and opposite moment to balance the gravity pull on the 20kg mass. [2]
b. Based on the information from the above graph, sketch its
corresponding net force-time graph on the axes given below (3]
Net force/ N
c. State and explain how you wou ld make the person's job easier in terms
of pulling lhe string and lifting the 20 kg load, [2]
Tim e/ s
2
d. By applying the method as discussed in (c), do you agree that the work
done by the person is smaller? Explain [2] b. State the focal length of the lens [1]
c. Hence complete the path for ray Z. (1)
d. With the lens placed at the distance as measured and shown on the
diagram, describe the changes in the nature of the image obtain if the
object is placed just 8.0 cm away from the lens [2]
e. The object was at rest wherf the string suddenly snapped. Calculate its
velocity when it had fallen through a distance of 5.0 m. [2)
4. A. B, C and D in the figure represent particles in a medium through which
waves are passing continuously in the direction indicated by the arrow.
A B c D

Describe the motion of the particles A , B, C and 0 when the wave [2]
3. An object, 0 is placed in front of a Jens of diameter 10 cm such that a a. is longitudinal In nature, and having an amp!itude.of 3 cm
sharp Image is formed on the wall
wall
0
b. Is transverse in nature, and having a frequency of 1 Hz.
a. Complete the ray diagram and denote on the diagram, F, the focus
point of the lens [3]
4
5. The figure below shows a stationary piston in a cylinder. The surface area
of ~he end of the piston 1s _4.0 cm 2, while over at the sealed end, it is 1.0
6. In a thermocouple thermometer where the reading of a voltmeter varies
cm . The sealed end can withstand a force of 100 N before it gives way. linearly wilh temperature, a voltage reading of 8.00 mV was obtained with
one junction at ice point and the other in a 200 c environment.
When the hot junction was taken out and placed into a container, the
reading was -1.20 mV. ..
a. What is meant by Ice point [1]
a . Why the pressure inside the cylinder increases when the piston is
pushed in? [3)
b. Stale one advantage of using thermocouple thermometer over the use
of a mercury in glass thermometer (1)
c. What is meant by the reading of a voltmeter lo vary linearly with
temperature [1]
b. The piston is pushed into lhe cylinder until the air inside it ca.n no
longer be compressed. Find the maximum force that can be exerted on
the piston before its sealed end gives way. (2)
d. Calculate the temperature of the container [2)
6
7. An experiment to show charging by induction uses a metal sphere n electric circuit is set up with a 12.0 V battery, a lamp, L1, and two
mounted on an insulated support. The sphere Is initially uncharged as
8
~ re 1 esistors R1 and R2. The potential difference across R1. as
shown in Fig. 7a below. ~!~~u~e~ by a v~ltmeter, is found to be 2.0 V. An ammeter, which is
connected to the circuit as shown in the diagram below, reads 0.50 A.
metal sphere
LI
RI
Fig. 7a R2
a. A negatively charged rod Is brought near the sphere, as shown in Fig.
7b below. f
[2]
a. Calculate the._resistance of the lamp, L 1.
Fig. 7b
State the cha rge distribution and explain the movement of electric
charges on the metal sphere that occurs as the rod is brought near it. [2]
b. Calculate the resistance of R1
[2]
b. The metal sphere is now touched at point A by a wire connected to
earth, as shown in Flg.7c. On Fig 7c, draw the charges on the metal
sphere. [1]
negajvely charge~ ?Iwire
rod \ _ connected
~ meanh
Fig. 7c
7 8
[1]
9. Two bar magnets are placed on a flat surface as shown in Fig 9. Ignoring
ii. it is necessary to include i~rnersion heaters in the system
the effects of the Earth's magnetic field.
State the magnetic poles at X and Y. [2)
X: ----------------------------
Y: --------------------
8 0-
b. someone suggests that the pipe should be insulated. Do you support ~t?
Explain [ )
Fig. 9
10. The diagram shows a section of a solar heating facility to provide hot water
for a household: It also has separate immersion heaters which will function
if an external power supply is connected to ii
11. Microwaves are waves in the electromagnetic spectrum.
a. State the name of the waves in the electromagnetic spectrum that have
1
wavelengths longer than microwaves. [ l
b Microwaves are sometimes used to send telephone messages over l~ng
The solar heating facility consists of solar panels placed outside the roof. distances. They are sent from larger aerial dishes on top of ~lgh
Connected to these panels we water pipes. The water in the water pipes is buildings or towers. There are often several re~eater or. booster st~t1ons
placed between the transmitter and the receiver, as illustrated in the
warmed by the thermal energy from the s.un and pumped into a water tank
in the house. Inside this tank, thermal energy is transferred from the wat er figure.
in the pipes to the water in the tank. The cooler water is also circulat ed
back to the solar panels. repealer receiver
transmitter station f.
a. Explain why:
i. it is necessary for the tube to be spiral and positioned at the bottom of State why:
the hot water tank - -[2) [1)
i. the aerial dishes are placed on top of high buildings or towers
[1)
ii. repeater or booster stations are necessary.
9 JO
SECTION B (30 Marks}
In this sectmn, answer BOTH questions 12 and 13. a. Explain how the waves are formed as shown on the graphical data. (3)
answer EITHER question 14 OR 15
12.The waves from earthquakes are detected by instruments called
seismometers. The diagr.am below shows a simple sei~l)1ometer setup. It
consists of a bar magnet (with its North, N and South, S poles as
indicated) suspended on a spring. The spring hangs from a metal rod that
transmits vibrations from the Earth. A computer monitors the current
across the coil. When there Is an earthquake, the magnet moves In and
out of the coil.
b. By drawing an arrow, Indicate on the coil the direction of the current in the
set up diagram when a S-pole moves into the coil. [1]
c. Describe whal the movement of the magnet will be (if any} when the
induced e.m.f. has its greatest value, at the point labeled A on the voltage- .
'\
From experimentation, it is found that the Richter scale, ML (measurement lime graph. (1 ]
used by geologist to capture the magnitude of any tremble on earth} and
voltage measured on the CRO are related by the following equation:
_ ML = lOxS
(4)" V'
d. Describe what the movement of the magnet will be (if any) when the
The graphi~I data below, as displayed on the CRO, was obtained during induced e.m.f. is zero. (1)
111
a particular day on 12 Jan 2010, where each pair of peaks (1 complete
oscillation of the same magnitude) represents 1 tremble.
For an earthquake to be major, M, has to be~ 5.0. M, has no unit.
VoltageN Graphical e. Calculate the number of major earthquakes that was experienced during
A Data ~~~ m
1.00 -- <________________ ~---------------------- --
f. Suggest 2 ways in which the seismometer could be made more sensitive,
so that II can detect smaller earthquakes. (2]
-1.
11
12
13. The diagram below shows a household electrical wiring circuit. The mains
cable, containing the live and neutral wires, is connected to a consumer b. The following appliances" are connected to the ring circuit in the
unit via meter M. At the consumer unit, the wires branch out into a kitchen.
number of parallel circuits. The diagram also shows the power circuit in
the kitchen. It is in the form of a ring circuit with three sockets tapped off Appliance Power Rating
the ring. Television 220V,600 W
Computer 220V, 2000 W
Microwave 220 V, 1500 W
i. If the appliances are all switched on, find lhe total current drawn from
the mains. [2]
30A 30A 15A SA E
To 22DV
mains supply
=
~ To slave
Ring circuil in
ii. Hence determine what will happen if an electrician replace the 30 A
the kitchen fuse with a 15 A fuse when one switches on all the appliances at the
same time. [1]
a. The rating of the stove is '220 V, 6000 W. Calculate the opera~ng
c. Given that the power sockets are arranged in a parallel manner in the
current required by the stove. Hence. explain why the stove is directly
connected to the mains via a separate circuit. [3] ring circuit, calculate the effective resistance in the ring circuit if all the
three appliances (television, computer and microwave) are used at the
same time. [3]
d. Explain why all the fuses are connected to the live wire. [1 ]
13
14
EITHER
14. The figure below shows an experimental set-up used for the The gra~hs of the total mass of pan against time and the temperature of
determination of the specific heat capacity of water and the specific latent water against time have been plotted as shown below:
heat of steam.
Total mat on lhe w eighing pn/g
A 20 V battery is iJSed. !he indicator on the rheostat, see figure 14b, can 980 '
only be tuned, In the direction of max R or min R, however its exact value 978
Is unknown. \ Tempcrature/"C
976
974
\ 100
972
\ 90
,,"'
v
970 80
i/
styrorooln 968 70
Copper~l'1- 966 60
I
Metal oOnlatot:r \ 50
~64
\ 40
;;
962
Welghlngpoo
980
958
.. ' 30
20 v
I
,o 20 30 40 50 60 70 80 90 10 20 30 40 60 60 70 80 90
"Tlrnehn\n Tnne after the cfroult wa5 switched onll'nlf'l
"\
Fig.14a a. Calculate the thermal energy gained by the _copper can from time =
10 min to time - 20 min stating the assumption that you have made
- ~
mlnR
Indicator of the
resistance on rheostat
'~
Fig 14b
The inner copper can has a mass of 300 g when empty. It contains 240 ~
of water. The copper can has a specific heat capacity of 0.40 J 0 c1 g
and its outer surface polished. b. Write an expression for the thermal energy gained by the water
from time = 10 min to time = 20 min with the mass of the water
The air between the copper can and the metal container is trapped by a being 240 g 111
layer of Styrofoam.
The circuit of the heater is switched on continuously while the readings of
the ammeter, the voltmeter, the thermometer and the beam balance are
recorded at regular time intervals.
15
16
c. Calculate the thermal energy given out by the heater from time= 10
min to lime = 20 min if the voltmeter and the ammeter readings e. Describe an experimen~ to find out how the time of heating the
have been 12 V and 3.5 A constantly. [2) water at room temperature to 10 C above room temperature, 1s
affected by varying the resistance of the rheostat. In your account
describe the procedure you use to obtain the readings [3]
d. Calculate the specific heal capacity of water using the results of
part (a) and (b). [2]
17
18
OR
15. The figure below shows a circuit set up to test whether electrical
woul~
On a certain day, the ammeter reads 0.003A. With only the table of
resistance of a material changes when temperature of the surrounding d. values shown above, which materials, A or B, you choose to use
changes. to predict the temperature of tho surrounding? Explain [2]
Figure 15
Two unknown material, A and B, are tested. They are each tested in tum,
by placing them between terminals X and Y. As the temperature changes,
the current readings on the anyneter are noted. The results are shown In e. You are given a new piece of material, C. You have been told that Its
the table.
resistance varies linearly with temperature from room temperature up to
100 c. Describe an experiment to verify this. In your account
Cornponent under test
I I O"C /A ht SO"C /A ht 75C /A l al IOO"C /A List the additional apparatus/ materials that you would need
A 0.900 0.800 0.500 0.100
B Describe the procedure you use to obtain the readings (4]
0.002 0.006 0.008 0.010
a. '
Using the figu_re 15 above, add in a voltmeter to it to show how ii is can
be used to measure the potential difference across XY. (1)
b. Describe briefly how you wpuld use the apparatus to obtain a value for
the resistance of the components A and B. (2)
c. From the above table, state the component in which the resistance
increases as temperature increases. [1]
END OF PAPER
19
20
Pai er I
I. B 2_ D 3. A 4. B 5. A 6. B 7. A 8. A 9. D 10. A Ne,t force/ N
11. D 12.B 13.D 14. A 15.B 16. A 17. D 18.A 19. D 20. B
21. B 22.D 23.B 24.A 25. A 26. c 27.D 28. c 29.B 30. A
31. c 32. B 33.A 34. A 35. c 36.C 37. c 38. c 39. D 40. c
16>-----....
SECTION A (SO MARKS)
Time/s
Answer a ll the questions in lhis section.
1. The velocit y-time graph of a moving object of mass 4.0 kg is shown
below.
f
Velocityi mis
10 - --------- -r----------_____ : _____________ _
... 2. Fig. 2 shows an axle A, of radius 14 cm, fixed to a wheel W of radius 10
cm, so that they rotate together, mounted on bearings which are not
40 ------I
'
- - - - - - - - --:-----
'
- - ; - - - - - - - shown. s
30 --1' -.-------------'' --------------~- --------------' wheel W
' axle A
10 20 30 Time/s
a. D~r~n~ the first 30 s, what is the displacement of the object from the
origin. _ .. [2) 20 kg
Fig. 2
A 20 kg mass hangs on a rope wound round the axle. A person is pulling
the string S, which is wound round the wheel.
a. State what Is meant by the principle of moment (1)
b. Based on_ the information from the above graph, sketch its
corresponding net force-time graph on the axes given below [3)
b. Calculate the force, F acting on the string S which produces an equal
and opposite moment to balance the gravity pull on the 20kg mass. [2)
2
c. State and explain how you would make the person's job easier in terms
of pulling the string and lifting the 2tl kg load. [2] a. Complete the ray diagram and denote on the diagram, F, the focus
point of the lens -- [3)
rii1aJ<'eJ~~:ouler;.,.. ,-_-
~fi'.ii~i16i.*1" b. State the focal length of the lens [1]
[aj!iufttie fprqj}~J2
lli-12:!.:"i!Il1J
d. By applying the method as discussed in (c), do you agree that the work c. Hence complete the path for ray z. [1]
done by the person is smaller? Explain [2)
d. With the lens placed at the distance as measured and shown on the
tJ:o'TW diagram, describe the changes in the nature of the image obtain if the
itn~~~_iot 0L~M.r9~MP.1itilt~~iiiiml'-~~,..-1'ii~.:tf.i~'hei,;h..~a1~;;... ;i'1n"'4 object is placed just 8.0 cm away from the lens [2]
, e.lI~l . ~ ~=~w- ~, ~v... ...
-...~ ~ce"PJ.ip1i?W;. m..fil1Ef'ftb.~
.. . t.,,bJ99i0~J:im;:;.M.~~lfili~cUT2rro:~
:t-":'\X"'""
ftJ1Y.hl!Ml.1.~Jo11 -l!J ..
bv1L~~kP.tt~a_&niflllfe@q~/fiG'i'"?IU'i1JiP'ifii~'fti_O:L" c.ii8'nff~Jj
e. Tlie object was at rest when the string suddenly snapped. Calculate its
velocity when it had fallen through a distance of 5.0 m. [2] 4. A, B, C and D in the figure represent particles in a medium through which
waves are passing continuously in the direction indicated by the arrow.
Q.~109;ccl'Energ'y,
~~-?Th 1 - " - ~~
']JlgJi .<= --mv1 ~ J;l:f c
. _,
~~ rr~ -z ~-#~~
A B D

Describe the motion of the particles A , B, C and D when the wave [2]
a. is longitudinal in nature, and h_aving an amplitude of 3 cm
3. An object, 0 is placed in front of a lens of diameter 10 cm such that a
sharp image is formed on the wall
b. is transverse In nature, and having a frequency of 1 Hz.
wall
the.parti11ies'Wi1f01ove-'Veaica11~:lifi"and'dowrf.atit"1wil{eiiWesp'ectlve
eH~l\-~Ji~tiff5~iiilD~i tier~~~ciJ.Jtar:!6".'.iti~ di~c~oht9f:?~V:~~~~~1:~na~L~br!i
ea~ _S!;l,OOJJ!i, eacn partide_w('iul_d-hcive-made o_n!)Qtnple_te.QSCill!l,flOn [1]
0
5. The figure below shows a stalionary piston in a cylinder. The surface area
of the end of the piston is 4.0 cm 2, while over at the sealed end, it is 1.0
[l ] for cm 2 . The sealed em.I can withstand a force of 100 N before it gives way.
-1 for ony missing
F
onow
-1 for missing '"F"
4
[2]
a. Why the pressure inside the cylinder increases when the piston is
pushed In? [3]
~~P.i!J.RWlrii~!iJ<I&-n1'5ue!'i~
~F~~at~IRmat~~'mj
~i!f&f'~t'!%1it&~Mawa'll\ifit&'e"aM1H'~.tl1"lRr..~.SSiira
1. An experiment to show charging by induction u.s~~ a metal sphere
b. The piston is pushed into the cylinder until the air inside it can no mounted on an insulated support. The sphere Is 1rntially uncharged as
shown in Fig. 7a.below.
longer be compressed. Find the maximum force that can be exerted on
the piston before Its sealed end gives way. [2) metal sphere
Fig. 7a
6. In a thermocouple thermometer where the reading of a voltmeter varies
linearly with temperature, a voltage reading of 8.00 mV was obtained with
one junction at ice point and the other In a 200 c environment. a. A.negatively charged rod is brought near the sphere, as shown in Fig.
7b below.
When the hot junction was taken out and placed into a container, the
reading was-1 .20 mV.
a. What is meant by ice point (1)
Fig. 7b
c. What is meant by the reading of a voltmeter to vary linearly with state the charge distribution and explain the movement of electric
temperature (1) charges on the metal sphere that occurs as the rod is brought near 1l.
[2]
~eBrol'J~--~e~~Jleifffi'-th)Jar~,i)r!he'.sP!i~lwmiRg!f~9-~:
. ~dt~!Th~~N\:'Itl'P_d11&~a~''~'ilrg:J~il!1i:~fia11~c~~f
~~ctro.i:i!!_n~_fi.fi~g!b&Ttn
6
b. The metal sphere is now touched al point A by a wire connected to
earth, as shown in Fi~.7c. On Fig 7c, draw the charges on the metal
sphere. [1)
negatively charged
rod
~
\ _
l1wU'e
connected
'earth
Fig. 7c
~Ci~]~\i'~~~m/tlf3"1!~;
8. An electric circuit is set up with a 12.0 V battery, a lamp, L 1, and two
identical resistors, R1 and R2. The potential difference across R1, as 9. Two bar magnets are placed on a flat surface as shown in Fig 9. Ignoring
measured by a voltmeter, is found lo be 2.0 V. An ammeter, which is the effects of the Earth's magnetic field.
connected to the circuit as shown in the diagram below, reads 0.50 A . State the magnetic poles at X and Y. [2]
X: ---------------------------
Y: ----------------------
e 0
LI
D
Fig. 9
R2
[2)
1O. The diagram shows a section of a solar heating facility lo provide hot waler
for a household. II also has separate immersion heaters which will function
if an external power supply is connected to it
f Cold water' tank
b. Calculate the resistance of R1. [2]
S p l,.nl
tube
The solar heating faci lity consists of solar panels placed outside the roof.
Connected to these panels are water pipes. The water in the water pipes is
warmed by the thermal energy from the sun and pumped into a waler tank
in the house. Inside this tank, thermal energy is transferred from the water
in the pipes to the water In the tank. The cooler water is also circulated
back to the solar panels.
8
a. Explain why:
i. i1 is necessary for the tube to be mi!fil and positioned at the bottom of
lhe hol water tank [2]
ii. it is necessary to Include immersion heaters in the system (1]
f
b. Someone suggests that the pipe should be insulated. Do you support it?
Explain [2]
11. Microwaves are waves in the electromagnetic spectrum.
a. State the name of the waves in the electromagnetic spectrum that have
wavelengths longer than microVfaves. [1]
b. Microwaves are sometimes used lo send telephone messages over long
distances. They are sent from larger aerials dish on top of high buildings
or towers. There are often several repeater or booster stations placed
between the transmitter and the receiver, as illustrated In the figure.
repealer receiver
transmitter station
State why:
i. the aerials are placed on top of high buildings or towers [1]
ii. repeater or booster stations are necessal')I, [1)
9 10
SECTION B (30 Marks)
Answer BOTH questions 12 and 13 in this .section.
Answer EITHER question 14 OR 15
a. Explain how the waves are formed as shown on the graphical data. [3]
12.The waves from earthquakes are detected by instruments called
seismometers. The diagram below shows a simple seismometer setup. It
consists of a bar magnet (with its North, N and South, S poles as
indicated) suspended on .a spring. The spring hangs from a metal rod that
transm its vibrations from the Earth. A computer monitors the current
across the coil. When there is an earthquake, the magnet moves in and
out of the coil.
Student can mentioned Faraday's Law, but flley still need to account for the -ve
peak voltage using lenz, if not -1
b. By drawing an arrow, indicate on the coil the direction of the current in the
set u di am when a S- ole moves into the coil. . [1]
filfliUfilfil"'fT~IP.W!ltfi:iJ.bfil'f5.f,{>ifs!W:'Qt
c. Describe what t_f:le movement of the magnet will be (if any) when the
induced e.m.f. has its greatest value, at the point labeled A on the voltage-
~mh,~m~
,,. ;..f!L~,?.~"'f''~~, "'F "'''t "'''"'"'"\'f ~rm<llr(iA'-&'".'mi),-g-~
.._(le._c,vY.cJ~Jn9KIJ9N!3J~:GUPN,_,. ;H'
,J..~,_,_QIJ~-~l-.l?l!Ji>-~J
l1l
From experimentation, it is found that the Richter scale, ML (measurement
used by geologist to capture the magnitude of any tremble on earth) and
voltage measured on the CRO are related by the following equation: d. Describe what the movement of the magnet y.till be (if any) when the
induced e.m.f. is zero. [1]
M, =10>< (s4)u v r11:~;!h)'91\'.~tri'$~1&mn'!t&fil'.i!Y>l.~tl~r11
e. Calculate the number of major earthq'uakes that was experienc~d during
The graphical data below, as displayed on the CRO, was obtained during tJ:ie time ela sed [2]
a particular day on 12111 Jan 2010, where each pair of peaks (1 complete
oscillation of the same magnitude) represents 1 tremble.
For an earthquake to be major,M, has to be<>: 5.0. Mi has no unit. Y~\f~ifli\~t~~~l1':~1tQY~9Wl~QVd@',V~fi'1
Graphical .:J.~ai1~.'&t~;r,ifit~'Y~tI5.w~,~1q~~'<2~1t~9il0~16:tJ1.~@
VoltageN Data
A
/ B
LO!> __ :- ::::::::::::-z::::::::::::::::::~::::::
f. Suggest 2 ways in which the seismometer could be mad~ more sensitive,
so \h<J.\ ii can detect smaller earthquakes. [2]
-1 0
II
12
13. The diagram_ ~elow sh?Ws a household electrical wiring circuit. The mains
cable, containing the live and neutral wires, Is connected to a consumer b. The following appliances are connected to the ring circuit in the
unit via meter M. ~t t~e consumer unit, the wires branch out into a kitchen.
num~er of pa~ll~I arcu1ts. The diagram also shows the power circuit in
the kitchen. It 1s 1n the form of a ring circuit with three sockets tapped off Appliance Power Rating
the nng.
Television 220 v. 600 w
Computer 220 v. 2000 w
Microwave 220V, 1500W
I. If the appliances are all switched on, find the total current drawn
from the mains. (2)
~;~~- -==- ~
JOA 15A SA E
~ To slove ii. Hence determine what will happen if an electrician replace the
30 A fuse with a 15 A fuse when one switches on all the
appliances at the same time. [1]
' circuit in
Ring
lhe kilchen
c. Given that the power sockets are arranged in a parallel manner in the
ring circuit, calculate the effective resistance in the ring circuit if all the
three appliances (television, computer and microwave) are used at the
a. The rating of the stove Is '220 V, 6000 W'. Calculate the operating same time. [3]
current reqwred by t.he s~ove. Hence, explain why the stove is directly
connected to the mains via a separate circuit. [3)
d. Explain why all the fuses are connected on the live wire. [1)
A safely pfecauti.on::0J)re~ent th~'iippJf~n<;:e~~tror!_i bec;oming :l!Y_eJ1f~ho~l~
tfiere be a11Y eregtriffiit'{a,ulf
13
14
EITHER
14. The figure below shows- an experimental set-up used for the
delel"fTlination of the specific heat capacity of water and the specific latent
Total mN on th weighing pan/g
980
.
heat of steam. 978
\
- Tempera.turerc
976
\ 100
I
A 20 V battery is used. The indicator on the rheostat, see figure 14b, can 974 /
only be tuned, in the direction of max R or min R, however its exact value 972
\ 90 v
/
is unknown. 80
970
70
/
968
60
I
966
96
\ so /
\ 40
I/
962
30 I
800
20
I
958
, 0 20 30 40 50 60 70 80 90 10 20 30 40 so 60 70 80 90
TimeJmln Time after the circuit was :switched on/min
a. Calculate the thermal energy gained by the copper can from time =
10 min to time= 20 min stating the assumption that you have made
[2)
Fig.14a
b. Write an expression for the thermal energy gained by the water
from lime = 10 min to time =
20 min with the mass of the water
being 240 g [1 ]
"'iR _......,.,._...ar-t'"'"oW~
resistance on rheostat
m}j~~~:ill~~Jr2'Hfit~~~R'.t2~~~
Fig 14b ~7tlleJ~9,tlJifl
_. }i~~Q~"ar1Ti:>'4awe<1
tr~L g~pgl[_tCJ!i.!'!!~r.H. =-- ""'~ L-~
The inner copper can has a mass of 300 g when empty. It contains 2~0 ~ c. Calculate the thermal energy given out by the heater from time ~ 10
of waler. The copper can has a specific heal capacity of 0.40 J c g min to time =
20 min if the voltmete and the ammeter readings
and its outer surface polished. have been 12 V and 3.5 A constantly. [2)
The air between the copper can and the metal container is trapped by a j:.::i'l\Jt . .... ~ , ' .
layer of Styrofoam. Eb(3:5:: ~~ \(1 O~qO) [1)
.EL5.~.5,2o9.~.l11
The circuit of the heater is switched on continuously while the readings of
the ammeter, the voltmeter, the thermometer and the beam balance are d. Calculate the specific heat capacity of water using the resulls of
recorded at regular time intervals. part (a) and (b). (2)
-- .,..r - -~ ...-.,:.~z4tict"2rr200
The graphs of the total mass of pan against time and the temperature of 4.8'~ it2.4.0,0:.fv~90,p8,.,:l.QQ;,c;:;li,,t,k_,.... .-5:'.::...... ..
water against lime have been plotted as shown below: ~~!475q_'J'1{1<g K).0B ef#~f75\I (i(fi;K)
15 16
e.
OR
15. The figure below shows a circuit set up to test whether electrical
y nt resistance of a material changes when temperature of the surrounding
descnbe the procedure you use to obtain the readings (3)
changes.
-~
Figure 15
Two unknown material, A and B, are tested. They are each tested in turn,
by placing them between terminals X and Y. As the temperature changes,
the current readings on the ammeter are noted. The results are shown in
the table.
Component under test I 11 OOC /A I at SOOC /A I at 7SC /A I at JOO"C /A
A 0.900 0.800 0.500 0.100
B 0.002 0.006 0.008 0.010
a. Using the figure 15 above, add in a voltmeter to it to show how it is can
~~1hWm7fitt'~~~r4fl~!t&-mmamt...~ffii!FO;CtJ:al be used to measure the potential difference across XY. [1)
, : _, 01atTthe- measureinerits;;.'k;'aric!ToiillfirFea1
~.".P{>'fi'i'aW'a '.1fi'it"rill'lliii""d ~~~ii1~rljitf~IJ1iak}lr'.Witl1'Wa,te~
b. Describe briefly how you would use the apparatus lo obtain a value for
the resistance of the components A and B. [2]
IJ&T&~'lJ.JiNl
1
Each of the BOLD Is 0.5m. Round off, eg 2.5 = 3
c. From the above table, state the component in which the resistance
Increases as temperature increases. [1]
d. On a certain day, the ammeter reads 0.003A- With only the table of
values shown above, which materials, A or B, would you choose to use
to predict the temperature of the surrounding? Explain [2)
13.[1)
s1nar1r tias"li. line~i''retation$hip:
l1J'-J'.''4e_?sed~Jcif.i>r:cii~ 1~~!
<!~~~fy_t~an:AwtiJefi'Js~a: li!J.i:v
4;pgii\jS.
17
e. You are given a new piece of material, C. You have been told that its
resistance varies linearly with temperature from room temperature Op to 03 Using Explain, in molecular terms, how the gas exerts a pressure within
100 C. Describe an experiment to verify this_ In your account h~~ - ~
List the additional apparatus/ materials that you would need ~G.?.i:fu]!~~Ifilf2&iiinlfo~r'rifi5fri.ffi5ti6n'Mf
Describe the procedure you use lo obtain the readings (4) rg~iftroBre:~~ru:aa:~~~wma~Wim
~$~iliJP.R'[f~fii.ThlJi.!.r.ijj~~c~otfo1:..ili\~ tt\WaifO~u''S'eXeffiiViF<i'rf8rclfiHJ
llJIP,Jl - ... ;~~~'Wl~~~~arita"WfMru~1ctt
; ;~;.Br~ffP]!];\'.Er'[liffi'P,ll twllffiil'ii:ifre'SWretsm~A:>!,1}klliJW~hit;iW'e'a
& T ~m~~tf{imp!)
~ .,, i~i1fiP.j). bfmaiiilmJM['stirflexe'tt'~nn'e'.~111'1l
i!
Sf\'ff!JmDJ -
iii. the purpose of the insulator [2]
t~t;~.~e~r:i1!Lili?~ .~.:.h.Jg~
'Bunsen
' / Burner
...... m$\. ru}~rgy: t9:lti'.li'-~~.Qdl~9
P'.("11
c. Describe the thermal energy transfer processes from the outside of
12"~!!-;-- ;;,i~; !!4~ ~0~1:\~s1;1o~~Ci'llli@ls9r-l:Wl:.. ~ the house to the pipe leading at point A. [3]
~:P~$J;
I -
Jii.iaf~j~~liQQID'.
-'f.: Ul
o -r e ciifil'.@~:,y~S:[i]
d. Suggest a reason why the water in the heating system is not released
into lhe water tank to be used as well (1]
Static charge
i. On the Fig 4b, draw the charges on the metal sphere. [1 ]
MusJ)j~oh'.Jbe~a..e~-~fu!e.iJu,a!Tiujppet<lCi:Liiliif::
c. The wire connected lo earth is removed. Then lhe negatively charged rod
is also removed, as shown in Fig 40. On Fig4D, draw the charges on the
metal sphere. (1)
END OF PAPER
19
20
Fig. 40
d. The support is made from an insulator. What is the purpose of this
stand? [1]
f
2f
f. Will the velocity In reality be greater or smaller thai:t (c)? Explain. [2)
151
f.
21
Kuo Chua Presbyterian School
Secondary 4 Express
Physics Paper 1
4
5
6. A h.e avy sack of weight W hangs :rom the end of a rope. l he sack is pulled sideways by a
horizontal force F and 1s held stationary. The tension in the rope is T. 8. A uniform 16 m long plank weighing 350 N rests on supports 8.0 m apart.
An 850 N man walks along the plank lo the right.
rope
F
Which force diagram gives the correctfvalue and direction for the tension T?
How far past point B can the man walk before the plank tips?
~
0.82m
1.6 m
c 2.5m
D 3.3m
9. Two similar cricket balls are released from a 20 m tower at t he same time. One falls from
the top, the other from half way up, as shown.
7. A force F is applied to a bent rod which is hinged at Pas shown. In wh ich diagram will F
produce the largest moment? o DD
DO D 20m
ooo
J
{
10
I~ I~
4m
Which quantity is the same for the both balls?
A B D
~
final speed
acceleration
c lime of interval
D increase in velocity
4E_Phy_S058_01_PE_2010
4E_Phy_5058_01_PE_2010
6 7
10. The apparatus in the figure below is set up at a constant temperature of 30 c, d = 12 cm. 12. An object p of mass 2.0 kg is pulled up a slope with a force of 10 N from point X to point Y
h = 10 cm. ' as shown in the diagram below.
Air is trapped inside the left-hand closed limb while the right-hand limb can be moved up
and down. The atmospheric pressure is 760 mm Hg.
t men:ory
l What is the work done against the forces opposing the motion along the slope if P moves
at a steady speed?
A 22 J
B 30 J
c 50 J
D 52 J
13. During a barbecue, why do we always cook food on top of the charcoal?
If the right-hand limb is rai sed such that the length of the trapped air, dis 11 cm, what is
the length of h? A To increase the rate of heat transfer by conduction.
B To increase the rate of heal transfer by convection.
A 9.0 cm c To increase the rate of heat transfer by radiation.
B 11.0 cm D To increase the rate of heat transfer by radiation and convection. .
C 14.6 cm
...
D 17.8 cm
14 . In the process of convection, energy is transferred.
11. Some gas is heated in a sealed container. Which of the following do NOT increase? A by the diffusion of molecules th~ough ~ fluid.
B because of density differences in a fluid.
A The number of collisions between the gas molecules and the container walls. c because of temperature differences in a solid.
B The force due to the collisions between the gas molecules and the container walls. D by the vibration of molecules about a mean position.
C The average kinetic energy of gas molecules.
0 The average distance between gas molecules.
4E_ Phy_5058_01_PE_2010 4E_Phy_50~_01_PE_2010
8 9
15. A thermocouple thermometer uses a voltmeter to measure the e.m.f. generated between 17. The diagram shows a water wave travelling in the direction of the arrow.
two junctions. The junctions are al temperatures 11 and t2 To calibrate the thermometer,
fixed points are needed.
As the wave moves forward which o f the following will happen?
f P and Q move from left to right.
A
What are the values of t1 and t2 when the thermometer is calibrated at the steam point? B P will go down, Q will go up.
c P will go up, Q will go down.
0 P will go up, Q will go up.
t1 t2
1
18. The graph represents a wave pulse travelling along a rope at 1ms in the direction shown
A o-uc o0c by the arrow.
B 0 c 100 c direction o: wavo tfavel
c 30c o 0c
p
D 3o 0c 130c
10 12 14
positionlm
16. It takes 2 minutes to raise the temperature of 1.7 kg of water by 40C using a 2.5 kW
heater. Assuming there are no heat losses, how long would it take to raise the
temperature of 170 kg of water by the same amount using a 5.0 kW heater?
A 80 minutes From the moment shown, what is the shortest time before poi nt P on the rope has a
B 100 minutes
displacement of 1.0m?
c 200 minutes
D 400 minutes
A 2.0 s
B 4.0 s
c 6.0 s
D 8.0 s
4E_Phy_5058_01_P E_2010 ~E_Ptiy_5056_01 _PE_2010
JO
19. A wave generator which makes 5 oscillations in one second is used to produce waves in a
ripple tank.
~ 21. The refractive index of water is 1.33.
Which of the following diagrams does not represent the behaviour of light as it travels from
water into air?
12 cm
A B
Which of the following values of frequency, wavelength and speed could the waves
have?
Frequency I Hz Wavelength I cm Speed I cm s1
A 0.5 12 6
B 5 3. 15
e 5 12 60
D 20 3 . 60
D
20. An obJect o is placed in front of a plane mirror MN as shown In the diagram
below.
I .
M_____ N
... eo- '' so.
~>r:::
alr "
X--~-------------~-----Y 22. Below are four statements about the uses of electromagnetic radiation.
2 3 4
Gamma rays are used in medical treatment.
Infra-red waves are used in sunbeds.
A student moves her eve along the line x:f to observe the Image o. Identify Microwaves are used in satellite TV.
the polntls> on X-rays are used in intruder alarms.
the line x:f where the image of o cannot be seen.
How many of these statements are correct?
A. 4 only A 1
B 3 and 4 only e- 2
c 1, 3 and tton ly c 3
D 1, 2 and 4 only D 4
4E.Phy_S0~8.01 _PE.2010 4E. Phy_5058_01 _PE_2010
12
13
23. The diagram shows a student looking at a plane mirror.
24. Two rays of light strike a diverging lens as shown below. The distance from the centre of
the lens, Oto F1 or to F2 , is the focal length of the lens.
XYZ Plane
mirror
f Which statement about the rays after they have passed through the lens is correct?
Which of the following shows the image observed by the student in the plane mirror?
A They appear to come from F1
B They appear to come from F 2
..:; They appear to come from 0
A D They m.eet at F2
IXYS 25. In the diagram below, John and Pete stand in an enclosed room with reflecting walls 1100
m long. John on the left fires a gun which makes a loud sound. Pete standing 800 m away
lxvz
B
to the right hears the gunfire sound and hears two echoes.
John Pete
c
lsvx ~
IOOm 800m
I ZYX
D
The speed of sound is 340 m/s. The time between the first two echoes is
A 0.588 s
B 0.294 s
~ 0.882 s
D 1.18 s
4E_Phy_5058_01_PE_2010
4E_Phy_505ll_01_PE_2ll10
14
15
26. Four processes are used to charge an isolated metal sphere. 2
29. A 0.4 m length of resisi ance wire with a cross sectional area of 0.2 mm has a resistance
012 n.
p The charged rod is_removed.
Which w ire of the same material w ill also have a resistance of 2 O?
Q The sphere is earthed by touching it.
R cross sectional
A charged rod is brought close to the sphere. length area
S T he earth connection is removed from the sphere.
A 0.2 m 0.2 mm2
In which order should these processes be carried out to charge the sphere? B o.2m 0.4 mm2
A
B

First
Q
Q
R
s
- p
R

Last
s
p

c
D

o.sm
O.Bm

0.4 mm2
0.1 mm 2
G R Q s p 30. In the diagram, 1, is the current supplied by a battery. lz and b are the currents in each
branch of the parallel arrangement shown.
D R Q p s
27. During a singl e lightning flash, 20 C of charge travels between a cloud and the earth in
0.02 s, across an average potential difference of 2.0 x 107 V.
How much energy is converted during the flash?
A 1.ox106 J
B 1.ox 106 J
c 4.0x108 J
Which is the correct statement concerning I,, 12 and 13?.
D 2.0x1010 J
28. An filament light bulb is supplied with a constant potential difference of 240 V.
A
13
'1 is equal to 12 but bigger than b.
12 is bigger than '3 but smaller than I, .

c 13 is bigger than 12 but smaller than h .
As the filament of the bulb heats up, its resistance D 13 is equal to !, but bigger than 12.
A decreases and the current flowing throug;i it increases.
6 decreases and the current flowing through it decreases.
C. increases and the current f lowing througt1it decreases.
D increases and the current flowing through it increases.
4E_Phy_50~_01_PE_20 1 0
4E_Phy_5056_01_PE_2010
16 17
31. The graph shows how the resistance of the thermistor varies with temperature. 33. In the circuit below, when the resistance of the variable resistor is decreased,
~hat Is the ammeter andvoltmeter reading when the temperature of the thermistor
1s 3oc1
.relstance lo
1000
I
800 I
'
400 I"\
20D " ....
,._
f the voltmeter reading decreases.
t emp~ralt.Jre / 00 A
2a 40 so so 100 B the voltmeter reading Increases.
C the ammeter reading decreases.
O the ammeter reading increases.
Ammeter reading Voltmeter reading
A 10mA / 4V
B 15 mA 4V
34. The diagram shows a small compass that is placed near a magnet to plot the magnetic
field.
c 30mA 2V
The reason for using a compass is that _ _ _ __
D 30mA 6V
~----
I
-----~-- ----- --------------------~I
I
32. Birds ?an stand on an overhead transmission line without su'fering any harm. Which of the
following statements best explain this?
I
I
'
1
I
I magnet 0 composs
A Their bodies have a very high resistance.
'
I
1 I
1 '
B Their feet are very good insulators. t-------------~-------------- ----- -----
C There is no potential difference between their feel.
b The spaces between their feather!! ;ict as insulators. the compass needle is a magnetic material that can be easily magnetized and
A
demagnetized.
B the magnetic field of the earth has not effect on the needle.
c the compass needle is a lreely suspended permanent magnet.
D the compass needle gets magnetized when placed in a magnetic field.
1
4E_Plly_5058_01_PE_2010
18
19
35. A student measures the voltage and frequency of an a.c. supply and obtains the following
trace on the oscilloscope screen. 36. The diagrams show the forces F between two wires carrying currents out of the page.
The magnetic fields close to the wires are also shown.
Which diagram is correct?
v A B c 0
cb ch. ~ 6 0 ~,
F
"""~ ~ /
/
~.
~ /
CV CV cp. cp
magneUc wire
field line carrying
current
If the screen grid has 1 cm squares and the controls have the following settings:
F F F F
y-gain: 3 mV I cm
timebase : 2 ms I cm
37. The figures below represent the magnetic field set up by current carrying conductor.
Which of the following correctly represents the voltage and frequency of the supply?
c
A
B

Voltage I mV
12
12

Frequency I Hz
16
62.5
16
-~--
- -- ----
-- ---
~
- -----
- -----........
- ... - -- - - -- --
--
6 ;
6 62.5'
0
II Ill
The dolled lines represent magnetic field. Which figure(s) show the correct pattern and
direction of the magnetic field?
A II only
8 II and Ill only
<.; I and II only
0 I, II and Ill
~E_Phy_50~8_01_PE_2010
20.
21
38. As switch S is closed, In what direction does the compass needle point and what is the
direction of the current through resistor R? 40. The diagram below shows a generator turning in the antH:lockwise direction.
0 NS~N
~
Compass needle d irection
A West - .- Cu rrent directio n thro ugh R
from 1 to 2
. West p
B ~
from 2 to 1 Q
c East from 1 to 2
East ~rom 2 to 1 Which of the following statements is true?
D
A The induced current in the coil JKLM is the maximum in the
39. A permanent magnet moving up and down on the end of a spring induces an e.m.f. in a position shown. .r
coil. B The induced current will change direction every 360 turn of the
coil JKLM.
C The magnetic field present will caused the coil JKLM to turn
continuously.
D The slip rings A and B ensure that the coil JKLM turn
continuously in the anti-clockwise direction.
Which of the following modifications, on its own, would decrease the maximum value of
the induced e.m.f.?
A increasing the number of turns in the coil
B increasing the strength of the magnet
C raising the coil
l:> raising the support of the spring
4t_f'l1y_5058_01_PE_2010
4E_Phy_5058_01_PE_2010
4E_P}fY_PE2_Pl_2010
ANS\VERS TO PAPER 1
1 A 21 B
2 B 22 B
3 c 23 c
4 D 24 A
5 B 25 'A
6 c 26 c
7 c 27 c
8 B 28 c
9 B 29 c
10 D 30 c
11 D 31 A
12 A 32 c
13 B 33 D
14 B 34 c
15 B 35 0
16 B 36 B
17 D 37 B
18 D 38 c
19 B 39 D
20 A 40 A
Kuo Chua Presbyterian School
Secondary 4 Express
Physics Paper 2
4
5
2. A body of mass 0.5 kg is projected horizontally, with a speed of 8.0 mis, from the (d) What is the speed of the body at B? [1]
top of a tower AD which is 45 m above the ground level DC. It moves with a
parabolic path and strikes the ground at C as shown in Fig. 2.1 .
i
45m
Path of body
/ (e) Whal is the ki netic energy just before it strikes the ground at C? (1]
Fig. 2.1
8 is t~e point on the path of the body 32 m above the ground level. The ground
level 1s taken as the zero level of potential energy. 3. Fig. 3.1 below shows a mercury barometer and a manometer filled with mercury
connected to a gas supply placed next to a meter rule.
(a) Calculate is the total energy of the body at A. (1 J
100- - ~-- -
(b) How much potenlial energy has it lost in l'lOVing from A to B? [1]
To g;tS f :=
uppty
(c) Calculate the kinetic energy of the body at B. [1 )
Fig. 3.1
(a) Determine the atmospheric pressure indicated by the mercury barometer in
cmHg .
.. .. .. .. ... ............ ............ .. ....... .. ..... ............ .. .... .... ........... .. .. .. ". .. [1 ]
<E..Pby_lol8_02_P~_2010
~E_Phy_SOSl_Ol_PE_lOI O
6
7
{b) Determine the gas pressure In cmHg. [1J
4. Fig. 4.1 shows an enclosed glass cell containing smoke and still air.
The smoke particles in the glass cell are observed through a microscope.
~--
(c) Calculate the pressure at A in Pa. (Density of mercury= 13 600 kgm3) [3]
(d) Explain in term of the behaviour of molecules, why lhe gas exerts a Fig. 4.1
pressure on the mercury. [2]
(a) Describe the behaviour of the smoke particles seen through the
microscope.
[2]
(b) What evidence does the observation provide about the air molecules?
Justify your answer.
[2J
E_l'l>y_lOSS_O?_PE_lOJO
4E_Pby_SOS8_02_PE_l010
8
9
5. (a) School caretakers often use trucks such as the one shown in Fig. 5.1
below lo move heavy objects around. The jesign of the truck makes it (b) Fig. 5.2 shows a demonstration of the effect of frictional forces on
easier to lift the load and the wheels make it easy to move the load from balancing a metre rule by moving two fingers A and B towards each other.
one place to another.
The 200 g uniform metre Jule is being supported by A and B. The reaction
forces of the fingers A and B on the rule are labeled as RA and Re
respectively.
~
Mass 200 g
.
-+!10.0cm~ ~20.0cm-:
Fig. 5.2
(i) Mark the position of the centre of gravity of the metre rule with "X". [1 ]
Fi g. 5.1
(ii) Calculate the reaction forces (RA and Re) exerted on the ruler. [4]
(i) On Fig. 5.1 mark and label the position of the pivot.
You should also mark the load force and the effort force needed to
lift the load. Show clear1y where they act and the direction in
[2)
which they act.
(ii) Explain, using the idea of moments, why the design or the truck
makes it easier to lift the load.
[2]
<E_Phy_lOll_Ol_PE_lOIO
4E_Phy_l0ll. ~_PE_20IO
10
II
6. Jn the circuit shown in Fig. 6.1, the variable resistor R is adjusted such that the 7. An electric lamp is labeled "250 V 100 w and an immersion heater is labeled
current flowing through the ammeter is 0.20 A. "250V, 2kW"
w on the lamp.
cr;:-1'
24V (a) Explain what is meant by the label "250 V 100
500
BOO
[2]
Fig. 6.1
(b) Calculate the current flow in the heater when it is operating normally. [2]
(a) What is the potential difference between P and Q? [1]
(b) What is the current flowing through R? [2] (c) Explain why It is dangerous to connect the heater to the main supply
without an earthing il. [2]
(c} What is the resistance of R? [2]
4E_Pt.y_505!_02_fE_:ro10
4E_Phy_505&_02_PE_lOJ0
..
12
13
8. Fig. 8.1 shows an alternating current input is connected to the two resistors.
(c) The CRO is used to measure the speed of sound as shown in Fi g. 8.2.
CRO
10.0m ___JI~
Y-input Microphone ~ ' Light
Mov ble
- -+--- Pulse gene<ator
D
CRO

g,_ wall
Fig. 8.2
The wall is stationary.and a pulse is generated by the pulse generator.
The CRO is turned on at the same time and the waveform is obtained as follow.
2on 600
'------ - - - - --0 I\; 0 - - - -- - -- - - - '
AC source
Fig. 8.1 (i) Base on the information, calculate the speed of sound if the time-
[1]
base of the CRO is set at 1 Oms/division.
The Y-input terminals of a Cathode Ray Oscilloscope (CRO) is connected across
the 20 n resistor.
The time-base of the CRO is set at 5 ms/division and the Y-gain is 2 V/ division.
(a) Calculate the peak voltage across the 20 n resistor. (1 ) (Ii) The movable wall is now set to move towards the pulse generator.
After 1 s, the waveform is obtained as follo\\f>. Determine the speed o f the
[2]
trolley moving towards the pulse generator if the CRO is set at 10
ms/division.
: ; : : : -t:- -
--t---t---t- --~ - ..
1 t t I 1
I t t I I I
(b) Determine the frequency of the AC source. (2) --:--:--:--:1:-
l > f >--~--->
I t I -- I
t 1 I I I I
--.-- t 'l--1-- -t- -t-
t I I t I I I
I I I t I I I
___....... - t--+-.. +----
1 I I I t O I
t t I I I I I
<E. Pby_SOSB_02_PE_2DIO
<b.~y_SOS8_02_PE_20 1C
14 15
Suggest a reason for the second sound to be softer. [1]
Section B (ii)
Answer all the questions from this secti:>n.
Question 11 has a choice of parts to answer.
9. Fig. 9.1 shows a man is watching a thunderstorm which is directly over a village.
Some distance behind the village Is a mountain which is about :.1.5 km trom the
(iii) Calculate the distance between the man and the thunderstorm if the
man.
time lapse between the two sounds is 4.4 s. [2)
(b) When the distance between the cloud and the ground is 400 m, determine
the amount of charges that flow during discharging, [2)
(i)
The flash is seen as a result of the effect of the electrical discharge on the gases
or rain droplets through which the charges travel. When the distance between the
cloud and the ground is 400 m, the average time for the lightning discharge from
the cloud to the ground is about 4.0 x 10-3 s. The amount of current produced
during the discharge is 20 000 A. The maximum power produced in every channel
of the flash Is about 1.0 x 108 W per metre of channel. Most of the power is [2]
dissipated in heating the channel to around 30 000 C. This heats 'up the air (ii) the typical potential difference between the cloud and the ground,
sufficiently to cause rapid expansion. The shock waves formed are then heard as
thunder. Travelling at 340 m/s, the thunder reaches the man after the flash is
seen.
(a) For each lightning flash, the man can hear a loud sound of thunder
followed by. a softer one.
(i) Why does the man hear two sounds? [1] [2]
(iii) the energy involved in the discharge when the flash is formed.
4E_Phy_SOSl_Ol_P6_1010
4E_Phy_SOS1_02_PE_2010
16
17
10. Fig. 10.1 shows a circuit diagram. The mains vcltage is 220 V. ..
(c) The electrician made a mistake in the construction of the lighting circuit.
F,
x -E==t---t-----,
Y, _ _ ___,..-...-----. (i) Suggest what is wrong with such a connection for the three bulbs.
33rv
150W
(1 ]
(ii) On the space below, draw a correct connection for the three bulbs. (you do
Lightlng DVD player
Circuit 20W
not have to draw the rest of the circuit diagram). {1]
Power Circutt
Hair Dryer
250W
220 V, 220 V, 220 V,
50W 50W SOW
Fig. 10.1
(a) Explain the use of fuses in the circuit. (d) Assuming the lighting circuit has been corrected:
(i) The whole system, including the lighting circuit, is switched on for 8 hours.
Calculate the cost of electricity if it is charged at $0.20 per kWh. (3)
[1]
(b} Some electrical appliances like hair dryer and DVD player do not have an
earth wire. Explain why?
(ii) Calculate the total current flowing through the whole circuit and hence
suggest a suitable fuse rating for F,. [3]
[1]
4E_Phy SOS8_02_PE_20JO
4E_Pby_m8_02_PE_2010
18
19
EITHER
11. Man~ cars are fitted with an air-bag, as shown in Fig.11.1. In a collision, the air- OR
bag inflates and reduces the effect of the Impact between the passenger and the In an experiment to determine the specific heat .capacity of? liquid,? 0.~2 kW
dashboard. immersion heater is used to heat the liquid. Ounng the heating, the hqul w~s
constantly stirred. The graph below shows how the temperature of the 1qu1
changes with time.
Temperatui-ef.C
~ .
__,..-
60
__
dashboard -..._____,
50
/
A
(a)

Fig.11.1
In a test of the air-bag, a heavy ball is used instead of the passenger. 40
/
The car is travelling at 14 m/s when It hits a wall. The air-bag inflates and
the ball takes 3.0 s to come to resl The ball has mass 5.0 kg.
(i) Calculate the averag.e deceleration of the ball.
(3)
30
/__ 150 200 250
...-Timc/s
[2) 0 50 100
(ii) Calculate the average force exerted on the ball.
(iii) Using ideas about acceleration, explain how the air-bag reduces the (a) Use the portion OA of the graph to calculate a value for the heat capacity
force on the ball during the test. [2) (3]
of the liquid.
Explain why the value you have calculated is bigger than the actual value. [1]
(b)
(b) If there was no air-bag, a large pressure would be exerted on the ball at
the point where it hits the dashboard. Explain why the liquid is constantly stirred during the experiment. [1)
(c)
(i) Define pressure. [1) What happens to the liquid along BC of the graph? (1]
(d)
(ii) The inflated air-bag reduces the pressure exerted on the ball. State (e} Explain, in molecular terms, what has happened to the heat supplied by the
two reasons why the pressure is reduced. (2] [2)
heater along BC.
(f) Calculate a value of the specific latent heat of vaporisation of the liquid if
109 of the liquid evaporates during the time B to C. [2)
End of Paper
46_Phy_50S8_02. PE_l010
s_Phy_soss_Ol_Ps_2010
Answer all of the following questions in the spaces provided:
(b) p.e. loss= mgh
1. (a) !t means the car is moving in a specific direction and for every one second = 0.5 x 10 x (45-32)
1t consistently moves 15 m. [1] ' = 65 J
(b) t = s/v (c) k.e. at point B = 241 -160 =81 J
= 12/15 [1]
= 0.80 s (2 sf) [1] (d) k.e. =% mV2
81 =y, x o.s x V 2
(c) s =area under the graph Vims'
V = 18 mis
20=Y.xtxv 15
k.e.= 241 J
t = (2 x 20) I (15) [1) (e)
= 2.67
= 2.7 s (2 sf) [1) 20
3. (a) Atmospheric pressure= 76.0 cm Hg (1 dp)
tis (b) Gas pressure= 86.0 cm Hg (1 dp)
(c) Pressure at A in cmHg = 35.0 + 76.0
=111.0 cm Hg [1]
(d)
15t-- - - Pressure at A in Pascal = hpg
= (111x102 )(13.6 x 103 )(10) [1]
= 15 0960
=1.5 x 105 Pa (2 sf) [1)
'(d) The gas molecules are in constant random motion colliding
with the surfaca"of the mercury. exerting a force on the surface [1]
and the force exerted per unit area on the surface of the
Us mercury is the pressure of the gas. [1 ]
0.80 3.5
(e)
Braking distance will be larger. 4. (a) The smoke particles are seen constantly tnovinq about randomly. [1]
[1]
The smaller particles are seen to move about more quickly that the
~s the braking force (fritional force between tyre and road) is constant, the bigger particles. (1]
increase 1n mass of the car will cause the deceleration of the car to
decrease (a= F/m). Hence, the braking distance will Increase. [1] (b) Evidence:
Air molecules possess energy and are moving about randomly and
Or When mass of the car increases, the initial also increases. Since initial is haphazardly. [1]
~he relucla~~e. for. an ob1ect change its state of motion, the car with
increased m 1t1al will take a longer distance to stop, g;ven the same initial
Justification:
speed. (1, 11 Each smoke particle is constantly bombarded by the air molecules
from all sides. These bombardments are not bal anced and give rise
to a resultant force that causes the smoke particle to move. [1]
4
5
5. (a) (i) Pivo t at centre of wheel [1]
Effort [1] at handle lo the rig ht and 7. (a) For the lamp to operate nor mally, the lamp requires a potential difference
Load d own wards at centre of gravity near centre [1] of 250 V across it. [1]
[max= 2]
[allow one error] The amount of electrical energy converts to light and other form of energy
such as thermal energy per second is 100 J. [1]
(ii) The effort is f urther from pivot [1]
So only a small effort is needed to produco:: a lciryer moment [1] (b) P= IV
I = 2000 / 250 [1]
=8.00 A (2/3 sf) [1)
(b) (i) at 50 cm mark (1]
(c) If the live wire comes into contact w ith the metal case of the heater
(ii) ~x0.7=2x0.3 [1] accidentally, the case will become 'live' with a potential of 240 V. (1)
RA = 0.86 N (f /3 sf) [1] Without a earthing, the current will flow from the high voltage w ire through
anyone touching the metal case and to the Earth. The person will get an
Rax 0.7 = 2 x 0.4 [1) electric shock. [1]
Ra= 1.14 N (2/3 sf) [1]
8. (a) Peak voltage =2x2
6, (a) p.d. = 24 - (80 x 0.2) ... =4.0V [1)
= 8.0V [1]
(b) f = 1/T
(b) V= RI =1/(4 x 5 x 103 [1]
8 = 50 x I = 50 Hz [1]
l=0.16A [1)
( c) (i) v = s I (V2)
current through R = 0.2- 0.16 . = 10.0 I [(6 x 0.001)/2]
= 0.040 A -[1) = 333.33 2
=3.3 x 10 ms' (2 sf) [1 ]
(c) V= RI
8 = (40 + R)x0.04 [ 1) (ii) Distance of the wall from the generator
R=160fl [1) s =vt
=333.33 x (5 x 0.01 )/2
= 8.33 rn [1]
Distance moved by the trolley in 1 s
=10.0- 8.33
=1.67 m
Speed of the trolley
= 1.67 / 11
=1.7 ms [1]
-'
7
9. (a) (i) The first sound is the direct sound of the thunder. The
second SJUnd is the reflected sound (echo) of the thunder 10. (a) A fuse protests equipment and wiring system against excessive current flow.
from the mountain. [1] It becomes hot, melts and breaks the circuit when current flow is greater than
its rated value. [ 1]
(ii) The sound energy was absorbed/Josi to the surrounding (b) It is protected by double insulation i.e. the wire itself has a layer of insulation
when the sound hit the mountain and was reflected back to plus the plastic casing of the appliance . [1]
the man.
[1]
(c) (i) Each bulb is not operating al its rated voltage i.e. NOT at its normal
brightness.
(iii) distance between thunderstorm and mountain
= speed x time OR When one bulb is out of order, the other two bulbs will also NOT light
= 340 x 4.4/2 up as they connected in series. [1]
=748m [1]
(ii) All bulbs must be connected in parallel [1]
distance between man and the thunderstorm
=2500 - distance between thunderstorm and mountain e.g.
=2500- 748
=1752 m [1 ]
(b) (i) Q= It
= 20 ODO x 4.0 x 10.J [1]
=80 c [1]
(ii) P =VI
1.0 x 10 x 400 =v x 20 000
8
[1]
v = 2.0 x 106 v [1 ]
(iii) E =V x Q
= 2 x 106 x 80 [1] (d) (i) Total power = (50 x 3) + 2200 + 150 + 20 + 250 + 80
= 1.6x108 J [1] = 2850 W =2.85 kW [1 ]
Electrical Energy consumed= 2.85 x 8 =22.8 kWh [1]
Cost= 22.8 x $0.20 =$4.56 [1]
=VI
(ii) Total P
l=PN
I= 2850 / 220 [1)
=12.9 A
= 13A (1]
A '15 A fuse' should be used. [1]
9
11 . (a) (i) a= (v-u)/t [1]
(f) By principle of conservation of energy,
=14/3 [1]
Electrical energy lost by the heater =Heat absorbed by the liquid
=4. 7 m/s2 (2/3 sf) [1]
Pt= mlv
[1)
(ii} F =ma or !'i x (i} [1) (0. 12 x 103) W "GO s 0.010 kg x Iv
=23 N (213 sf) (1 ] 5
Iv= 6.0 x 10 J kg-
1 [1]
(iii) It takes a longer time (of impact) for the ball to come to stop (1]
This results in smal~acceleration (1]
(b) (i) pressure= force I area or P =FIA or force on unit area [1]
(ii) larger area [1]
smaller force (1)
OR
(a) By principle of conservation of energy,
Electrical energy lost by the heater= Heat absorbed by the liquid
Pi= CiJO [1]
3
(0.12 x 10 ) w x 100 s = c x (so c -3o c) !11
c = soo J c1 111
(b) Part of the heat from the heater is lost to the surroundings. [1]
(c) To ensure the temperature of the liquid is uniform throughout
the liquid. [1]
(d) The liquid boils. [1]
(e) The heat absorbed is used to break the intermolecular bonds
between liquid molecules [1]
and to push back the atmosphere in order to escape as
vapour. [1]
Methodist Girls School
Secondary 4 Express
Physics Paper 1 Page 2 of 17
A van moves from point A to B along a straight road at an average speed of 2 mis. It . Page 3 of 17
then returns from point B to point A at a higher average speed of 4 mis.
5 A bullet of mass 0.1 kg is shot into a wall. The depth of penetration is 10 cm
r-~~-=--~__.~~=---=---~
I
0.1 kg
What is the average speed ol the van for the whole journey?
l __ __
A zero If the resisting force ac:ing on the bullet by the wall is 9 800 N, estimate the speed of
the bullet, v before entering the wall.
B 1.33m/s
c 2.67 mis A 120 mis
0 3.00mls B 140 mis
C 160 mis
2 Jardena participates in a 100 m race. In the first 4 seconds. she increases her speed
2
with a constant acceleration of 2.5 m/s . She then maintains her speed until she O 180 mis
finishes the race. Jardena can finish the 100 m race in a time of
6 A , B, C, 0 and E are five positions on a smooth curved rail. Position C Is directly
A 8.Ss across E as shown in the diagram.
B 11.5 s
c 12.0 s
A
0 12.5 $
3 Which of the following statements concerning mass and weight is correct?
A At a particular place, weight of an object is directly proportional to it mass. E
B Both mass and weight are measured in kilogram.
c Both mass and weight do not change with location.
D
0 Both mass and weight have directions pointing to the ground.
4 A horizontal force F pushes a block of mass m along a horizontal rough plane ai a A marble released from rest at A slides along the rail. The speeds of the marble at B,
constant speed v . What is the work done against friction during time t? C, D and E are v,, v,. v, and v4 respectively.
Which of the following about the speeds is correct!
F
A -xvxt
m A v, > v 3 > v2 > v1
m B V1 > V2 > V3 >Ve
B -xvxt
F c v. = v, > v, > v,
c Fxvxt D V3 > v. = v, > v,
D m xv x t
7 The pressure of a gas ~oes not depend on the
A average speed of the molecules.
8 frequency of molecular collision per unit area of the wall.
C mass of each molecule.
0 molecular collisions among the gas molecules.
Physics Paper 1 Sec 4 Prelim Exam 2010
Page 4of17
8 A manometer is conneded to an air pump as siown. Page 5 of 17
12 metallic object
.'<' -- - lldllmmlm_ _. .. f- woodeg poor
~:
A metallic object is placed on a wooden floor at room temperature. When we touch
them, we feel that the object is colder ;han the floor. Which of the following statements
1m
are correct?
(1) The temperatures of the object and the floor are lower than our bod y
temperature.
(2) The temperature of the object is lower than the temperature of the floor.
(3) The object is a better conductor of thermal energy than the noor.
A (1) and (2) only
B (1) and (3) only _
C (2) and (3) only
Given Iha~ the manometer is filled with waler of density 1 000 kg/m3 and the D (1), (2) and (3)
almosphenc pressure Is 100 000 Pa, what is the air pressure p caused by the air
During a campfire, we sit around In a circle beside a fire. Our faces feel hot. Thermal
pump? Take g; 10 N/kg_ ' 13
energy is transferred from the fire to our face mainly by
A 100 000 Pa
B 110 000 Pa A convection.
C 200000 Pi! B radiation.
C radiation and conduction.
0 210 000 Pa
D radiation and convection.
9 Which of the following statements. best describes the upper fixed point?
14 A double-glazed window has two sheets of glass separated by a layer of air.
A The upper fixed point is the temperature at which w ater boils. Thermal energy is conducted and convected through the layer of air. The amount of
B The upper fixed point is the temperature al which pure water boils. conduction and convection varies with the thickness of the layer of air, as shown in t he
e The upper fixed point is the temperature at which pure waler boils under grar;h.
standard atmospheric pressure.
D The upper fixed point Is Iha temperature at which pure water boils in vacuum. thermal energy
transfer
10 After thermal energy is transferred to a substance which may be a solid a liquid or a through a ' total energy
gas, what must happen to the substance? ' double glazed transfer by
window '', conduction end ,
A The temperature of the substance must increase . :onvectio~_,,,.- by convection
B The kinetic energy of the molecules of the subst~nce
must increase. .. , .-"
c The potential energy of the molecules of the substance must Increase
0 The internal energy of the substance must increase. .
11 Which of the following pairs of substances have different specific heat capacities? - ------ - -- - - - ----- ----------
A Alcohol at 2oc and alcohol at 3oc. Which thickness of air produces the smallest energy transfer, and why?
B Al~mln_ium in the form of a rectangular block and aluminium In the form of a
cyl1ndncal block. A 0.5 cm because there is little convection
C Liquid naphthalene and solid naphthalene. 8 1.0 cm because the total thermal energy tr;insfer is least
0 Water in a large tank and water in a cup. C 1.5 cm because the total thermal energy transfer is small and conduction is low
D 2.0 cm because there is little conduction
dist Girls' School Physics Paper 1 Sec 4 Prelim Exam 2010
Sec 4 Prelim Exam 2010
Phvsics Paper 1
Page 6 of 17
Page 7 of 17
15 A layer of oil on the surface of hol soup can reduce lhe rate of temperature fall of the Question 18 - 19
soup because
A transverse wave travels along a string as shown in the followmg figure.
A oil is a bad conductor ol lherrnal energy.
'B oil has a smaller la!ent heal of vaporisation than water.
C the layer of oil prevents lhemal energy transfer by convection.
0 lhe layer of oil reduce the rate of evaporation of the soup.
16 A musician plays a violin. The string of the violin vibrates and sound trarsmits through
the air.
Which of the following correctly describes the waves produced?
A Longitudinal waves are produced in the string and in the air_
B Longitudinal waves are produced in the string and transverse waves are
produced in the air.
c Transverse waves are produced in the string and in the air.
0 Transverse waves are produced in the string and longitudinal waves are 18 If the frequency of the wave is 12 Hz, what is the velocrty of the wave?
produced in the air.
A 1.0mis
,-- l
17 A wave on a string is produced by a vibrator. B 4 O mis
The waveforms at time t = 0 and t =. 0.03 s are as shown in the diagrams. l? 4.8 mis
D 12 mis
I
direction of wave motion 19 If the frequency of the wave is doubled, what will be the amplitude and the wavelength
of the wave? Assume the speed of the wave remains ~nchanged
r - - - -
t= Os Amplitude Wavel ength
1! L___
vibrator
A 0.1 m 0.2 m
I B 0.1 m 0.4 m
-
1-
]""""'
c 0.2m 0.4 m
~ O.Bm
p 0.2m
t =0.03 s --
20 I ~ound transmitter l
Which of the following statements about the wave is/are correct?
(1)
(2)
(3)

The amplitude of the wave is 1 cm.


The wavelength of the wave is 2 cm.
The period of the wave is 0.04 s.

r
LE~ flaw
A
:J
' ultrasound receiver
There is a flaw in a steel bar AB. A transmitter installed at end A sends an ultrasonic
pulse to end B. Two refiected pulses are then rec,eived by a rece~er also installed ~l
A. (1) only end A The reflected pulses are received 1.5 x 10 s and 2.0 x 10 s after the pulse rs
B (2) only emitted.
c (1) and (3) only What is the distance of the flaw from end A? (The speed o( sound in steel is 5 200 mis)
D (2) and (3) only
A 0.39 m
B 0.53 m
C 0.76 m
0 1.04 m
Melhodist Girls' School Physics Paper 1 Sec 4 Prelim Exam 2010 Methodist G1<ls; School Physics'"Paper 1 sec 4 Prelimexam2010
!,\'\
Page 8of17
Page 9of17
21 A loudspeaker is connected to a signal generabr as shown in the diagram below.
23 P, a, R and s are four objects placed al the same level above a plane mirror of length
1) cm as shown in the figure.
loudspeaker
... .......... --. ::. ------- .: .... -- --d~ - ---R~ ........st.
p,
!
x. ~
' '
'
: :
. .
... ---~-- ------- ~- - ... ---- '-
mnmm;>I~ -- ~ cm 4 cm ~ 4 4 cm ~I
The output frequency of the signal generator is increased from O Hz to 20 kHz while
the output amplitude is kept constant. Which of the four 0 bjects can be seen in the mirror by an observer al position X?
Which of the following graphs best shows the varlatlon of the loudness of the sound
emitted by the loudspeaker with the frequency as heard by a normal hum'an ear? A p and Q on1y
e Q only
A f B c Q and R only
0 Rand Sonly
Loudness Loudness
24 lhe following diagrams show the refracton of light in four different liquids.
Which liquid has the highest refractive index?
~~~~~~~~~~~~~~----,
A .B
air
20 kHz
~Hz
Frequency Frequency
c
Loudness Loudness
I
I c
I air olr
l
20kHz Frequency 20kHz Frequency
22 Which of the following statements concerning the image of an object formed by liquid (3)
renection in a plane mirror is incorrect?
A The distance of the image from the mirror is the same as the distance of the
object from the mirror.
B The image can be formed on a screen.
C T he image has the same size as the object
D The image is erect.
Physics Paper 1 ----- s-ec 4 Prellm Exam 2o10
Methodist GirlS' School Physics Paper 1
~1.\
Page 10 of 17
Page 11 of 17
25 F and F' are the foci of the converging lens. C and C' are both al a distance of two
times the focal length from the lens. ..
Two uncharged metal spheres, not touching one another, are suspended by means of
27
' cotton thread. A positi'1ely charge rod is brought near.
c
I I
F
I [;
I
F'
I
C'
l
I
... + + + +
t1:=::::
++
+ ... +
Which diagram shows what happens to the spheres?
In order that a magnified image of a luminous object is formed on a screen, where
should the object and the screen be placed?
.. . ' ~ ._ r: ~ . -
;...-P,os ition'Of''obj eet ~ .,, Position oi screen -
A Beyond c Beyond C'
8 Beyond C Between F' and C' + + + + t .. + + .. +
c Between F and C ., Beyond C' ti::::;::~:: ti: ::=:.
+ + ... + + + + ++ +
D Between F and C , Between F' and C'
c D
26 The diagram below shows an experimental set-up to study the inage formed by a
converging lens of focal length 15 cm. The object is an illuminated arrow " '-.~ placed a
distance of 30 cm away from the lens. The screen is moved to obtain a sharp image.
lens
.,,;- -- - --
! I
Which of the following diagrams shows the shape of the image on the screen as seen
by the observer?
(f ;1, 3
)
I
I
~(4) I
x------- Q _J
The diagram above shows an electric field line XY and a positive charge _Q placed on
the field line. The d irection of the electric force acting on Q is best represented by
c D
A (1)
B (2)
15 (3)
D (4)
Methodist Girls' School Physics Paper 1 Sec 4 Prelim Exam Z01f
M~ihodist Gi;ls' School Sec 4 Prelim Exam 2010
Page 12of17
29 The e.m.f. of a dry cell is 1.5 V means that
Page 13 of 17
A 1.5 C of charge passes through the cell in 1 s.
B The cell appl'es a force of 1.5 Non each coulomb of charge passing through it
32 l'he diagram below shows the circuit formed when a rice-cooker is plugged into a
mains socket.
C The cell supplies 1.5 J of energy to the charge passing through it in 1 s.
'Ci
CVm~
The cell supplies 1.5 J of energy to each coulomb or charge passing through it.
2.1 kW healing element
30 live wire
At which point in the circuit is the current the smallest? ------,
'
l supply
Neutral wire
B l 10A fuse :'____r______________
. ~ ! .
I Metal case
I There is one fault in this arrangement. The fault is that
0
A the earth wire should not be connected to the metal case.
31 The diagram shows the circuit for a hair-dryer_ e the ruse value is too low.
~
--' ----- - --
the ruse should not be in the neutral wire.
0 the switch should not be in the .ive wire.
fan
33 which of the following diagrams correctly shows the pointing direction of a compass
p when it is placed near two bar magnets as shown?
A B
Q
heater
::::::JJ
heater
9N
The fan has a power rating of 0 . 1 kW and the heaters each have a rating or 0.4 kW.
The cost of electricity is 8 cents I kWh.
What is the cost of running the dryer for two hours with switches P and Q closed and
switch R open?
:::m
c
=:sJ 0
=~=:Im
A 1 6 cents
B
c
0

3.2 cents
6.4 cents N
8.0 cents
(D
N
L_ =:w
1od1st Girls' School
Physics Paper 1
Sec 4 Prelim Exam 2010
Methodist Girls' School - - -- - - ----~ --
Pny&ics Paper 1 ' - - - - -- ----so:e;;:c:-:4;-pD:r;;;ielim Exam-io10'"
Page 14 of 17
34 An object X is brougtit near (but not touching) a bar magnet which is placed on an Page 15of 17
electronic balance as shown in the diagram_ X may be
36 The diagram shows a brass rod and an iron rod beside each other at lhe bottom of a
(1) a bar magnet; rectangular coil.
(2) a iron bar:
(3) a copper bar_
r- --- - - -
1
i
I I
I

rectangular
co l
~rass rod iron rod
What happens when a direct current passes through the coil?
A Only the brass rod is magnetised.
B Only the iron rod is magnetised.
If the reading of the balance decreases, what can X be?
C The two rods attract each other.
A (1)only O The two rods repel each other.
& {1) and (2) only
37 In diagram 1 below, a galvanometer deflects to the right with a reading of 10 A
C (1) and (3) only
0 (2) and (3) only
35 A bar magnet attached to a spring balance is hung vertically above an electromagnet
as shown in the diagram below >
r ------ - --- -- - - - ,
I
1
l
The north pole of the same magnet Is now pushed towards the same coil from the
bar magnet other side at a faster rate as shown in diagram 2 ab~ve.
Which of the following statements is correct?
A The deflection on the galvanometer will be less than 10 A to the left
B The deflection on the galvanometer will be more than 10 A to the left.
c The deflection on the galvanometer will be less than 10 A to the right,
D Tl\e deflection on the galvanometer will be more than 10 1iA lo the right
What will happen to the reading of the spring when switch S is closed?
A The reading will be changing periodically.
B The reading w ill decrease.
C The reading will increase.
D The reading w ill remain unchanged
Physics Paper r- Sec 4 Prelim Exam 2010
?>2.-=\ ~Physics Paper 1
Methodist Gir1s' School
Page 16of17
Page 17 of 17
38 In the diagram below, the portion XY of the soft iron core of the transformer is
removabl e. 40 A simple a.c. generator produces an alternating e.m.f. as shown.
0 -
-
..
.o I \ I \
>
~ 0 1.0
Tlmo / 1
L _ -\1/0
J -
'.S:
I .0 --- -
---1 --1-
I
-2,0
Which of the following Is correct when XY Is removed?
A The brightness of the lamp defreases. The speed of the generator is doubled.
B The brightness of the lamp increases. Which graph best represents the new output?
c The brightness of the lamp remains unchanged.
D The lamp stops lighting. A e
39 The diagram shows an a.c. generator.
Coll
.......
c 0
I
l___,__:____ 0,., _ __
Which of the following statements is true?
A Lncreaslng the speed of 1he coil increases the induced voltage but not the
f requency.
B I ncreasing the speed of the coil increases the output frequency but not the
induced voltage.
C The induced voltage is zero when the coil is at 90i to the position shown In the
diagram.
D The induced voltage is zero when the coil is in the position shown in the
diagram.
End of Paper
uxlist GiMs' sciloor------ - Physics Paper 1 Sec 4 Prelim Exam 2010 Methodist Girls' School Physics Paper 1 Sec 4 Prelim Exam 2010
~1'\
Methodist Girls School
Secondary 4 Express Page 2 of 17
Physics Paper 1 Section A [50 marks] Page 3 of 17
Answer all the questions in this section in the spaces provided. F
' ..
2 below shows part of a prefabricated bridge structure being lifted into pos1llon.
2
A skydiver of mass 70 kg jumps from a plane. The graph In Fig. 1 shows how the
T~g~ structure Is made of cone<ete and steel. It Is held in position at each end by. two
cranes. The structure has a mass of 300 000 kg. The centr~ of mass of the bndge
vertical velocity of the sky-diver varies with time. Parts of the graph 1ave been labelled
s\Jucture is :;.o rn from one end and the length of the structure 1s 25.0 m.
A, B, C, D and E.
Take g =10 Nllc;g
40
30
20
I 10
l (a)

0
0 10 15 20 25
Fig 1
JO J5 40 4.5 50
Time ls
Al point A on the graph, the skydiver has acceleration equal lo the acceleration
due lo gravity of 10 mis'. (a)

Fl . 2
What is the weight of the prefabricated bridge structure?
Calculate the resultant force acting on the skydiver at this instant.
Weight,, _ _ _ _ _ _ {1)
(b ) Draw and label on the diagram, the tensions T,, T, in the cables of cranes 1
Resultant force= - - - - - - (1) and 2 respectively. [ 1I
{b) (i) Describe the type of motion the skydiver is experiencn g from B to C. Calculate the tensions of each cabte.
(<::)
(1)
(ii) What is the resultant force on the skydiver between Band C?
(1)
(c) Describe and explain the motion of the skydiver from C until she lands at E [J)
- - -- - - - - - - - - - - - - - - ------------
---------- - - - - -------------------(2)-
(d) Another skydiver, of the same mass, falls from the same height and deploys her
parachute after the same duration as the earlier diver but uses a parachute with
a smaller surface area. On the same grid in Fig. 1.1, sketch a graph to show
her motion. (2)
MettiOclTst Girls' School Physics Paper 2 Sec 4 Prelim Exam 2010
Methodist Glrls' School
Page 4of17
Page 5 of 17
3 Fig. 3 Siows a tube dipped into a bowl of water. Water has been drawn up the tube by
a pump. 4 (a) Fig. 4.1 sMws a ray of light passi'lg thro_ugh water into air.
(Density o'f water = 1 000 kg/m'): Take g =10 N/kg.
Emergent ray
(i) Explain, in terms of the properties of light, why the light changes its
direction as shown.
Bowl Water
[1]
(a) Three points are labelled X, Y and Z. Which pqJnt{s) is/are at atmospheric
(ii] If the refractive index of the water is 1.35, calculate the angle made by
ores sure?
the emergent ray with the normal.
(1 )
( b) What is the pressure due to the column of water XZ.?
A n g l e = - - - - - - (2)
(b) The experiment is repeated using a semi-circular glass block as shown in Fig
Pressure=-- - - -- - [2) -~ .
(C) What is the pressure at X when the atmospheric pressure Is 100 kPa? Normal:
Pressure=---- - - - (1)
(d) The cross-sectlonal area of the tube is 5.0 x 10-s m2 Calculate the mass of
water in the 0 .20 m length of the tube.
Light raY.
Fig 4.2
(i) State why the light ray does not change direction as it enters the glass.
mass=--- - - - - [2]
[1)
thod1st Giris' SChOoi Phy$iCS Paper 2 Physics Paper 2 Sec 4 Prelim Exam 2010-
Se<: 4 Pretim Exam 2010 Methodist Gl~s School
?,3,}
I ., j -~ .
Page 6of17
(ii) Explain why the light ray does not emerge from the straight edge AB of Page 7 of 17
the glass block.
6 Fig 6.1 and Fig. 6.2 show two view\ of a vertical wire corrying a current up through a
horizontal card. Points P and Q are marked on the card.
lr..t..-
[1]
Q
(Iii) The _glass has a rnfraclive index of 1.60. Calculate the largest angle that
the light ray would need to make with the normal to allow the light ra'f to
emerge from the s:raighfedge AB of the glass block. P, J-----1--
l :
JC vertical wire
Angle= - - - - - - [2] ~----......:...F~i9~~6.~1
1
_ _ _ _ _ _ _____ F~lg~._6_.2
-omoow.o~"~
_______
5 Fig. 5 represents waves being produced in a ripple tank.
(a) On Fig. 6.2,
(i) draw a complete magnetic field line (line of force) through P and indicate
Ripple tank A B its direction with an arrow. [11
(ii) draw an arrow through Q to indicate the direction in which a compass
placed at Q would point. [1]
(b) State the effect on the direction in which compass Q points if
(i) the current in the wire is increased,
[1]
---- -------- ----
Wave
(ii) the direction of the current in the wire is reversed.
generator Direction of waves----;,. [1)
Wavefronts
Fi . 5 Fig. 6.3 shows the vlew from above of another vertical wire carrying a current
{c)
up through a horizontal card. Point W is 1 cm vertically above the top surface of
(a) Calculate the wavelength of these waves given that 1.0 cm on lhe figurt:
represents 5.0 cm. the card.
venical w11e earrymg
Wavelength= _ _ _ _ _ _ (1) current
{b) It takes a wavefront 1 second to travel from A to B. What is the frequency of the
waves? m
1 cm
Fig. 6.3
Frequency=-- - - - - (1) State the magnetic field strength at S, T and W in terms of the magnetic field
strength at R. Use one of the alternatives, weaker, same strength or s tronger
(c) Calculate the speed of the waves.
for each answer.
at S -- ----------
Speed"-- - - - - [2) at T
(31
M\'thodist Girls' School Physics Paper 2 Sec 4 Prelim Exam 2010 atW
Methodist Girls' Sctiool
Page 6of 17
Page 9of17
7 Carissa wants to use an electric ~ettle to bo~ water. Some . f
manufacturer of the electric kettle Is glven below. in ormation from the Fig. 8.1 below shows a circuit whose output potential difference. VAiR varies with the
s surrounding air temperature. Fig. 8.2 shows how the resistance of the air temperature
1 &
--..,_.) I Specific latent heal of wp<>riulion of steam produced
. Rate of steam escaping from kettle

- - - - -

-~~
~~_sis l
sensor varies with temperature.
Rale of energy loss throu~ the surface of lh~ke;tl~~~ri;;t;.;i;; 50J/s _j Resistance I 0
e.m.f. of a.c.supply
.-.1..--~~;_:_ _ _ __ 220V j
(a) EKplaln briefly why the surface of the ket:le is chromed silvery.
6.0V
- - - - -- - - --- - - - - air
[1 )
'----------------- temperature
sensor
(b ) Explain why the filament is nfot placed high up in the kettle.
Fig. 8.2
Fig. 8.1
(11
Name the input transducer that can be used as an air temperature sensor.
(a)
11!
(c) Calculate the power of the filament of the kettle.
..
(b)
Show that at a temperature of 20 c. the potential difference across the air
temperature sensor N,.,R) is 4.0 V
[21
Power of the filament = 131
(d) If Carissa wants to increase the power of th fil1
or use another longer one? Explain. e ament, should she cut it shorter (c~
Calculate the current in the circuit when the air temperature is 20 c.
Express your answer in units of mA.
C u r r e n t = - - - - - - (2)
--------------------~ How much power is dissipated in the 1 000 0 resistor when the air temperature
(d)
is 2o'C?
Power=---- - - - 121

)disl Girls' School Physics Paper 2------- ..- - -- - --
Sec 4 Prelim Exam 2010 P~ysics Paper 2
"?>o~ Methodist Girls' School
Page 10 of 17
Class: _ _ (
Section B (30 marks)
A nswer all the questions from this section_ Ques lion 11 has a choice of parts to answer.
9 Life on Earth is made possible by the energy from the Sun as electromagnetic
radiation. We use this fonn of radiation in many applications from micro () wave
transmissions in radar to gamma (y) wave emissions from nuclear power generation.
The most familiar part of the spectrum is light, the part that is visible to our eyes. Light
occurs over a band of frequencies, as do all other phenomena in the spectrum such as
ultraviolet radiation. Shown below is a table of the spectrum with frequencies and their
associated phenomena.
Phenomena y X UV vis IR UHF VHF
Frequency I Hz 1021 1018 10' 10" 1 o" 10 10 109 109
(a) Calculate the wavelen~th of infra-red (IR) or thermal radiation, given that the
speed of light is 3 ~ 10 mis.
[2]
(b) The energy in the spectrum is directly proportional to the frequency. How many
times more energy do UV rays at one end of the visible (vis) spectrum have,
compared with IR rays at the other end?
[1)
(c) -Ultraviolet (UV) radiation is recognised in three parts, each with its own
wavelength. There are:
UVA= 315-400 nm;
UVB = 280-315 nm and
UVC 100 - 280 nm.
The ozone layer in our atmosphere .most!~ absorbs UV radiation in the
frequency range 3.0 101 Hz to 1.0 10 5 Hz. Which parts of ultraviolet
radiation reach the Earth's surface?
[3)
Met110dist Girls' School Physics Paper 2
Page 11of17
Page 12 of 7
(d) The solar constant, or amourt of radiation received from the Sun per second at
the top of the atmosphere, is measured as a unit of power P, 1357 W. How Fig. 10.1 shows the essential features of a home-made d.c.motor. WXVZ is a wooden
10
much energy would this be per 24 hours? frame.
-- - - -- - - - -----,
{2]
17% scattered
I ""'
Solar constant,
1357W 33% reflected &
absorbed
30%of1Rlo
outerspace
I
I 11% reflected
I
--- /:
70% oflR
absorbed
{e)

l MAUW
50% received
WWWSk Ubi44 HZ
50% of the solar constant actually reaches the surface of the Earth. Of this
amount, 11 % is reflected ,baek into space The rest is absorbed by the surface
and emitted as Infra-red radiation. Only 30% of the emitted IR goes directly Into
out to space. Find the 11alue (In Walls) of the balance absorbed by, and
available for, heating the atmosphere.
Fig. 10.1
(a) What is the polarity of A?
(1]
{2]
Whal would the direction of rotation of the coil WXYZ as viewed from the front
{b)
be?
(1}
{c) wtiat is the function of the brushes?
(11
(d) Stale the effect of replacing wooden frame WXYZ to a soft-iron cylinder.
ii<list Girts' Schocii- Physics Paper 2 Sec 4 Prelim Exam 2010
Pa!;Je 13 of 17
Page 14 e>f 17
If the resistance of the variable resistor is increased explain and
(e)
change in tne rotation of the motor. state the 11 EITHER
"
t Fig. 1 1.1 ~hows a cylindenhat contains gas with the same pressure as the air outside.
The piston is held at its present position throughout and no gas escapes.
[3]
(f) lhfthe ba ltery risthreplaced by a 50 H:i: a.c. supply, state and explain the change in
t e ro1a1ion e> e mote>r.
Explain how the molecules of the gas inside the cylinder exert a pressure on the
(a)
pressure sensor.
_,_________ _ (2)
- - - - -- -- - - --(2_)_
(g) State one advantage of using electre>magnets instead of permanent magnets.
The cylinder is cooled from BO c to 30 c. State and explain how the values of
(b)
each of the following quantiles changes after cooling.,
(1]
(i) The average speed of the gas molecules.
[1 )
(Ii) The average dista'rice between the gas molecules.
(1)
(iii) The frequency of collisions of the gas mi lecules with the walls of the
cylinder.
11I
(vi) The pressure on the pressure sensor.
(1]
Methe>d1st G1~s sc~--- ----- -o~---
Physics Paper 2 ~ysics Paper 2
Page 15of 17
(c) Describe the use of a manometer in the measurement of gas pressure.
In your accoun~
give a labelled diagram of the apparatus,
state the readings that are taken,
explain how pressure of a gas can be obtained from the readings.
'
(4)
Melhodist Girls' School Physics Paper 2 Sec 4 PreIIm Exam 2010
MGS 20 1U Prelimi ru.ry E)(aminatlon -Physics 5058
Secondary4 (b) Pressure = height x density x g
PapEr1 = 2.0 x 103 Pa
(c) Pressure at X =9.8 x 10 4
Pa
(d) Pressure = (mass x g) I area
= 0.01 kg
4 (a) (I) Speed of light increases from denser medium (glass) to less
dense medium (air)
(li) sin (unknown angle) I sin 45 =refractive ind~x
sin (unknown angle) =1.35_x sin 45
unknown angle =72.7
(b) (l) Angle of incidence as it enters glass is zer
(ii) Ang le of incidence in gl<i$S at 45 >critical
(iii) Largest angle =critical angle
{a) Resultant force criticai"angle =
38.7'
5 (a) Wavelength
(b) (I)
(b) Frequency
(ii)
(c) Speed of waves
(c)
6 (a) (i)
(d) (ii)
2 (a)

(c) Ta~ing"""'~
~Z!j~17 aboul_A; 7 (a)
Ant1clockw1s~moment = Clockwise moment ater will be able to sink to displace tt1e less dense,
(b)
T 2 x 25.0 m = 3.0 x 106 N x 5.0 m
T, =6.0 x 105 N
Since Power = (mass of water x latent heat) I time + power loss
(c)
T, + T, =3_0 x 1066 N = 4 x 10- kg x 2520 x 10' J/kg s + 50 J/s
T, =2.410 N
= 1 060W
3 (a) Y&Z (d) Since p = V 2 / R, the power of the filament can be increased by cutting the
filament shorter
to reduce its resistance.
2
8 (a) Thermistor 11
(b) =2000 0 Either
=4 v
(a) Air molecules move around randomlv at high speeds, and collide with the
(c) Current = voltage across circuit I resistance of circuit wall of the pressure sensor,
= 2.0mA they exert a force on the pressure sensor per unit area upon collision,
hence exerting a pressure on the oressure sensor.
(d) Power =(Current through 10000 resistor) 2 x Resistance
= 4.0 x 1cr3w (b) (i) Average speed of gas molecules decreases as temperature ~reps
OR thermal energy is removed, lowering the average kinetic
Paper 2 energy of gas molecules
Section B (30 marks)
(ii) Average distance between gas molecules re
9 (a) Wavelength of light =speed of light I frequency since the number of molecules and vol~
=3 x 10.s Hz the same.
(b) Factor difference =100 {h1) l'requency of coilision with the walls of
because average speed of the gas mole
(c) Range of wavelength of UV radiati~n absorbed by aim area of walls of the cylinder remains the sa
from 1x 107 mto 3 x 10'7 m
Parts of UV radiation that rea (iv)
Mainly UVA and
UVB > 300 nm (3
(c) Diagram;
(d) Brief description of proc
Relevant readings to 'b
How readings are pro
(e)
OR
(a) (i)
10 (a)
(ii)
etic force acting on the coil is reduced. As a result, the
f the coll decreases.
(f) If the battery is replaced by a 50 Hz a.c. supply, both the current through
the coll and the magnetic field In the solenoids are both reversed at the
same time.
So the direction of rotation of the coll will remain unchanged.
(g) The motor can work in both a.c. and d.c. OR magnetic field strength from
magnet can maintain or stronger when electromagnet is used.
3
:,t;\
~arne: Register Number: Class: Which of the lollowing is correct?
A.
Scalar quantity
mass
Vector quantlly
speed
.
s. fo-ce displacement
~
J. temperature acceleration
D. kinetic energy distance
2. The bob of a slmple pendulum Is displaced to one side and released. T~e ~lion
during its swing is shown in t he graph. What is the frequency of lhe osctllaliOn of
Nan Chiau High School the pendulum?
Outancc frompoin< ofrer..sc
Preliminary Examination Three 2010
PHYSICS 5 058/1
Secondary 4 E~press 1 hr
MaximLm Marlls: 40 14 Sept 2010 Tuesday
,'f4N~Nt.t..' Hf(.}( SCJIO<JLl(All 1.'WUU fll(Jlt :110tX 1>0,.,WC'.Jf':.tt:H"1tl 1a#O<H. .VA.Y .:J'h.4U ln)JIJCJl()o ltA..wC.Ml.C' 11/~JflC'jll'}(lt HAIC'll1'li HJGH~/I 0 l.0 2.0 ) .0 4.0
,lqltClf/At..'ltlOllJ('KO()(lll.C.4.11&dllOGHKN004~0fll.#('llJOtflOfOO(/f.AYORAl'/W'~$("JIO<Jf.Jl,tl(OllWl_,SOll00&.1'N'C"an."U/Gll$0/l004.JI
14NC#IM' JtlOIKMOO(/f,Alf Ol&IVJOQI J('lfOOt lt.MIQICkC.'l/IOflJtM.tO(.Jt.4V<MfU #l~lt.#CJfo,IP_,.a'li01'411CMkC:IDGR~JI
1C4)11"1CAS"MltJt.SClllOOll.l!Jl'C.UU' wr;M SCMVOC.~tlAICIOClt '"''A:'ft:M4U .llP#K900lJilllll<"Mtl' .-.~W,,:CllilMU#ICll IOf!UOLll A. 0.25Hz
1Wtc...-KIOl~M"OIMUlflGRSOJOHAJICXUli.wlHl'K'lfl'*.llAltCHWD'lltt:llSOl'QOMt1an..C1HJltlOJrOOl.1'NIC'ltJACl1t/QllOICKJUI
"JtllClllM llK.lt,JQIOQ( /W>'C11't..U /11!;/f IC1l'OU&. /ll,4,'l(dMU'mt;ll lt'7fOOI. """'C'm.tV lf1C# ~NNlcm.v IVO.r.Hc;l)(.HMIC'MA(;1tlCH ICN'OOC."
1'111.KClfUll #1Q; IOIOOI.. ,;AJl(.'.l(U(I H;G.Cf ICltOOL>IAllCNIACI RIGlf~HNIC"llUV If~ J<:lrOOLIJAH("MArr HIOH ICHOOl.>IANC~V1'1CH l<ilflOL II 8 0.5Hz
l'(4Jlo t;l((,til JfKM IOIOOI "411 Clt.l.W Hl(JH ICll">Ol.l>'AHCHUUH!'JK IC'NOOL!IAH Cblofl.I Hffl.llU'll"'!>1A...' CJG.!l.I HIClH SCHOOi, NA/lltlatC:ltJVll KllO(X.#
c. 1.0Hz
INSTRUCTION S TO CANDIDATES 0. 2.0Hz
Wnle your name. class and index number al lhe top ol lhis page. A r has an oil drip As the car moves. ii drips o~ a't a regular rate. leaving a trail
3.
1~~ls on the road: Which of the followlnsi diagrams of lhe cafs trail of spots
There are 40 queslions in this seclion. Answer all questions. For eec::h question, there ~hows the car continuously slowing d1;1Nn?
are four possible answe1S. A, B, C and D. Choose lhe one you consider correct and
Si
.record your choice in soft pencil on the Op:ocal Answer Sheet (OAS).
INFOR MATION FDR CAN DIQATES

A.
[:, _ ____._._
_;_
Fil<b
....)
f'mbh
Each cor-ect answer wia score one marll. A marl< will rot be deducted for a wrong B. SI'' ___ i
answer. Any 10ugh working should be done in this booklel
Answer all questions. v---... 3
2
Take g "1Cmls 0t g" 10N/kg unless otherwise s<ated.
c. $ta>1 Fim:t.h
I . . ... . . .I}
Setter. Ms Elisabeth Oh
D. S<ort I
F1Msh
This question paper c onsists of 13 printed pages. lncludlng 1hls cover page. I
..l}.- .-. - -- - -.}
2
4 ihe
!.!"''}:.
velocity.time gr~ pti above repre$ents the motion of a particle along a straight

7.
An object X of ma~s 3.0 kg moves up a rough slope with an applied force of 30 N
a
from pointP to point with a uniform speed as shown in the diagram below.
-,~~
10
~ 8rn
p
- 10
The frictional force i s - - - -
20 ....... . .... .
A ON c. 18.0 N
B 12 N o. 22.5 N
Which of the following statements are troe?
=
I. The part~le's veficiry is fl'IOmentarily zero at t 5 s. , 8.
Ten identical steel balls were Immersed in a measutl:lg cyilnder containlng 20cm'
11. The particle comes ba<:k lo i\S starting point in 12 s of water. If the mass of each sleet baD is 27 g and the dens1\y of steel is 9 glcm.'.
Ill. The parti.cle Is furthest away lrom the starting poini at t 12s = what is \he new reading of the waler le~et?
IV. The particle travels n t~,e opposite direction between t =5 sand t = 12 s.
A. 3 cm' c 30 cm'
A.
B.
I and IV only
II and Ill only
c.
D.
I. II and IV only
I, ~I and IV only
B- 23 cm' o. 50cm'
tn 1116 diagram below. an object Is placed on a slope. Given that the object did
9.
s. Two forces X arni
Y acting !>n a ban produce a resul!anl foroe Ron the ball not topple over. which is the most likely position of its cenlle of gravity?
Which of the foitov.ing diagrams correctly shows lhe resultant force R?
. V ~
y
~.
c 0
10.
Which of the following does !lQl affect the pressure. beneath Ille surface of a
liquid?
A. Density of the liquid
6 Depth of lhe liquid
6. A t~y cart of mass 1 l<g moves wilh a constant velocity of 2 rrJs to the right 0 a c Strength of the gravi:atlonal field
D Surface area of the liquid
hon:tontat table. Which of the following statements Is correct ? ' n
A The !able exer1s a force of 1O N upward on the toy cart
B. The toy cart exens a force of 1 N downward on lhe table.
C There Is !rictlonal force of 2 N acting on the toy cart.
O. There ls a resultant force of 2 N to the right on the toy car1.
4
3
11. A manomelor Is used lo measure the pressure of a gas supply as shown Jn the
diagram. Given !hat the .atmospheric pressure Is 76 cmHg. what is the pressure 15. Which or the lollowing statements bust explains why drops of liquid rorm on the
of the gas supply? [deosrty of mercury: 13600 kg/m3; density o1 oil: 900 kgfm~ outside of a glass of ice water on a hot da'{?
. "
A. Water molecules in the air are all/acted to tl>e surface of the glass.
~~:i,vi--- r
8. Water molecules In the air l/ar1$fer L'lermal energy lo the glass.
C. Waler molecules in the ice water gain energy from the hot air.
30 cm ~ 28 en C'J Water molecules in the Ice water conduct through the glass to the outside
of the glass.
. --~~[-_ ...f____ ... 16. Ether is a highly volatile liquid. Air is blown into tho tube containng some ether.
Underneath the beaker of ether Is a smaD pool of water. Which of the following
'o11 correctly describes what happens?
A. 100 cm of mercury
B 98 cm or mercury
98 cm of oil
D. 77.5 cm or mercury
12. The diagram shows a simple hydraulic jack. Which of lhe folowing alterations will
enable heavier loads to be ~fted?
IOad
Ether Water
h)ld1'11UliC plaron w
nuld
A. Evaporales Evaporates
B. Evaporates Freezes
di ameter of piston w diameter of piston Z c. Freezes Evaporates
A. doubled , halved D. Freezes Freezes
B. doubled f remains the same
c. halved doubled 17. A hot object is added to a container of water al 'room temperatJre. The water Is
D. re mains the same halved continuously stirred while !he hot object is immersed in It. Which of the following
....,......,
temperature-time graphs best shows the temperature changes that follow?
13. A oas .has a volume of 1000 cm al 100 kPa Whal Is lhe pressure or 1hls ga$
when II ls C:Ompt'essed very slowly to 500 cm?
A. n ...,_,...
I~ 1~
A. 500 kPa c. 100 kPa
a. 200 kPa D. 50 kPn
14. lo the Brownian motion experiment. the tiny smoke particles are observP.d to 't'w
......
move In a random zig-zag manner. This is due 10
A. the convection cu11en1 In air. 0. ,...,.._.
c.
1~ ~
8. the smoke par1icles being loo fight
C,
D.
uneven bombardmenl of air molecules on lhe smoke particles.
the average Internal energy ol the air molecules changing. ___-;:;:-
.
T-
"""'
5 6
18. A thermccqup:e lheri'l'Cmeter indicates 0.1 mV at ice point and 2.. ,
What Wiii be !he t&~perati,;e when the thermocouple .... "_mV at 500 C. Water waves travels in lhe direction shoWn.
mV? u ..imometerindicates 3.0
.... aoc
- - - - \lt3VC O<OHOO
9 sscc '
0.
ll!.
: :~ino plant mana.~r neech .tJJ mett 150C !o'.9 .,-: =ap e<>o1>er to seli 1o
melti manuf~urer. ;he
copper is at an Initial le.mpetature Ot1sc anci ils Which one of !he iollowing correctly shOWS 1he direction of motion of lhe !hree
te ng point 1s 1083 C. How.much ~rmat energy is required to raise the water particles X. Y, Z al the instant shown.
of~~~)' the copper to its melting point? (copper has a specific heat
x y z
A 6.2x 10'J
B. 6.3 x 1!11 J A. 4 ~ 4
C. 7.7 x 101 J B. .l. t .I.
O. 7.9x 1ctJ c. t l t
20. The energy r~<!Ulred to change ~quid water into water vapour al the
0. +-
- +-
The diagram beJ<?W shoWs a hammer about to strike a moveable piston.
temperature is c.alled latent heat ol V"'""risalion w.....do lh' same 23 .
....- - es is e.1ergy do?
A inc~eases the average speed of the water molecules
8. s~1ts the water molecules ln!o their separate atoms
c. ~rses the temperature or the air near the water
0. increases the average separation of the water molecules
21. The diagram
of 500 Hz. below shows a wave generator 11'.at emits
. a wave with a frequency As the piston moves suddenly when struck, which type of wave pulse is
generated in lhe air inside lhe cylinder?
A. electromagnetie
wCIW<""-~ -~ 0 .1 m f..-
~~~----- 1
8 transverse
c. refracted
D . longitu<final
24. Wh!ch of the following slatemenls best descnbes an electroma9netic ..;~ve with a
How long dof:s it lake for the wave lo travel a cistance of 2000.m?
long wavelength?
A. 20 s
f. It has a low fr.ncy and can travel in a vacwm.
B. 30s
40 s
B. II has a high frequency and can travel in a vacuum.
0. c. ti has a low frequency and can only travel 1n a vacuum.
o. It has a high frequency and can only travel rn a vacuum.
25. In an eJq>erirr:enl to measure the speed of sound, a student stands 40 m from a
wall and bangs two pieces of wood together. The moment he hears lhe echo, he
bangs them together again. He does this 50 times. The time taken lof 50 bangs
is 12 s. Which caJculation gives the speed o1 sourid in air?
12 40r50 40x2x50 4Cx2t12
A B c D
40Y.50 12 12 50
7 8
26 The diagrarn sN>ws a hoozontal ray of Nght Is Incident on an inc~ned plane mirror.
26. A positively charged rod 1s placed near the brass cap of an electroscope. The
mirr r
gold leaf fhps away from the brass plale because
fref}9ctea ray
"
brass rod
What ls t'le angle of incidence?
bran plate
A. 39 c. 52
B 45 0. 75
A. positiVe charges are repelled downwa1ds from the brass cap causing both
27. :wo uncharged melilt spheres are hanging by a thread as shown. A positive rod
1s now brought near it
the gold leaf and brass plate to be pos11Jvety charged and repel each
other.
B. electrons are repelled downwards from the brass cap causing both the
gold leaf and brass plate to be negaUvely charged and repel each other.
C. eleclrons are attracted upwards lo the brass cap causing both the gold
: :I leaf and brass plate to be positively charged and repel each other.
D. posiUve charges are attracted upwards to the brass cap causing both the
Which of lhe following d~ram shows what happens to the spheres? gold leaf and brass plate to be negativel1 charged and repel each other.
A.. c. 29. The resista~ of a cylindrical conductor of length L with a Cioss-sectlonal radius.
r, is R. The resistance of another similar conductor with a length of 2L end Y. r
I :~I dd - + :!Id6
will have a resistance or_.
A.
B.
1/8 R
1/2 R
c.
ll
4R
SR
6J D. 30. If a current of 10 A flows through an electric heater for an hour and converts 8.64
MJ orelectrical energy into heat energy. Then the potential difference across the
::I a . . . ,d b
.. t + -
heater is ___.
A.
a
664V
240V
c.
0.
144 v
100V
31. What will be the voltmeter reading In the circuh shown in figure below?
12\'
A 2..4 v

20
o_ 20

-lJllt--
-~
c.
f()
6.QV
B. 3.0V D. Mv
32.
We earth an electrical appliance to preveni - - - - - -
a soon circuit anij causing an eleclric shcck to the user
3!>. Preclse measunng . .
instruments o """'
1equire shock absorbers to eNminate small
of shod<
A. . affect the results of an expeTllMn1 "' ,....
8. lhe fuse from blowing vibrations thal can . ma net that repe:S 3 magne:IC platform
C. the metal casing of appllance from being charged absorber that can be used is afn lowielectro t~ps would provide the greatest hft lo
placed ariove il Which ol the ol ll9 se
0. excessive current from flowing the platform?
33. An electnc heater is rated al 2 kW. Electrical energy costs 8 cents per kWh. Wnat
is the cost of using the heate< for 3 hotJtS?
A. '.\\:a:,ixoc
pbtlon:>
r.. ~~~ ......
1t.;lonr.
34.

A.
B.
6 cents
15 cents
<;:
D
24 <:ents
48 <:ents
Tne diagram below shows the magneUc field of 2 magnets placed close together
with 3 compasses placed at 3 different locations.

to [f
~ ~l'f'I""'
f J)
B. ~Mxtic
- plJtform
[f ::1..
.J.
. . f

D -!-
nt through two wires. The currents
36. The f igure shows the oirect1ons o cur'te d and X Is equidistant from both wi res.
through both wires are equal In magn1 u e ~
Whalis the direction of L..,e magnetic field al point X .
Which set of compasses shows the correct directions of the compass needles?
A. B. c. 0. x
CDCD8 CDCD8 CDCD8 CDCD8
2 3 2 3
A:
&
Upward
Downward
2 3 2 3 ~ To the left
o. fo the right
$7. The diagram shows ~ be am of electrons entering a magnetic field. Whal will be
the direction of the field?
A. T awards the bottom of the page
~ T awards the top of the pag~
c. Out of the page
D. Into \he page
Region with
magnetic field
38. You are gilfen a bar magnet and a looped co~ of wire. Which or the following
...ould induce an em1 in the coil?
I. Moving the magnet toward the coil -
II. Moving the coil away rrom the magnet.
Ill. Turning the coil about a vertlcl.1 axis. ,,
A. I only C, I & II only
B. II onty 0.1. 11& Ill
39. In which dilection mus\ U\e wire be moved to induce a current as shown in !he
d iagram?
A. Vertically upward
8 Verticalydownward
C. Horizontally towards the south pole
0. Honz.ontally towalds the north pole
40. The diagram shows an a.c. generato<. Wilen the speed of the rotation of the toop
Is doubled. how does the penod and maximum voltage of the ovtput e.m.f.
change?
period maximum voltage
A- halved halved
e halved doubled ,,,..
c. doubled halved
0. doubled doubled_..,
13
,....;!ctioo - ...,,, i.Jcturcu ""'u":;.'1ions l:>U-marks) (b) It is eslimaled :hat. dun.19 the skid, the magnitude ol the deceleration of the car Is 8.5 mis'.
Answer ill! questions In the spaces provided. Determine, 131
{I) the speed v of the car bef0<e the brakes are applied,
-
1. A truck Is rolUng down a very sroght iOcline as stlown It travels at consl.arit speed.
<> D
(a) Explain why, although the truck Is on an Incline, It does no; accelerate. [!) V"----
(1)
the lime Interval between the hazard appearing and the brakes being applied.
(ii)
(b) Next,there
why lhe slope
is now the incline is Increased. A s a resu1Iof lhs, the truck now accelerates. Expl.,in
ofaccelerallon. [1)
time mtecval = ___ _
2.
~ ~:.:0~1;"U: :::'1~g,!=I~ al~'!~alght road atspeed v. A hazard suddenly appears (c)
(iI
Hence, determine.
the tolal amount of thermal and souod enetgy produced due to the braking as the car comes to 121
coming lnlo oper~tion, !he car,::,",,, lorwar~ 'z-d::rd appeatlng and the brakes on the car a halt,
force applied, lhe front wheels of the car leave skid ~ of 29.3 m. With a constant braking
illustrated in Fig. 2.1. ma 5 on the road lhat are 12.8 m long, as
-nl;-B
po9lion ol car
'16dmar1<
l total thermal ana sound energy = ____
121
the power of the brakes.
I 29.3 m
Fig. 2.1
12.Bni
(a} From the dinformation. p rovided s k e1ch a d.rstance - time graph of the cal's molioo from the ..
the ha73 121
' appears liU the car comes to rest ume power 2 _ _ __
3. One type of weighing machine, known as a steelyard, ls illustrated In Fig. 3.1. The two sliding
weights can be moved independently along the rod. With no load on the hook and the sliding
weights at the zero marl< on the metal rod, the metal rod ls horiz.ontal. The hook is 4.8 cm from
the plvol
me\11 rod
I
hool< 2.5 N sfiding wei!lht
Flg.3.1
' - ~tedi \ r,g .. 1
A sack of nour is su~9er-.;..~:~ !.!":m the hO.:lk )!'I 'Hder to return trie rn;,:~al rod to lf'le honz.ontal Some smoke particies are viewed throu~ a rrncroscopr., as !IJ.1'~ .. . r . o. .
5.
pos1fon. the 12 Nsliding wa.;;ht is mov~d 84 cm alor1~ !he rod and the 2.5 N weigh: is moved
72 cm. A
(a) CalC'Jlate the weight of the sack ol flour.
wei9htofllour"~--
.....
...... _ lT--- _.
-~
Fig. S.1

~uc.:11
Brownian motion is observed. Suggest ar.d eY,plain why Brownian motion provides evidenca for
121
(b) Suggest why this steelyard would be lrrpreclse when weighing cbjects with a weight of 111 (a) the movement of molecules as assumed in the kinetic theory ot 93ses.
aboct 5 N.
4. A su~marlne with a hatch cover of area 2.5 m 2 desGends to a depth of 70 m below \he surfaGe 121
tained e""lne contains la<ge panicles of soot Suggest why ll\c
of water. (density of !he water is 1050 kg/m'; atmospnertc pressure is i.O x 10' Pa) Calculate (b) Smo~e tram ~ poorly maJ" ..~
Brov;olan mollon of such large par\icles ;s undetectable.
(a) the p<usure on the-submarine at a depth of 70 m. (21
- The refractive lndcx of the glass
Fig. 6 .1 shows two raysoflighl DE and PO enterios the pnsm.
6.
of lhe prism is 1.49.
(b) \he fore:>- on the outside of the cover. 121
force" - - - - - fig. 6.1
(a) cAf"ain wh~ r<ayuE does nolundergcl"bending'I (1) The resistances P and 0 are 20Cv n and 50CO n respectively. A voltmeter is connected in
s. parallel with the 2000 n resisl.Or and ; therrr.lstor Is connected in parall4'1 with the 5000
n resistor as sl\oWll In F'g. 8.1.
{b) Calculale the angle or relrac1ion al F. [2]
6.0 v -\
~---~1 - - - 1-----~
20oon sooon
a119le of refraction ,.
(b) On Ftg. 6.1. complete the path of ray QR. (2)
7. 7 .d1 ofshowsl"'ht la graph of an el~trom':!lnetlc wave l'avelllng in a vacuum drawn to scale
Fslgpee.
( ~ n vacuum. c = 3.0 x 10 rrJs) Fig. 8.1
(3)
(a) Tt:e voltmeter reads 3.6 V when the temperature of lhe thermistor Is 19 c. Calculate the
res:Slance of the lhennislor at 19 c.
distaoco
(a) Usirtg Fig 7.1 , determine the frequency of the electromagnetic wave. (2)
Resistance of thermislor at 19 c = ____
(3)
State and explain qualllatlvely the change in the reading of the voltmeter as the
(bl
temperature of the thermistor is ralsecl.
frequency"
(b) Suggest the !we of eleetromagnellc wave shOwn. (l]
(c) Stale two possible harmful effects of electromagnetic waves. l2J
9. Fig. 9.1 shows a car battery baing cilarsed from a :~~O v a.c. mains suppl)'. [2]
j -- :- - +-----
' : I
l;...
200 V ;
! ~1J~
... . ,.
.I
car
battery
energy used: _ _ __
ll ~l
l '~ ,.-- - ..,u,..~J
Fog. 9 1
(a) INl"l'f is the~ necessary? 11)
(b) The average charging current is 2.0 A and the battery takes 12 hours to cha/ye fully. [2]
Calc~iate the charge that the battery stores when fully charged.
Charge : - - --
(c) The battery hai; an electromotiva force (em.I) of 12 V. Stale what is meant by 2n 111
electromotive force of 12 v.
idi) In lh space below, draw a circuit 01.,gr5m to show how lwo 6.0 V lamps should be IZ)
connected. to a 12 v battery so that both tamps glOw with noimai brightness.
(ii) Tile power of each lamp Is 8.G W. Calclla:e the current in the circuit. 12!
current=~---
specific talent heat of~ of ice : - . . -
Sf!> _ : LOnt_ _;ured ' - __;ns [3t - l tal rs of the sall'le si>:e containing lhe
Answer i.B QUes11ons in 1he spaces provided. , Fig 11 3 shows a temperature-time graph of two coo ne
(b)
Answer only one of lhe two altemalive questions in Ouesti~ 12. sa,;,e ~olume of hot water being lefl to cool In a laboratory.
temperature
10{a) Some crushed Ice at O c Is placed In a funnel together with an electric heater, as shown In Fig.
11.1.
fr====~~ supply
Fig. 113
(2)
EXJ>Jain why the lemperalure-time graplls do no\ have constant gradient
( i)
beal<ar------~ (2)
Fig. 11.I lime p11s for both conlainers and
Stale a reasoo for the difference in the temperature-- gra
(ii)
suggest one difference between two containers.
The mass of water coltectld In the beaker in a measured lnteNal of tlme is dete rmlned with the
heater switched oN. The mass is then found with the healer switched on. The energy supPled
to the healer is also measured. For bolh measurements of the mass, water is not collected until
melting occurs at a constant rate. The data shown In Fig. 11.2 are obtained.
Mass of water I g E.nergy suppfied to Time interval I min
-..heater I J
16.6 0 10.0
64.7 18000 5.0
Fig.~- --.-
(i) State why crushed Ice is used instead of large pieces of iee. [1
(io) Slate why the mass of waler is determined with Ille heater switched off. ["
(Iii) Suggest howl\ can be determined lhat lhe Ice iS meltM'lg al a constant rate. ('
(iV) Calculate a value !or the specific latent heat of fuslor of ice.
l 1(a) f he circuit in Fig. 11.1 shQws an eleclcomagnelic relay being used to ewHch an ele~'.ric mater
>n and off. The rela1 coil has a much greater reslstMce than t~e potential divider. (i) w~,e n the slid>19 contact is at N, the trac: on tha oscUloscope is a horizor,tal line passing 11'.
ph1olsd lror: through the cenlre of lhc screen. Explain.
a1maHJre
'
m

_, pover suppiy
urJ1 r--'"-
. GV lor molor lll
rf~L- r " ~ (ii) The sliding cmtact is moved f rom N to M. Sketch the new trace on the CRO in t he diagram
~[M); moour
belcw.
.. , , I ,. J
[__-L------R \ _
SWil t.'!: J "
' \
__J
~---"
relay corf.:
"
'\
(.ont~l\'.-15
Fig. 11.1
(i) The relay operates when there is a potential difference of 3 V across the coil. On Fig. 10.1, !11
mark the po~ition of the slider of the potential tii'<ider when the relay j11st operates.
(ii) Describe how the relay closes the contacts In the mntor c ircuit [31
The 10V battery is then replaced with a 20V. 50 H2' a.c supply. Sketch the new trace on the [2)
(iii)
CRO in \he dagram below.
State two ways of modifying the set-up so that the relay can operate even when t~e potential 121
difference across the coll ls lesser than 3V.
(b) Fi!J. 11.2 ~hows two lenninals Mand N of :l ootential divider (potentiometer) connected
10 a 1OV battery. N Is aJso connected to one cf ~h~ two Y-lnput terminals o: a cathode-ray
.
osc.llloscope. The otherY-inpot terminal is connected iv !~e sliding contact cfth6 ootentia:
divider (potentiometer). (time-base selling is 10 ms/div; Y-9<ii!1 is 5 V/div)
J__
10 v
__ ,_
Ell-
12(ai) Sound Is a loajlidinal wave-:-Explain, with the aid ~fa dagram, what Is meant by this iJ
statement '<
(ii) Describe how lhe reflection of sound may produce an echo [1
Focal length = - - - -
(in) Describe how an echo may be used to measute the depth or the sea. In your ac,;ounl, (3
llsl the apparatus/equipment required,
state the readings lo be taken,
explain how the dej>th of the sea is calculated from the readings.
You may use a stmple dial)ram to aid your descriptl~n and exptan~Uon.
(b) Fig. 12. 1 shows an object, 0 and the virtual image, I, formed by a converging lens. Complete \~
the ray diagram to determine the position of the corwerglng lens and focal length of the
converging lens. label the converging lens 'L'.
120015t~r;:~hrye ~;~:~~~~ ~hs~~~~~r;~~~~~~r~~ ;~:~,
[2)
OR
12(a) Describe an e~periment to show the lorce on a current~ariyin(I conductor. In your account. (1) The step-up transforme rth3s
(4) calculate the number o1 urns on '
give a labelled diagram of the apparatus,
transformer.
indicate the direction o! lhe force and
describe how you would moo ty your set-up 10 increase the magnitude of the force.
Diagram:
No. ol turns in secondary coll"' - - -
[21
Why is electrical energy transmitted ;at high vol\age?
(1i)
(b) Fig. 12.1 is a block diagram of an electrical energy supp~/ system. using the output of a coal
fired power station.
c 11 D 21 c 3f D
s 12 c 22 c 32 c
3 A 13 B 23 D 33 D
lfatl&m!,5.Si(,n c 24 A 3<\ B
powe< oulf,aiS ~:~up
oo.Aput step- oulput
r,
5 0
~
,,,
14
B 25 c 35 0
36 a
"""' ln:msotnt...olf al 32000V tr1'n&tormer
16 B 26 A
Al 11oov ai 240V L_ 6 A
27 A 37 0
7 a H 6
h,;o.12.1 8 0 1B c 28 c 38 ()
19 A 29 D 39 A
6
() Stale the type of vollage sup~y required In a lrans.'1>rmer. Explain. 121 20 0 30 B 40 ll
H> D
Name: Register N11mber: Class:
----- Section A: Str uc:tured Questions [50 marks}
Answer fill questions in the spaces provided.
1. down a very slight
A truck is rolling
-
. Incrme as shown. It travels al constant speed.
() ( )
Explain why, although Uie truck is on a~ Incline, it does nol accelerate. [1]
(a)
Forward force down the slope= friction force
Nan Chiau High School therefore no resultant force OR forces balanced
Next. the slope of lhe incline is increased. As a resull of this, the truck now accelerates. Explain [1)
Preliminary Examination Three 2010 (b)
why there is now acceleration.
Forward force down slope> friction force
PHYSICS therefore there ts a resultant force OR forces unbalanced
505812
SecoJJdary 4 Express
1 hour 45 m ins . . h d t oristant speed v A hazard suddenly
Maximum Marks: 80 2. A car of mass 800 kg is travelling alOng a stra19 I roa a c . d a ~ring and the driver
f 14Sep 2010, Tuesday
appears in front o1 the car. 2 ~ec?nds elapse bei:::~o~a~":"rdist:~e of 29.3 m. Wllh a
Jl.tNCK'-tUHIQ,,l(;ffOOl,NNti O(UU>l/etrK1"00tJt~Of1.AtllflUH SOJOf>L/llA.'I CllUI.' litr:JHKH'JOL1'.t"tCH/.I(/ HIGH~ KA.-<ICHL4U Jf.IGHSCJIOl}J, 11
JI~ CH!AtJ HIGNSt:Ht'W'Jlltl.AN CJIUtlHIQ.JISDMOLIWI OftAUIWHSOIOOl../1',AJI ClllAV1/IG>f.SClfl>OL /WI CHIAV Nl<;lfSC7t0t:M..IWI Q'Ltll NIGHSCffO()/. >o" applying the brakes. Durin_g this time, thhe calr mf~h car leave skid marks on the road that are
Jo'ANClfJAfJSfl'GKJClll'X:l..f.NAN Clntl(I HJCHSC1/ooLNAJICNt.IU llJGHSCHO()l.JJIOllAf!Jfl'(J>I ~IW'OIUUHJGJftOloot. Jl.A,"IQfJArl HIOH$0t()()L .v cor.stant braking force applied, the front w ee so e
}11.IJl~t:Jf)Olf SCIK>CLlN CkMIJHIOfl.k:NOO.tllAJICJtUlJllJQH:llOOLJlcJI OlltllJ/lfi'R JCJIOO(JIAJ<JCHU llk>GNsaJOo.t.Jl.l..'IQIUCI /11/G,,.SOIOOL>J "
HANC.W..lllfJ'GN1QIOQLN.1.JvClri.i,'Hlfl#~OQlHANCN~jJHXHfSCHOOLNA/IC-Ht41JJ/J<JKJ!CllrH)l;NA'tCH/ACl'KXttO/Oul,.'tA}ICIUAUH10HSOJ0"1,/'I 12.8 m long, as illustrated in Fig. 2.1.
H~cw:i.tr,,.K1N~OOL~CllUUHtr.NSC1lOOL~CJN14UHIGH~l'AJICJWr!JIJOJIKHOOL/IAJIC!tutl~JfSCffGOl.~~NJ.4t'JIJ(';NSCJfOOS.,'11
JJAJICHUUH,,;;t.satoo:.11,,wcff1AU~S01(J()LHAJldl1AL"HK;HSCHOOL1'A>IOU4UJllGH':CHl>.lt-"'C~tlHJGN~oL11.vtcNuUK!ti.'fJX:H(,J()L/ll
NAX~CH>Glt$0AA)J.ll,AJIC'Jf/Al!JiJUffKllOOJ..NAJtlC'll"4UHIOhKHOOLJ'..,,,_nf1,UJfn4HKHMl;llAJ10flAUHJt:N5Cf900L.'HClat(!lf/CH:r.tHOOLI# p0si'lioa ol car skid mark
v.~n hazar<l appearS
INSTRUCTIONS TO CANDIDATES J I
~~1~~~~--~~m~~~~r--1~2~.8~m::--;
Write your name, class and register number in-the spaces provided at the top of this paper.
Write your answers In blue or black pen.
29
Do not use highlighters, glue or correction fluid.
Section A: Short-answer Questions (50 marks] Fig. 2.1
Answer all questions in the spaces provided.
(a)
.
From the lnfomiation provided. sketch a distance - time graph of the ca~s motion from the lime (2]
Section B: Structured Questions f30'marks] lhe hazard appears till !he car comes to rest.
Answer fill questions in the spaces provided. For Question 12, answer only ~ of the tw
alternative questions. y, m _axes labelled correctly
disl.ancelm
INFORMATION FOR CANDIDATES 42.l 'h. m - a II three correct
All working must be shown clearly in ink. y, m - correct shape distance aod lime values
The number of marks is given In brackets [ ] at the end of each. question or part question. 29.3 _.,. ______ _
Unless otherv.;se stated, eXpress your numeric answers to 3 significant figures and take th '
acceleration due lo gravity to be 10 mis'. '
I FOR EXAM~NER'S uSi .
''
'
~
' - correct shape
y; 111
Setler: Ms Elisabeth Oh
j Total
-l<'-----!:--------. limcls
This question paper consists of 17 printed pages, including this cover page. 2
2
(t>) Oeternine,
(i)
Ille speed v of the car beloro the bra~e; are applied,
v =29.3/2 72cm.
"
position, L'ie 12 N shdir.g weight is moved e4 cm along Iha rod and lhe 2.5 N weight S moved
' fl)
v =14.65 mis (1mJ ';1CC8pt 14.7 (~i Calculate the weight of lne sack o! ffour. {3)
t1rn fer i.:C~!':~ principle of moments!
(ii)
lhe de ce:erAtion of the car when the brakes were app/1eU
w x 4.8 {2.5 x 72) + (12 x 84) !1m)
(Jf.
y, x 14 .65 x 1 = 12.8 (1m) W =248 N or 247.5 N [1mJ
1" 1.75 s (b) Suggest why this steelyard would be imprecise when weighing objecls with a wei9h1 ot (1)
accept 1.74 s If 14.7mls was used; ALLOW ECF abouts N.
a = o- 14.65 / 1 [1mj The sliding weights would be moved very small distances.
a"' 8.38 mis' 4. A submarine with a hatch cover of are~ 2.5 m' descends to a depth ot 70 m below the surface
otwater. (density cf the water is 1050 k\j/m~ atmospnenc pressure 1s 1.0 x 10'Pa) Calculate
d ecel eration = 8. 38 mis ' 11m)
(a) the pres~ure on the submarine al a depth of 70 m, (2j
P: Palm+ pgh
(c) Hence, determine,
(i) the total amount of lhermai and d 1.0 x 10 + (1050 x 10 x 70)
a hal~ soun energy pro1uced due to Ille bak1ng as lhe car comes to (1m!
121
'h m v2 = Y. x 800 x 14.65' (1 m) =835000 Pa [1m)
~ 85800 J (3sf) or 86400 J (3sf) (if s1udent used 14.7) [1mj
AlLOWECF (b) !he force on Ille outside of lhe cover. [2)
F = 835000 x 25 (1m}
(Ii) the POWGr of lhe brakes. F = 2090000 N or 2087500 N [1m)
8580011.75
12]
f1mJ Some smoke partlcies are viewed through a mi.:roscope, as iBustraled in Fig 5 . ~ .
49000 W (3sf) e r 49400 w
[1m] ALl.OW ECF
3. One type of weighing machine known as a . 1 d , . . -
weigh!!; can bo move~ lndepe~denUy along~':';:' ~s llustrated in Fig. 3.1. The two sr;c;nQ
welg~IS a1 the zero marl< en the metal roe :he I .I ;L~. r.o load on lhe hook and 1he sliding
tlis pivot. ' me a ,O!f !S hortZ011121. The hOok is 4.8 cm from
,,~1 ==: -rr':
UJ".....---
5rt~ c~il
Fig. 5.1
(a} Browni an mot ion Is observed. Suggest and explain why Brov,nlan motoon provides evidence for 121
the movement of molecules as assumed in the kinetic theory of gases.
Brownian motion is the unequat I unbalanced collision rates of the air
molecules on the smoke particles 11m1
This provides evidence that gas mole~les move in continuous, random
motion 11ml
(b) ~
a
Smoke from poo!fy ma#)ta;neo eoginE COntafns
Brownian motion of such large particles Is undet ~ particles of soot Suggest why tt.e
(2]
The largo smo ke particles a.-e heavier/ cc ~e.
Th" collislons d ue to the air molecul more massivelNive more ln&rtla (1m] (a) Using Fig 7.1, determine IM frequency or 1he electromagnetic wave. (3]
es cause ont1 sman . Student measures the wavelength a 3.0 - 3.1 m (1m)
or the amount o f anergy from the air art; movements/little acceleration
pprtlcles lo m ove vlslbly(1m) P cles are not enough to cause the heave soot V f x wavelength
f =
3.0 x 10 (im] I wavelength 1x10'0 Hz (1m)
6. Fig. 6.1 shows two rays of llgh1 nE and PO
of the prism is 1.49. enlring lhe prlom. The refrac.Uve u>el.,,; ul the glass (bl Suggest lhe type of electromagnetic wave shown. {1)
Microwave
s (c) State two possible harmful effects of electromagnetic waves. (2)
Shortening of lifespan/ premature aging I damage to vital moleculesl damage to DNA I
cancer
(Any two)
o. The resistances P and 0 are 2000 n and 5000 n respectively. A voltmeter is connected in
parallel with the 2000 n resistor and a thermistor Is connected in parallel with the 5000
Fig. 6.1 n resistor as shown in Fig. 8.1.
20000 50000
(a)
Explain why Ray OE does not undergo betiding?
The angle Of incldenc.. is 0. .
(ii
(b) Calculate the angle or refrac:~on at F.
1.49 sin 30 1.0 sin r
!2J
(1m) Fig. 8.1
r,. 44.2
(1m] (a) The wltmete1 reads 3.6 V when the temperature of the thermistor is 19 c. Calculate the [41
(b) resistance of the thermistor at 19 c.
On Fig. 6.1. complete the path of ray QR
The ray wlll be along the side B C With .arrow (1) 2000 I (2000 + (115000 + 1/RJ "') x 6 = 3.6_
7.
Fig. 7.1 shows a graph of an electromagnetic wave . . [1m) . (1m) [1m)
(speed of lighc in vacwm. c 3.0 x 10 mis) traveling'" a vacwm drawn to scale.
electric R =1a20 n [1m]
ne..
slrPf\g\h
(b) State and explain quafitatively the change in the reading of the voltmeter as the (31
temperature of the thermistor is raised.
distance
ras t emperature risesl, resistanc:e of Cthennistorl decreases t1m1
either resistance of parallel combination decreases
or p_d_across s kO resistor and thennlstor dec:reases 11m1
voltmeter reading Increases [1m) (Award only when explanation of correct)
Fig. 7.1
6
9.
Fig. 9.1 shows a car battery being charged rrom a 200 V a.c. mains supply. 10(a) Some crushed ice at O c is pla~ed in a funnel together woth an electri!: heater. as shown ill Fig.
10.1.
11 i- -- ~~; --;
J 1.) - -
. ' t/
200 V ?.:'. ' car
battery
l 1
"- j
5 "t "T
---- -----~
-------
(a) Why is the diode necessary?
Fi9 9. 1
be- - . -- -. -~ Fig 10.1
To chang e/convert the Induced altemaling current to a direct cur rent (1]
(b) The avera~ charging current Is 2.0 A d the b The mass of water collected in the bea~er in a measured interval of time Is dete'l'!lined with the
Calculate tile charge that the batte an attery lakes 12 hours to charge fully. [2] heater switched off. The mass Is then found with the heater switched on. The energy supplied
ry store:; when fully charged.
to the heater is also measured. For both measurements of lhe mass, water is not collected uni~
a " I x t - 2 x 12 x 3600 11 mJ melting O::curs al a constant rate. The data shown in Fig. 10.2 are obtained.
a" 86400 c (1mJ -Mass of w a g
l e +Energy
/ supplied toTnmelntma_11 m~il'
(c) The battery has an electromotive rorce (
electromotive force Of 12 v. 1) f y
e.m. 0 12 Slat e what Is meant oy an
>----,~-~-l--- 16.6
Heater switched off
~ater I J
---a-- I 10.0 - -
(1) Heater swi!_~64.7 18000 5.0--- - _J
12 J Of Work fs done by the battery j d Fig 10.:.? -
circ uit. n nvm9 one coulomb of charge round a complete
(di) (I) State why crushed ice Is used instead of large pioces or Ice. 11)
In the space below, draw a cl~uit diagram ID 5 h0 . Ins id e of large pieces c ould be w ell below freezing point
ccnnec1ea to a 12 y battery so that both ,w h".' IWo 6.0 V lamps should be
1amps 9.ow with normal boigh!ness. (1] OR smaller air gaps if piece.s smaller
OR batter contact between heater and Ice
Sho W>s iitles conneaion -of tw r OR t o ensure heat from heater o nly goes to the Ice
batterfrfm}. o. amps using th,_:c_o!Teet circult Ynil>cl for bulb arid
OR largei: surface area
DO NOT ACCEPT ice melts f~ter or oven heat distribution
(ii)
(Ii) State why l)le mass or waler !s determined with the healer switched off. Ill
The power or eacti lamp is 8.0 W. Calculate the current In the circuit. t o k now how much w ater m olts doe to heat gain from U't e 5urro undings
(21
P ~ Vx I Suggest toow it can be determined that the ice is m"lting al a constant rate. 111
8 "' 6.0 x I [1mJ The no. of droplets from funnel per unit time Is the same
I : 1.33 A (1m} OR same amounuvolume/mau of water colleed in every minute
(iii)
Calculate the energy used by the two lamps when bot~ are Iii r0< one hour. (iv) C:alculate a value for the specific latent heat of rusion ol ice. [3)
E =P " ! " 8 ~ 2 x 3600 [1 m]
(2j m i>'SS melted by heater in 5 minutes =
64.7 - I'/, x 16.6l =56.4 g 11m>
E =57600 J f1m) 56. 4 ;( 10- x L : 18 11ml HO ECF
L = 320 t.'..I k9-1 11ml
Section B : Long Structured Questions (30 marksj Fig. 10.3 shoo..'S a tem,:;:!rature-Ume graph of two containers of the same StZe contalnlng the
Answer fill questions In the spaces provided. same volume of hot waler being left tc cool in a laboratory.
Answer only one of lhe two alternative questions in Question 12.
8
w1nperatuta
1 "i(~) Th_: c.n:;un u1 Fi~. 11. a6t.OWS M -=t:l.t:Vt)t.tgrteu.... .........j be!f't\i ---~~swncrr..n'l"'-...ctrlc n.
r.11 m1.:l off. The refay ~oil has a mu~h ~re~ter resislancc than the potential divider.
pivoted icon
cum~ture
1 power supply
'r:~~ _rm.r~~1'
-11,___ __._~__j
_.,......,
~\,"" . ~-
~~~~~:::r::.t::,e~:;i;:~ ;~phs do not have constant gracftenl
(i)
(2;
passes] s temperature of the water decreases {1mJ [r.o! as time I '
r~l'Y cOre cOntacts
This is du& to small&r temperature gradle t Id
water and surroundings [ 1 m) n ec~slng temperature difference betw, Fig_ 11.1
(ii) (i) The relay operates when there is a potential difference ol 3 V across the coil. On Fig. 11.1. [1]
Suggest two differences between the two
cooling. contamers that accounts for the different 1ate cl
(2) mark the posl~on of the slider of the potential divider when the relay Just operates.
M aterial of container A Is a good conductor of the Mark In the MIDDLE
poorer conductor of thermal energy e pl astl t
9 cs
rm,ar
tflergy e.g metal while B Is a
/ yro oam. [1mJ \Ii) Describe how the relay closes the contacts In the motor circuit. [3)
Container A has a dull I .,ugh / black surf When the current flowing In the coil is large enough 11m1, the core Is
white surface (respecti~ely) m) ace while Container B has shiny J sme>oth 1 magnetized strong enough to induce magnetism In the Iron armature a nd
11
attract the Iron armature 11m1
The anticloclCWise moment of the iron armature about the pivot 11m1
causes the contact$ to close.
(iii) State two ways of modifying the set-up so that the relay can operate even when the potential ('2]
difference across the coil is lesser than 3V.
' Increase no. of turns per unit length of the coll (1 ml [not no. o f coils)
Position the iron armature nearer the electromagnet OR the contacts are closer together
(1m)
(Oo not accept use an iron core because the rtlay core Is likely already an Iron core]
(bl Fig. 11.2 shows two terminals Mand N of a potential divider (potentiometer) connecled
10 a, rn v battery. N is also connected lo one of the two Ylnput terminals of a cathode-ray
c>Cllloscope. The other Y-input terminal is connected to the s~ding contact of the potential
dNider (potentiometer). (time-ba~e setting is 10 ms/div: Y-gain Is 5 V/dN)
~1
1 onedMsi>n
..,..
M
~
10V '
_._
I cathode-ray
MCiiloscope
T
F1g. 11 ,2
{i) When the sliding contact is at N, the trace on the oscilloscope is a horizontal line passing (11
through the centre of the screen. Explain.
10
Th@ volt""" appl d
,.- oe across the terminals of 11'.c CRQ 'V .
on. is,, ar.d the t1me,..bclse fa switched
!Do not accept " the voltage AT N Is OV"' . '
POl;1!S] - vo 11age/ pd is alw~ys ACROSS or lle~.vecn two Answer only~ of the t-0 allemaHve q~ns in Question 12.
The slicli;1g COlltat : ...,~ '':im N to M Sk
be~'!. - " .. . etch the new L'<lee cn lhe CRO .., the diagram ,,, EIThER
12(ili) Sound is a lo119'tudm3I wave. Explain, Willi the aid of a diagram. what is me;;nt by this (3)
- T-Tr'""

statement
(1m)
-
i . The direc;tion of vibration of the particles of the medium Is parallel to the d ireclion of
wave travel. [1m)
: I Energy Is transferred from one particle to !he ne>et as the particles of the medium vibrate
I back and forth In a ..erles of compressions and
rarefacllon. (1m]
horizontal line 2 div above
I ' .J....- ~.
-1- - ~LL
the centre line (tmj
(ii)
(m)

Describe how the niftection of sound may produce an echo.


An echo Is produced when sound reflec1S off a smooth/flat and hard surface.
Describe how an echo may be use<! to measure tile depth of tne sea. In your accuunt.

[ t]
(3]
(i) The 1ov battery is then raplaced with a 20V 50 Hz ~st the apparatus/equipment requited.
CRO in tne diagram below. a.c supPly. Skelch the new trace on the (2) state the readings to be taken.
explain how the deptll of the sea Is calculated from Ille reaoings.
Show 5 complete waves across the screen with correct amplitude df Ycu may use a simple diagram to aid your description and explanation.
an requ~ncy List of apparatus/equipment
u.ltrasound transmitter (1/2m)
'J1T1 II !I receiver (1/2m)
!I II I CR0 [112m)
~ I
Readings to be taken
Using time-base of CRO, detennine time interval between transmitting and
r..ceivlng the ultrasouqd pulse (1/2m)
;
-u \J u 'U i Depth of sea
depth or sea "' S;:-ud of ultrasound x Um~ interval I 2 (1/2m]
Appropriate Diagram: (1/2m)
II
t2
(b)
Flg. 121 shows an object. 0 and lhe virtual 1
..
1he ray diagram lo delenrine the posllion or t'::ge, I, ro'!"ed by a.converging lens. Complete
(3)
converging lens. Label the converging lens 'L'. convcrgrig lens and focal length of the
OR
Labelled correct position of lens [1/2m) and focal le lh 1 12(a) Describe an experiment to show the force on a current-carrying conductor. tn your account, f4J
2 correct rays (with arrows) [2mj- not dotted at the";fghtlp~cel give a labeDed diagram of the apparatus.
"' s or no arrow minus 1m Indicate the direction' of the force and
dcooribe how you would mnrflfy your set-up to increase the magnitude of the force.
Diagram:
- -::::---~
two poles of magnet (1/2m)
straight conducting wire (1m)- must specify material is a conductor
DC supply (112m)
Switch {1/2m)
Correct direction of force according to students' current direction and direction of
magnetic field (1/2m)
Increase voltage of DC supply/ Increase current in the conduclfng wire/coil/ Increase
the strength of the magnets [must mention at least lWO. 1m)
Focal length "' 5.0 cm [Use of D.C Motor max 3 marks because there is a simpler and more straightforward
experiment, without use of split rtng or carbon brush<!$ to show force on a current-
carrying conductor.]
{b) Fig. 12.1 is a block diagram of an electrical energy supply system, using the output of a coal-
fired power station.
..-- - - - , lransmisso" ..-- --,
power OUlptd step-up ou1pul slep~n output
coos.1.:mer
5.tarioo at 11oov transiotmer al 32000V translot-. at240V
Fig. 12.1
(i) State tne type of voltage supply required in a transfO<"mer. Explain. [2]
A.C [1/2m) alternating current sets up chnglng magnetic ftux In the secondary coll (1 m)
which Induces emf and current l112m) In the secondary coll.
(i) The step-up uansformer has 1200 lurns on lhe prima~/ coil. Using the values In Fig. 11.1, {2]
calculate the number of tums on its secondary cod. Assume that the lransformer an Ideal
transformer.
Ns/Np~VsNp
Ns,. 1200 x 32000/1100 [1m) 5 35000 {34909 rounded up and to 3sf) !1m]
No. or turns In secondary coll " _ __
(ii) wt.( Is electf1cal energy transmitted at high voltage? {2)
Transmission current will be low (1m), this will reduce power loss duo to joule heating
according lo P = l'R. (1 m)
13
14
QUEENSWAY SECONDARY SCHOOL James is measuring the length of a pendulum in an experiment. What is the
PRELIMINARY EXAMINATION 2010 reading?
PHYSICS
A 84.4 cm
PAPER 1 (MULTIPLE CHOICE) 5058/1
6 84.5 cm
c 84.7 cm
D 85.0 cm
Level : Sec 4 Exp ress Time: 1 h
Total Marks: 40
Inst ructions to Candidates:
Write in soft pencil.
2 The diagram shows part of a micrometer screw gauge. What is the reading
Do not use staples, paper clips, highrighters,-glue or correction Ouid. shown?
Write your name, class and index no. on the Answer Sheet.
DO NOT OPEN THIS BOOKLET UNTIL YOU ARE TOLD TO DO SO.
0 2 3
10
INFORMATION FOR CANDIDATES
mm
There are forty questions in this paper. Answer all questions.
For each question there are four possible answers A, B, C and D.
Choose the one you consider correct and record your choice in soft pencil on the separate
Answer Sheet. A 3.08 mm B 3.12 mm C 3.58 mm ~ 3.62mm
Read the instruction s on the Answer Sheet very carefully.
3 Two identical balls, one on Earth and the other on the Moon are thrown
Each corre~t answer will score one mark. A mark will not be deducted for a wrong answer. Any upwards with the same fQrce at the same time. Which of the following statements
rough working should be done in this booklet. is tr ue?
Take the value of g to be 10 rn/s2 , unless stated otherwise.
1 J.
o
Ball A
0
Ball B
On tho Earth On the Moon
A Ball A will travel a greater distance as the gravitational field strength on Earth
is greater than that of the Moon.
~ Ball B will travel a greater distance as the gravitational field strength on Earth
is greater than that of the Moon.
--------------------------- c They will travel the same distance as they have the same mass.
This qu estion paper consists of 1 sprinted p;ges.---- - - - D They will travel the same distance as an equal force is applied to the balls.
[Turn Over]
4 A stone is thrown vertically upwards with a velocity of 5 m/s. After time t, it reaches
a Which of the following shows a sphere in stable equilibrium?
the original position. Neglecting air resistance, the time required for the ball to
reach the highest position is A B c
A St B 2.51 C -0.5t 6 1/3 t
For Questio n 5 and 6. A force F pulls two blocks connected by a string as shown
below:
a- 9 A force F is applied to a bent rod which is hinged at P as shown. In which diagram
will F produce the largest moment?
~ ITT;,'.,, >
<c
5

smooth
If the blocks are moving with acceleratio n of 2 m/s2 , find the force F and the
tension T in the string, ,
J 4m
A B

J
4m
I?
D

I~
A F= 10N, T = 16N
a,. F = 16N, T = 10N
1o A skier of weight 650 N is pulled up a 30 inclined slope at a constant speed by a
c F = 10N, T = 10N
tow rope which is acting 40 to the inclined slope. When he has moved 15 m along
D F = 16N, T = 16N -
the inclined slope, what is the gravitational potential energy gained?
6 If the string suddenly snaps, find the acceleration of the blocks. A 650 x 15 x sin 30
e. aoo x 1s x cos 40 600 N
5 kg b lock 3 kg b lock c 650 x 15 x cos 30
-[) 600 x cos 40 x 15 x sin 30
A 0 m/s2 2 m/s2
B 2 m/s2 O m/s2
c 0 m/s2 5.3 m/s2
D 2 m/s2 5.3 m/s2
7 A uniform beam is pivoted at its midpoint. Cube P is placed on the beam as shown.
Where should cube Q , of the same density as P, be placed to balance the system?
A B c t5
p
14 The outlet of a glass syringe is sealed so that air is trapped below the piston.
11 A 50 kg cyclist on a 10 kg bicycle speeds up from 5 m/s to 10 mis. W hat was the
total kinetic energy before acceleration? What was the total kin etic energy after piston
accelerating?
glass
syringe --'-.i=:;,;;,;c=r--.
~- 2500 J before acceleration, 625 J after accelerating
750 J before acceleration, 3000 J after accelerating trapped air
c 625 J before acceleration, 2500 J after accelerating
D 3000 J before acceleration, 750 J after accelerating _ _ _ ____-_-_-_-_+--- seal
hot
water
12 The diagram shows a girl lifting a box of weight 100 N from a low shelf to a high
shelf.
Which of the following explains why the piston begins to rise when the syringe is
placed in hot water?
A The molecules of trapped air are getting bigger.
1.5m B Convection is occurring inside the syringe.
low shelf
c The glass is expanding.
D Ti!ie trapped air molecules are hitting the piston more often.
0.5m
15 The air in a large paper bag is heated and the bag is then found to rise through the
How much work is done by the girl? surrounding cooler air. This is because - - - - - -
A 50 J 100 J c 150 J [) 200J
/\ heat always rises
B the mass of air in the bag has increased
13 Four different liquids are poured into identical measuring cylinders. The diagrams c the air in the bag has become less dense
show the heights of the liquids and their densities. Which liquid csuses the largest D the mass of the paper bag has decreased due to heating
pressure on the base of its measuring cylinder?
A B c D 16 Which factors increase the rate of evaporation of a liquid?
Increasing its Increasing its Increasing its depth
temperature surface area f
A yes yes yes
B yes yes no
1ocmt c yes no yes
D no yes yes
2.0 gl cm' 0.7 gl cm' 1.6 g/ cm' 1.1 g/cm'
6
5
20 As a sound wave progresses at 340 m/s, regions of compression and rarefaction
. . . ~='~
17 The diagram represents a section through a particular type of building board. appear. One such typical waveform is drawn below. The frequency of the sound
wave is
~
paper boards ~ polystyrene
A
.a.
170 Hz
340 Hz
340 mis
aluminium foil c 680 Hz
D 1360 Hz
---.-!I 0.5 m i"~---
Which one of the following shows why the board provide good heat insulation? ! i
~A-
Aluminium foil Is Expanded polystyrene is 21 The diagram shows the electromagnetic spectrum, with the blue and red ends of
a good reflector. a poor conductor. the visible spectrum marked.
'
B a poor renector. a i oor conductor. Which section of the spectrum has waves which have the longest wavelength?
c a good conductor. a good renector. blue red
I I I11~~ ~
D a good conductor. a poor reflector.
18 A transverse wave is represented by the graph below.
not drawn to scale
8 A c I I
E
~
i ~+--~--+---F--..,-+--'lr--1---.,_-+--->t---+-
i< 1
22 Which of the following is NOT a property of microwaves?
A They are used for radar communications.
B They travel at the speed of light in vacuum.
Given that the velocity of propagation of the wave is 8 mi s. how long does it take
c They can be absorbed by water and fats in food.
n They are longitudinal waves which have long wavelengths.
the crest to travel from A to B ?
A 0.1 s a 0.2 s c 0.3s D 0.4 s 23 The diagram below represents parallel rays from a distant object passing through
lenses L1 and L2. The rays emerging from L2 are parallel.
19 A vibrator sends ripples across the surface of water. The ripples become further
apart as they travel away from the source. This shows that t h e y - - - - - -
A decrease in frequency
B increase in freQuency
C slow down
1 'J speed up
The principal foci of L1 and L2 are respectively
A F, and F3
B F2 and F.
C F1 and F
1 ~ F2 for both lenses
24 The diagram illustrates an arrangement in which the image and the object 26 A positively charged rod is brought near two neutral spheres X and Y. which are in
coincide. What is the focal length of the lens? contact. Y is momentarily earthed. Z is then removed. What happens to the charge
distribution on X and Y? J."~~~~~~~~
30cm '
Scm~ image
P:
-- -----------t -
plane converging lens ob'ecl
++++
z
mirror j
x y
A. 5cm B 15 cm C 20cm D 25cm A Negative Neaative
B NeQative Positive
c Positive Positive
25 A camper is standing between two cliffs, 400 m apart., in a valley. He claps his
hands, and after some time he hears both echoes at the same time.
D Positive Neaative .
27 A cell is connected to a resistor. What is the e.m.f of the cell equal to?
A the potential difference across the resistor for each unit of current
B the power produced in the circuit for each unit of charge that passes
C the work done in the circuit for each unit ofcharge that passes
D the work done in the circuit for each unit of current
28 In the circuit below, all resistors are equal. If current at P is 1.0 A, current from
battery is
.... A
A 1.0 A
B 2.0A
c 2.5A p A
0 3.0A

Now he walks 165 m to one side and claps his hands again. Given that the velocity I I
of sound in air is 330 mis, what is the time interval between hearing the first and
A A
second echoes?
A 0.6 s B 1.0 s c 1..2 s D 2.0 s
10
9
32 In each of the circuits below, a short circuit occurs.
29 Which of the following graphs correctly shows the relationship between the
resistance R of an ohmic resistor with cross sectional area, A? In which circuit would the fuse blow and make the circuit safe to repair?
A RIO . B RIO
A B
live lll(E;
.11ho.r.t shprt
crtc.u1t; clrt::Ofl.
l)eyjj'!ll ne1JVJ.d
Alm2 A/m2
C RIO R/O
c D
nve uve ---
short stwrt .
citcult ptcutt.!
Alm2 A/m2 neutral neutra~ _
0-----------~
30 Three similar light bulbs are connected to a constant voltage d.c. supply. Each bulb
operates at normal brightness and the ammeter registers a steady current. The 33 What is the mistake in the household circuit?
filament of one of the bulbs breaks. What happens to the ammeter reading and to
the brightness of the remaining bulbs? L wire
A' 1---...----..--~
Meler
Nwire Main Switch Figure 30
A There should be an earth wire connected to the bulbs.
B The neutral and the live wire in the socket should be interchanged.
A increases increases c The position of the earth wire in the socket is wrong.
B unchanoed unchanoed
11 The main switch should be installed at the live wire.
c decreases unchani:ied
34 Sonar waves are emitted from a surface vessel to determine the depth of the sea.
The emitted signal and its reflection from the sea bed are displayed on the
oscilloscope. The horizontal speed of the oscilloscope trace is 6 cm/s. The speed
31 The diagram shows a resistor connected to a cell of e.m.f. 2 V. How much heat of sound in water Is
energy is produced in the resistor in six 1200 m/s. What Is the depth of the sea?
seconds?
A 200m
A 2.5 J B 300m
B 4.8 J
c 10J c 600m
D 60 J D 2400 m
38 A girl turns the handle of a small a.c. generator four times each second. The
35 The diagram shows a LOR in a circuit. generator produces a maximum output voltage of 0.5 V.
,.-----111-~ Which of the following graphs ~est shows this?
What could this circuit be used for?
4 to switch on the lamp when it begins to get dark
B to switch on the lamp when light is being detected
c to make the lam p flash on and off ().25
D to make the battery charge up in bright daylight output
vollageN
0 1-t-+-t--='=""+-+--1
36 When an electron enters a magnetic field, it is deflected in the direction as shown
~.25r
below. When a positively-charged particle P enters the magnetic field from the left,
it will be deflected
-
I'

r------------ ct
'
!
'

;
~
'
'
:'_____ .................. _...... ____:'
,..

magnetic field
electron

39 The figure shows a simple electric motor with a solenoid connected to a battery.
An armature (with attached coil plane) is also connected to the same battery via
the split ring commutator. What will be the motion of the coil plane as seen by the
observer after switch S is closed?
A to the top
B to the bottom
C out of the paper
D into the paper.
37 X and Y are wires carrying electric currents at right angles to the page. P, Q and R

are plotting compasses. Any effect of the Earth's magnetic field has been ignored.

x CD CD CD
p Q. R

y
s
What is true about the direction and size of the currents? '7" obseiver's
v
A It will rotate in the clockwise direction.
B It will rotate in the anti-clockwise direction.
A same laroer in X than in Y
C It will not move.
B same smaller in X than in Y O It will rotate in the clockwise direction and then the anti-clockwise direction
<: different repeatedly.
r=;f I 14
13
Queeosway l;ec Sch
Preliminary 20 I0 (Physics 4E)
J.C 11. B 21. D 31. 8
40 The step-down transformer as shown in the diagram below has 600 turns and 400 2.C 12.B 22. D 32.A
turns in the primary and secondary coils respectively. ignoring any power loss, find 3. B 13. c 23. D 33. D
the input voltage and current. 4.C 14. D 24. D 34. c
5.B 15.C 25. D 35. B
6.C 16. B 26.A 36.A
7.D 17.A 27. c 37.A
8. D 18. D 28. c 38. A
9.C 19.D 29.D 39.A
10.A 20.B 30.C 40.C
12V6W
Voltage /V Current/ A
A 220 0 .33
B 220 0.75
c 18 0.33
0 18 0.75
- END OF PAPER 1 -
(d) CaJculate the average value of acceleration between time 2 s and
Section A 6 s.
Answer all the questions in this section.
A parachutist jumps from an aircraft and falls through the air. Fig. 1 below
shows a graph of the velocity of the parachutist against lime as he
descends.
Average acceleration =........................ [ 1 ]
Velocity/ms1
50
40
f ./
.....-- \
2 Fig. 2 shows a piece of cardboard suspended from a hole at one corner at
A. The weight of the piece of cardboard is 0.10 N and the centre of gravity
isatB.
30
/ \
20
/ \
...
I \
10
0
I
0 2 4 6 8 10 12 14
---
16 18
Time/s
Fig.1
(a) Describe the motion of the parachutist between time Os to 12 s .
.. ... .. . .. .... .. . .. .. .... ... . .. . .. .. . .... .. ... .... .. . .. .... .. ...... ...... .... ..... .. [ 1 I
Fig. 2
(b) State the value of the terminal velocity before and after the The piece of cardboard turns because of the moment of the weight
parachute opens. about point A.
Before: (a) State what is meant by the moment of a force.
After : [ 1l
(c) Estimate the displacement of the parachutist travelling at the
terminal velocity before he released the parachute. . ................................................................................... [2]
Displacement = ................... [ 1 )
"
2
(b) The p iece of cardboard in Fig. 2 swings freely and comes to rest. In (b) the kinetic energy of the object at C,
the box below, sketch a diagram of the piece of cardboard in its
final r est position. Mark points A and B on your diagram.
Kinetic energy =........................ ( 2 ]
(c) the speed of the object at C.
[ 21
(c) Explain why the piece of cardboard remains at rest in this position.
Speed= ............ .. .......... [ 1 J
(d) The actual speed of the object at C was found to be 2.0 mis.
Determine the frictional force acting on the object as it moved fro'm
Alo C.
[ 21
3 A small object of mass 2.0 kg moves along a track as shown in the
diagram below. The speed of the object at point A is 5.0 mis. The length
of the track AC is 2.5 m.
c
1.4 m
1m
B friclionAI force =........................ [ 2 ]
Assuming that the track is smooth, calculate
(a) the gain in potential energy of the object from A to C,
Gain in potential energy =........................ ( 1 ]
5
4
5 Fig. 5 shows a metal disc attached to an insulator handle and placed on a
4 A series of waves are generated by a vibrator operating at 10 Hz. The charged plastic plate.
waves travel from region A to region B, which is at a different depth,
through a straight boundary PQ. The distance between two successive
crests of the waves in region A is 0.03 m while that of the waves in region
B is0.02m.
charged plastic
Fig. 5
{a) On the above diagram, draw the distribution of charges on the
metal disc. [ 11
(b) - The top surface of the disc is then touched with a finger. Explain
what would happen to the charges on the metal disc?
........................................ ............................................ [ 2 J
(a) What is the speed of the waves in region A?
(c) After touching the disc, it is then lifted high above the plastic plate.
What type of net charge does the metal disc possess now?
--- .. .. .. .. [ 1 1
Speed= ................... [ 1] {d) The metal disc has a net charge of 2.6 nC. The time it takes for the
metal disc to have a net charge is 4.0 s. Calculate the average
(b) Which region is deeper? Explain. current during this time .
.. :................................................................................. ( 21
Average current = ................... [ 2]
(c) . On the diagram above draw waves on region A and region B. [2l
7
6
6 Fig. 6 shows a simple electrical circuit of a rented room. The rating of each (iii) Why is the above circuit inefficient? How do we maximize the
appliance is shown in the diagram. usage of electricity?
Air c;onditloner [ 2]
200V.2000W
(b) Suggest a danger posed by this circuit and a safety feature that can
Llghti unit be installed to prevent any electrical hazard.
2oov,~oow
....... :............................................................................ [ 2)
Fig. 7 .1 shows a circuit containing a 12 V filament lamp and a 12 V power
supply.
Fig. 6
12V
(a) Calculate
(I) the total current in the circuit.
voltmeter 1 voltmeter 2
1 -- ---1 v ..._______.
Current =................... [ 1 ) x y
(ii) the value of the resistor in the circuit so that the lighting unit and the
Fig. 7.1
air conditioner can operate at correct voltage.
The variable resistor XY is made from a long resistance wire. The sliding
contact C moves along the wire from X tc#Y.
(a) Stale and explain what happens to the readings of the three meters
as C moves from X to Y. Give numerical values wherever possible.
(i) Volmeter 1
Resistance = ................... [ 2 ]
.................................................................................... [ 2 J
(ii) Voltmeter 2
8 Fig. 8.1 below show a simple DC motor. The ends or the single loop coil
ABCD are soldered to copper pieces X and Y which make contact with
springy metal strip P and 0. A cell is connected across P and Q. Fig. 8.2
shows an end view of the arrangement.
.................................................................................... [2]
(ii) Ammeter.
.............................i ........................................................... .
.................................................................................... (21
(b ) Fig. 7.2 shows an incomplete graph of the current in the lamp
against the potential difference (p.d.) across the lamp. One point
has been plotted, where the p.d. is 4.0 V. " Fig. 8.2
Why does side AB experience a force?
0.30- --------------~-------------------------
.
''
'' .................................................................................... [ 2 J
0 4.0 2.0 (b) On Fig. 8.2 draw an arrow on the side A to show the direction of the
force. [1J
p.d.N
Fig. 7.2
(i) Sketch the shape of the graph on Fig. 7.2. (c) Explain why the coil still rotate in the same direction after half a
[1]
revolution.
(ii) Explain why the graph has the shape that you have drawn.
.................................................................................... [ 1]
.................................................................................... [ 3]
JI
(d) The si:z:e of the constant force acting on AB and CD is 3.0 N each.
There is a moment acting on the coil that varies with time as the Section B
coil turns. The distances AD and BC are both 0.065 m. Calculate
the maximum moment created on the coil.
Answer all the questions in this section.
Answer only one of the two alternative questions in Question 11.
Using the article below, answer the following questions.
Optical Fiber
Its simplest terms, fiber optics is a medium for carrying information from
one point to another in the form of light. Unlike the copper form of
transmission, fiber optics is not electrical in nature.
A basic fiber optic system consists
of a transmitting device, which
generates the light signal; an
optical fiber cable, which carries
the light; and a receiver, which
accepts the light signal
transmitted. The fiber itself is
Maximum moment = .................... .
passive and does not contain any
active, generative properties.
Optical fiber systems have many advantage!? over metallic-based
communication systems. These advantages include:
Large Bandwidth, Light Weight, and Small Diameter OWhile today's
applications require an ever-increasing amount of bandwidth, it is
important to consider the space constraints of many end-users. It is
commonplace to install new cabling within existing duct systems. The
relatively small diameter and lightweight of optical cables makes such
installations easy a'hd practical, and saves valuable conduit space in these
environments.
Non-Conductivity DAnother advantage of optical fibers is their dielectric
nature. Since optical fiber has no metallic components, it can be installed
in areas with electromagnetic interference (EM/), including radio frequency
interference (RF/). Areas with high EM/ i~ude utility lines, power-carrying
lines, and railroad tracks. All-dielectric cables are a/so ideal for areas of
high lightning-strike incidence.
Although the material used in fiber optics are relativo affordable, installing
the cables are expensive. In addition, they are more fragile than wire and
are difficult to splice.
Exlracted from:
http:/twww.cominqcablesystems.com/web/coneqe/fibertutorial.nsf/introfro
(a) State the principle behind the workings of an optical fibre .
.......................................................................... .......... [ 1 l
13
12
(b) State four advantages in using optical fibre in communication .
.................................................................................... [ 2)
(c) Describe how electromagnetic interference ca11 affect conventional
metallic wires .
.............................. ~ ......................................................... .
......... :.......................................................................... [ 2] 10 Fig. 1O shows a simplified model of how transformers are used in
transmitting power over long distances.
(d) The following diagram shows a light ray in air hilting the surface of
a glass block. The normal at the point where the light ray hits on the
surface is represented by a dotted line. The glass block has a
refractive index of 1.5.
D
a.c.
Fig. 10
The table below shows the readings on some of the meters.
8 V1 12 v
A1 0.60A
Explain what happen to the light ray as it hits side AB and side CD.
VJ 9.6 v
When necessary show you workings clearly. [ 5]
A3 0.75A
(a) Describe the principle of operation of an iron-cored transformer.
...................................................................................... ...
~
............................................ ........................................ ( 2]
14 15
{b) In Fig. 10, all lamps are identical. Calculate the power of lamp Q.
. 11 Either
{a) Describe how the height of a liquid column is used to measure the
value of atmospheric pressure in pascal.
Power= ........................ : 2) In your account,
give a labelled diagram of the apparatus,
{c) Transformer Y has 1000 turns in the primary coil and 50 turns in the state the readings that are taken,
secondary coil. Calculate the reading on voltmeter assuming the v, explain how the atmospheric pressure in pascal is calculated
transformer is 100% efficient. from the readings.
V2 = ................... ,.... [2)
(d) Should the reading on A2 be larger, smaller or the same as the
reading on A1? Give two appr?priate reasons for your answer.
,
................. .......................................................................
.................................................................................... ( 3) ............ ........... ........................ ........ ............................. ( 4)
(e) In some industries, transformers used are immersed in oil that does
not conduct electricity. Explain the purpose of using the oil.
.................................................................................... [ 1)
16
(b) A man uses a steam wallpaper stripper as shown below to help him (i) Calculate the time taken for the water to begin to boil.
prepare his living room for re-decoration.
pl ale
plug
Time'" ........................ [ 1 ]
cable (c) The appliance is used until 50% of the water has been turned into
steam. How much energy ls required to bring about this change of
state?
heating element
When the appliance is used, water is he'ated until it boils and
produces steam. The plate is then held against the wall and the
steam released from the pipe slackens the paper. The following
information is shown on the appliance:
Energy = ........................ [ 1 }
Power rating 2.5kW
Voltage 240V (d) . Explain latent heat of vaporisation in terms of molecular motion.
Capacity 20 litres
Specific heat capacity of water= 4200 Jkg1c1
Specific latent heat of vaporisation = 2260 000 Jkg1
Tap water at 20 'C is used lo fill the tank. One litre of water has a
mass of one kilogram.
.. .................................................................................. [2}
(I) What is the mass of water that fills the container?
......................... .................................... ....................... [ 1]
(ii) Calculate how m uch energy must be absorbed by the water to raise
its temperature to boiling point.
Energy= ........................ [ 1)
18 19
(ii) is at its lowest point.
(a) A student sets up an experiment as shown in Fig. 11 below. She
claims that it can be used to measure current flowing from tenninal .................................................................................... [ 1 ]
A to terminal B through the solenoid when a d.c. source is
connected across AB. (c) Describe an experiment to demonstrate that e.m.f. acts in such a
direction so as to produce effects that oppose the change causing
it.
In your account,
draw a diagram of the apparatus,
describe the procedure,
explain how the induced e.m.f. opposes the change.
Fig. 10 B
(i) Suggest how this set-up can be used to measure current.
............................................................................... : .... [ 3)
(ii) Suggest one method to increase the sensitivity of this set up.
.... ............................................................. ................ [ 1 J
(b) A and Bare now connected to a centre-zero galvanometer. The
magnet moves in and out of the solenoid. Explain whether there is .................................................................................... [4]
- any current passing through the solenoid when the magnet
(i) enters the solenoid,
.................................................................................... [ 1 J
The End
21
20
Mar king Scheme (c) KE =Y:i m v2
Preliminary Examination 2010 17.0 =Y:i x 2.0 x v2
Sec 4E Paper ~ v=4.12mls(1]
Section A (d) Loss in KE ::: Work done against friction
Y, x 2.0 x (4.122 - 2.02 ) = F x 2.5 ( 1 ]
1(a) Parachutist is undergoing decreasing acceleration [1). F =5.2 N [ 1]
(b) Before - 50 mis [ Y:. ] 4(a) v= 10x0.03 =0.3 m/s
After - 5 m/s [ Y, ]
(b) Region Al.11 It has longer wavelength [1].
(c) Displacement= 50 x 2 [ Y,]
= 100mi[Y,]
(c)
{d) Average acceleration= (40 - 20)/4 [ Y. J
= 5 m/s2 [ Y:i l
2(a) Moment of a force is the product of the force [ 1 J and the
perpendicular d istance from t he pivot to the line of action of the
fo~ef1t -
OR: Moment of a force refers to t urning effect [ 1 J of a force.
(b)
[ 1l
The two dots/marking are along
the same vertical line [ 1 ].
A and B are marked properly [ 1 J.
5
(c) The e.g. is directly below the pivot [ 1 ]. Hence there is no moment
ill due to this weight about A. metal disc
3(a) Gain in GPE = mg llh charged plastic plate
= 2,0 x 10 x 0.4
= 8.0 J [ 1 J
(b) The metal disc is earthed and electrons flow down to the gr ound [ 1 ] .
(b) K.E. at C =KE at A-Gain in GPE The ositive char es will remain attracted to the ne ativel char ed
last 9 at 1 .
=Y:. x 2.0 x 5.0 x.5.0 - 8.0 [ 1 1
=17.0J(1]
' (c) Positive charge [ 1 ]
2
8(a) '
AB experiences a force as the m agnetic field of the cpil interacts [1]
(d) I= 011 with the magnetic field of the perm anent magnet (1).
= 2.6 x 109 14 [ 1 ]
=6.5 x 1010 A [ 1 ] (b) Downwards [1].
6(ai) Total current= 2000 / 200 + 400 / 200 [ 1/2) (c) Due to momentum of rotation (1) and split ring commutator !11 which
=12 A [ 1/2] helps to change the direction of th e current in the coil (1).
(ii) V= 240 - 200 = 40V [ 1 ] (d~ Max moment created on the coil is when the coil is horizontal.
R =VII
= 40/ 12 Fxd + Fxd = 3.0 x 0.065/2 + 3.0x0.065/2 [1]
= 3.33 !l [ 1] = 0.195 Nm [1]
(iii) The resistor generates heat energy [ 11 making the circuit ineffecient. Section B
Replace th e resistor with a bulb of the sam e resistance [ 1 ].
S(a) Total Internal Reflection [ 1 l
(b) Damaged i nsulation, overheating of cables or any other dangers[ 1 J.
Any relevant safety feature [ 1 ). {b) Light weight I V. ), large bandwicilh [ Y, ], small diameter [ 'h ], dielectric
nature [ 'h ].
7(ai) The reading decreases from 12V (1) to a minimum value because
voltmeter 2's reading increases [1] as resistance of XY increases. (c) External sources from other cables or equipment might induce current in
the metal wires [ 1 ]. This current might affect transmitting signal in the
wires r 1 J.
(ii) The reading increases from OV [1] because resistance across XY
Increases 11. (cij When the light ray hits side AB, it bends towards the normal [ '12 l
because glass is denser than air [ '/2 ).
(iii) The current decreases [1) because the total resistance in the circuit
increases [1]. n =sin i/sin r
1.5 = sin 45/sin r
(bi) ..[ = 28.1 [ 1 J
.l\ngle of incidence hilting sida CD is 28.1 .
[1)
I/A n = 1/sin c
=
1.5 1/sin c
0.30. ----------- c=41.8.{1}
Since angle of incidence is smaller than critical angle and light ray is
travelling from denser to less dense medium ! 1 J, the light ray will be
refracted out [ 1 J of the glass block at side CD.
4.0 12.0
p.d.N
(ii) The resistance of the filament increases as the temperature lncrsase:go_
ill.
4
3
10(a) At the ~rimary coil._the alternating current (1/21 creates a continually (bi) 20kg(1]
changing magnellc field [1/2). This changing magnetic filed is linked to
the secondary coil [1/21 which induces a current in the secondary coil (ii) Q =mc0
[1/2]. = 20 x 4200 x (100-20)
= 6720 000 J [ 1 1
(b) P=VI (iii) Q =Pxt
= 9.6 x 0.25 (1) = 2.5x 1000 x t
=2.4 w (1] t = 6720 000/(2.5 x 1000)
=2688s[1]
(c) Vs I Vp =Ns I Np
9.6 / V2 = 1000 I 50 [1]
V2 = 192 V [1)
(c) Q=ml1
(d) Smaller (1) = 10 x 2260 000
Reasons: 1) Vz is greater than V1 and assuming no power loss = 22 600 000 J [ 1 l
A 2 is smaller than A1 [1].
(d) During vaporization, latent heat is used by the molecules to break the
2) By using smaller current in power transmission energy loss bonds of attraction between the water molecules [ 1 J. Energy is also
can be reduced (1). required for the molecules to overcome external atmospheric pressure
to escape into the atmosphere C1 J.
(e) To cool down the transformer OR To prevent oxidation OR To protect OR
~OR To insulate the iron core to prevent eddy currents [11.
11(ai) When current flows from terminal A to terminal B through the solenoid, a
Either magnetic field in and around the solenoid is produced with the upper end
11 (a) being a N-pole [ 1 J. The suspended magnet is attracted to the solenoid
metre rule and moves towards the solenoid C1 J, extending the spiral spring. The
vacuum amount of extension indicates the size of the current [ 1 J.
Correct set-up [ 1 ]
Apparatus labelled (ii) Any one of the following [ 1 ]
correctly [ 1 } Increase the number of turns in the solenoid.
Use a spiral spring that is less stiff.
Use a stronger magnet.
(bi) There is a change in magnetic field [ 1/2 J, hence current is induced
L!lll.
::
(") No current is induced [ 1/2 J as it is momentarily at rest and there is !!Q
change in magnetic field [ 1/2 J.
Using a metre rule, record the vertical height of the mercury column .
Was shown above. Using the fomiula, P = hpg [ 1 J. calculate the
pressure in pascal.
6
(c)
solenoid la coil or wi~er
..
S N
permanent
magnet

Correct diagram [ 1/2 ]


Apparatus labelled
correctly [ 1/2 )
~---1 t >-------'
A galvanometer B
When the south pole of a permanent magnet moves into a solenoid I 'h J
tne galvanometer deflects to the left [ 'h ]. This means that current flows
from B lo A and the end of the solenoid near to the South pole of the
magnet is a South pole [ '!. ].When the magnet leaves the solenoid [ Y. J.
the galvanometer deflects to the right r 'h J. This means that current flows
from A lo B and the end of the solenoid near to magnet when the magnet
leaves the solenoid is a North pole ( 'h ).
The End
2
1 The dimension of a rectangular block of wood is measure as 130 mm, 4.0 mm and
3 .21 mm.
St. ~argaret's Secondary School
What are the measuring instruments used to obtain such readings?
Preliminary Examinations 2010 (i) metre rule
(ii} vernier calipers
FouMed 1842 (iii) micrometer screw qauqe
A (i) and (ii) only
B {i} and {iii) only
Secondary 4 Express c (Ii) and (iii) only
D (i), (ii) and {iii)
17th September 2010
Duration 1 hour
Total Marks : 40
2 Given that Newton's Law of Gravitation is given by F = Gm,m 2 where m, is the
READ THESE INSTRUCTIONS FIRST r2
~ass of particle 1, m2 is the mass of particle 2, F is the gravitational force, r is the
Do not open this Booklet until you are told to do so. distance between the two particles and G is the universal gravitational constant.
Write in soft pencil. Which of the following is the unit for G?
Write your name and index number on the answer sheet provided.
Do not use staples, paper clips, highlighters. glue or correction fluid. D
There are forty questions on this paper. Answer all questions. For each question there are
four possible answers A , B, C, and D.
Choose the one you consider correct and record your choice in soft pencil on the Answer Question 3 and 4 based on the information given below.
separate answer sheet.
A stream is 30 m wide and its current flows southward at 1.5 mis. A toy boat is
Read the in structions on the Answer Sheet ver y carefully. launched with a velocity of 2.0 mis eastward from the west bank of the stream.
Each correct answer will score o ne mark. A mark will not be deducted for a wrong answer. 3 What is the magnitude of the boat's resultant velocity as it crosses the stream?
Any rough working should be done in this booklet.
A 0.5 mis B 2.1 mis C 2f> mis D 3.5 mis
When necessary, assume the acceleration due to gravity, g to be 10 m/s2.
4 How far southward will the boat have traveled upon reaching the opposite bank?
A 18.0 m B 22.5 m C 30.0 m. D 37.5 m
This question paper consists of 15 printed pages [Turn over
5 Two stones, A and 8, are thrown horizontally from the top of a cliff. Stone A has an
initial speed of 15 rn/s and stone B has an initial speed or 30 m/s. There is 8 The diagrams show a rectangular box with inside measurements of 5 cm x 6 cm x
negligible air resistance. 4 cm. The box has a mass of 40 g when empty. When filled with a liquid, it has a
total mass of 220 g.
Compared to the time it takes stone A to reach the ground, the time it takes stone B
to reach the ground is liquid
A half as great.
B the same.
C twice as great.
D four times as great.
Scm
mass= 40g total mass = 220 g
6 An object is pulled at a constart speed across a rough surface. Which of the
following statements about friction Is true? What is the density of the liquid?
A The magnitude of the frictional force is greater th2n the driving force.
A 220 g I cm 3
B The magnitude of the frictional force is less than the driving force. (5x6x4)
C The magnitude of frictional force Is equal to the dri.;ing force. 220 - 40 I
B g cm 3
O The magnitude of the frictional force Is equal to the resultant force. (5x6x4)
(5x6x4) I
c 220 g cm
3
7 The diagram shows a system in equilibrium (5x6x4)g/cm3
consisting of an object of weight W that hangs 0
220-40
from two ropes. The tensions in the ropes are T1
and T2 .
Which of the following are correct values of T1
-
9 The diagram shows a uniform metre rule of mass 0.60 kg balanced horizontally.
and T2? The mass of A is 0.40 kg.
T1 T2
w T
A T2 sin 60
cos60 -
w
B Tz sin 60 - -
sln60
40 cm
w w
c ---
tan60
- --
cos60
D
w
- -- -
w
-
tan60 sin60 What is the tension T in the string?
A 6.7 N
6
10 The diagram below shows an object tilted such that it is at the point of falling over.
14 When a bullet of mass 20 g moving at 400 mis strikes a fixed wooden block of
Where would be the likely position of the centre of gravity of the object? thickness 0.030 m, it emerges wiltl a speed of 200 mis as shown in the diagram.
~ Iii)------+
400 mis 200 mis
0.030 m
What is the retarding force of the wood?
A 66.7 N Bl 13300 N C 40000 N D 53300000 N
11 Water of density 1000 kglm3 fills up to 30 cm in a container. The base of the
container is broken accidentally and water starts to leak from a tiny hole of area
1.0 mm 2 To prevent water from leaking, a sticky tape is used to cover the tiny hole. 15 Gas inside a cylinder is heated slowly to a higher temperature. The pressure inside
the cylinder remains constant as the piston moves outwards.
Determine the minimum force that the tape must be able to withstand.
A 0.003 N B 0.3 N C 30N D 3000 N air
gas
b '"' b
O+
,,,,,, ~
molecule
12 The diagram shows a 2 kg block, starting from rest, slides 20 m down a frictionless d' ~
inclined plane from Xto Y, dropping a vertical distance of 10 m.
Tl~ ..
cylinder
How do the speed of the gas molecules and their rate of collision with the piston
20m compare with their initial values al the lower temperature?
ho= lOm
~j --------------- 2kg Block A
speed of molecules
greater
rate of collision
greater
r
B greater reduced
Determine the magnitude of the net force on the block while ii is sliding.
c greater same
A 1N B SN <) 10 N D 20N
D same greater
13 An object is thrown downwards with a speed of 10 mis from a cliff that is 120 m
above ground.
If air resistance is negligible, determine the time taken for the object to fall to the
ground from the cliff.
A 4.0 s B 4.1 s c 4.8 s D 6.0s
8
16 Which of the following contains the molecules with the'fighest average speed?
19 A cup of hot coffee of mass 200 g is at an initial temperature of 80 c. Given that
wind, at an initial temperature of 20 ' C, is blowing across the cup at a rate of
1.0 kg/s. After moving over the cup, the temperature of the wind increases to 25 c.
ice cube
at - 10'C
water at room
temperature
wood at room
temperature

TI
steam
at 11oc
A B c D
17 The d iagram shows a section through a particular type of building board. If the specific heat capacity of coffee i~4iO
kJ/kgC and that of air is 200 J/kg'C,
f compute the time necessary to cool the cup of coffee to 50 'C.
../" , . . _, _ _ __ ., -~f i,/ Aluminium foil
Compressed oO,.OQ Oo6ooooson 0 QO' A 0.024 s B 4.8 s c 24 s D 40 s
paper boards'-..,.~-qoo <too~ cra.
0 o,o~cs_ ?~0o9o f - Expanded polystyrene
0
- - -- - - h.. Aluminium foil
'
Which statement best explains why such a board provides good thermal insulation? 20 Which of the following in~reases when a liquid becomes a gas at its boiling point?
A the average kinetic energy of the molecules
Aluminium foil is a Expanded polystyrene is a
B the molecular size
A good conductor poor reflector
C the molecular spacing
B poor reflector poor conductor
D the total number of molecules
c good conductor good reflector
D good reflector , poor conductor
18 Fang Fang tried to use a thermistor as a thermometer. She found that when the
temperature was 200 c.
the resistance of the thermometer was 250 and when n
the temperature was 50 'C, the resistance of the therm:>meter increased to 500 n.
What would be the temperature when the resistance of the thermistor is 600 D?
A -60 'C B -40 C c -1 oc D 10 C
10
21 The diagram shows the motion of a wave in a ripple tank.
23 Astronaut 1 uses a hammer to mend a satellite in space. Astronaut 2 is r.eart
There is no air in space.
4
height/cm
0.25 0.5 hammer
time ls astronaut 1 astronaut 2
What are the amplitude and frequency of this wave? Compared with the sound heard if they were working on Earth, what does aslroni
2 hear?
amplitude I cm frequency I Hz A Aloudersound.
A 2 2 B A quieter sound.
B 2 4 C A sound of the same loudness.
c 4 \ 2 D No sound at all.
D 4 > 4
24 Two plane mirrors are placed facing each other with a pin 0 held upright betw1
them as shown in the diagram.
22 A vertical stick is dipped in and out of the water at P. It takes two seconds to travel
from P toQ.
4.0cm
Determine the separation between the two images formed in the mirrors.
A 6.0 cm B 8.0 cm C 10.0 cm D 12.0 cm
Which of the following statement is tue? 25 The wavelength of yellow sodium light in vacuum is 5.89 x 10- 7 m. The speed of
light in glass with an index of refraction of 1.5 is
A The frequency of the waves is 1.5 Hz.
B Distance X is the amplitude of the waves.
A 4 x 10-7 mis B 9 x 10-7 mis C 2 x 108 mis o 3 x 108 mis
C Distance Y is the wavelength o' the waves.
D The wave formed is a lonaitudinal wave.
12
26 The image of an object formed is smaller than object when the object is placed
19 cm from a converging lens. Placed at a distance of 17 cm, the image of the 30 A potential difference of 10 V exists between two points, A and B, within an electric
object becomes slightly bigger than the object. field.
What is the approximate focal length of the lens? What is the magnitude of charge that requires 2.0 x 10-2 J of work to move it from A
to 8?
A 9cm B 10cm C 16cm D 18 cm
A 5.0 x 102 C B 2.0 x 10-1 C C 5.0 x 10-2 C 0 2.0 x 10-3 C
27 A microwave and an X-ray are traveling in a vacuum.
31 In the circuit shown, all the lamps are identical.
Compared to the wavelength and period of the microwave, the X-ray has a
wavelength that is
A longer and a period that is shorter.
B longer and a period that is longer.
C shorter and a period that is longer.
O shorter and a period that is shorter.
What will happen to their brightness if switch S is closed?
28 In the table shown, which region A, B, C or D is used in dental Inspection?
:r
I Gamma I A B c D I
Microwave I Radio wave A
L1
decreases
L2
increases
L3
decreases
~ goes off increases decreases
c goes off increases remains unchanged
29 A student tries to charge a metal cylinder in the following ways:
D increases decreases remains unchanged
I
positively
charged rod
LlY'"
insulating plate
32 The resistance of wire A is 4 n. Wire B, made of the same material, has twice the
length and half the radius of Wire A.
(I) bring a positively charged rod near the cylinder Determine the resistance of Wire B.
(II) touch the cylinder with a finger
(iii) remove the rod A 320 B 16 0 c 80 D 40
(iv) remove the finger.
After this process, the cylinder would most likely be
33 Which device is designed to allow a small direct current (d.c.) to control a large
A uncharged. direct current (d.c.)?
B positively charged. A a relay
IJ
14
34 The diagrams show the possible wiring to a heating element.
36 Four plotting compasses are placed in the magnetic field of two identical ba
Which one shows the correct arrangement of wires? magnets as shown in the diagra(Tl.
A Which compass is shown pointing in the wrong direction?
B
B
tllll'!!!llm~-+---
L----++.....:....+ - L
..._____..~_-_.4---- N
c:: tU~llllffill\--+---- L
'-----++--+- N
.____..,..._-_,-t---- E

A
e l.__N_ _____,s I e
D
c
37 Two long parallel wires, separated by a distance d, carry equal currents I toward the
ttllll!lllm~\--+--- N top of the page, as shown in the diagram.
ffll\"U\1111'!>-+F==!-- L
L----+-1~:::+- E ~---->----N
,____ ___..~--_,-1---- L ~--I.::J---- E
I I
35 The diagram shows a 3-pin plug. -d-
The magnetic field due to the wires at a point halfway between them is
A directed into the page.
B directed out of the page.
r. zero in magnitude.
D directed to the right.
If the brown wire carries a current of 4 A when the appliance is switched on, what
would be the currents in the yellow/green and blue wires?
yellow ~ind green I A blue/A 38 A square loop of wire lies in the plane of the page. A decreasing magnetic field is
directed into the page. The induced current in the loop is
A 4 4
e. 0 4 A clockwise.
c 4 0 B anti clockwise.
D 0 0 c out of the page.
& not present.
15
~
39 A thin solid condu::tor with sides PQRS is moving at constant velocity v, at right
angles to a uniform magnetic field 8, directed into the page as shown.
x x x x XB x x x
x x x x x x x x
Q
x Pl IR x
x x x x s rx x x x
v
x x x x x x x x
f
Which side of the conductor has the greatest concentration of electrons?
A p B Q c R D s
40 A transfonner is to be used to provide a 10 V output from a 100 V supply.
100V ""-' "-'10V
primary coil secondary coil
What are suitable numbers of turns for the primary coil and for the secondary coil?
number of.turns on number of turns on
the primary coil the secondary coil
A 100 1000
B 200 110
c 400 490
D 800 80
Name: .. . .. . . .. . . ... . .. . . . .. .. . . .. . .. ... . . . . . .. .... . . .. . .. .. Reg. No. . . .. . .. . .. . . "''"" ................
.. Section A
Answer all the questions in this section.
St. Margaret's Secondary School
1 Fig. 1.1 shows an archer drawing back a bow to shoot an arrow horizontally
towards a target. When he releases the arrow, it is accelerated over a distance of
Preliminary Examinations 2010 ~.40 m by the bow. It leaves the bow with a speed of 80.0 m/s, heading on a direct
line to the centre of the target which is 16.0 m away.
Assume that air resistance is negligible and that the arrow hits the target.
f<J~'llded 1842
Secondary 4 Express
17t11 September 2010
I
Duration 1 hour 45 minutes
Total Marks : 80
16.0 m
READ THESE INSTRUCTIONS FIRST
Fig. 1.1
Do not open this Book let until you are told to do so.
(a) Calculate the time taken for the arrow to travel the distance of 0.40 m from
Write your name, register number and class in the spaces at the top of this page and on release by the archer until the arrow just leaves the bow. Assume that the
any separate writing paper used. arrow has constant acceleration over this time.
Write in dark blue or black pen.
You may use a soft pencil for any diagrams, graphs or rough working.
Do not use staples, paper clips, highlighters, glue or correction fluid.
Section A
Answer all questions.
Section B
Answer all questions. Question 11 has a choice of parts to answer.
Write your answers to Section A and to Questions 9 and 10 in the spaces provided on the
timE# = s (2)
Question Paper. Write your answer to Question 11 on the lined pages and, if necessary,
continue on the separate Answer Paper provided. (b) Calculate the magnitude of the acceleration of the arrow over the 0.40 m in
At the end of the examination, fasten all your work securely together. part (a).
Candidates are rem inded that all quantitative answers should include appropriate units.
Candidates are advised to show all their working .n a clear and orderly manner, as more
marks are awarded for sound use of physics than for correct answers.
The number of marks Is given in brackets [ ] at the end of each question or part question.
2
When necessary, assume the acceleration due to gravity, g to be 10 m/s
2
acceleration ....................................m/s (2)
lhis question paper consists of 19 printed pages [Turn over
<;
...
(c) Sketch a speed-time graph of the motion of the arrow in Fig. 1.2 below. Your
graph should start at the time when the archer releases the arrow and end 2 Fig. 2.1 shows a child sitting on a child seat and Fig. 2.2 shows a simplified drawing
after the arrow has hit the target Label your graph appropriately, indicating of the same child seat. The total weight, W, of the child and the seat is 12.0 cm from
clearly when the arrow leaves the bow and when the arrow hits the target. C.
(4]
Fig.1.2
A B
Fig. 2.1 Fig. 2.2
(a) Draw on Fig. 2.2, the position of the force(s) that will hold the child seat in
equilibrium. Indicate clearly the direction of this/these force(s). [1]
Given that the total mass of the child and the seat is 10.8 kg, the length of BC is
70.0 cm and the horizontal distance AB is 60.0 cm, calculate
(b) the total weight, W of the child and the seat,
weight N [1]
6
(c) the force(s) needed to keep the child seal in equilibrium when the child is on
3 Fig. 3 .1 shows water held by a dam. The maximum depth of the water is 120
it. Water flowing through the pipe i'asses through the turbine and generates electric
3
The density of water is 1000 kglm
force N
force (if any) N [4)
Fig. 3.1
(a) Determine.the pressure due to the water at the base of the dam.
pressure = ................................... Pa
(b ) Explain why the dam is thicker at the base than at the top .
............................................................................. .........................................................
(c) (i) Determine the speed of the water hitting the turbine.
speed m:s
~
8
(ii) If the turbine is only able to convert 60 ;, of the supplied energy into
elec1rica1energy, calculate the power generated, given that the rate of 5 Fig. 5.1 shows a ray of monochromatic light incident on an interface of air and com
water hitting the turbine is 3600 kg/s. oil al an angle of 35. The ray is lransmilled through parallel layers of corn oil and
g lycerol and is then reflected from the surface of a plane mirror. located below and
parallel to the glycerol layer. The ray 1hen emerges from the corn oil back into the
air at point P. The refractive index of com oil is 1.'17.
Incident ray
Air
power = ................................... w [2]
4 Fig. 4.1 shows a sealed syringe that contains gas, at atmospheric pressure and
many very small dust particles su~pended in the gas. The piston moves freely along Corn oil - - r - -
the cylinder without any friction. No gas can escape.
piston
C: I Glycerol- - r - -
Fig. 4.1
Fig. 5.1
(a) Explain why the dust particles stay suspended in the gas and do not fall to
the bottom of the syringe.
(a) Calculate the angle of refraction of the light ray as it enters the com oil from
air.
...................................................................................................................................... {2]
(b) As the syringe is heated, the piston moves outwards and stops moving when
the temperature is steady.
Using the kinetic model, explain
angle = ................................... [2]
(I) why the piston moves outwards,
(b) The ray does not bend at the corn oil and glycerol interface. Explain why.

.......................................................................................................................... (1 I
0 0
(ii) why the piston stops moving.
...................................................................................................................................... [2]
10
(d) Will the reflected ray from the m irror ever go through total internal reflection
at point P if we vary the angle of the incident ray in air? Explain your answer. 7 Fig. 7.1 shows an electrical circuit containing four resistors R1 , R 2 , R 3 and R
.
4
connected to a 12 V battery of n~gligible internal resistance.
,.o-oO-uu U uoo., ,.ooouoH
12V A
1,
...................................................................................................................................... [3)
A 1 =2n
A,.sn
6 Fig. 6.1 shows a negatively charged rod lowered into an uncharged metal can
standing on an insulating slab.
B
Fig. 7.1
(a) Calculate the effective resistance of the circyit.
I
Fig. 6.1 Fig . 6.2
(a) Indicate on Fig. 6.1, the distribution of charges produced on the can. !11
(b) The outside of the can is then touched with a wire connected to earth in
Fig. 6.2.
effective resistance = 0 [2)
Explain the effect this will produce on the charges on the can?
(b) Determine the value of 11 , 12 and 13 .

................................................................- .................................................................... (2)
(c) Can the same effect in part (a) be produced if the insulated charged rod is
replaced with a metal rod held in the hand of a student?
Explain your answer.
'1 ................................... A
12 A
(2)
b = ................................... A (3]
12
8 Fig. 8.1 shows two identical wires, W 1 and W2. positioned 5 cm from one another. "' Section B
They carry identical currents in the direction indicated.
Answer all the questions from this section. Question 11 has a choice of parts to answer.
9 Read the article below and answer the following questions.
The diagram below shows how electricity is transmitted through a national grid
system to our home using high alternating voltages.
275kV
supergrid or 400 kV
grid system
132 kV
-Scm-- - 5 cn1-~-- 8 c m -
grid supply intermediate
Fig. 8.1 Fig. 8.2 point substation
25 kV step-up supergrid bulk 33 kV
(a) From Fig. 8.1, identify the <firection of the force which W2 experiences as a transformer supply point
result of the current in W1.
..............................................:.............. [1] 220V HOME
(b) Fig. 8.2 shows a third wire, W 3, , , .. 1g a smalleF.current, being placed 8 cm The reason to have a national grid system is to have only a few large power
from W2. stations generating the electricity for all consumers nationwide.
Explain qualitatively the forces on W 2 as a result of the currents in W1 and Electrical energy is generated by power plants and is then sold to end
W 3. (You may make reference to Fig. 8.1) consumers by retailers. The electricity distribution networks allow the delive~
of the generated electricity to consumers. The rapid industrialization in the 201
century made electrical transmission lines and grids a critical part of the
economic infrastructure in most industrialized nations.
The transmission grid allows large generation facilities such as hydroelectric
darns, fossil fuel plants, nuclear power plants, etc. run by large public and
private utility organizations to produce large quantities of energy and then
deliver it to distribution networks for delivery to retail customers for
consumption .
...................................................................................................................................... [4]
Electricity is usually sent over long distance through a combination of overhead
power transmission lines and buried cables.
A transmission grid is made up of power stations, transmission circuits, and
substations. Energy is usually transmitted on the grid with 3-phase alternating
current (AC). The voltage level on the bulk power transmission system is
typically between 115000 V and 765000 V. Energy may also be tran~mitted
using high voltage direct current.
At the generating plants the energy is produced at a relatively low voltage of up
to 25 kV, then stepped up by the power station transformer to a higher voltage
for transmission over long distances to grid exit points (substations).
14
It is necessary to transmit the electricity at high voltage to reduce the
(d ) Suggest one reason why, in a real transformer, the power drawn from th
percentage of energy lost. For a given amount of power transmitted, a h igher supply exceeds the po""f!r dissipated in the secondary circuit.
voltage reduces the current and resistance losses in the conductor. Long
distance transmissio'n is t ypically at voltages of 100 kV and higher.
Transmission voltages up to 765 kV AC and up to +/-533 kV DC are currently
..........................................................................................................................................
used in long-distance overhead transmission lines.
..................................................................................................................................... r
Source: http :/le n .allexperts .com/g/Phys i cs-1358/e lectri c ity-voltan e.htm
(e) A tnwn is 5.0 km from the nearest electricity substations. Two conductors ar
used to connect the t own to the substation. l;:ach metre length of eac
conductor has a resistance or 1.2 mn.
(a) Give two reasons why electricity is transmitted at high alternating volt ages.
Calculate the power loss in the conductors when the current through them
40.0 A.
...................................................................................................................................... [2]
(b) Name the object Y in the system.
power loss = ................................... W [:
...................................................................................................................................... [1]
(c) It is given that the step-up transformer in the system increases the a.c. 10 fig. 10.1 shows a coil being rotated in a magnetic field between two magnets.
voltage of 25 kV to 275 kV. Fig. 10.2 shows how the vollage between terminal T1 and T2 varies when the coil i
rotated at a constant speed.
(i) What is its turns ratio?
~ VN Output graph
~ ~--.!3,--- x
I - _____,...--- - ----r---<>-.,---
T1 T2
z
turns ratio [1] Fi g. 10.2
(ii) Calculate the current in the secondary coil if the power input to the
primary coil is 400 W.
What assumption have you made in this calculation?
---c=J-Load
Fig. 10.1
(a) Complete Fig. 10.1 by showing the necessary connections lo the load an
_______ .,.. [2] indicate the direction of the current.
I -::.hol '\tn1 u tir:::rnuinnc
(b} z
Which points X, Y or on the output graph in Fig. 10.2 could represent the
.. 16
11 Answer on the lined pages at the end of the booklet
voltage when coil is at the position as shown in Fig.10.1?
Explain your answer clearly.
EITHER
........................................................................................................................................... Fig. 11.1 shows a skateboarder of mass 54 kg about to descend a curved ramp in a
skate park.
..........................................................................................................................................
''''''''''''''''''''''"''''''"'''''''''''''''''''''''''''''''"'''uooooooooooouooooooooooooooo,.oo
..........................................................................................................................................
D
(c}

oooooooOOluooo"''"'"'''''''"'"''''''''""'''''''"'''''''''''"'"''''''"''''"''''"'"""'''''"'"''*'*"'''''''' ''''"'''"*
f
Explain briefly why the output graph in Fig. 10.2 has such a shape.

(3]
- --------------;: __
....:::-: B
__,.___-..,,,,
Fig. 11.1

...:::
1.6111
_ ..............____________________ ..
.......................................................................................................................................... The skateboarder can be assumed to be a rigid body during the motion.
........................................................................................... ............................................
').,
(a) At which of the points A, B, C or D is the acceleration greatest along the
(2) track? Explain your choice. [2]
'''''''"'''''''''''''''''''''''''''''''"'''''''''''"''''''''''uooooooo1uoooooo-oo
(b) The skateboarder is initially at rest. Neglecting the frictional force acting on
the ramp, calculate the speed of the skateboarder at B, (2)
(d} Draw on the same axes )n Fig. 10.3 a new outi:ut graph when the speed of
rotation of the coil is halved. (2] (c) The skateboarder has just enough energy to reach D because of friction. The
total length of the track between A and D is 8.0 m.
VIV (i) Determine the frictional force along the ramp, assuming it to be
original output
co~~ ~
(ii) If the skateboarder wants to go back to point A, with what minimum
speed should he leave point D. [3]
tis
Fia. 10.3
17
1b
OR
Fig. 11 2 illustrates an experiment in which the electrical energy used to produce a
,
................................................................... ..............................................................................................
measured rise in temperature of a liquid can be determined.
r...
d.c. supply
0000000000000f000000000000000000000000000000000000000000000o00000000000000000000000o000000000000000000000000000H000000000000000000000000000000000000000000000000
0
voluneter roo . . . . oooooooooooooooooooooooooooooo+oo o oooooooooooooooooooooooooooooooooooooooooooooooooooOOOOIO
..............................................................................................................................................................
'''''"'nooooooooooooooooooooo04oOOOOOOoOOooooooooooooooooooooooooooooooooooooooooooooOooOoooooooooooooooooooooooooO
...................................................................................................................................................................
...................................................................................................................................................................
...................................................................................................................................................................
stopwatch
...................................................................................................................................................................
..................................................................................................................................................................,
Fig. 11.2
...................................................................................................................................................................
(a) Define the specific heat capacity of a substance. [1)
...................................................................................................................................................................
(b) Explain why the heater is usually placed at the bottom of the container. [2)
Explain why the temperature of the liquid will eventually slop increasing even
.....................................................................................................................................................................
(c)
though the current is still passing t>irough the healing coil. [2)
..........................................................~........................................................................................................
(d) Explain why the specific heat capacity of the liquid calculated will be more
accurate if the liquid is first cooled to about 5 c below room temperature
and the current passed until the temperature is about 5 c above room
temperature. [1]
The power rating obtained from the readings of the ammeter and voltmeter is
...................................................................................................................................................................
(e)
240 W. After 16 min, the rise ir temperature for 6 kg of liquid is 10 c.
Calculate the specific heal capacity of the liquid. [2]
.....................................................................................................................................................~.............
6
(f) The specific latent heat of vaporisation of the substance is 2.26 x 10 J(kg
5
and its specific latent heat of fusion is 3.36 x 10 J/kg. ...............................................................................................................................................:...................
Why is the specific latent heat of vaporisation so much higher? [2]
...................................................................................................................................................................
...................................................................................................................................................................
hits target
St. Margaret's Secondary School leaves bow +H-++-i-+++-H
ll I 1 II
Preliminary Examinations 2010
Secondary 4 Express
Paper1
c
A
A
D
B
D O 0.02 0.04 0.06 0.08 0.10 0.12 0.14 0.16 0 .18 0.20 0.22 tis
c
A 2 marks for correct shape graph with clearly labelled values
c 1 mark for correct labelling of graph (leaves bow & hits target)
D 1 mark for correct calculation of distance travelled by arrow during flight
(0.40 m for ace portion; 15.6 m for the 80 m/s portion)
Paper2
2 (a) Upward forces at A and B. [1 ]
1
s=-(u+v)t
2 (b) W =mg= 10.8 x 10 ~ 108 N [1]
(a) 0.40 = ~(O + 80.0)t (1) (c) ..L distance of W from B = (70.0 cos 45") - 12.0
2
= 37.5 cm [1]
t = 0 .010 s [1]
Taking moments about the B,
CM=ACM

v = u+ at
FA x 60.b =108 x 37.5 (1] Fs = 108- 67.5 == 40.5 N [1]
(b) 80.0 = o+ a(0 .01O) [1)
FA = 67.5 N [1]
a = 8000 ml s2 [1]
3 (a) P =(120 x 1000 x 10) [1] =1200000 Pa [1]
(b) Pressure increases with depth [1] so a thicker base is needed to withstand
t he greater force per unit area at the base. [1]
(c) (i) loss in GPE gain in KE
mgh Y:z mv2
10 x 100 Y,xv2 (1)
4
(Ii) Power generated =3600 x 10 x 100 x 60% [1]
=3600000 x 60% (b) The can will be positively charged. [1] The excess electrons will move from
= 2160000 w [1) the can to the earth. [1]
(c) ' No. [1) As metal rod and the hand are conductors of electricity the electrons
4 (a) The dust are kept in suspension as they are continuously hit by the air will flow from the rod to the hand and to ea1111 arnJ lhH rod will be neutral. ['I)
molecules/particles [1] which are at constant random motion and moving at
high speeds. [1] 1 1 1
-=-+-
Rh 20 5
(b) (i) As the gas is heated, the gas molecules gain K.E/speed up and hit
the piston with a greater force and more often [1], hence pushing the 7 (a) 5
=-
piston outwards. 20
(ii) As the piston moves outwards the volume of the gas increases and R11 =4!1 [1]
the rate of coll:sion decreases, hence pressure decreases. [1) The
piston stops moving~hen lne pressure inside equals the pressure Rr= 2 + 4 = 6 0 [1]
outside. (1]
I,= VT
sini RT
n= - -
sinr
' (b)
12
. 5 (a) 1.47 = sin35
sinr
i1J 6
=2.0A [1]
r =23 [1]
4 Ii= l3
(b) Both have the same refractive indices [1) therefore the speed of light does
not change when it reaches the interface and no bending will be observed.[1] 12 + 13 = 5 12 =2.0 A
12 = 0.4 A [1]
(c)
Incident r"y
b =1.6A [1 )
(1)
8 (a) W2experiences a force toward w,.
Or
Comoll W2experiences a force to the left.[1)
(b) W2experiences a force of attraction to W 1. W 2 also experiences a force of
attraction toward W 3 [1]
Pion"'"'' However the force between W2 and W3 is smaller as the distance between
W2 and Wa Is larger than between W, and W 2 and the current in W 3is
smaller. (1]
(d) No. [1] The net force on W 2 is one of attraction toward W 1 (1)
The incident ray from air can only enter the com oil at max. 90 . this will give
but the ma~nitude is reduced due to the presence of w 3, as compared to the
a refracted angle= critical angle of com oil. [1] But for total internal reflection force expenenced in Fig. 8.1. (1)
to occur, the incident angle at P needs to be graater than the critical angle of
com oil. [11 1
5
6
9 (a) Since power= voltage x current, a high voltage means a small current
flows ii the cable, so less power l:>st in the cable as thermal energy. [1] .1ther
Alternating voltages will produce a ,changing magnetic field which is
needed for a transformer to work. [1l 11 (a) A [1], the component of weight along the slope is greatest at A and friction
has no effects on the skateboarder yet. [1]
(b) GPE.., = KEe
(b) Step-down transformer. [1]
m3h = y, mv2
. VP 2 gh
Tuns ratio= V 2x10x1.8 [1]
s v = 6.0 mis [1]
25kV
(c) (i) Turns ratio 1s 1:11 [1] (c) (i)
275 kV Energy lost= mg (1.8-1.6) = 54 x 10 x 0.2 = 108 J [1]
11 Frictional Force = Energy lost
Dist
(ii) Power o utput power input 108
275000 x Is 400 [1]
8.0
Is 1.45 x 103 A [1]
=13.5 N [1]
Assumption: transformer is 100 % efficient [1]
(ii) Energy needed at D GPE.., +WO,
(d) Heating effect of the coils GPEo + KEo = GPE.., +WO,
or y. mv2 GPEA - GPEo +WO,
Heating effect in the soft-iron core due to eddy currents [1) 0.5 x 54 x v2 108 + 1oa [2J
v = 2.8 mis [1]
(e) Combined resistance= 2 x (5000 x 0.0012)" 12 n [1)
OR
Power loss = 12R = 402 x 12 (1] =19200 W [1]
11 (a) Thermal energy needed to increase the temperature of 1 kg of the substance
by 1C (1K) '
10 (a) Slip ring5 [1) and carbon brushes [1] correctly drawn and labelled.
Current is clockwise In coil. [1) (b) So that the liquid can be heated up by convection. [1]
Hot liquid will expand and become less dense, moves up. Cool liquid on top
(b) is denser, sinks, to be heated up [1]
Point Z [1) I T "th
Point xindicates the max output w1en the coil is at a honzonta pos1 ion w1
maximum cutting of the magnetic field lines. ..
Point y indicates no output as the coil is in a vertical pos1t1on whereby there
(c)
As temperature of liquid rises, more energy is lost to the surroundings. [1]
Temperature will stop rising when the rate of thermal energy absorbed by
is no cutting of magnetic field lines. [1] . . liquid equals to rate of thermal energy given out to surroundings by the hot
When the coil is inclined as shown, there will be _some culling of magnetic liquid. [1 )
field lines which is responsible for an output value between the max and
zero. [1] (d) Energy received by surroundings equals to energy given out to surroundings
[1 J
(c) The output graph shows a continuous curve - this is due to the continuous
turning of the coil. [1 ) h. d (e) Pt=mc0
The graph has positive and negative values at allemate ph~se - t 1s ~s ue 240 (16 x 60) = (6) c (10) [1]
the current changing direction every half cycle as the coil rotates in the c =3840 Jl(kg c)0
r11
magnetic field - effect of electromagnetic induction. [1m]
(f) In latent heat of vaporization, more energy needed to break liquid bonds and
(d) The peak output value is halved [1} and the period is twice. [1] move them much further apart. [1]
Energy also needed to lift molecules into atmosphere (work done against
J SECTION A : (40 marks)
Each question 1s provided with four possible answers (A, B, C and D). Select the most
appropriate answer and shade your choice on the Optical Answer Sheet proVlded
ST. PATRICK'S SCHOOL A student tries to find out the thickness of a page of a hardcover story book or about
two hundred pages. Which one of the following is the most accurate way of
PRELIM INARY E XAMl NATIONS 2010 estimabng the thiekness?
A Use a micrometer SCfew gauge to measure a single page of the book
PHYSICS 5058 PAPER 1 3 SEP 2010 B Use a pair of vernier calipers to measure the book from cover to cover and
SUBJECT
divide by the total number of pages.
SECONDARY 4 EXPRESS 1 HOUR C Use a micrometer to measure the thickness of all the pages of the book and
L EVEL D URAT ION:
divide by half the total number of pages.
D Use a ruler to measure the thickness of all the pages Of the bool\ and divide
by the total number of pages.
INSTRUCTIONS TO CANDIDATES
DO NOT OPEN THIS BOOKLET UNTI L YOU ARE TOLD TO DO SO. Which one of the following statements about the period of a simple pendulum is
correct?
1. 'Write yow name, daU and index n1..mbef on the cover page of this Booki.t and the Optic11I
Answer Sheet in the spaces provtded On tne Optical Answer Sheet. K la also required thot
A It decreases when the length increases.
you WRITE and SHADE your index number (please a sk the Invigilator if you are not sure).
B It does not change when the pendulum is brought up to a vety high
2. There are Forty (40) que&tions in this paper. Answer ALL questions. mountain.
c It doubles when the length doubles.
3. FO< each question there are Four (4) posStble answers (A, 8, C or DJ. ChooH the O n e (1) you D It Is independent or the mass of the pendulum.
~ co<recl and recoro you d>ooc. ., son pencil on !he Optic.J Answar Sheet.
4. My rough working should be done In 1t.s 0u..uon Pa~r
An object falls freely from rest at point X to the ground at point z. Express the
5. Throughout the paper, the accelerati on d ue to gravity on Earth 1s taken as 10 m/s2 unless stated distance between Y and Z 1n terms of a.
otherwise_
6.
7.

Catc::u&ators may b9 uHd where nec.essary Whe re numeric.Al a nswe rs are not u.act give
answers to !!)!tt ()) slgnlncant figures
At the end of the exam1nahon. arrange yout answers to Section C In o rder Tie your Question

t=0 s
t: 1 s

x
y
-

i a
Paper loosely on top of your writing p aper using the string provided. A a
B 2a ?
a. Submit tho Optic al An1wer Sheet and thl1 booklet SEPARATELY at the end of the examination
c 10-a
D 20 - a t=2s z

4 An astronaut has a mass of 60 kg on Earth He can jump 1 m high on the surface of
Earth.
Which of the following statements is correct about the distance he can jump on the
moon and the corresponding reason? Assume that the acx:eleration due to gravity
on the moon 1s 1.6 mis'.
Distance he can jump on moon Reason
A higher than 1 m his mass is less on the moon
B higher than 1 m his weight Is less on the moon
This pa~r cottsirts of H prillt~d pag~ including th~ cover page c lower than 1m his mass is more on the moon
0 lower than 1m his weight is more on the moon
2
Two ice skaters at rest at a skating rink are pushed by a 200 N force exerted by a 9 The figures below show the posobon of the centre of gravity of each standing lamina.
third person as shown in the figure_ As a result, they move together with an Given that the laminas have the same we1ght. which or the laminas shOwn below is
accelerabon of 1.6 mls2 . If the mass of A is 50 kg, what is the mass of B? the most stable?
AB A c
&~ ----~----
OON , ~ -_ 16
_ m/,
a ______ ..,.s
gr~~--ice ~U - -
-e----
c
---f]-
A 50 kg B 62_5 kg 75kg D 125 kg B 0
6 Ten identical steel balls, each of mass 24 g, were ccmpletely immersed in water In a
3
measuring cylinder containing 30 cm of water. The reading of the water level
3
changed to 68 cm .
What Is the density of steel?
3 3
A 0.36g.'cm' B 0.67g/cm3 C 3.64 g.tcm D 6.67 g.'cm 1Cl What are the energy changes 1n hydroelectric power production?
A electrical -> kinetic ..... potenual
-: A uniform metre rule is balanced at its midi)Oint P on a pivot. A 1O N weight is hung B electrical ..... potential ..... kinetic
from the rule at 11Brious positions_ Which moment or force about pomt P s not c kinetic ..... potent1al ..... electrocal
posstble? t> potential ..... kinetic ..... electrical
A ONm B 1Nm C 5Nm 0 IONm
A car of mass m has an engine which can dehver power P. Whal is the minimum
time in which the car can be aocelerated from rest to a speed V?
A load of 9 O N 1s placed on a light rod which is suspended from two Newton meters
P and Q as shOwn In the figure. The length of the rod is 36 0 cm. A mv+P C m..J + 2P
B P+mv 0 2P + m..J
p ~ Q
Water and mercury are placed in either stde of a manometer. Which of the following
diagrams correctly shows this? (Diagrams are not drawn to scale.)
waterilll
mercury f:J.v j
>-"'~
9.0 N
Which of the following gives the correct readings of both Newton meters?
Reading of P Reading of Q
A 3.0 N 6.0 N B c 0
B 4.S N 4.5 N
c 6.0 l'I 3.0 N
0 9.0N 9.0N
3 4
13 The figure below shows a simple mercury barometer. 1S Fibre-glass covenngs laid on the floor. walls and ceiling of a house can greatly
reduce heat lost to the surroundings. Fibre-glass consists of a large amount of fine
glass fibres.
If the base area of the trough is 1.5 x 10.J m'. find the orce exerted on the base of
the barometer (Density ot mercury = 13 600 kg m .J)
A 1.6x 1o' N C.. 6.9x 107 N
Et 1.8 x 102 N o a.ox 10' N
14 Two flasks are connected via a tap T as shown in the diagram below. One of the
flasks contalns a gas while the other a vKuum. Which of the following is/are the reason(s) for this?
~ I The specific heat capacity of glass fibres 1s small
:-:::::==: _::i T i---~ ...
11 - A
B

II
Ill
The glass fibres INl:h trapped air are very poor conductors of heat.
Fibre-glass reflects heat radiation.
lonly
ll only
C
0
I and II only
I and Ill only
\
\
; vacuum
17 There are two parallel walls facing each otller, INlth one wall hotter than the other.
Which of the following wil allow heat to be transferred most qu1C1dy from the hotter
to the cooler wall by radiation?
When tap Tis opened, which of the following properties does not change?
Hotter wall Cooler wan
I Temperature, A dull and black dull and blaek
II Density B dull and black shiny
Ill PressurE; c shiny dull and blaek
IV Mass 0 Shiny Shiny
s IV only
Ill and IV only
c
0
II, Ill and IV only
All of the above 18 A student investigates the changes that may take place as wax solidifies. Which of
the following describes these changes?
15 Which statement correctly explains what happens to a used spray paint can 1f It 1s /I. A transfer of energy and a change in temperature.
thrown into a fire? B A transfer of energy but no change temperature.
c No transfer of energy and no change in temperature
A It will not explode since the rema1 ning paint is not flammable at all. o No transfer of energy but a change in temperature
B It may explOde as heat from tne lire may cause the remaining paint to
expand and explode.
C It Will not explode as the high pressure within the can prevents any
evaporation of the paint even under extreme heat
O It may explode as evaporated molecules of the remaining paint may result 1n
a pressixe too large for the can to handle.
5 6
19 Wh1cl1 of the follOINing solids will produce a convection current when placed 1n a 22 Two copper cans are filled with boiling water as shovm. One can is lagged with
beaker of water. of density 1 kg/ml. which is at room temperature? cotton wool The temperature of the water in each can is taken every minute.
Density (kg/ml) Temperature (K)
I 0.6 280
II 0.6 350
Ill 1.4 280
IV 1.4 350 hot water
A IV only c II and Ill only coppe~n~.(;'!~~~~
B I and IV only D II and IV only
can 1 can 2
20 Julian pours some 95'C water into a cup of instant noodles. The final temperature of Which graph best shows the results obtained?
the mixture is lower than 95'C. Which of the following statements isl are possible
explanation(s)? A c
The temperature of the noodles is lower than that of the hot water. Thus ten;:b
C)
rature(
energy is absorbed by the noodles. '
II Energy is lost to the surroundings.
room ' ....... can 2
Ill The mass Of the mixture is larger than the total mass Of the water and the temp - -------can 1
noodles.
C t and II only
O time
A lonly
B llonly o II and Ill only B D
te~=~ature(
'C) te~:~rature('C)
21 The positions of the mercury threads in a thermometer is as shown.
room
. ' room ' , ,, ..)..
temp '\~ ....... ._... can 2 temp ',__ can 1
n2 - --
o time 0 bme
boiling
23 A student is given a thermometer which reads -1' C for ice point and 99 C for the
steam point He measures the temperature of water before and after heating. Which
water
of these is correct?
First reading Second reading Calculated temperature rise
A tootow too tow. correct
What 1s the distance between each 1 c mark on the thermometer? B too high too high too low
c too low too high too high
A 0.22cm B 0 .24cm C 2.0cm o 2.2cm D too high too low correct
7 8
~~ 1lle diagram shows a plane muror QR inclined at 110' to another plane mirror PQ. 27 The following diagram illustrates circular wavefronts radiating from a point source O
R
p
What Is the angle of reflection at mirror QR?
e so c so 0 70
z:. The figure below shows parallel rays from a distant obJect and passes through lens If the time taken for a wavefront to travel from N to P is 1 O s, and the wavelength of
L1
and L,. the waves is 2 m. the speed of the waves, in mis 1s _ _ _--:
A 0.20 ~ 0.80 c 1.00 0 1.25
ta Which of the following applications use gamma rays?
I detect flaws in metals
II killing cancer cells.
What are the pnnopal foci of L, and L2? Ill radio broadcasting
IV stenhsing
A x, and X2 C X2 for bOth lens
e x, and x, O X,and Xl
A II only c I anCI II only
B IV only D I, II and Ill only
26 Which of the followrng does not change as light passes through the glass window
pane? 29 Which one of the following graphs shows the correct relationship between the
frequency, f, of an electromagnetic wave and its wavelength, J..?
light=:::-..
rays-...=:::
A c
A direction B frequency C velocity 0 wavelength
B D
9 10
2
30 Judy stands 55 m from a wall and knocks two wooden blocks together. When the 33 The length and sectional area of a metal wire are 2 m and 5 cm respectively. If the
frequency of knocking is 3 knocks per second, the echo of a knock 1s heard at the resistance of this wire is 3 O. whlCh of the following wire made from the same metal
instant of the next knock. will have a resistance of 6 0?
What 1s the speed of sound in air? Length Im Sectional Area I cm'
A 2 2.5
A 165 mis B 330 mis c 340m/s D 495 mis B 2 10
c 4 2.5
) 4 10
31 A poSttively Charged rod is brought near two neutral spheres X and Y, which are in
contact. Y is momentarily earthed. Z is then removed.
34 In the circuit below, all resistors are equal
R
p R
z
++++ -------1 11
R R
What happens to the cnarge distribution on X and Y?
If airrent at P is 1.0 A, what is the current supplied by the battery?
x y
A negative negalive A 1.0A lil 2.0A C 2.5A D 3.0A
8 negative positive
C? positive
' positive
D pos1uve negatiVe
35 In whlCh of the following orcu1ts is the lamp hghted?
32 A ptotting compass is placed above a wire.
A c
M N
When a current flows from M to N , what direction would the needle In the compass
point?
CD CD 8
A B c
8 D

B D
II 12
36 The cost of running a 2 kW heater and two identical light bulbs, rated at 1oo W 40 A time base voltage is applied to the X-plates and the trace of the output voltage
each, for 4 hours is $ 2.20. Find the cost of running the same heater and three such from a transformer connected to the Y-plates is obtained as shown.
tight bulbs for 2 days, each day for 6 hours.
c

A $5 82 IJ $6.90 $7 18 0 $7.57
l7 A house-ov.ner replaced a blown fuse for the lights of the house. When the lights
were switched on, the new fuse also failed T'le house-owner then put in another
fuse with a higher rating then the pre111ous two Why is this not a sensible thing to
do? What will be the new trace if the time-base os hatved?
A The fuse has already melted be<:ause the raung was too high.

B Fuses allow the circuit to work only if the rating is exactly right.
c Using a fuse with too high a rating would cause electric shocks.
D A fuse with a higher rating might allow the circuit to work, but the fault would
not be corrected.
A B c 0
36 Which of the following statements describes an example of induced magnetism?
ii A bar magnet attracts a piece of soft iron
a A bar magnet loses its magnetism if 1t is repeatedly dropped.
- End of Paper -
C Two North poles repel each other, but a North pole attracts a South pole.
D A bar magnet, swingiog freely, comes to rest poinbng in a North-South
dtrectoon.
39 A multi-turn coil is rotated in a magnetic field Which set Of changes would increase
the voltage by the greatest amount?
speed of rotation of number of turns of strength of
the coil colt magnetic field
A decrease increase decrease
B increase decrease decrease
C': increase. Increase decrease
D increase' increase increase
13 14
Name : Index No: Class:
SECTIO N A : (50 marks}
Answer ALL questions in this section. Show your working and write your answers in the
ST. PATRICK'S SCHOOL space provided.
PRELIMINARY EXAMINATIONS 2010
1. A lorry, of total mass 3000 kg, was travelling on a road at a constant speed of
40 mis. The driver sees a wall ahead of him and starts to decelerate his
vehicle at a constant rate.
SUbjed PHYSICS 5058 PAPER 2 Date 25AUG 2010
(a) Given that the maximum braking force between the lorry and the road
Level SECONDARY 4 EXPRESS Time 1HR45 MIN surface 1s 0.72 times the weight of the lorry,
INSTRUCTIONS TO CANDIDATES
\IJ calculate the maximum braking force,
DO NOT OPEN THIS BOOKLET UNTIL YOU ARE TOLD TO DO SO.
1. Wnt& your name. class and dass index number at lhe top of this page and on any writing pap8f used
For Section A : Answer ALL questions. Write your answers 1n the spaces provided.
3. For Section B: Answer ALL questions on lhe writing paper provided. Questioo 13 is an EITHER I
OR QUESTION SELECT ONLY ONE PART OF THIS QUESTION
Maximum braking force=---~--- I1I
4. Throughout the paper, the acceleration due to gravity on Earth IS taken as 10 rn1s2 unless sblted
otherwl. . (h) the minimum braking distance that the lorry takes to come to rest,
and
5 Al The End Of The ExamlnaUon: 00 NOT DETACH any HCIJOOS from lhs paper. Arrange yOU<
iinswtra to Se1:1ion II in order. Tit yQ\!r Question Paper loosely on top of your writing pper
INFORMATION FOR CANDIDATES
FOR EXAMINER'S USE
The number of marks is gtven 1n brackets [ J at the end of each quesbOn '
pan question. Section A
Candldates are rem1~ed that ALL quantitative answers should indude
appropriate unils. Section B
Minimum braking distance = ____,__ _ 12 I
Candidates are advlsed to show all their working in a clear and ordet1y
manner. as mOfe ma~s are a'W'arde-d for sound use of phySfca than for Qn.11 (iii) hence, the time which tJ-e lony takes to come to rest.
rorrect answers.
Cala.ilat's mey be used where necessary. Where numerlcal 1nawers
ilre not er.act, glve nwers to Three (3) signmcant figures Qn.12
Qn.13
TOTAL --S-o
T ime=-- - - - - 11 I
100% Grade
{II; Give a possible reason why, ir. reality, the driver takes a longer time to
stop the lorry than the time calculated in the previous part.
Parent's I Guardian's Signature : - - - - - -
l;!lls.Eriiiii'ti-~,s"'~18 printtdpages. -t~.,:!#;:~~.!...... (1l
SL Patrick's School Page 2of18
2. The figure below shows a kite of mass 3.5 kg flying at a con stant speed of (bJ Calculate the final temperature reached by the tea and ice.
4 mis In the sky. The force of the wind can be taken as a single force of 75 N
acting at an angle of 40 to the horizontal.
Final temperature = _ _ _ __ (2)
(c) What mass of ice cube is needed to further reduce the temperature of
the mixture by 15 C?
(a) Explain why the resultant force on the kite is zero.
I1I
Mass= _ _ __ _ [3]
(b ) By means of a scaled diagram. find the magnitude o f the air resistance
acting on the kite.
4. The figure below shows a simple hydraulic press. The cross-sectional area of
the left limb and right limb are 3500 cm2 and 800 cm2 respectively. Take
atmospheric pressure= 1.0 ><10' Pa, density of water= 1000 kgtm'. density
of oil = 920 kglm'
A force of 24 N is applied on the right limb, which is sufficient to raise a heavy
load on the large piston.
Air resistan c e = - - - - - ( 3)
3. Two ice cubes. each of mass 0.02 kg, are dropped into a cup of hot tea,
initially at a temperature of 45 c. It can be assumed that no heat is lost to the '~I Calculate the pressure exerted by the small piston on the liquid at A
surrounding. and B.
Heat capacity of tea = 900 J/K :
Specific heat capacity of water " 4200 JlkgK ;
Specific latent heat of fusion of ice = 336000 Jlkg.
~ State the energy req~ired to lower the temperature of the tea by 1 c.
Pressure at A = - - - - - [ 1 l
_ _ _ __
Pressure at B = I1 I
Ene~y=
I1 I
St Patrick's School Page 3of18 SL Patrick's School Page 4of 18
(b) What is the max1roJm load that can be lifteo by this force? (aj Determine the value of
(I) 9. and
(fl) J'l.
Maximum load = - [2]
9= _ __ _ _
(c) Explain how your answer to par. (bl will change 'f
I1 I
p (t 1
(i) It e cross-sectional areas cf both limbs are doubled.
111) Assuming that the sheath has a refractive index of 1.00, what is the
minimurr value of an~ le a for total "ntem.:: refraction to take place?
I2 I
(ii) the liquid .n the hydraulic press Is changed to water.
[1 l
a= (t )
{rl) Ap'n from the assumption t!iat the iquid is incompressible, state one
other assumption in performing tre calcu:a;ions for the hydraulic press
above. Wrtv would it be desirable for the core of the fibre to have a high
refractive index?
I1I
[' J
s. Light enters the core of an optical fibre with refractive index of 1.38 as shown
in the figure below. The figure is not drawn to scale. (i1) State two advantages of using optical fibres rather than copper wires for
carrying telephone communicatior.s other than the cost inccrred.
sheath
(2 J
St Palnck"s School Page 5 ol 18 Page 6ol18
6. The figure below shows a simple thermocouple connected to a very sensitive (cj Can the sound pulse be heard by a human being? Explain your answer.
m'.llivoltmeter.
[2 I
8. An electrostatic generator is used to produce sparks. The belt carries
negative charges to the dome. making it negatively charged.
L M metal
dome metal
negatively _ discharge
When !Unction L and M are placed inside melting ice and boiling water charged ball
respectively, the millivoltmeter registers a reading of 25 mV. With junction M
still placed in boiling water, junction L is placed into a beaker of liquid. The
belt
reading on the millivoltmeter changes to 15 mV. Calculate the temperature of
the liquid
wire connecting rod to base
(o) Before a spark is produced, the discharge ball becomes positively
charged.
Describe and explain the movement of the electrons in the discharge ball
Temperature" _ _ _ __ and the conducting rod as the ball becomes charged.
[ 21
1. A sonic tape measure is used to measure the length of an empty hall. There
is a time interval of 0.36 s betweer the transmission of a sound pulse, of
wavelength 1O mm, and receiving an echo. 12 I
la) Given that the speed of sound in a;r is 340 mis. find the length of the
1.l>i Mark an X on the diagram, to show where there is the rnost positive
hall.
charge on the discharge ball. I 1I
Le n g t h " - - - - - 12 I
{b) Explain why this method cannot be used to measure the length of the
hall if it is not empty.
I1I
SL Patrick's School Page 7 of 18 St. Patrick's School Page 8 of 18
9. The figure below shows a fisherman catching a fish_His left hand acts as the (b) As the fisherman lifts 1..p the fish, he gets into a position similar to that
pivot while his right hand exerts a force F to lift the fish above the water shown ir the figure below
surface. The fish has a mass of 600 g_
The figure below shows a simplified diagram of the one above.
Ft..----.
. !
0.4..........._..m
Explain how the new position of the fishing rod affect the size of tt.e
new fcrce. F', exerted by the fisherman.
t
weight of fish

1.5 m
pivot
[2 J
l'l) Calcu:a'.e tne force, F, exerted by the fisherman. {ii) I' the end of the rod where the fish is hanging is 0.6 m vertically
above tJe horizontal, calculate F'.
f; (2 J
f'; - - - - - [3 J
St i>alriclc's School Page 9of16 SL Patrtclc's School Page 10 of 18
10. A conductor connected to a d.c. source is suspended from two insulating SECTION B : [30 marks]
threads and placed between two magnets, as shown in the figure below.
Each question is 10 marlls. Answer All quest10ns m this section. Question 13 is an
EITHER I OR QUESTION. SELECT ONLY ONE PART OF QUESTION 13. Show your
working and write your answe~ in the writing paper provided
11. {a) An ex:periment was set up as shown in the figure below.
(a) The switch Is closed.
(l) State the direction in which the conductor will move.
200 turns
[1 I
(JI) It was observed that the conductor did not move in the direction it is A student followed one of the steps of the experimental procedure and
supposed to. Suggest a reason why this is so. found that the galvanometer shows a deflection even though a d.c.
supply was used. He also reafises that when he stopped carrying out th.e
step, there is no longer any deflection in the galvanometer.
l1 I (i} What was the action which the student dtd to enable the
(Ill) ...Suggest a way to ensure that the conductor moves in the direction galvanometer to have a deflection? [1I
11 should be moving.
(ii) Explain why this action results in the galvanometer showing a
deflection. [2I
[1I
(b) Describe how the magnets should be repositioned so that the conductor
moves to the right.
l1I
SI Paltiele"s School Page 11 of18 St. Pa11ick's School Page 12 ol 18
(b) Another student designed a circuit which indudes a transformer. His 12. Engineers investigating the performance of a small d.c. motor used in the
objective is lo tum on both bulbs when switch S is dosed The a c. arrangement shown in the front and side View diagrams below. As the output
supply 1s connected in series with bulb Y. The primal) coils I and II eaeh shall turns. the thread wraps itself evenly around the shall and the load is
have the same number of turns and are wound in opposite d1rect1ons. raised at a constant speed from the noor.
His setup is shown in the figure below.
(-i) Explain why bulb X does not light up when the switch is closed. [2] The following measurements were made during the experiment.
(ff) State one modification which can be made to the primary coils Diameter of output shaft 7.0mm
which will result in both bulbs lighting up when the switch is closed. [ 1] Mass of load 0.50 kg
Height through which load is raised 2.20m
(c) High voltage cables are used to transmit electrical energy from the Time taken sos
power station to the households through 2 transfomers. The power
station produces electricity at a voltage of 25 kV. The voltage in the 1$.l Calculate each of the following quantities, assuming that tlie thickness of
cable 1s 300 kV while the voltage supplied to the households is 240 V. the thread can be ignored.
Both transformers are assumed to be ideal.
(i) The number of revolutions of the shall to raise the load through
High voltage cables 2.20m.
Power
ECJ \
Transformer A
CJ====i
Transformer B Households

(II) The speed of rotation of the output shaft 1n revolubons per second.
\Iii) The useful power of the motor.
station
liv) The power input 1f the motor has an efficiency of 72%.
(i) Calculate the tum ratios (primary to secondary) of both
transformers and state what type of transformers they are. (2 ] (b) At the instance when the load was raised to a height of 3.5 m, the thread
snapped.
(II) The total resistance of the high voltage cables is 1000 0 and the
electrical power generated by the power station is 450 kW. {I) State the Principle of Conservation of Energy.
Calculate the power loss in the high voltage cables. [2J
(Ii) Determine the speed of the load when it crashed onto the floor.
SI. Palrle1<'s School Page 13of18 SI. PalriCl<'s Scllool Page 14 of 1a
(c) -he simple machine has been modified such that a handlebar is useo to Question 13 is an EITHER I OR question.
rotate the shaft to lift the load by holding onto the grip. You are to do EITHER Part I OR Part II.
Either
13. Part I
output
shaft A light-dependent resistor (LDR) is used as part a sensing unit for an alarm
that will sound when the light intensity Is too low.
thread
(a) Comment on how the resistance of a LDR changes with different
intensity of light. [1I
12.0 V
Giving a reason. explain the effect on the force required to rotate the
shaft when the following modificatiors are made independently.
(I) Increase the length of the handlebar. (1)
/1;) Reduce the diameter of the grip. (1) :-------------:
' LDR :--'ip-- - ;
-----------
To alarm
INith the rheostat set at ~ the potential difference across the alarm is
being monitored for 3 minutes. The data collected is shown in the table below.
It is observed that the alarm starts to sound only from the second minu1e.
p.d. across
nme/min LOR resistance I Cl Current flow I A
alarm/ V
1 0.48
2
- 8 .0
3 30
-
,
(b) Copy and complete the table above. [ 3)
(t) Explain, in detail, the purpose of the rheostat in the circuit and what
happens when the resistance of the rheostat is ircreased. [ 2)
School Page 15of18 St Patrick's Scllool Page 16 o' 18
(P) The LDR has a maximum resistance range of 150 Q while the rheostat (b) The figure below shows a schematic diagram of the balloon.
can be adjusted to have values between 0 - 100 0.
(I) By considering the entire balloon and its carriage as a single object.
What is the maximum setting for the rheostat, such that the alarm will copy the diagram below and draw labelled arrows to represent the
not be disarmed? [3J forces acting on the balloon as ~ is moving upwards. [1J
(e) Why IS 1t not safe to adjust the resistance of the rheostat to a value
close to zero? [1J
OR
Elastic cable which attaches
13. Part II
the balloon to the ground
In New Zealand, the hot-air balloon is an aerial ride which uses a helium
balloon to bring tourists up to the sky. The unladen weight (weight without
{U~ If the tension of the elastic cable is 650 N when the unladen balloon
passengers) of the balloon is 4QQJ)J. The speed at which the balloon drifts up
is controlled by an elastic cableattached to the ground. The balloon is lifted is moving up at the time indicated by AB in the speed-lime graph,
by an upthrust due to the density of helium. This upthrust is constant calculate the upthrust of the balloon. [ 2J
throughout the motion of the balloon.
!ff!) Calculate the tension in the elastic cable when the balloon is
(a) The figure below shows the speed-time graph of the balloon. Point D moving up at constant speed. I2I
indicates the bme when the hot-air balloon comes to rest at the peak.
{c) Explain the effect on the tension of the elastic cable on a very windy day. [1J
speed (mis)
End of paper
B
15 .
10
5
A
0
50
... 100 150 200 250
time(s)
300
(II Calculate the maximum height reached by the balloon. I2 J
(II) Determine the initial acceleration of the hot-air balloon. I2J
SL Palrick's School Page 17of18 St. Palnck's School Page 18 of 18
Answer for SA2 Physics 4E 2010 3. (a) Energy required = 900 J [1l
Paper 1 (b) Energy gained by ice Energy lost by tea
2 ((0.02) (336000) + (0.02) 900 (45 - O) [1)
1 2 3 I 4 5 6 7 8 9 10 (4200) (0 - OJ]
~ D D B c D D c A D e 25.3 C [1)
11 12 13 14 15 16 17 18 19 20
c A D A D B A B B c {c) Energy gained by new mass of m {336000) + m {4200) (25.3 -
21 22 23 24 25 26 27 28 29 30 ice 15)
A A A B D B B A D B 379260m J [ 1]
31 32 33 34 35 36 37 38 39 40 Energy lost by mixture 2 (0.02) (4200) (15) + (900) (15)
A B A c c B D A D A 16020 J [1l
Mass of ice required 16020 I 379260
0.0422 kg [1l
Paper 2
Section A
4. {a) Pressure at A 24
In general. 0.08
no/wrong unit. deduct 1 mark 300 Pa [1]
wrong sig fig.m deduct 1 mark Pressure at B 300 Pa [1l
1. (a) (i) max braking force 0 .72 x 3000 x 10 (b) Maximum load 300x0.35 [1]
21600 N I1I 105Nor10.5 kg [1I
{ii) 21600 x d 0.5 (3000) (40)2 [1l {c) (i) Th\'![~ Will!>~ ng change. ( 1]
d 111m(ecf) (1] T his is because a resultant smaller pressure at A due to the
Ded1.:ct 1 marl< if a negative sign was not used when using increase in area of the small piston will still lead 10 the same
gradient to find answer. .maximum load as the large piston has also twice its original area. [1I
{iii) 0.5 (40) (t) 111 (ii) There will be no change as water is incompressible as well. [1)
5.55 s (ecf) (1 l
(d) The assumption is that the weight of the pistons are negligible. [1 I
(b) Reaction time of the driver [1l or
There is no air trapped in the liquid.
2. (a) The kite is moving at a constant speed . [1]
or 5. {a) {i) sin40
1.38
The kite is moving with zero acceleration. sin O
e 21.s [1]
Correct diagram I1I
Correct arrow for resistance (1 ) {ii) I} 62.2 [1]
(b)
c = . -'(- l )
sm
1.38
46.4 or 46.5 [1 l
Air resistance 59 N [1 ]
Accept 57 - 61 N (c) With a high refractive index. the critical angle will be smaller. thus it wul
be easier for total internal reflection to take place. [1J
or 8. (a) Electrons from the metal discharge ball move from left to right and down
So that 0 is smaller and a will l>e larger. the conducting rod. 11 I
This <S due to the force of repulsion from the negatively-charged metal
(d) Any2: dome, which resulted in the electrons moving down to earth. (1 ]
There is less signal loss in transmission through optical fibres than that The 2 mafRs are mdepen;JtJnt.
through corwentional copper wires.
Optical fibres can carry nore information than conventional copper (b) [1 l
wires
Transm1ss1on can take place faster. metal
Signal can be transmitted over a longer distance. dome metal
Lower energy loss during transmission. [2 J negatively _ diseharge
charged ball
6. Temperature difference belt
_!2.x 100
25
60 C [1 l
T emperature of liquid 100 - 60
40C [I I -
9. (a) Taking moment about the pivot,
7. (a) 2 x length of room 340 x0.36 [1l
sum of dockw1se moment sum of a nti-ciockwise moment
ll!flgth of room ;; 61.2 m { 1]
F x 0.4 6 x 1.5 [1l
(b) The wave will bounce off the things in the room such that many pulses F 22.5 N f1I
of echo will be received. I1 I (b) (I) The anti-dockwise moment of the hsh about the pivot decreases
(c) Frequency of wave 340
(0.5) since the perpendicular distance decreases [0.5). 11 I
Thus. F' is smaller than Fas a smaller dockwise moment [0.5] is
0.01 required to overcome the smaller anti-dockwise moment (0.5]. [ 1)
34000 Hz I1I
It is necessary to show frequency is 34000 Hz. (ii) Perpendicular distance
Wave cannot be heard as it is above the audible frequency of human J(tsY-(o6f
beings. l1I 1.37 m I1I
The 2 marks are independent.
sum of dod<.wise moment sum of anti-dockwise moment
F' x 0.4 6 x 1.37 I1I
F' 20.6 N [1 I
10. (a) (i) The conductor will move upwards. [1 l
{ii) The weight of the conductor Is greater than lhe force due to the
magnetic field and current [1l
(i ii) Use stronger magnets, or
Increase the current of the circuit, or
Increase the current of the circuit by increasing lhe voltage of the
circuit !1 I
(b) Place the south pole above the conductor and north pole below lhe No. of revolutions 22
conductor. [1 I 002198
100 [1l
Sectio n 8 (ii) Speed 100
11. (a) (i) Adjusting the resistance of the variable resistor to vary the 5
20 rev/s (ecf) [1]
current flowing in the circuit. [1 J
(Ill) Useful power (o.sx1ox2.2)
(ii) When the current through the primary coil changes. the magnetic
fied through the iron core will change in strength. [1l 5
The changing magnetic field w1ll 1nduce an e.m.f., which results in 2.2W I1I
an induced current in the secondary coil. [1)
The 2 marks are independent. (iv) Power input IOO x2 2
72
(b) (i) The magnetic field in coll I and II cancels each other out. [1 J 3.06W(ecf) [1l
This will result in no change in magnetic field cutting the
secondary coil, which results 1n little or no current induced in the (b) (I) Principle of conservation of energy states that energy cannot be
secondary coil, according to Faraday's Law. (1 l created or destroyed but converted from one form to another. [1]
The 2 marks are independent. No half marl< to be given
(ii) Change the coil orientation in either of the primary coil such that it (ii) By conservation of energy,
will produce a south pole at 8 or 0 .
or ~(o.sXv)' (o.sx10X3 :s) [1l
Remove one of the primary coils v 8.37 mis [ 1l
or
Double the number of turns on one of the primary coils [1] (c) (i) Force required w ill be smaller [0.5) as the longer distance to the
pivot requires less force to produce the same tu ming effect [0.5]. [1 J
(c} (I) Transformer A 25 The 2 half marks are independent
300
0.0833 or 1 : 12 [0.5) (ii) No effect on the force. I1I
step-up (0.5]
Transformer B 300000
240 13. (a) Resistance of LOR decreases with increased intensity of light. [1l
1 250 or 1250 : 1 ( 0.5)
step-down [ 0.5 J (b) [ 3]
(ii) 450000 p.d . across
Time/ mi n LOR resistance In Current flow I A
alarm/V
300000
1.5 A I1I 1 I 15 7.2 0.48
I
Power loss (1 :5)'(1000)
2 20 8.0 0.4
= 2250W [1)
3 30 9.0 0.3
2 ~(7x~o-')
12. (a) (I) Height raise in 1 rev
(c) The function of the rheostat is to adjust the sensitivity of the alarm. [1 l
When the resistance of the rheostat increases, the resistance of the
0.0220m [1 J LDR has to be higher in order to trigger the alarm.
or
When the resistance of the rheostat increases, the intensity o' light will
have to be lower in order to trigger the alarm. (1)
(d) A minimum p.d. of 8 V across the LOR is required to activate the alarm.
Max p.d. across rheostat 12 8
4v [1J
_R_xl2 4 [1 )
R+150
R 750 (1)
(e) The current will be too large, especially in situations when the intensity
of light is high. [1)
13. (a) (I) Maximum height
.!.(2so+sox1s) (1 J
2
2480 m [1J
(II) Acceleration 15
100 [1 J
0.15 m/s2 (1I
(b) (I) 2 forces correct: 0.5 mark I1I
3 forces correct: 1 mark
Ignore other forces.
(ii) Upthrust - 400 650 {40Xo.1s) (1 )
1056 N (1)
(Ill) 1056 - 400 tension 0 !1I
656 N I1I
(c) Tension will increase (0.5) as it will need to balance the additional force
(0.5] due to the wind. (1]
482
Index Number. Class:
Fig. 1a shows the scales of a pair of vernier calipers when its two jaws are
fully closed. Fig. 1b shows the same scales when 1fil) pieces of vanguard
TEMASEK SECONDARY SCHOOL sheets are placed between the two jaws wh ich are tightly in touch with the
0 Level Preliminary Examinations 2010 stack of vanguard sheets.
4
PHYSICS 5058101
Paper 1 Multiple Choice 1 hour
1111111 " I .. 1111_,11111
0 5 10 10
Additional Material: Multiple Choice Answer Sheet
Fig. 1a Fig.1b
READ THESE INSTRUCTIONS FIRST
What is the thickness of one vanguard sheet?
Do not open the booklet until you are told to do so.
A 3.47 x 102 cm .
Hand in this booklet at the end of the paper. B 3.54 x 10'2 cm
C 3.57 x 102 cm
Write your name, index number and class in all the work you hand 1n 0 3.61x102 cm
W rite in soft pencil.
2 A car travelling at 10 mis is found to have an oil leak. It is then braked
There are forty questions on this paper. Answer all questions. uniformly to a stop after moving 15 m. From the point it braked, 31 oil
marks are found on the road.
For each question there are four possible answers A, B. C and D.
Choose the one you consider correct and record your choice in soft pencil on the What is the rate of the oil leak in drops per second?
separate Answer sheet.
A 2 B 5 c 10 Wl 20
Read the instructions on the Answer Sheet very carefully.
Each correct answer will score one mark. A mark will not be deducted for a wrong 3 Fig. 3 shows a trolley that starts rolling down a sloping smooth runway
answer. connected to a flat rough floor.
Any rough working should be done in this booklet.
(Take the gravitational field strength of the earth, g, to be 10 Nlkg)
Fig. 3
Which of the following velocity-time graphs best illustrates the motion of the
trolley?
velocity velocity velocity velocity
This document consists of 16 printed pages and O blank page. B c D
481 482
---------- - - - - - - - - - - - - - - - - - - --- - - - - - - -
483 484
4 An object of weight 12 .0 N is attached to one end of a rope and hung from 7 The diagram shows the brake pedal of a car.
a ceiling. The rope is pulled to one side by a horizontal force of 16.0 N.
liZi!l~~~~ Ceiling
Rope
)
If a force of 800 N is applied at the pedal, what is the force F transmitted to
12.0 N the brakes?
What is the tension in the rope attached to the ceiling? A 128N B 5000 N C 10 000 N D 32 000 N
A 14.0 N B 18.0 N C 20.0 N D 28.0 N
8 The diagram shows a balancing toy which consists of t he aeroplanes and
the weight pivoted on a stand.
3&ropla.MS
5 A 4.0 kg wooden block is pu lled along a horizontal ground from rest. and a -
force of 15 N is required to produce an acceleration of 2.0 m/s2 . The force is ~
then changed and produces a constant speed of 5.0 m/s on the wooden
block on the same ground. What is the magnitude of the new force?
A SO N B 7.0N C 8.0 N D 10.0 N weight
6 The diagram below shows two solid cylinders made of two different If slightly tilted, the toy does not turn over b ut returns to its original position
materials. One cylinder has a diameter that is twice as b ig as the other. instead. Th is is because
The height of both cylinders is the same.
A The centre of gravity of the toy is slightly below the pivot.
x B The centre of gravity of the toy is fixed exactly at the p ivot.
~
C The centre of gravity of the toy lies at the centre of the weight.
D The centre of gravity of the toy moves between the aeroplanes.
9 A block of mass 2 kg is at rest at a height of 5 m on an inclined plane. The
length of the slope is 10 m. It is then allowed to slide down the inclined
If the weight for both cylinders is W, what is the ratio of the density of the plane.
2 kg
larger cylinder to that of the smaller one?
A 1 :2 B 1:4 c 2: 1 D 4 :1
I 5m
W///////////#////////////////1 W//##////////.WAW##/#///////////////Q////////////~
When the block reaches the ground, it moves at a speed of 5 mis. What is
the work done against friction on the inclined p lane?
A 25 J B 75 J c 100 J D 175 J
483 484
- - --- - - -- - - - - - - - - - -- - - - - - - - - - - -- - - ' - - - - - - - -
485 486
10 At the recent Youth Olympic Games, the performance data of four weight- 14 A ray of light travels from medium X, through air. and enters medium Y.
lifters were recorded as shown below. Which weight-lifter generated the The refractive index of medium X and medium Y is denoted as n , and n,
largest average power? respectively.
ht-Lifter Wei ht lifted/N Hei ht liftedlm Time taken/s
. 6:. 120
140
1.6 0.6
x
8 1.8 0.8
l
c 160 1.8 1.0 ~'
D 200 1.6 1.2
Air
11 A column of liquid M floats on water in a manometer of uniform cross-
section area. y
Which relationship correctly describe the magnitude of n,and n, ?
A 1 > n ... > ny;
8 n , > n,. > 1
n,, > n, > 1
water c::
0
n , > 1 > n ,.
3
If the density of water is 1000 kg/m , what 1s the density of liquid M?
15 A ray of light is incident on each of the 3 glass devices shown below. The
3 3
A 50 kg/m3 B 500 kg/m C 1000 kglm 3 D 2000 kg/m critical angle of glass is 42.
12 The pressure of 1O m of water in a lake is equal to the atmospheric
pressure of 76 cm Hg. An air bubble rises to the surface of the lake which
3
is 20 m deep. The volume of the air bubble is 6 cm at the bottom of the
lake.
centre
(1) (2) (3)
What is the volume of the air bubble at the top of the lake?
12 cm3
3 3
A 2cm3 8 3cm C D 18cm In which of the d1agram(s) is/are the path of the light ray correct?
A (2) only.
13 Light waves change their wavelengths when traveling from one medium B (3) only.
7
into another. In an experiment, a ray of red light of wavelength 7.0 x 10 m C (1) and (3) only.
is incident from air into glass of refractive index of 1.50. What is the D (2) and (3) only.
wavelength of the red light in the glass?
7 6
A 2.0x10"8 m B 4.5x101 m C 4.7x10" m D 1.1x10" m
485 486
- - ---------- -- -----~----
487 488
16 At a particular instant, a transverse wave of frequency 12.5 Hz 1s travelling 19 Joey stands 1n front of a large wall. She claps two pieces of wood together
to the right, as shown in the diagram below. At that instant, the regularly at an interval of 0.8 s. She can hear the echo of the first clap
displacement at point P is zero. when she makes the third clap.
~C~++t-~
If the speed of sound in air is 340 mi s, what is the distance between her
and the wall?
~c-r-r- "7~istance A 136m B 272 m C 408m D 544m
What 1s the shortest time elapsed before the displacement at point Q
becomes zero?
20 Fi g. 20 shows a model that demonstrates the behaviour of gas molecules.
The steel balls which represent the gas molecules are made to bounce
A 0.0 1 s B 0.02 s c 0.04 s D 0.08 s
about in the glass tube by a vibrating piston. A small weight is placed on
the movable disc to maintain equilibrhJm.
17 The image of an object seen through a converging lens L is virtual,
magnified and upright. The focal length of the lens is as shown In the Movable disc
diagram below. Positions B and D are both 1 focal length away from the
optical centre. Steel ball
L
~
A B c D
j/ibrating piston
focal local - Fig. 20
lerY;j1h leoglh
The speed of the vibrating piston is now increased. State the effects this
change has on the inter-particle distances of the steel balls and on the
In which of the following positions, A, B, C or D. should the object be
placed? pressure exerted on the wall of the glass tube.
Pressure I lnter-carticle distances
A Increases I Increase
18 Fig. 18 shows the electromagnetic spectrum, w ith the blue end and red
end of the visible spectrum marked. ~ Increases Remains constant
c Remains constant - 1 Increase
D Remains constant - I Decrease
blue -----.. - - - - red
D
visfble
Fig. 18
Which section of the spectrum has waves that can make water molecules
vibrate vigorously?
487 488
--
!
489 490
21 When a liquid evaporates, some of its molecules escape from the surface 24 The resistance of an electric iron is 200 0. What is the resistance of
and the temperature of the liquid changes Which of the following describes another electric iron if it has half the power rating for the same supply?
the escaping molecules arid the change i11 temperature of the liquid?
A 500 B 100 0 c 2000 D 400 0
Escaping molecules Temperature of the liquid
A Low energy Increases 25 A constant-voltage source 1s connected with an ammeter and three
B Low energy Decreases resistors as shown below. The circuit can be completed by a movable
C High energy_ Increases contact M.
O High energy ~ Decreases ,
22 X, Y and Z are three identical metal cylinders standing on an insulating
plate and are in contact as shown in the diagram below. A positively
charged rod is held near X. What is the charge on each cylinder 1f Y is
removed while the charged rod is held in place?
..4- - ~ -
When M is connecled to X, the ammeter reads 0.6 A. M is then connected
+++~ x y z first to Y and then to Z, what are the ammeter readings?
AtY AtZ
A 0.2A 0.1 A
I
Positively
charged rod
Insulating plate

8
c
D

0.3A
0.6A
1.2A

0.2A
0.6A
1.8A
x y z
A - ' Neutral +
B Neutral - +
c - + +
D - - + -
23 A circuit has a current of 0.32 A. Given that the charge of an electron is
1.6 x 10-19 c, how many electrons will pass a fixed point in the circuit in
1 second?
A 1 x 10 18 B 2 x 10 18 :; 5 x 1018 D 5.12 x 10 18
489 490
------------------- - - -- - - - - --
491 492
26 The diagram shows a potential divider circuit with two identical lamps. 28 The following diagrams show different connections of a heater to a mains
supply. Which connection is the most dangerous?
LIVE Metal casing
Metal casin
p
Lamp 1 Heating element Heating elemenl
x
EARTH A B
Lamp 2
LIVE Metal ca sing
LIVE Metal casin
Healing element
What happens to the brightness of the lamps as the contact X is placed at NEUTRAL NEUTRAL
the point P of the potential divider?
EARTH
Lam 2 EARTH
c D
,29, A 2 kW heater and a 1 kW fan is connected in parallel to a 240 V mains
supply. Suggest a suitable rating for the main fuse in a household circuit
that can be used to protect the heater and the fan.
27 A signal is applied to a cathode ray oscilloscope and the trace appears as
shown in Fig. 27.1.
A 12 A B 12.5 A C. 13 A D 15A
30 An uncalibrated mercury-inijlass thermometer is shown below.
30mm
Fig. 27.1 Fig. 27.2
Which of the following adjustments could obtain a trace in Fig. 27.2?

,_
i
20mm

T
-10c
i
100c
120 mm
(1) Double the setting of the time base.
(2) Halve the setting of the time base: What is the temperature when the tip of the mercury thread is at T ?
(3) Double the frequency of the input signal.
(4) Halve the frequency of the input signal. A 1 C
B 10 C
A (1)or(3) c 11 C
B (1) or(4) D 23 C
<;; (2) or(3)
u (2) or(4)
491 492
493 494
31 The diagram below shows a thermocouple thermometer with the reference 34 Four bars, all exactly the same size, are placed with one end 1n boiling
junction placed in an ice bath at oc and the other junction A in steam at water. The time taken for the temperature of the other end to increase by
100C the e.m.f measured is 6.0 mV. 2C are measured.
Cu
~ \I
Junction A Material of bar Time for 2 C rise I s
Fe aluminium 10
) copper 5
I cork 800
Reierenc~ -k
1-t.; Ice bath
foam 1200
Junction
........_...
",r.
;;.,1 (0 ' C)
To make a large metal tank for storage of hot water with the least heat
loss, which materials should be used for the tank and its insulation?
Junction A 1s now removed from the steam and is placed in a liquid bath at I Tank Insulation
a constant temperature, with the reference junction remaining in the ice. A Aluminium Cork
The e.m.f measured is now -1.S mV. B I Aluminium Foam .
c I Cooper Cork
What is the temperature of the liquid? -ts I Coooer Foam ....
A -25 C B 25 C c 75 C o 125 C
32 8000 J of energy is used to heat up a 4 kg copper block from 1o0c to 1s 0c.
What are the heat capacity and the specific heat capacity of the copper
block?
35 Two permanent magnets are placed parallel to each other on a flat
He at c apacity I (J I K) Specific heat capa city I (JI (kg K)) horizontal surface. A plotting compass is placed on the spot as shown on
A 400 400 the diagram. In wh ich direction will the pointer of the plotting compass
point?
B 400 1600
e 1600
- 400 ,
s
0 1600 - 1600 A
B
-
+-

N
c t
33 A gas heater increases the temperature of 2.5 kg of water by 20 in 10. c
minutes. How long will it take for the same heater to cause the same 0 ~ s N
temperature nse for 15 kg of oil if the specific heat capacity of water is
twice that of oil?
A 20 minutes B 30 minutes C 60 minutes D 120 minutes
493 494
495 496
36 An iron rod is attracted to an electromagnet as shown_The spring meter 1s 37 Fig. 37 shows the top view of three parallel wires. The movable wire P is
firmly attached to the table and the iron rod pulls the spring meter by placed near wires Q and R which are both fixed in position al equidistance
means of an inelastic thread. The iron rod can move towards and away from P. The three wires each carry current of 1 A flowing in the direction
from the coil XY on the rollers below it Neglect frictional force between the shown.
iron rod and the table.
In which direction will wire P move?
Inelastic thread B
rheostat
Spring meter\ Iron rod
I c:::==::J f:--Lj () () J
A+-0- c
I I x y
Q l 0R
Which g raph below shows the variation of the rheostat reading with respect
D
to the spring meter reading when the switch is closed?
Fi g. 37
Rheostat reading I 0
A
38 The figure below shows the structure of a moving-coil loud-speaker. The
coil is free to move inside a cylindrical magnet. Fig. 38.1 shows the side
view while Fig. 38.2 shows the front view of the loud-speaker.
Spring meter reading I N
Side View Front View
Fi g. 38.1 Fig. 38.2
From the direction of current, which statement is correct?
A The coil rotates clockwise.
B The coil moves anti-clockwise.
C The coil moves forward.
1' The coil moves backwards.
495 496
497 498
39 Fig. 39.1 shows a magnet being pushed perpendicularly toward an iron coil 40 The diagrams show the relative motion between a magnet and a solenoid.
which is connected to an axis with a rigid light plastic rod The axis can A galvanometer is connected to the solenoid to show the presence of the
rotate freely about the stand. Fig. 39.2 shows the top view of the set-up. induced current.
Which diagram shows zero reading in the galvanometer?
-
Stand
-.....
--~
1::-----~ ~ ~
1::-----1
s
Fig. 39.1 Fig. 39.2 ..--.o
-A
~~
B
--
Which of the statement is incorrect?
A
B

The axis rotates clockwise when the N-pole of the magnet is pushed
toward the coil.
The axis rotates anticlockwise when the S-pole of the magnet is

$~ ~.
pushed toward the coil. ~ 0
C The speed of rotation of the axis increases with the speed of the
magnet.
O The speed of rotation increases when two iron coils are used instead
of one.
497 498
-- - . ---,--
499 500
Temasek P2 - Section A (SO marks)
Answer all the questions in this section 2 A uniform rod PQ of length 80.0 cm and weight 2.0 N is placed on the pivot as
Tal<e gravitational field strength on Earth to be 10N/kg. shown in Fig. 2 below. A spring balance is attached to the other end of the rod.
A load of 8 O N is placed 20.0 cm from the spring balance.
A parachutist with mass 70 kg jumps from a hot air balloon which is 4000 m
high. When he is 2000 m high, his falling speed is 60 mis At this height, he Spnng balance
pulls open the parachute and lands with a speed of 4 m/s.
(a) Calculate the change in the potential energy of the parachutist from the
time he jumped out of the hot air balloon to the time just before he opened his
parachute.
Change in potential energy =.................. [1 J
8.0 N
(b) (i) Calculate the work done against air resistance from the time he Fig. 2
jumped out of the hot air balloon to the time just before he opened his
parachute. (a) Calculate the reading on the spring balance in order for the rod to balance
horizontally?
Work done = . ... .. .. . ... .. . [3)
(ii) Hence, calculate the air resistance acting on the parachutist. Assume
the air resistance acts constantly. Reading on spring balance =............................ (2)
(b) Determine the magnitude and direction of the force acting on the pivot.
Air resistance =............... [2]
(c) Calculate the work done against air resistance in the last 2000 m of his
fall. Magnitude = ..........................
Direction = ............................ [2)
(c) If the 8.0 N weight is gradually moved along the rod towards Q , wh ile the
rod being kept horizontal, state and explain the change to the reading of
Work done =............... [2) the spnng balance.
(d) Explain the difference between (b) (i) and (c).
................................................................................................................. [2} ................................................................................................................. (2)
499 500
- -- -- - - - - - -- - ---- - - - -- -- - - -- -- - - -- - - --
501 502
3 Fig. 3.1 shows a mercury manometer connected to a tank containing gas A, 4 A small lamp L hangs above a marble floor, AB, as shown in Fig. 4.
and Fig. 3.2 shows the same manometer connected to another tank containing
gas B .
Plane mirr~~~
~
0
Gas A Gase
L
A Marble floor 8
Fig. 3.1 Fig. 3.2 Fig. 4
(a) Given that the atmospheric pressure is 76 cm Hg, determine, in cm Hg, Draw two light rays from the lamp that are reflected onto the floor to show the
(i) the pressure of gas A. range of reflection of light of the mirror. [2]
5 Fig. 5 shows parallel rays of light incident on a thin diverging lens. The points
labelled F show the principal focus on each side of the lens. [2]
Pressure of gas A= ......................... [1 ]
(ii) the pressure of gas B. F F
Fig. 5
Pressure of gas B =......................... (1]
Complete Fig. 5 to show the light rays after they pass through the lens.
(b) Tank A is connected to gas tank B by a U-shaped mercury manometer as
shown in Fig. 3.3. Using your answer in (a)(i) and (a)(ii). draw the mercury 6 Very large containers are used in some industrial processes and it is
level in each limb of the tube and indicate the difference in height between necessary to monitor the depth of liquid in the containers. This can be done
the two mercury levels. (1] using pulses of ultrasound. Fig. 6.1 shows a container and a device that emits
and receives ultrasound. Some of the ultrasound emitted is reflected back from
the surface of the liquid and no ultrasound is reflected from the walls of the
container.
Gas A Gas B
Ultrasound
transmitter and
receiver.
Fig. 3.3
Fig. 6.1
501 502
503 504
(a) Why 1s ultrasound used for this purpose rather than ordinary sound? 7 Fig. 7 illustrates a device, based on expansion of matter, designed to give an
audible warning when the temperature inside the cylindrical tank nses to a
particular value. The glass tube G, closed at one end, contains a sample of dry
air enclosed by a mercury thread. There is a small gap between the two
........ ........................................................................................... [1) electrical contracts C which are sealed through the sides of the glass tube .
( ====G=-M=e""rc"'dt'~
(b) A pulse of ultrasound emitted is displayed on a cathode ray oscilloscope Battery
(CRO) screen as shown m Fig. 6.2. The renected pulse is also displayed
on the screen. The time base of the CRO is set at 2 ms/cm.
~Bell
Tank
I =1:
l e
Fig. 7
(a) Explain briefly what will occur as the temperature inside the tank nses until
1 cm the warning is given.
.............
1 cm
Fig. 6.2
(i) Explain why the first pulse displayed on the CRO screen has greater
amplitude than the second pulse.
. .................................................................................................................(3)
........................................................................................................... (1)
(b) Using the kinetic theory of matter, explain why the volume of the air inside
G changes much more than that of the mercury thread.
(ii) Given that the speed of ultrasound through the liquid is 1500 mis,
calculate the depth of the liquid.
.................................................................................................................. (11
(c) (i ) When the atmospheric pressure increases, what observation will be
made about the position of the mercury thread?
Depth of liquid= ............................ (1)
........................................................................................................... [1 ]
503 504
505 506
(ii) What effect will an increase in atmospheric pressure has on the 9 Some pure ice placed in an insulated beaker is warmed up by a heater from a
temperature at which the warning is sounded? temperature of -2 C. Fig. 9 shows how its temperature varies with time.
Assume that the rate of heat supply is kept constant.
Temperature I C
........................................................................................................... [1]
~ ---- ----------- -......... -- .. -. -- .... - - - --- - - . - -..
8 Fig. 8 shows an electric circuit powered by a 12.0 V battery of negligible
resistance .
0 A p B Time / s
4.00 3.00
- 1_ -- . - -. - - - - -- ---- - - -- - --- . - -. - -- --- - - - -- . - ------ -..
-2
Fig. 9
(a) The temperature of the ice remains constant at OC for some time, as
shown from point A to point B in the graph, despite heat being continually
Fig. 8 supplied.
Determine the ammeter and the voltmeter readings when
State the changes to the physical property of ice and to the heat supplied
(a) the switch S is open.
that explain this observation.
..................................................................................................................(2)
Ammeter reading= ......................... (1) (b) State the content of the beaker at the time indicated by point P.
Voltmeter reading= ......................... [1)
(b) the switch S is closed. . ................................................................................................................. [1)
(c) Explain why the temperature rise from -2 C to 0 C is faster than that from
0C to 2 C.
Ammeter reading= ......................... [1] .................................................................................................................. [1 J
Voltmeter reading = ......................... [1]
505 506
507 508
10 Fig. 10.1 shows two horizontal plates X and Y connected to a source of high 11 Fig. 11 shows the structure of a d.c. motor which uses the moment of the
potential difference through a switch. The X plate has a small hole at its centre forces acting on the wires in the coil to tum.
The Y plate is earthed.
s
x magnet ..... f-~ -;1 magnet
't---1 J
I
Fig. 10.1
(a) On Fig. 10.1 , draw the electric field lines between the parallel plates. [1]
(b) A negatively charged oil drop is introduced into the region between the 11-- j.-- ----'
plates through the small hole as shown in Fig. 10.2. Fig. 11
s
(a) Explain how a current flowing 1n the coil produces a turning effect on the
x coil.
Oil drop e
y
Fig. 10.2
(i) By adjusting the source's potential difference, the oil drop is made to
be stationary.
Explain why the oil drop can remain stationary in air.
........................................................................................................... [1]
(ii) More batteries are added to the circuit to increase the source's .................................................................................................................. (3)
potential difference.
State and explain the observation that you would make about the oil (b) Explain why
d rop. (i) the spht-nng commutators are used in the d.c. motor.
. .......................................................................................................... [1)
...........................................................................................................(2) (ii) sometimes, the motor does not start when switched on .
(iii) State what happens to the charge on the oil drop when 1\ makes
contact with the Y plate.
. .......................................................................................................... [1]
........................................................................................................... [1]
507 508
- - - - - - ---- - - - - - -- - -- -- -- - - -- - - - - - - - - - - -------
509 510
Section B (a) After the heater was switched off, the temperature of liquid X continued to
Answer all the questions from this section. rise for a while before it started to fall. Explain why this was observed.
Question 14 has a choice of parts to answer.
12 Fig. 12 shows an experimental set-up to find the specific heat capacity of liquid
X. 0.25 kg of Liquid X at room temperature was kept in a glass cup and heated
by an immersion heater which has a power of 30 W .
.................................................................................................................. [2]
(b) Using data from the graph, calculate the specific heat capacity of liquid X.
(Ignore any thermal energy lost to the surroundings).
immer~n
healer
-- -- lid
Specific heat capacity of liquid X= ......................... [2]
liquidX----+- (c) Explain the term 'specific heat capacity of liquid X' using the value you
have calculated in part (b).
Fig.12
.................................................................................................................. [1 ]
The heater was switched on for 320 s and then switched off. The temperature
variation of liquid X was recorded and plotted against time as shown below_ (d) State and explain one precaution you might take in order to reduce the
thermal energy lost to the surroundings.
.................................................................................................................. (2)
(e) If the student carrying out the experiment forgot to cover the cup with the
lid, state and explain whether the value obtained will be higher or lower
than the value obtained in part (b).
................................................................................................................. [2]
(f) Using the same axes shown in the graph on page 12, sketch a new graph
to show how the temperature of liquid X wou ld vary with time if the
experiment is repeated using 0.50 kg of the liquid. [1)
509 510
-- - -----------------------------------~--------------
-'
511 512
13 (a) Fig. 13.1 shows an ideal transformer (b) The core of one transformer of the above type is cut in two as shown in
Fig. 13.2 and used as part of a design for a water depth indicator. The
bottom half of the core is fixed. The top half of the core is attached to a
pivoted beam and is free to move.
!3eam
240 v
a.c. inputL
!
Fig. 13.1
(i) Explain how the transformer works.
Fig. 13.2
An experiment on the water depth indicator was carried out with the above
set-up. An a.c. voltage is input into the transformer. The water depth was
varied and the voltmeter reading was taken. The results were shown in the
following table.
Water depth I cm 5.1 5.0 4 .9 4.8 4.7 4.6
Voltmeter reading I V 2.6 2.5 2.2 1.7 1.0 0.1
........................................................................................................... (3)
(i) Explain why the voltmeter reading falls as the water depth decreases
(ii) The turns ratio of the transformer is 0.75. An a.c. voltage of 240 Vis
input into the transformer, calculate the output voltage.
Output voltage= ....................... (1]
........................................................................................................... (3]
(iii) State one feature of the iron core that makes the transformer ideal. (ii) The 0.1 V value is difficult to read reliably. State and explain one way
in which a larger voltage could be obtained for the same water depth.
........................................................................................................... (1]
........................................................................................................... [2]
511 5 12
~--------------- - - - - - -- - - - - - - - - -- - - - - - - - - -----
513 514
14 Either (b) Explain how and why the fuel leak affected the motion of the rocket.
A rocket of mass 105 kg was launched vertically into the sky from the ground.
Some time later, a leak in the fuel tube leading to the rocket engine was
detected. T he leak significantly reduces the upward force exerted on the rocket
by its engine. The rocket continued to ascend even after all its fuel had been
used up. The rocket, after reaching the maximum height, then fell freely from
the sky. Knowing that the launch was unsuccessful, the ground control .................................................................................................................. (1)
destroyed the rocket in mid air using a pre-programmed explosive device. The
motion of the rocket is represented by the velocity-time graph in Fig 14. (c) Explain why the rocket continued to ascend even after all its fuel had been
used up.
Velocity I mis
50
401---+--t--,1!"--!--~--,.-;.--_,..-_,...___.,__+-_..._--+
~l---+---1"-!--+--+---"o.--;--....._-+---<;---+--+---.
201--~----'---l---'--"..,._--I-_,_+-,__-+--+---+ ................................................................................................(2)
10 .......,._..__,__,__.....__.___,_ +-.3i.--+-!---'---+---+
o 1t---+-1---_;___:_ _ _.._~_.__....__.__ _ Time I min (d ) State the time at which he rocket reached the maximum height.
-10 I---+---'----'---+-~-+--".--+---+ .................................................................................................................. (1)
-20 l---+--+--'--!---1----'- + ---1--+-;...-..:>i..--+---l
-30 I---+--+--+--+---'-.,._-'---+-'--+--+---+
(e) Calculate the maximum height reached by the rocket.
401---+--+-'---'---i----'- +--'--+-!---'--+---+
~01--~----'---'---'----'-~-'----'--'---'---'---'
Fig. 14
(a) Calculate
(I) the acceleration of the rocket before the fuel leak was discovered.
Maximum height "' ......................... (2)
(f) Calculate the height from the ground at which the rocket was destroyed.
Acceleration = ......................... (1)
(ii) the initial lifting force of the rocket.
Initial lifting force "' ......................... (2) Height from the ground= ......................... [1)
L
513 514
515 516
14 OR (ii) Explain why the light ray behaves differently at room temperature and
Fi g. 14.1 and Fig. 14.2 show respectively the side-view and front view of an at-80 'C.
optical fibre. The core 1s made of glass of refractive index 1.47 and the
cladding is made of plastic of refractive index 1.45. Both refractive indices are
measured at the room temperature.
Cladding
'
1---------+ Core
- Cladding
........................................................................................................... [2]
l+ Cladding Core
(c) Optical fibres are used in medical field when 1t 1s necessary to look inside a
Fig. 14.2 pat1enrs stomach as shown in Fi g. 14.6 A bundle of light fibres is used to
Fig. 14.1
pass light into the stomach. A second set of fibres collects the light
reflected inside, and this allows the doctor to examine the intenor of the
(a) Calculate the critical angle of the glass. stomach as shown in Fig. 14.7.
Viewing device
~
..... Light from strong
source
Critcal angle =......................... [2]
(
(b) It 1s known that the refractive index of the cladding is related to its
temperature. Fig. 14.3 shows how the refractive index of the cladding I
changes with the temperature. The refractive index of the core is given for Inside patient
comparison. Fig. 14.6 Fig.1 4.7
Refractive Index
Give two reasons to explain why many thin fibres are used to make a
bundle of light fibres instead of a few thick fibres.
\\~--------
f Core
Cladding
- ) Temperature I c
-100-80-50 0 so 100
......................................................... ......................................................... [2]
Fig. 14.3
(i) Fig. 14.4 and Fig. 14.5 show a ray of light entering the optical fibre at (d) State another application of optical fibres and one advantage of using
the room temperature and at -80 C respectively. Complete the path optical fibres in such an application.
of the light ray in each diagram to show what happens to the light ray
upon hitting the core-cladding boundary inside the optical fibre. [2]
At room temperature At - 80 C
Cladding Cladding
~,.._-------{+ ' Core )~--------1+ ' Core
.................................................................................................................. (21
Light ray t+ Cladding
Ltght ray L; Cladding
Fig.14.4 Fig. 14.5
515 516
Sec 4 Express Physics Prelim Exam 2010
=
(ii) Work done force x distance moved 1m for method
=
1 274 000 f x 2000 1m for final answer.
f = 637 N
Marks Scheme
Air resistance = 637 N
(c) Work done against air resistance 1m for method
A =70x10x2000 + Yo x70x60 2 - Y. x70x42
1 c 11 B 21 D 31
= 1 525 440J
2 c 12 D 22 A 32 c =1 500 000 J or 1.5 x 106 J 1 m for final answer.
3 D 13 c 23 B 33 B
4 c 14 B 24 D 34 B
(d) Air resistance increases, 1m
5 8 15 c 25 A 35 B Because of a larger area of contact of parachute with 1m
6 B 16 B 26 D 36 D
air.
7 8 17 c 27 A 37 c
8 A 18 c 28 c 38 c 2 (a) Taking moment about P,
9 B 19 B 29 c 39 B For rod to be in equilibrium,
10 A 20 c 30 A 40 D Total clockwise moment= Total anticlockwise moment
2x40 + 8x60 = Fx80 1m
F= 7N 1m
(b) Total upward forces =total downward forces
7 + Fp = 2+8 1 m magnitude
FP = 3 N downw<1rd 1 m direction
Section A
(a) Change in height= 2000 m (c) Reading of the spring balance increases. 1m
Change in potential energy As 8.0 N load moves towards Q, its clockwise moment
= 70x10 x 2000 = 1400000 J 1m about P increases. Thus, the spring balance needs to
exert a larger upward force so as to aeate a larger 1m
(b) (i) KE at 2000m height 1m for KE anticlockwise moment to balance the now larger total
= Yo mv2 = Yo x70x602 = 126 000 J calculation clockwise moment.
Work done against air resistance 3 (a) Pressure at A = 76 cm Hg + 30 cm Hg = 106 cm Hg 1m
=Change in potential energy- KE at 2000m height
= 1400000 - 126000 1m for method (b) Pressure at A= 76 cm Hg - 1O cm Hg= 66 cm Hg 1m
= 1274000 J 1m for final answer.
(c)
1m
Total energy at hot air balloon The height
Gas A difference between
= =
mgh 70x10x4000 = 2 800 000 J
the two mercury
Total energy at 2000rn height levels must be
clearly labeled.
= mgh +Y. mv2
2
= 70x1 Ox2000 + 0.5x70x60
= 1 400 000 + 126 000 = 1 526 000 J
Work done against air resistance
=
= 2 800 000- 1 526 000 1 274 000 J
518
517
519 520
4 L' 7 (a) Volume of dry enclosed air increases. 1m
[2m) 1m for drawing The mercury thread is pushed towards C.
each of the two light The mercury thread joins the contacts in C. The 1 m
rays with equal external circuit is closed, causing the current to flow 1 m
incident and and the bell to ring.
reflected angles.
(b) The intermolecular force (bond) between air particles is
much weaker than that between mercury particles. 1m
Hence, it is much easier for air particles to move further
apart to occupy a bigger volume.
A
;
p B (c) (i ) The mercury thread moves towards the left. 1m
!
!.- Range of reflection of light _ ,J (ii) Temperature needs to be higher in order for the 1m
warning sound to be activated.
8 1m
5 (a) (i) Ammeter reading = ~ = __!2_ = I.7 IA 1m
R 3+4
1 m for centre ray Voltmeter reading =O V
passing without
bending.
..) I I I
(II -- = -+--~
R,.,. , 3 9 +3
R,.,, = 2.40
1 m for 2 light rays
diverging outward Ru>tI = 2.4 + 4 = 6.40
from F.
Ammeter reading = I = !:.. = E_ = I .88A 1m
R 6.4
2.4
V,.,., = . x12 = 4.SV
64
9
6 (a) Ultrasound has a distinct frequency and will not be 1 m Voltmeter reading= - - x 4.5 =3.3 75 "'3.38V 1m
confused (interfered) with any surrounding sound. 3+ 9
(bl (i) Some of the sound energy is absorbed by liquid X 1m
and walls of the container. 9 (a) Ice changes from solid to liquid state I Ice melts. 1m
Heat supplied is changed to the potential energy 1m
O R Some of the sound energy is converted to the KE of
the molecules of liquid X and that of the molecules of between water molecules.
{l>) A mixture of solid and liquid . 1m
the wall of the container.
(c) Ice has a smaller specific heat capacity than water. 1m
As the reflected sound has less energy . so its
amplitude is smaller.
10 {a) Parallel electric field lines that are equally spaced out 1m
with arrows pointing downward.
") 2d -
v= - ~bOO=
2d ~d=4.25m
( II 1m
I 3x2x10-' (b) (i) The negatively charged drop is attracted to the 1m
plate X which has a higher potential. Thi s upward force
is equal to the weight of the oil drop that is acting
downward.
519 520
i-- r- -~
, 521
I
I (ii) Oil drop accelerates I moves upward. 1m (f)
The upward electric force is greater than the weight 1m
I
I
of the oil drop as the electric field is stronger.
T!Mlperaturefc
[1]
I
(iii) The charge will be discharged to the ground. 1m
11 (a) When current flows in the coil, a magnetic field is 1m
formed around the coil.
This magnetic field interacts with the magnetic field 1m
between the two magnets to produce a downward force
on AB and an upward force on CD, according to
Fleming's Left-hand Rule.
These two forces combine to produce a rotating effect 1m
on the coil.
(b) (i) The split-ring commutators allow current in the coil to 13 (a) (i)Vue to magnetic effect of electric current, a magnetic
change direction after half a turn, so as to keep the coil field is produced in the primary coil when current flows
rotating in the same direction. 1m through 1t . [1}
Magnetic hnes of force from the primary coil are cut by
(ii) It does not rotate as the coil is in a vertical position. the secondary coil. The soft iron core concentrates
At this point, the current is cut off because the split- the magnetic lines of force inside it [1]
ring commutators are not in contact with the carbon As alternating current supply is used in the primary
brushes. 1m coil, it will create a continually changing magnetic field
which will induce a current in the secondary coil. [1]
Section B
12 (a) The liquid around the heater is at a higher temperature, [1] (ii)Output voltage= o/. x 240 =180 V [1]
so there is still heat flow to the thermometer.
(ii1l Laminated soft iron core. [1]
As there is a distance between the heater and the
thermometer, it takes some time for heat to reach the [1] (b)(i) Depth decreases. wooden block in lowered. [1]
thermometer The top half of the iron core raises and the gap size
between the top and bottom half of the iron core
(b) mce =Pt increases. [1]
0.25 x c x 54 =30 x 320 (1] The flux linkage will then decrease. I
c = 711 J/(kg C) [1 ] The efficiency of the transformer decreases. [1]
(c) 711 J of thermal energy (heat) is needed to increase [1] (ii) Increases supply voltage. [1]
the temperature of 1kg of liquid X by 1 C. Same turns ratio, hence output voltage increases. I
Output voltage is proportional to the input voltage. [1]
(d) Cover the glass cup with a layer of insulation, so as (1]
to reduce heat loss by conduction. (1] Or:
Reverse tum ratio I more secondary turns. [1]
(e) Higher value of c. [1 ] Output voltage increases as the ratio of secondary
Heat is lost by convection and evaporation, so the rise voltage to input voltage is proportional to the turns
in the temperature of liquid X is smaller. [1] ratio. [1]
521
- - - -- - - - - -- .. - - - - - - -- - - - - - - -- - - - -- - - - -
14 (a)(i) Acceleration: If the angle of incidence at the core-cladding
a= (v-u)/t boundary is greater than 42.8,
a= (20- 0) I (1x60) = 0.33 ms2 [1}
(ii) Initial lifting force= mg+ ma
= 10x 10 + 10x 0.33 [1]
=1.03x106 N [1] [1 ]
(b) -The acceleration of the rocket decreases,
-because the lifting force decreases. [1]
(iii) At room temperature, the refractive index of the
(c) At the moment the fuel had been used up, the rocket cladding is slightly less than that of the core. Total
possess kinetic energy. (1] internal reflection occurs and the light ray is
The air resistance causes it to continue to ascend with transmitted in the core along the optical fibre. [1]
deceleration until its velocity decreases to zero. (1]
At -80C, the refractive index of the cladding is
(d) 9 minutes after launch. [1] equal to that of the core. The cladding and the core
are like a same medium, thus the light ray passes
(e) Maximum height reached through them without bending. [1 ]
='/:zX20x(1x60) + 'hx(20+50)x(3x60) + 'hx50x(5x60) [1]
=14400 m [1] (c ) -More rays of light can be transmitted into the stomach
to make it brighter. (1]
(f) Height of descent= 'h x 30 x (3x60) =2700 m
-If a few of the optical fibres fail to work, the instrument
Height of the rocket when destroyed can still be used with the remaining optical fibres. [1]
= 14400-2700 = 11700m [1]
(d) -Optical fibres are also used in telecommunication. [1]
14 OR
-A lot of information can be transmitted at the same [1]
(a) sin C = 1/ n time.
sin C = 1/1 .47 [1]
c =42.8 [1 ]
(b) (i) At room temperature
C ladding
(ii)

..~"'''"'
,
At -ao0c
If the angle of incidence at the core-cladding

[1]
boundary is smaller than 42.8,
"'"'~ [1]

You might also like